Superpage
Colon

Authors: Safina Ahmed, M.B.B.S., Vidya Arole, M.D., Nalini Bansal, M.D., Adrian C. Bateman, M.B.B.S., M.D. , Phoenix D. Bell, M.D., M.S., Bindu Challa, M.D., Lina Chen, M.B.B.S., M.Sc., M.Med., Wei Chen, M.D., Ph.D., Kenrry Chiu, M.D., C.M., Yevgen Chornenkyy, M.D., M.Sc., Mona Deerwester, M.D., M.Sc., Chien-Kuang Cornelia Ding, M.D., Ph.D., David K. Driman, M.B.Ch.B., Andrew L.J. Dunn, M.D., Anna Sarah Erem, M.D., Ashwini Kumar Esnakula, M.D., M.S., Michael Feely, D.O., M.J. Fernández-Aceñero, M.D., Ph.D., Jennifer Findeis-Hosey, M.D., Giby V. George, M.B.B.S., Raul S. Gonzalez, M.D., Hanni Gulwani, M.B.B.S., Catherine E. Hagen, M.D., Saroona Haroon, M.B.B.S., Christopher Hartley, M.D., Lewis A. Hassell, M.D., David Hernandez Gonzalo, M.D., Masoumeh Peykan Heyraty, M.D., Rachel Horton, D.O., Zarrin Hossein-Zadeh, M.D., Aaron R. Huber, D.O., Shilpa Jain, M.D., Albina Joldoshova, M.D., Maria Kamal, M.D., Tulsi Kantu, Raj P. Kapur, M.D., Ph.D., Gagandeep Kaur, M.D., Enoch Kuo, M.D., Lili Lee, M.D., Subramanya Sakaleshpura Mallikarjunappa, M.B.B.S., M.D., Carolina Martinez Ciarpaglini, M.D., Ph.D., Blaine A. Mathison, B.S., Gustavo Moreno, M.D., Byoung Uk Park, M.D., Dustin W. Parsons, M.D., Ameya Patil, M.D., M.S., John D. Paulsen, M.D., Nat Pernick, M.D., Isma Perveze, M.D., Amber Petrolla, M.D., Maryam Kherad Pezhouh, M.D., M.Sc., Alexandros D. Polydorides, M.D., Ph.D., Varsha Prakash, M.D., M.S., Bobbi S. Pritt, M.D., M.Sc., Nicole E. Putnam, Ph.D., Amy Rapkiewicz, M.D., Luisa Ricaurte Archila, M.D., Arvind Rishi, M.D., M.B.B.S., David F. Schaeffer, M.D., Saba Shafi, M.D., Pauline Shih, M.D., Charanjeet Singh, M.D., Lei Sun, M.D., Ateeqa Mujeeb Ullah, M.D., Benjamin J. Van Treeck, M.D., Neha Varshney, M.D., Monika Vyas, M.D., Elliot Weisenberg, M.D., Kwun Wah Wen, M.D., Ph.D., Martha M. Yearsley, M.D., Feng Yin, M.D., Ph.D., Lizhi Zhang, M.D.
Former Editors-in-Chief: Raul S. Gonzalez, M.D.
Deputy Editors-in-Chief: Raul S. Gonzalez, M.D., Catherine E. Hagen, M.D., Aaron R. Huber, D.O.
Editor-in-Chiefs: Raul S. Gonzalez, M.D., Debra L. Zynger, M.D.

Copyright: 2003-2024, PathologyOutlines.com, Inc.

GI related: Jobs, Fellowships, Conferences, Cases, CME, Board Review

Related chapters: Anus & perianal, Appendix, Small intestine & ampulla

Editorial Board oversight: Naziheh Assarzadegan, M.D. (last reviewed November 2023)
Page views in 2024 to date: 38

Aberrant crypt foci
Definition / general
  • Earliest neoplastic lesion of colon
  • May predict future adenoma or carcinoma (Am J Gastroenterol 2006;101:1362, Am J Gastroenterol 2005;100:1283)
  • Endoscopy: aberrent cryptic foci are stained darker than normal crypts with Methylene blue using high magnification chromoendoscopy, which helps in obtaining a targeted biopsy
Prognosis and treatment
  • ACF are markers of increased colorectal cancer risk, particularly those with dysplastic features (J Surg Oncol 2008;98:207)
Microscopic (histologic) description
  • Crypts are 2 - 3x larger and more dilated than normal crypts, may have a foci of dysplasia
  • ACF can be subclassified as dysplastic, nondysplastic or mixed type
  • Dysplastic ACF resemble adenomas
Molecular / cytogenetics description
  • Dysplastic ACF are present both in familial adenomatous polyposis (FAP), which is the result of a germline mutation of the APC gene; and in sporadic colorectal carcinomas, due to mutations of KRAS (Am J Pathol 2005;166:1069)

Actinomycosis
Definition / general
  • Infection of large intestine by actinomycotic microorganisms
  • Colonic actinomycosis is a subset of abdominal actinomycosis, but some sources do not discriminate between actinomycosis of abdomen and pelvis
  • Other common sites are oral cervicofacial actinomycosis ("lumpy jaw", see secondary chronic osteomyelitis), skin, thoracic and pelvic (cervix, kidney, ovary)
Epidemiology
  • Rare, but abdominal actinomycosis comprises 15 - 20% of actinomycosis in humans
  • Males are more commonly affected
Sites
  • Most common site is cecum, most often with concurrent ileal disease
  • Some cases are secondary to disease in appendix or female genital tract
Pathophysiology
  • Due to infection by anaerobic or microaerophilic, saprophytic bacteria of genus Actinomyces, a normal commensal of mouth, large intestine, vagina
  • Disruption of mucosa is necessary for disease - causes are trauma, surgery, diverticular disease, ingestion of foreign body, or less commonly other inflammatory processes
    • In surgery related cases, disease may appear months after perforation leading to appendectomy or bowel resection, but without evidence of actinomycosis at the time
  • In some cases, no inciting event is identified; rare cases may originate through hematogenous spread
  • While actinomycotic colonies may dominate the histologic picture, it is likely that companion, or "co-pathogenic" bacteria may also be necessary for disease to occur, possibly by lowering oxygen tension
Etiology
  • Actinomyces israelii is the most common etiologic agent; however, at least five other species are implicated in human disease
  • Using comparative 16s ribosomal RNA, some bacteria traditionally associated with actinomycotic disease have been reclassified as Aracanobacterium, Actinobaculum or Cellulomonas
Clinical features
  • Usually indolent disease, but more severe in immunocompromised patients
  • Patients usually present with non-specific symptoms including abdominal pain, fever, change in bowel habits, sensation of a mass
  • Imaging studies reveal a mass-like lesion, cystic lesion or an abscess, often suggestive of malignancy
  • May coexist with other inflammatory lesions of colon with fistula or sinuses
  • Clumps of yellow bacteria known as sulfur granules are characteristically present
  • No cases of human to human transmission have been reported and Actinomyces has not been cultured from nature
Diagnosis
  • Usually by tissue biopsy or resection where the characteristic microorganisms are seen
  • Abdominal actinomycosis is rarely suspected clinically and most patients are diagnosed after major surgery
  • Diagnosis by microbiologic culture is uncommon as strict anaerobic conditions are necessary and antibiotics interfere with growth in culture
Prognostic factors
  • Most patients respond well to therapy
Case reports
Treatment
  • Prolonged high dose antibiotics
  • Even with advanced clinical disease, most patients can be effectively treated with medical management alone; however, this is very uncommon with abdominal disease
Gross images

Images hosted on other servers:
Missing Image

Sigmoid colon mass

Microscopic (histologic) description
  • Characteristic clumps of basophilic filamentous bacteria in a vaguely rosette-like configuration surrounded by acute inflammatory cells are characteristic
  • Acute inflammation is accompanied by dense fibrosis described as "woody"
  • Eosinophilic clubs may be found at periphery (Splendore-Hoeppli phenomena)
  • Granulomatous inflammation may be present
Microscopic (histologic) images

Contributed by Asmaa Gaber Abdou, M.D.
Missing Image Missing Image Missing Image

Intestinal actinomycosis



Images hosted on other servers:
Missing Image Missing Image

Bacteria and sulfur granules

Missing Image Missing Image

Colonies of actinomyces

Missing Image

Gram+ staining

Missing Image

Filamentous microorganisms (GMS)

Positive stains
  • Gram positive, GMS, basophilic with H&E
Negative stains
Differential diagnosis

Acute self limited colitis
Definition / general
Essential features
Terminology
  • Acute self limited colitis
  • Infectious colitis
ICD coding
  • ICD-10: K52.9 - noninfective gastroenteritis and colitis, unspecified
Sites
  • Large intestine
Pathophysiology
  • Most commonly associated with bacterial enterocolitis (Int J Mol Sci 2020;21:4748)
  • Bacterial virulence factors include:
    • Adherence to epithelial cells
    • Enterotoxins
    • Invasion factors
    • Cytotoxicity
  • Adherence:
    • Via fimbriae or pili
    • Process of adherence destroys the microvilli brush border
  • Enterotoxins:
    • Toxins bind to cell membrane, enter cells, activate massive electrolyte secretion (cholera toxin, E. coli heat labile and heat stable toxins produce traveler's diarrhea)
    • No white blood cells in stool
  • Invasion factors:
    • C. jejuni, Salmonella and Shigella species, E. coli and Y. enterocolitica are organisms that invade the tissue and cause epithelial injury and death
    • These organisms invade via microbe simulated endocytosis, then cause cell lysis and cell to cell spread
    • Tissue invasion also results in the production of inflammatory cytokines that trigger an acute inflammatory reaction
    • Cytotoxins are polypeptides that cause tissue injury by inhibiting protein synthesis, disrupting tight junctions and depleting adenosine triphosphate (ATP) within cells
    • Endothelial injury results in the activation of the coagulation cascade that ultimately leads to an ischemic colitis pattern of tissue injury (Odze: Odze and Goldblum Surgical Pathology of the GI Tract, Liver, Biliary Tract and Pancreas, 3rd Edition, 2014)
  • Cytotoxicity:
    • Shiga toxin, enterohemorrhagic E. coli
Etiology
  • Wide variety of pathogens, most commonly bacterial organisms such as Campylobacter jejuni, Salmonella, Shigella species, E. coli and Yersinia enterocolitica (Odze: Odze and Goldblum Surgical Pathology of the GI Tract, Liver, Biliary Tract and Pancreas, 3rd Edition, 2014)
    • Clinical manifestation of acute self limited colitis is related to the ability of microorganisms to invade mucosa or produce enterotoxins
    • Ingestion of preformed toxins (Staphylococcus aureus, Vibrio cholera, Clostridium perfringens)
    • Infection by enteroinvasive organisms which invade and destroy mucosal epithelium cells
    • Infection by viral organisms (cytomegalovirus [CMV], herpes simplex virus [HSV], HIV, etc.)
Clinical features
  • Abdominal pain, watery or bloody diarrhea (Int J Mol Sci 2020;21:4748)
  • Causes symptoms within hours (including explosive diarrhea)
  • Sudden onset, early fever, often with numerous (> 6) bowel movements daily
  • Complications of dehydration, sepsis, perforation can occur secondary to potential massive fluid loss and loss of mucosal barrier
  • Inflammatory process usually resolves completely within 2 - 4 weeks
Diagnosis
  • Stool cultures (Gastroenterology 1994;107:755)
  • Colonoscopy with mucosal biopsy
  • Imaging studies can show nonspecific thickening of the bowel wall; however, they are seldom performed
Laboratory
  • Peripheral blood and fecal leukocyte count
Radiology description
  • Limited role since inflammatory abnormalities are nonspecific (e.g., colonic wall thickening)
Prognostic factors
  • Depends on specific underlying infectious agent (Gastroenterology 1994;107:755)
  • Immune status of host
  • Generally good with appropriate therapy
  • Commonly self limited disease (2 - 4 weeks)
Case reports
Treatment
  • Supportive therapy with rehydration
  • Rarely may require antibiotics or steroids
Clinical images

Images hosted on other servers:

Colonoscopy

Gross description
  • Mucosal erythema, erosion, friability (Case Rep Infect Dis 2012;2012:810943)
  • Ulceration, erosion, pseudopolyps, hyperemia
  • Exudates may be present (pseudomembranes)
  • Mass lesions are unlikely
Microscopic (histologic) description
  • Inflammation of lamina propria (active much more than chronic), edema, hemorrhage (Br Med J (Clin Res Ed) 1984;289:270)
  • Usually lacks features of chronicity:
    • Crypt architectural distortion
    • Basal lymphoplasmacytosis
    • Pyloric gland metaplasia
    • Paneth cell metaplasia in the left colon
  • Neutrophil induced epithelial injury (cryptitis or crypt abscess)
  • Severe cases have crypt abscesses, extensive necrosis, hemorrhage and microthrombi
  • Over time, neutrophils disappear (within second / third weeks) with persistent monocytic infiltration
  • Colonic lymphocytosis can be seen in resolving phase
  • Epithelial vacuolization, nuclear disarray, lymphocytosis, intranuclear inclusion can be seen in viral etiologies
Microscopic (histologic) images

Contributed by Lili Lee, M.D.

Active colitis

Cryptitis

Sample pathology report
  • Colon, cecum, endoscopic biopsy:
    • Active colitis (see comment)
    • Comment: The cecal biopsy shows colonic mucosa with a variable degree of crypt architectural distortion and neutrophil mediated epithelial injury, diagnostic of active colitis. No granulomas or evidence of dysplasia is identified. The findings are nonspecific and may be seen in the setting of intestinal infection, medication / drug associated injury and idiopathic inflammatory bowel disease. Definitive diagnosis will require synthesis of all available clinical, endoscopic, radiologic and pathologic evidence.
Differential diagnosis
  • Inflammatory bowel disease:
    • Chronic mucosal injury, crypt distortion, basal lymphoplasmacytosis, pyloric gland metaplasia (Cureus 2017;9:e1817)
  • Irritable bowel syndrome:
    • Type of functional gastrointestinal disorder
    • These problems cause the digestive tract to be very sensitive and change how the bowel muscles contract; the result is abdominal pain, diarrhea and constipation (Inflamm Bowel Dis 2018;24:2479)
  • Adverse drug reaction:
    • Apoptosis and intraepithelial lymphocytosis could be observed
Board review style question #1

A 35 year old, previously healthy female complained of right lower abdominal pain and intermittent diarrhea. The clinical impression was acute appendicitis. An appendectomy was performed. Grossly, the appendix appeared normal. Histologically, there was patchy chronic and acute inflammation as shown in the image above. Which of the following statements about this infection is correct?

  1. A patient with this infection is likely to die within 6 months
  2. Long term antibiotics are the treatment of choice in this patient
  3. The patient most likely has a colitis and requires supportive therapy
  4. This infection may be disseminated and tests to discover additional foci are mandatory
Board review style answer #1
C. The patient most likely has a colitis and requires supportive therapy. This patient most likely has CMV colitis demonstrated by nuclear and cytoplasmic inclusions, commonly referred to as owl eye inclusions, which are Cowdry type A inclusions. CMV colitis can mimic chronic idiopathic inflammatory bowel disease, graft versus host disease and acute appendicitis.

Comment Here

Reference: Acute self limited colitis
Board review style question #2
Paneth cell metaplasia is an indicator for chronic mucosal injury in which part of the colon?

  1. Ascending colon
  2. Cecum
  3. Sigmoid colon
  4. Terminal ileum
  5. Transverse colon
Board review style answer #2
C. Sigmoid colon. Paneth cells are normally absent in the left colon. When present, they serve as an indicator for chronic mucosal injury. However, Paneth cells are normally present in the right colon; hence, sigmoid colon is the correct choice. If Paneth cell metaplasia is seen in the sigmoid colon, this is an indicator for chronic mucosal injury.

Comment Here

Reference: Acute self limited colitis

Adenocarcinoma
Definition / general
Essential features
  • Most common primary colon carcinoma
  • Typically arises through chromosomal instability pathway (70 - 80%) or microsatellite instability pathway (10 - 15%)
  • Stage is most important prognostic factor
Clinical features
  • Increased carcinoma risk in patients with polyposis syndromes, Lynch syndrome and inflammatory bowel disease
  • Right sided tumors cause anemia, weakness and fatigue
  • Left sided tumors cause change in bowel habits (diarrhea or constipation)
  • Superficial tumors only rarely cause lymph node metastases due to distribution of lymphatics in colon
Diagnosis
  • Generally discovered on colonoscopy and confirmed on biopsy
Prognostic factors
Case reports
Treatment
  • Surgical resection is generally required unless tumor is small and confined to a polyp
  • Adjuvant therapy given for patients with lymph node metastases
  • Neoadjuvant therapy often given for rectal carcinomas
Gross description
  • Usually single, polypoid or ulcerated mass
  • May cause serosal puckering if muscularis propria is involved
  • Right colon tumors tend to be polypoid and exophytic, while left colon tumors tend to be annular, encircling lesions
Gross images

Images hosted on other servers:
Missing Image

Early, flat tumor

Missing Image

Flat, small adenocarcinoma

Missing Image

Invasive mucinous adenocarcinoma

Missing Image Missing Image

Exophytic lesion

Microscopic (histologic) description
  • Usually well or moderately differentiated gland forming carcinoma with marked desmoplasia, particularly at edge of tumor
  • Glands often cribriform and filled with necrotic debris (dirty necrosis), in both primary and metastatic sites
  • Inflammatory cells and scattered neuroendocrine cells are common (Pol J Pathol 2005;56:89)
  • Intramural venous invasion may be easier to identify using an elastin stain (J Clin Pathol 2002;55:17)
  • Well differentiated:
    • 15 - 20% of all carcinomas
    • Well formed glands or simple tubules with uniform, basally oriented nuclei
    • Somewhat resembles adenomatous epithelium
  • Moderately differentiated:
    • 60 - 70% of all carcinomas
    • Tubules may be simple, complex or slightly irregular
    • Nuclear polarity lost
  • Poorly differentiated:
    • 15 - 20% all of carcinomas
    • Less than 50% gland formation
    • Majority of tumor (excluding advancing edge) consists of sheets of cells without gland formation
    • Usually right sided (Hepatogastroenterology 2004;51:1698)
  • Note: preoperative histologic grading is not accurate (J Med Assoc Thai 2005;88:1535)
Microscopic (histologic) images

Contributed by Raul S. Gonzalez, M.D.

Low power

High power

Dirty necrosis

Dirty necrosis

Cribriforming

Cribriforming

Desmoplasia

Desmoplasia

Lymphovascular invasion

Lymphovascular invasion



Contributed by Semir Vranic, M.D., Ph.D. and Beverly Wang, M.D.

Poorly differentiated adenocarcinoma

Adenocarcinoma



Images hosted on other servers:
Missing Image

Whole mount scan

Missing Image

Moderately differentiated

Missing Image

Dirty necrosis in gland lumens

Missing Image

Venous invasion


Missing Image

Serosal penetration

Missing Image

Detached carcinoma cells

Missing Image

Signet ring morphology

Missing Image

Lymph node metastasis

Negative stains
Molecular / cytogenetics description
  • Most commonly mutated genes include APC, TP53 and KRAS
  • Molecular classification of carcinomas has been proposed (Histopathology 2007;50:113)
  • Tumors can be screened for microsatellite instability via immunohistochemistry for MLH1, MSH2, MSH6 and PMS2
Videos

Histopathology colon adenocarcinoma

Sample pathology report
  • Sigmoid colon, resection:
    • Adenocarcinoma, moderately differentiated (see synoptic report)
Differential diagnosis
Board review style question #1
    Which of the following is an official WHO recognized subtype of colorectal adenocarcinoma (per the 2019 classification)?

  1. Adenosquamous carcinoma
  2. Clear cell carcinoma
  3. Cribriform comedo carcinoma
  4. Low grade tubuloglandular adenocarcinoma
Board review style answer #1
A. Adenosquamous carcinoma

Comment Here

Reference: Adenocarcinoma
Board review style question #2
    Which of the following is true about colon cancer?

  1. Commonly mutated genes include APC, TP53 and KRAS
  2. Most cases are poorly differentiated
  3. Most cases are positive for CK7 and negative for CK20 and CDX2
  4. Superficial / early tumors metastasize often
Board review style answer #2
A. Commonly mutated genes include APC, TP53 and KRAS

Comment Here

Reference: Adenocarcinoma

Adenoma overview
Definition / general
  • Sometimes referred to as conventional adenoma to be distinguished from serrated lesions
  • Benign, premalignant neoplasm composed of dysplastic colorectal epithelium
Essential features
  • Premalignant lesions
  • At least low grade dysplasia; absence of true invasion
  • Subtypes include tubular adenoma (most common), villous adenoma, tubulovillous adenoma and advanced adenoma
Terminology
  • Conventional colorectal adenoma
ICD coding
  • ICD-O:
    • 8210/0 - adenomatous polyp, NOS
    • 8210/2 - adenocarcinoma in situ in adenomatous polyp
    • 8211/0 - tubular adenoma, NOS
    • 8212/0 - flat adenoma
    • 8221/0 - multiple adenomatous polyps
    • 8221/3 - adenocarcinoma in multiple adenomatous polyps
  • ICD-11: 2E92.4Y & XH7SY6 - other specified benign neoplasm of the large intestine & tubular adenoma, NOS
Epidemiology
  • High incidence in populations with diets typical of high income countries with a sedentary lifestyle
  • High risk population overlap with those of colorectal adenocarcinoma (see Etiology)
Sites
  • Colon
Pathophysiology
  • Adenoma - carcinoma sequence: genetic changes that occur before morphologically identifiable tumor formation, including a small set of driver genes (APC, CNNTB1, KRAS, SMAD4 and TP53) (Proc Natl Acad Sci U S A 2013;110:1999)
    • APC genetic alteration results in reduced degradation of beta catenin and dysregulated WNT signaling (Science 1997;278:120)
      • Inherited (constitutional) APC alterations lead to familial adenomatous polyposis
    • Activating KRAS mutations leads to growth dysregulation through MAPK pathway (Br J Cancer 1997;75:341)
    • SMAD deletion leads to disruption of TGFb growth inhibitory pathway
    • Alteration of PTEN or activation of PIK3CA disrupts PI3K pathway, inhibits apoptosis and promotes neoplastic cell survival (Nat Commun 2016;7:11971, Nat Commun 2014;5:4961)
    • Alteration of TP53 allows the cells to survive DNA damage and other cellular stresses (Int J Cancer 1995;64:47)
  • Small subset of adenoma acquires defect in DNA mismatch repair genes (predominantly hypermethylation of the MLH1 promoter) (Gastroenterol Hepatol Bed Bench 2017;10:S117)
Etiology
Diagrams / tables

Images hosted on other servers:

Adenoma carcinoma sequence

Clinical features
Diagnosis
  • Usually asymptomatic and takes 5 - 20 years to progress to carcinoma
  • Detected by screening or surveillance colonoscope
  • Usually small (mm) in size
  • Pedunculated, sessile, slightly elevated, flat or depressed appearance
Prognostic factors
  • Most adenomas do not progress through the adenoma carcinoma sequence
  • Risk is associated with:
    • Higher number of lesions
    • Larger size
    • Higher proportion of villous architecture
    • Extent of high grade dysplasia
Case reports
Treatment
  • Major treatment: endoscopic biopsy or resection
  • According to NCCN Guidelines for Colorectal Cancer Screening (version 2.2021), if pathology identified:
    • Low risk adenoma: < 2 polyps and < 1 cm
      • Repeat colonoscopy in 7 - 10 years
    • High risk polyp:
      • High grade dysplasia present
      • Villous / tubulovillous histology
      • 3 - 10 adenomatous polyps (serrated lesions are discussed in different section)
        • Repeat colonoscopy in 3 years; if negative, repeat colonoscopy in 5 years
        • If positive, treat according to the pathology finding
    • Large colorectal polyps (> 1 cm in size) without invasion:
      • If pedunculated, colonoscopy in 3 years
      • Sessile morphology with no high risk endoscopic features for invasive cancer:
        • If complete resection and no unfavorable risk factors, colonoscopy in 1 - 3 years
        • If incomplete resection, referral for surgery evaluation or expertise in management of large colorectal polyps
        • Sessile morphology with high risk endoscopic features, even if no invasive cancer identified by pathology - surgical evaluation or expertise in management of large colorectal polyps
  • Reference: National Comprehensive Cancer Network: NCCN Guidelines - Colorectal Cancer Screening [Accessed 18 October 2021]
Clinical images

Images hosted on other servers:

Giant villous adenoma

Several adenomas

Gross description
  • Specimen is received in formalin, additionally labeled transverse colon polyps and consists of 2 soft, irregular, red-tan tissue fragments (0.5 x 0.3 x 0.2 cm in aggregate)
  • Specimen is received in formalin, additionally labeled right colon and consists of multiple tan to white, soft, irregular mucosal tissue fragments (0.8 x 0.2 x 0.2 cm in aggregate)
Gross images

Images hosted on other servers:
Polypoid tumor with gyrated surface sitting on a short stalk

Polypoid tumor with gyrated surface sitting on a short stalk

Polypoid tumor

Polypoid tumor

Microscopic (histologic) description
  • Tubular adenomas:
    • Conserved crypt architecture with variable elongation of the crypts and an increased number of glands
    • At least low grade dysplasia: hyperchromatic nuclei, nuclear spindling and stratification, loss of cell polarity
    • Decreased numbers of goblet cells and absorptive cells
    • Small (< 25%) villous component is acceptable
  • Tubulovillous adenoma:
    • Similar to tubular adenoma but with 25 - 75% of villous component
    • Villous component: architecture that resemble small intestinal villi
  • Villous adenoma:
    • > 75% of villous component
  • Advanced adenoma:
    • All adenomas with a size > 10 mm
    • With tubulovillous or villous architecture
    • With or without high grade dysplasia
    • Highest risk of synchronous of metachronous adenomas
  • Rare subtypes:
    • Paneth cell rich subtype (more common in proximal colon or in younger patients) (Hum Pathol 2009;40:872, Sci Rep 2016;6:26129)
    • Squamous components might be present as morules or squamous metaplasia (Pathobiology 2005;72:269)
    • Other rare morphological findings (Histopathology 2021;78:348):
      • Clear cell metaplasia or clear cell change
        • Note the clear or vacuolated cytoplasm are not mucin
      • Osseous metaplasia or heterotopic ossification
      • Neuroendocrine differentiation
        • Neuroendocrine hyperplasia
        • Neuroendocrine metaplasia
        • Neuroendocrine cell proliferation
        • Composite intestinal adenoma microcarcinoid
        • Mixed neuroendocrine - nonneuroendocrine neoplasm (MiNEN)
        • Mixed adenoma - neuroendocrine tumor (MANET)
      • Signet ring cell-like lesion
  • Histology grading:
    • 2 tiered system: low grade versus high grade
    • Criteria for high grade dysplasia:
      • Architecture: marked complex glandular crowding with glandular irregularity; cribriform architecture; intraluminal necrosis; can be observed at low power
      • Cytology: substantial loss of cell polarity, marked enlarged nuclei with prominent nucleoli, dispersed chromatin pattern, atypical mitotic figures (Eur J Gastroenterol Hepatol 2002;14:183, Colorectal Dis 2015;17:682)
  • Should be no evidence for invasion, however, pseudoinvasion (epithelial misplacement) could sometimes be seen due to prolapse (Mod Pathol 2015;28:S88)
    • Features favoring pseudoinvasion / epithelial misplacement:
      • Signs suggestive of previous epithelial trauma (extracellular mucin, hemorrhage or hemosiderin)
      • Focus appears to be continuous with the surface epithelium with similar cytology
      • Signs of mucosal prolapse such as muscular proliferation
      • Acute necrosis of the surface
    • Features favoring adenocarcinoma:
      • Isolated glands without accompanying lamina propria
      • Budding
      • Vascular invasion
      • Poor differentiation in morphology
    • Concurrent review by more than 1 GI pathologist is suggested if high grade dysplasia is present
Microscopic (histologic) images

Contributed by Chien-Kuang Cornelia Ding, M.D., Ph.D., Kwun Wah Wen, M.D., Ph.D. and Enoch Kuo, M.D.

Tubular adenoma

Low grade tubular adenoma

Tubular adenoma with high grade dysplasia

Focal high grade dysplasia

Virtual slides

Images hosted on other servers:

Tubulovillous adenoma

High grade dysplasia

Molecular / cytogenetics description
  • Not routinely performed, although majority of cases will have chromosomal instability; a subset (~25%) of cases will show TP53 mutations
Videos

Tubular adenoma

Colon dysplasia

Villous adenoma

Sample pathology report
  • Colon, hepatic flexure polyp, biopsy:
    • Tubular adenoma with focal high grade dysplasia
  • Ascending colon, polyp, endoscopic mucosal resection:
    • Fragments of tubulovillous adenoma (see comment)
    • Comment: No high grade dysplasia is identified. Specimen fragmentation precludes assessment of specimen margins.
Differential diagnosis
  • Reactive colonic mucosa:
    • Smaller nuclei with basal orientation
    • No or less significant hyperchromasia or pseudostratification
    • More abundant cytoplasm and mucin
  • Invasive adenocarcinoma:
    • At least invasion through muscularis mucosa into the submucosa
    • Desmoplasia, single cells
  • Traditional serrated adenoma:
    • Sawtooth luminal / surface contour
    • Ectopic crypts characterized by aberrant budding crypts
Board review style question #1

Which of the following genetic changes is considered a driver mutation in colonic adenoma?

  1. BCL2
  2. KIT
  3. RB
  4. TP53
Board review style answer #1
D. TP53. Alteration of TP53 allows cells to survive DNA damage and other cellular changes. It is found in ~25% of adenomas.

Comment Here

Reference: Adenoma overview
Board review style question #2

According to 2021 NCCN guidelines for colorectal cancer screening, which of the following is a feature of low risk colonic adenomatous polyp(s)?

  1. 2 adenomas
  2. High grade dysplasia
  3. Tubulovillous histology
  4. Villous histology
Board review style answer #2
A. The number of adenomatous polyps considered to be a low risk feature is < 3.

Comment Here

Reference: Adenoma overview

Adenoma with invasive carcinoma
Definition / general
  • Malignant colorectal polyps are colorectal adenomas containing invasive adenocarcinoma that extend through the muscularis mucosae into the submucosa and do not extend into the muscularis propria (Am J Gastroenterol 2020;115:1751)
  • They are classified as pT1 in the current TNM classification system in the 8th edition of AJCC Cancer Staging Manual (CA Cancer J Clin 2017;67:93)
Essential features
  • Colorectal adenoma with invasive carcinoma represents the earliest form of carcinoma because submucosal invasion leads to further risk for lymphatic and vascular metastasis even if the polyp / tumor has been completely resected (Gastroenterology 2004;127:385)
  • Colorectal adenomas with intramucosal carcinomas limited to the surface epithelium, lamina propria or muscularis mucosae are staged as carcinoma in situ (Tis) and should be excluded from this category (Am J Gastroenterol 2020;115:1751)
  • On a limited biopsy specimen when a clinical mass lesion is identified, we can diagnose invasive carcinoma in a colorectal adenoma if we see marked desmoplasia, infiltrative pattern, poorly differentiated morphology or lymphovascular space invasion (LVI) (Gastroenterology 1995;108:1657)
Terminology
  • Also called malignant epithelial / colorectal polyp, cancerous polyps
ICD coding
  • ICD-10: C18.9 - malignant neoplasm of colon, unspecified
Epidemiology
  • Prevalence of malignant polyps is between 0.2% and 12.0% (average: ~5.0%) in endoscopic polypectomies (Endoscopy 1995;27:153)
  • Incidence is increasing due to more efficacious colonoscopy screening programs, which are fundamental in the prevention of colorectal cancer and the treatment of some advanced polyps (World J Gastroenterol 2010;16:3103)
  • Malignant polyps are also classified based on the depth of invasion, which is the most important feature; in 1985, Haggitt et al. put forward a classification system for pedunculated and sessile polyps based on the depth of invasion of adenocarcinoma (Gastroenterology 1985;89:328)
  • Risk factors for finding invasive carcinoma in colonic adenomas depends on
    • Size of adenoma
    • Villous component in adenomatous polyp
    • High grade dysplasia has a 35% risk of having carcinoma (versus low grade dysplasia, which has only a 6% risk)
    • Age of the patient (older patients are at higher risk of having malignant transformation of their colorectal polyps)
Sites
  • Any part of the colon
Pathophysiology
  • 95% of colorectal cancers arise from adenomatous polyps and follow adenoma - carcinoma sequence, which is an indolent process taking many years to progress after a stepwise collection of genetic alterations
  • Sessile serrated adenoma is presumed to be the precursor of right sided adenocarcinomas with high levels of microsatellite instability (MSI-H) (Am J Gastroenterol 2012;107:1315)
Clinical features
  • Asymptomatic to rectal bleeding
Diagnosis
  • Colonoscopy with polypectomy and histopathological evaluation
Radiology description
  • Polyps are divided endoscopically by their size into (Am J Gastroenterol 2018;113:303)
    • Diminutive: < 5 mm
    • Small: 6 - 9 mm (account for > 80% of polyps encountered during colonoscopy and have little overall risk for advanced histology [0.8 - 1.6%] and malignancy [0 - 0.1%])
    • Large: ≥ 10 mm
  • Colon polyps > 10 mm have a 22.9% likelihood of advanced pathology, while those lesions that are 30 mm carry a 60% risk of high risk pathology (Gastrointest Endosc 2012;75:1022)
Prognostic factors
  • Risk factors leading to lymph node metastasis or local recurrence from residual malignancy following polypectomies are (Endoscopy 2013;45:827)
    • Higher histologic grade (poorly differentiated or undifferentiated carcinoma, signet ring cell carcinoma)
    • Tumor ≤ 1 mm from the resection margin
    • Lymphatic / venous vessel involvement
  • Histologic factors that have adverse prognostic factors for distant metastasis are as follows (Mod Pathol 2017;30:1299)
  • In en block resections of pedunculated or nonpedunculated colorectal lesions with submucosal invasion, the pathologists need to measure and report the depth of invasion, distance of the tumor from the deep and lateral surgical resection margins, in addition to prognostic histologic features, such as degree of differentiation, presence or absence of lymphovascular invasion and tumor budding
Case reports
  • 43 year old woman with a 2 cm pedunculated polyp in the descending colon showing poorly differentiated adenocarcinoma, invading into the submucosa and demonstrating a lymphatic invasion (World J Surg Oncol 2021;19:269)
  • 60 year old woman with rectal bleeding and a smooth 0.8 cm polyp in the cecum, with a pathology revealing an infiltrating adenocarcinoma arising within the hyperplastic polyp (Am J Gastroenterol 2005;100:S211)
  • 88 year old man presented with ischemic colitis and incidental 8 mm ascending colon polyp with invasive adenocarcinoma was found in a single section from the ascending colon (Cureus 2021;13:e13928)
Treatment
  • Polypectomy with complete removal of the malignant colorectal polyp; prevention of colorectal cancer progression is the primary goal
  • Endoscopic resection can provide complete resection and obviate the higher morbidity, mortality and cost associated with alternative surgical treatment
  • Complete excision with conventional endoscopic snare polypectomy for < 1 cm adenomas, which accounts for 80 - 90% of colorectal polyps (Am J Surg Pathol 2018;42:1083)
  • Cold or hot snare polypectomy (with or without submucosal injection) to remove 10 - 19 mm nonpedunculated lesions
  • Endoscopic mucosal resection (EMR) or endoscopic submucosal dissection (ESD) are used for > 20 mm nonpedunculated colorectal polyps but are usually only available in specialized centers
  • Benefit of EMR and ESD is the ability to obtain 1 large piece of tissue, avoiding fragmented excision with clear margins and making it easier to orient, section and evaluate margins
  • Colectomy with lymph node dissection is recommended for pedunculated polyps with any unfavorable histology, invasion into submucosa of bowel wall (Haggitt level 4) and any sessile / flat adenomas with invasion (Haggitt level 1 - 4)
  • First follow up surveillance colonoscopy is 6 months for larger colorectal polyps ≥ 20 mm and the interval to the next colonoscopy is at 1 year and then 3 years (Am J Surg Pathol 2018;42:1083)
Gross description
  • Grossly the polyps are described as polypoid (pedunculated or sessile) and nonpolypoid (flat or ulcerated) subtypes according to the Paris classification (Gastrointest Endosc 2003;58:S3)
Microscopic (histologic) description
  • Adenocarcinoma can arise in adenomatous (tubular, tubulovillous or villous), serrated (sessile serrated adenoma / polyp or traditional serrated adenoma) or hamartomatous polyps
  • For malignant pedunculated polyps, submucosal involvement by carcinoma has been divided into 4 Haggit levels (head, neck, stalk and beyond stalk in the submucosa) (Gastroenterology 1985;89:328)
    • Level 1: invasion limited to head of pedunculated polyp
    • Level 2: invasion extends to neck of pedunculated polyp
    • Level 3: invasion extends to stalk of pedunculated polyp
      • Note: levels 1 - 3 have the lowest risk of metastasis (< 1%)
    • Level 4: invasion of submucosa in bowel wall proper (beyond the stalk of pedunculated polyp)
      • Note: level 4 has the highest risk of lymph node metastasis, up to 27%
  • For malignant sessile polyps, submucosal involvement by carcinoma has been divided into superficial, mid and deep levels (Kikuchi levels SM1, SM2 and SM3) (Am J Gastroenterol 2020;115:1751)
    • SM1: invasion into upper third of submucosa
    • SM2: invasion into middle third of submucosa
      • Note: SM1 / SM2 are associated with low risk of metastasis; reported to be 0% for SM1 and ~10% for SM2
    • SM3: invasion into lower third of submucosa; the greatest risk of lymphatic spread, up to 25%
Microscopic (histologic) images

Contributed by Albina Joldoshova, M.D. and Naziheh Assarzadegan, M.D.

Pedunculated tubular adenoma

Tubulovillous adenoma

Tubular adenoma with invasive adenocarcinoma


Tubular adenoma

Invasive adenocarcinoma

Lymphovascular space invasion

Poorly differentiated adenocarcinoma


High tumor budding score

Pseudoinvasion

Pseudoinvasion

Molecular / cytogenetics description
  • Sessile serrated adenoma more often shows high levels of microsatellite instability and MLH1 hypermethylation in sporadic cases (Am J Gastroenterol 2012;107:1315)
Sample pathology report
  • Colon, ascending polyp, hot snare polypectomy:
    • Invasive adenocarcinoma, moderately differentiated, arising in a tubular adenoma (see comment)
    • Negative for lymphovascular space invasion
    • Low tumor budding score
    • Cauterized stalk margins negative for dysplasia or carcinoma
    • Immunohistochemistry for mismatch repair proteins will be reported in an addendum
    • Comment: Tumor invades into submucosa at the head of the polyp (depth on invasion is 0.5 mm). The distance to the deep margin is 2 mm. Complete excision of this lesion is considered adequate treatment; therefore, completeness of excision should be ensured clinically, if not already achieved.
Differential diagnosis
  • Adenoma with pseudoinvasion (misplacement of benign or dysplastic glands) (Cancer 1974;33:206):
    • Presence of dysplastic glandular epithelium of the mucosa beneath the muscularis mucosae in colonic polyps mimicking a submucosal invasion
    • Usually occurs in large polyps (> 1 cm) (especially those with long stalks) and is mostly found in polyps of the sigmoid colon
    • Overall, it has lobular architecture and crypts with smooth and rounded edges
    • Nondysplastic or same grade of dysplasia as adenoma at the surface
    • Usually surrounded by rim of lamina propria
    • Absent desmoplastic stromal response
    • Hemorrhage or hemosiderin deposition
    • Sometimes acellular extracellular mucin associated with ruptured dilated mucinous cysts and inflammatory response may be noted; need to differentiate from an invasive mucinous carcinoma (colloid), which will have mucin pools with malignant cells, a feature lacking in pseudoinvasion
  • Colonic adenomatous polyps involving submucosal lymphoglandular complexes (Am J Surg Pathol 2018;42:1083):
    • Lymphoglandular complexes (LGCs) are lymphoid follicles, present in close apposition to lamina propria or muscularis mucosae or submucosa; rarely, colorectal adenomas involve submucosal lymphoglandular complexes, simulating invasive adenocarcinoma with associated extensive lymphoid response and presenting a diagnostic pitfall
    • Tumor is contained within the lymphoid tissue and lack infiltrating single cells / small clusters, poorly formed, fused and irregular glands, solid tumor nests, desmoplastic reaction and lymphovascular invasion
  • Localized colitis cystica profunda:
    • Glandular, nondysplastic epithelium in submucosa
    • Overlying mucosa: usually ulcerated or hyperplastic
    • Usually rectal; seen with prolapse or after irradiation
Board review style question #1

What is your diagnosis based on the image above?

  1. Tubular adenoma with high grade dysplasia
  2. Tubular adenoma with invasive adenocarcinoma
  3. Tubular adenoma with invasive adenocarcinoma with associated mucin production
  4. Tubular adenoma with pseudoinvasion
Board review style answer #1
D. Tubular adenoma with pseudoinvasion. This represents a tubular adenoma with pseudoinvasion, which shows misplacement of dysplastic glandular epithelium into submucosa mimicking a submucosal invasion. Pseudoinvasion will show associated stromal hemorrhage, hemosiderin deposition and acellular extracellular mucin with ruptured dilated mucinous cysts. Answer A is incorrect because of the additional features of pseudoinvasion. Answer B is incorrect because tubular adenomas with invasive adenocarcinoma will show true invasion of neoplastic glands beyond the muscularis mucosa into the submucosa. Answer C is incorrect because abundant expression of extracellular mucin within the tumor is not shown.

Comment Here

Reference: Adenoma with invasive carcinoma
Board review style question #2
Which of the following are poor prognostic factors for malignant colorectal polyp?

  1. Large polyp size
  2. High grade dysplasia
  3. High tumor budding, lymphovascular invasion, higher histologic grade, positive margin and submucosal invasion > 1 mm
  4. Villous component in adenomatous polyp
Board review style answer #2
C. High tumor budding, lymphovascular invasion, higher histologic grade, positive margin and submucosal invasion > 1 mm are the adverse prognostic factors for distant metastasis. Answer A is incorrect because while a risk factor for finding an invasive carcinoma is higher (46%) in larger colorectal polyps (> 2 cm), this doesn’t have an adverse prognostic factor for distant metastasis or local recurrence. Answer D is incorrect because a risk factor for finding an invasive carcinoma is higher in colorectal polyps with a more villous component. Answer B is incorrect because while high grade dysplasia has a 35% risk of having carcinoma (versus low grade dysplasia, which has only a 6% risk), it is not a poor prognostic indicator.

Comment Here

Reference: Adenoma with invasive carcinoma

Adenoma-like adenocarcinoma
Definition / general
  • Well differentiated subtype of colorectal carcinoma with good prognosis that resembles villous adenoma on the surface (Anticancer Res 1998;18:2649)
  • Formally recognized by the WHO
Essential features
  • Very well differentiated subtype of colorectal carcinoma
  • Can mimic tubulovillous adenoma on biopsy, making diagnosis challenging
  • Very good prognosis, with metastasis uncommon
Terminology
Epidemiology
  • Approximately 3 - 5% of colorectal carcinomas have villous / adenoma-like features
Clinical features
  • Low rate of metastasis to lymph nodes and other organs
Diagnosis
  • Biopsy may resemble villous adenoma, so clinical / endoscopic correlation is essential
Gross images

Images hosted on other servers:

Villous tumor with central ulceration

Microscopic (histologic) description
  • Invasive carcinoma with architectural and cytologic features resembling villous adenoma
  • May have traditional invasive component or invade by pushing
  • Mucinous features often present
  • Epithelial islands in desmoplastic stroma is a helpful finding (Am J Surg Pathol 2004;28:1460)
  • May also have a component of conventional adenocarcinoma (Hum Pathol 2021;107:9)
Microscopic (histologic) images

Contributed by Raul S. Gonzalez, M.D.

Villous adenocarcinoma of colon



Contributed by Semir Vranic, M.D., Ph.D.

5x

10x

10x

10x


CK7

20x

CDX2

CK20



Images hosted on other servers:

Abrupt transition from benign to malignant mucosa

Molecular / cytogenetics description
  • 24% of cases have microsatellite instability
Sample pathology report
  • Transverse colon, resection:
    • Adenoma-like adenocarcinoma of the colon, well differentiated (see synoptic report)
Differential diagnosis
  • Villous adenoma:
    • Can be hard to distinguish on biopsy (subtle clues include gland distortion and desmoplasia)
Board review style question #1

A patient presents with a colon mass, which is biopsied and then excised. What is the best diagnosis?

  1. Adenoma-like adenocarcinoma
  2. Colorectal adenocarcinoma, NOS
  3. Pure mucinous adenocarcinoma
  4. Tubulovillous adenoma with pseudoinvasion
Board review style answer #1
A. Adenoma-like adenocarcinoma

Comment Here

Reference: Adenoma-like adenocarcinoma
Board review style question #2
Which of the following is true about adenoma-like adenocarcinoma of the colon?

  1. It always demonstrates microsatellite instability
  2. It can be difficult to diagnose on biopsy
  3. It has a high rate of lymph node and liver metastasis
  4. It is always pure, without any component of conventional adenocarcinoma
Board review style answer #2
B. It can be difficult to diagnose on biopsy

Comment Here

Reference: Adenoma-like adenocarcinoma

Adenosquamous carcinoma
Definition / general
  • Rare (0.1% of colonic carcinomas) WHO recognized epithelial malignancy of colon with glandular and squamous elements (Am J Surg Pathol 1978;2:47)
Essential features
Sites
Pathophysiology
Clinical features
Case reports
Treatment
  • Surgery with adjuvant treatment
Clinical images

Images hosted on other servers:

Circumferential obstructive mass

Bleeding circumferential lobulated mass

Gross images

Images hosted on other servers:

Cross section

Microscopic (histologic) description
  • Resembles conventional colorectal adenocarcinoma but with areas of squamous differentiation, either admixed or distinct
Microscopic (histologic) images

Contributed by Raul S. Gonzalez, M.D.

Adenosquamous carcinoma



Images hosted on other servers:

Collision-like areas

Composite-like areas

Adenosquamous carcinoma

Malignant squamous cells with keratinization

Sample pathology report
  • Ascending colon, biopsy:
    • Adenosquamous carcinoma (see comment)
    • Comment: The malignancy demonstrates both glandular elements and squamous elements. The former is positive for CDX2 by immunohistochemistry and the latter is positive for p40 by immunohistochemistry.
Differential diagnosis
Board review style question #1

Which of the following is true about adenosquamous carcinoma of the colon?

  1. Always arises in the setting of HPV infection
  2. Has a good prognosis
  3. Often presents at an advanced stage
  4. Represents approximately 10% of colorectal carcinomas
Board review style answer #1
C. Often presents at an advanced stage

Comment Here

Reference: Adenosquamous carcinoma

Adenovirus
Definition / general
  • Disease by adenovirus is most common in the upper respiratory tract, but also causes pneumonia and conjunctivitis, and may affect the genitourinary tract, liver, central nervous system or other sites
  • Adenovirus is a common cause of pediatric diarrhea, although at least half of infections are subclinical
  • The colon may be involved as part of systemic infection
  • Most cases are community acquired but nosocomial outbreaks occur
  • Infection is associated with immunodeficiency including HIV infection, solid organ and hematopoietic stem cell transplantation, congenital immunodeficiency (Hum Pathol 2010;41:1777)
  • There are over 50 serotypes with seven species that affect humans
  • In intestinal disease, serotypes 31, 40, and 41 are mostly reported in infants; 2, 3, and 5 mostly in children
  • It is not well understood why specific serotypes are associated with specific syndromes; however, differences in mode of transmission and viral tropism likely play a role
Pathophysiology
  • Adenovirus is ubiquitous; transmission is by respiratory droplets, fomites, fecal-oral route
  • It is a hardy virus and prolonged survival in the environment is possible
  • Adenovirus is a nonenveloped, lytic double stranded DNA virus (CDC - Adenoviruses)
  • Adenovirus enters the cytoplasm after binding to a receptor and then is transported to the nucleus where replication occurs
  • Cell rupture leads to dispersion of viral particles, cytokine production and an inflammatory response
  • Chronic or latent infection, usually involving lymphoid tissue, may occur
  • Virus may be shed for months to years after infection
Clinical features
  • With only very rare exceptions, diarrheal disease is mild and self-limited in immunocompetent individuals
  • Infants may develop watery diarrhea, fever and vomiting that lasts 8-12 days (Gastroenterol Clin North Am 2001;30:779)
  • Subclinical disease is frequent
  • Significant morbidity or mortality can occur in immunocompromised patients, with symptoms of fever, weight loss, abdominal pain
  • Other infections, especially CMV, may coexist with adenovirus infection in immunocompromised patients
  • Nosocomial outbreaks may occur; appropriate hand hygiene and isolation procedures effectively prevent this
  • Infection may cause ileal or cecal intussusception in children
Diagnosis
  • Characteristic inclusions in biopsy material are highly consistent with infection; immunohistochemical stains provide confirmation
  • PCR, viral culture and stool electron microscopy may be used for diagnosis
  • Serologic studies may also be obtained
Prognostic factors
  • In immunocompromised patients, high viral load by PCR is associated with a poorer prognosis
Case reports
Treatment
  • Generally only supportive care
  • Cidofovir has been used in immunocompromised patients, but significant toxicity may occur (Biol Blood Marrow Transplant 2007;13:74)
  • Immune reconstitution, if possible, is usually curative
Microscopic (histologic) description
  • Surface epithelial cells, especially goblet cells, are infected
  • Cowdry type B nuclear inclusions with enlarged, homogeneous, smudgy basophilic nuclei (smudge cells) are more common than Cowdry type A inclusions, which are eosinophilic to amphophilic with nuclear halos
  • Usually present are necrotic cells, apoptotic bodies and cellular debris with a mononuclear cell infiltrate and generally mild architectural distortion (Histopathology 2015;66:467)
Microscopic (histologic) images

Images hosted on other servers:
Missing Image

Infected cells with
irregular amphophilic
nuclei

Missing Image

Cells have eccentric
nuclei and vacuolated
cytoplasm

Missing Image

Adenovirus immunostain

Positive stains
  • Adenovirus immunostain
Electron microscopy images

Images hosted on other servers:
Missing Image

Intranuclear inclusions
with (inset) regular
pattern of arrangement


Adhesions
Definition / general
  • Serosa based fibrous bands of scar tissue that cause colon to connect and adhere to nearby structures, typically other viscera
Essential features
  • Fibrovascular scar tissue of the colonic serosa, usually secondary to injury or prior surgery
  • Can distort anatomy and lead to complications such as obstruction, herniation or ischemia
Sites
  • Can occur anywhere in GI tract, commonly between bowel segments or abdominal wall and operative site
Etiology
  • Typically due to injury, such as prior surgical procedures, infection (i.e. peritonitis) or radiation damage
  • Also Crohn's disease or serosal endometriosis
  • Rarely congenital
Clinical features
Diagnosis
  • Typically observed radiologically or during surgery; can also be seen microscopically
Treatment
Clinical images

Images hosted on other servers:

Between loops of intestine

Microscopic (histologic) description
  • Cellular fibrous connective tissue containing vessels and nerves; may contain fat and smooth muscle clusters (J Pathol 2000;192:67)
Microscopic (histologic) images

Contributed by Raul S. Gonzalez, M.D.
Serosal adhesions Serosal adhesions

Serosal adhesions



Images hosted on other servers:

Attached to intestinal
serosa and
linking viscera

Sample pathology report
  • Ascending colon, resection:
    • Segment of colon with reactive change and prominent serosal adhesions
    • Margins of resection unremarkable.
    • Four benign lymph nodes.
Board review style question #1

Which of the following is true about colonic serosal adhesions?

  1. Abdominal obstruction is a rare complication
  2. Most are congenital
  3. They are only detectable microscopically
  4. They can be caused by prior surgery
Board review style answer #1
D. They can be caused by prior surgery

Comment Here

Reference: Adhesions

Allergic colitis
Definition / general
  • Allergic proctocolitis: benign disorder of blood streaked stools in otherwise healthy-appearing infants
  • Occurs in infants and children related to food, particularly cow's milk in infants and soy or eggs in older children
  • Both IgE and non-IgE related mechanisms (Allergol Immunopathol (Madr) 2009;37:36)
  • Some clinical cases of food allergy in young children are eventually diagnosed as inflammatory bowel disease (Med Wieku Rozwoj 2006;10:475)
  • Italian study of allergic proctocolitis: atopy patch tests positive in 100% infants, multiple positivity in 50%; sensitization for breast milk in 100%, cow's milk in 50%; also soy (28%), egg (21%), rice (14%), wheat (7%) (BMC Gastroenterol 2011;11:82)
Clinical features
Case reports
Treatment
  • Removal of cow's milk from infant or (if breast fed) mother’s diet
  • Elemental diet based on amino acid formula typically resolves symptoms in 2-3 days, GI eosinophilic inflammation within 6 weeks (Pediatr Allergy Immunol 2007;18:360)
Clinical images

Images hosted on other servers:

Endoscopic findings

Microscopic (histologic) description
Microscopic (histologic) images

Images hosted on other servers:

Eosinophilic infiltration

Differential diagnosis

Amebic colitis
Definition / general
Essential features
  • Due to invasive infection with Entamoeba histolytica
  • Transmission primarily by ingestion of E. histolytica cysts in fecally contaminated food or water (fecal - oral); also by sexual contact (oral - anal)
  • Associated with fever, abdominal pain, tenesmus, diarrhea (with or without blood), dysentery
  • Amebae may disseminate to the liver and other organs (see Entamoeba histolytica abscess)
  • See flask shaped ulcers; rarely inflammatory mass (ameboma), perforation
  • Amebic trophozoites invade the submucosa, undermining the overlying mucosa; no amebic cysts in tissue
  • Trophozoites with pseudopod projections, ingested RBCs in cytoplasm, small round nucleus with dot-like karyosome and peripheral rim of condensed chromatin
  • Trophozoites are CD68 negative, strongly PAS positive
  • Treat invasive disease with metronidazole or tinidazole; also use paromomycin to eradicate luminal cysts
Terminology
  • Amebiasis, amoebiasis
ICD coding
  • ICD-10:
    • A06.0 - acute amebic dysentery
    • A06.1 - chronic intestinal amebiasis
    • A06.2 - amebic nondysenteric colitis
    • A06.3 - ameboma of intestine
    • A06.9 - amebiasis, unspecified
  • ICD-11:
    • 1A36.0 - intestinal infections due to Entamoeba
    • 1A36.00 - acute amoebiasis
    • 1A36.01 - amoeboma of intestine
    • 1A36.0Z - intestinal infections due to Entamoeba, unspecified
Epidemiology
  • Occurs worldwide but more common in tropical and subtropical regions (Open Forum Infect Dis 2018;5:ofy161)
  • In developed countries, affected patients are usually immigrants, travelers, men who have sex with men (MSM) or institutionalized individuals
  • Microscopy based detection methods overestimate the prevalence of E. histolytica due to the presence of the morphologically identical amebae, E. dispar, E. moshkovskii, E. bangladeshi and E. nuttalli (CDC: Amebiasis [Accessed 22 April 2022])
  • 1 of the top 15 causes of diarrhea in children; diarrhea is a leading cause of death in children < 5 years of age (Lancet Infect Dis 2017;17:909)
  • Millions of cases are estimated to occur worldwide each year; estimated 67,900 deaths in 2015 with 15,500 deaths in children < 5 years of age (Lancet 2016;388:1459)
  • < 5 deaths per year reported in the United States from 1990 - 2007 (Am J Trop Med Hyg 2011;85:1038)
Sites
  • Colon; cecum is the most common, followed by right colon, rectum, sigmoid and appendix (BMC Gastroenterol 2021;21:367)
    • Terminal ileum is usually spared
    • Skip lesions are common
  • Hematogenous dissemination may occur, usually to the liver (see Entamoeba histolytica abscess); also lungs, brain
  • Rectovesical fistula and fistulous involvement of the skin may also occur
Pathophysiology
  • Entamoeba histolytica cysts are ingested from fecally contaminated food or water; sexual transmission also occurs (see Diagrams / tables)
  • Cysts are resistant to gastric acid (and chlorine in water supplies)
  • Excystation occurs in the small intestine to release trophozoites
  • Trophozoites are invasive and multiply by binary fission
  • Estimated that 20% of infections are associated with intestinal wall invasion
  • Susceptibility to invasive infection appears to be host dependent (Trends Parasitol 2011;27:254):
    • Polymorphisms in leptin receptor affect susceptibility
    • Risk factors for severe disease: malnutrition, malignancy, alcoholism, immunocompromised state, pregnancy, young age
  • Cell adhesion and killing by E. histolytica trophozoites (Trends Parasitol 2011;27:254):
    • Gal / GalNAc lectin on E. histolytica's surface binds galactose and N-acetyle-D-galactosamine residues found on O linked sugar side chains of host colonic mucin; degrades intestinal protective mucous barrier and allows penetration of epithelium
    • Parasite secretion of proteinases, contact dependent cell lysis and apoptosis and the formation of amebapores result in host cell death
  • E. histolytica ingests remnant erythrocytes (hemophagocytosis)
  • Some trophozoites undergo encystation through signaling pathways, completing the cycle
Etiology
  • Due to E. histolytica infection
Diagrams / tables

Contributed by Centers for Disease Control and Prevention

Life cycle of Entamoeba histolytica



Images hosted on other servers:
Missing Image

Model of mechanism for cytotoxicity

Clinical features
  • 80 - 90% of individuals with E. histolytica infection are asymptomatic (Can J Gastroenterol Hepatol 2018;2018:4601420)
  • Symptoms range from mild diarrhea (most common) to severe dysentery
    • Subacute onset; develops over 3 - 4 weeks with worsening diarrhea and abdominal pain
    • Symptoms may also develop acutely and mimic acute abdomen
    • Cases may occur where symptoms develop months after infection
    • Young children may develop intussusception or necrotizing colitis, that may lead to perforation
    • Rare complications are toxic megacolon, fulminant necrotizing colitis, colonic amebomas, perianal fistulas
  • Disseminated disease is more common in men (see Entamoeba histolytica abscess)
  • Morphologically similar species E. dispar is nonpathogenic (does not invade); the pathogenic potential of E. bangladeshi, E. moshkovskii and E. nuttalli is not fully understood
Diagnosis
  • May be suspected based on epidemiologic factors, patient symptoms, radiologic or colonoscopic findings (Open Forum Infect Dis 2018;5:ofy161)
  • Definitive diagnosis of amebic colitis is made by finding trophozoites that have invaded the intestinal mucosa
  • Patients with amebic liver abscesses usually have antiamebic antibodies (see Entamoeba histolytica abscess)
    • Cyst aspiration is sometimes performed; although it is unusual to see parasites, the absence of other microorganisms supports evidence of amebic liver abscess
Laboratory
  • E. histolytica infection is usually detected via stool microscopy (ova and parasite examination), stool antigen or stool nucleic acid testing (Open Forum Infect Dis 2018;5:ofy161)
    • Positive stool exam is not definitive for amebic colitis, since 80 - 90% of infections do not cause symptoms
    • Stool microscopy detects cysts or trophozoites that are morphologically consistent with E. histolytica and other morphologically identical species, but cannot usually differentiate them
    • Some antigen detection or nucleic acid amplification tests (NAATs) can distinguish E. histolytica from similar appearing amebae
    • NAATs are the most sensitive detection methods performed on stool
    • Stool microscopy findings:
      • Trophozoites measure 10 - 60 micrometers and possess a single nucleus with even peripheral chromatin and a pinpoint, often centrally located karyosome
      • Ingested erythrocytes pathognomonic for E. histolytica infection; otherwise, needs to be distinguished from nonpathogenic species (e.g., E. dispar)
      • Cysts are spherical, 10 - 20 micrometers in diameter; mature cysts have 4 nuclei and chromatoid bodies with bluntly rounded ends
  • Serologic testing for E. histolytica antibodies supports the diagnosis of amebiasis but cannot differentiate current from past infection
    • Most useful for diagnosing disseminated disease
Radiology description
  • Evidence of colitis may be seen on CT (Jpn J Radiol 2021;39:558):
    • Acute colitis: nonspecific diffuse wall thickening, submucosal edema with stratified or target patterns (contrast enhanced CT)
    • Chronic colitis: prominent thickened wall; may mimic malignancy
    • Most common pattern: involvement of cecum and rectum with sparing of the terminal ileum
  • Plain film radiographs using barium may reveal ulcerative changes
  • Ameboma: localized wall thickening with minimal contrast enhancement on CT
  • Fulminant necrotizing colitis:
    • Prominent thickened wall on contrast enhanced CT with a thin enhancing rim
    • Contiguous involvement or skip lesions
  • Ultrasonography and CT can be used for detection of liver abscess
Radiology images

Images hosted on other servers:
Missing Image

Fulminant colitis and hepatic amebiasis

Case reports
Treatment
  • Amebic colitis is treated with both an amebicidal and luminal agent (Open Forum Infect Dis 2018;5:ofy161)
    • Metronidazole or tinidazole amebicides, to target invading trophozoites
    • Paromomycin to eradicate luminal cysts and prevent relapse / transmission to others
  • Complicated disease may also require surgical resection of perforated / necrotic bowel, fluid resuscitation, broad spectrum antimicrobials for peritonitis
Gross description
Gross images

Contributed by Centers for Disease Control and Prevention

Ulcerative intestinal amebiasis

Diffuse ulcerative amebic colitis

Amebic colitis with perforation



Images hosted on other servers:
Missing Image

Ulcer

Microscopic (histologic) description
  • Lesions begin with superficial mucosal necrosis and progress to erosions and ulcers (AFIP: Topics on the Pathology of Protozoan and Invasive Arthropod Diseases [Accessed 22 April 2022])
  • Overlying inflammatory exudates consist of necrotic material, fibrin and inflammatory cells, and often contain E. histolytica trophozoites
  • Chronic cryptitis and crypt architectural distortion may be seen
  • Mature ulcers are flask shaped, with a base that is broader than the apex; undermines the overlying mucosa
    • Ulcer crater is composed of cellular debris, fibrin and trophozoites
    • Minimal inflammation initially; later includes neutrophils, histiocytes, lymphocytes, plasma cells and occasionally eosinophils within and around the ulcer
    • Amebic trophozoites are best appreciated at the border of viable and necrotic tissue
  • Trophozoites of Entamoeba histolytica are 10 - 60 micrometers and may resemble macrophages
    • Round to oval with pseudopod projections
    • Commonly surrounded by a halo in FFPE tissue preparations; caused by retraction during fixation or by parasite toxins
    • Cytoplasm is abundant, vacuolated, pink-purple on H&E, and may contain ingested red blood cells
    • Nuclei are small and round with a prominent rim of peripheral chromatin and a small, dot-like central karyosome
  • Amebomas are comprised of granulation tissue, chronic inflammatory cells and fibrosis
    • Clusters of trophozoites are usually seen near submucosa ulcerations
  • In trichrome stained stool specimens, peripheral chromatin and discrete karyosome are supportive of E. histolytica / E. dispar (Pritt: Creepy Dreadful Wonderful Parasites Blog - Answer to Case 546 [Accessed 13 October 2023])
Microscopic (histologic) images

Contributed by Centers for Disease Control and Prevention, Bobbi S. Pritt, M.D., M.Sc. and Blaine Mathison

Classic flask shaped ulcer

Edge of flask shaped ulcer

Invading trophozoites

Trophozoites invading muscle


Amebic trophozoites

Trophozoite in blood vessel

PAS positive trophozoites

Trichrome stained stool specimen

Virtual slides

Images hosted on other servers:
Missing Image

Ulcerative amebic colitis

Positive stains
Negative stains
Sample pathology report
  • Colon, cecum, biopsy:
    • Intestinal amebiasis (see comment)
    • Comment: The biopsy shows mucosal ulceration with an overlying fibrinous exudate. Within the ulcer bed and exudate are numerous trophozoites of the protozoan pathogen, Entamoeba histolytica, which extend into the submucosa but not into the underlying muscular layer. The trophozoites are readily identified by their abundant dense bubbly cytoplasm and small round nucleus with peripheral rim of condensed chromatin and central dot-like karyosome. Some trophozoites contain ingested erythrocytes. The trophozoites are highlighted with PAS, which stains their cytoplasm deep magenta.
Differential diagnosis
  • Appendicitis:
    • Acute inflammation
    • No infiltrating trophozoites
  • Balantidium coli (Balantioides coli):
    • Flask shaped ulcers resembling amebiasis
    • Large (40 - 200 micrometers) ciliated trophozoites invading the mucosa and submucosa
  • Crohn's disease:
    • Active chronic colitis
    • No infiltrating trophozoites
  • Histiocytes (also known as tissue macrophages):
    • Present in various inflammatory and infectious conditions
    • Similar size to Entamoeba histolytica trophozoites
    • Large, often reniform nucleus (versus small round nucleus of E. histolytica)
    • CD68+
  • Nonpathogenic ameba
  • Pseudomembranous colitis:
    • Mucopurulent exudate erupts out of crypts to form a mushroom-like layer of karyorrhectic debris and neutrophils that adheres to the epithelial surface
    • Crypt necrosis
    • Superficial lamina propria with neutrophilic inflammation and some capillary fibrin thrombi
    • No infiltrating trophozoites
  • Pyogenic abscess (in liver)
  • Tuberculosis
  • Ulcerative colitis:
    • Active chronic colitis
    • No infiltrating trophozoites
Board review style question #1

A 47 year old man living in Mexico presented with 3 months of gradually increasing abdominal pain and diarrhea. The initial diagnostic workup was unrevealing, so a colonoscopy was performed, which revealed several well demarcated ulcers in the cecum and ascending colon. Biopsies of the ulcers were obtained and above is a representative image of the histopathologic findings. What is the diagnosis?

  1. Amebiasis
  2. Balantiosis
  3. Signet ring carcinoma
  4. Toxic drug injury
Board review style answer #1
A. Amebiasis

Comment Here

Reference: Amebic colitis
Board review style question #2
Which of the following histochemical stains is useful for highlighting invasive trophozoites of Entamoeba histolytica?

  1. Alcian blue
  2. Giemsa
  3. Masson trichrome
  4. Mucicarmine
  5. Periodic acid Schiff
Board review style answer #2
E. Periodic acid Schiff

Comment Here

Reference: Amebic colitis

Amyloidosis
Definition / general
  • Extracellular deposition of amyloid protein, often around blood vessels
Essential features
  • Amyloid deposition in the colon, confirmable with Congo red
  • Usually around blood vessels, which can lead to vascular injury
Terminology
  • Localized (limited to the colon) or diffuse (present in numerous organs)
Epidemiology
  • Can be primary, secondary, hereditary or endocrine related
Pathophysiology
  • Overproduction of amyloid protein (AL, AA, ATTR, etc.) due to various causes
  • Senile amyloid is often present in GI tract of elderly patients (Pathol Res Pract 1994;190:641)
Etiology
Clinical features
  • Gastrointestinal involvement is seen in most patients with systemic amyloidosis
  • May be asymptomatic or cause bleeding, obstruction, perforation or abnormal motility
  • Amyloid tumor may clinically resemble carcinoma (AJR Am J Roentgenol 2002;179:536)
  • Uncommonly, amyloid is localized to colon and does not require systemic treatment (Amyloid 2003;10:36)
Diagnosis
  • Can diagnose with rectal biopsy that includes submucosa (85% sensitivity), though amyloid deposition may be initially discovered in a resection specimen
Radiology description
Case reports
Treatment
  • If systemic, depends on type of amyloid but generally targeted at the cause (myeloma, kidney failure, etc.)
Gross description
  • Mucosa may be normal or finely granular
Gross images

Images hosted on other servers:

Amyloid tumor (above)
and adenocarcinoma
arising from villous
adenoma (below)

Microscopic (histologic) description
  • Amyloid present in blood vessel walls and muscularis propria; may be subepithelial; may cause ischemic changes or frank hemorrhage
Microscopic (histologic) images

Contributed by Raul S. Gonzalez, M.D.
Colonic amyloid Colonic amyloid

Colonic amyloid



Images hosted on other servers:

Submucosal vessel involvement

With Congo red stain

Congo red stain
highlights vessel
wall and free
submucosal amyloid

Congo red stain

Subepithelial
deposits resembling
collagenous colitis

Positive stains
  • Congo red (stains deep pink and demonstrates apple green birefringence, as in other body sites)
Sample pathology report
  • Colon, splenic flexure, biopsy:
    • Amyloidosis (see comment)
    • Comment: The biopsy shows amorphous eosinophilic material present around submucosal blood vessels. On Congo red stain, the material demonstrates apple green birefringence.
Differential diagnosis
Additional references
Board review style question #1

Which of the following stains would be positive in the amorphous perivascular material seen in this colon polyp?

  1. AFB
  2. Congo red
  3. GMS
  4. von Kossa
Board review style answer #1
B. The material is amyloid. It would stain a salmon pink color with Congo red and demonstrate apple green birefringence.

Comment Here

Reference: Amyloidosis

Anatomy & histology
Definition / general
  • Colon: the terminal portion of the gastrointestinal tract situated in the abdomen and the pelvis
  • Includes the cecum, ascending colon, transverse colon, descending colon, sigmoid colon and rectum
Essential features
  • Colonic mucosa has 2 predominant functions:
    • Absorption of water and electrolytes from the nutrient poor chyme that passes into the colon from the ileum
    • Production of mucus to lubricate the fecal material
  • Above functions are accomplished by 2 predominant cell types:
    • Absorptive columnar cells absorb water and electrolytes, line the surface epithelial layer and are the major cell type of crypts in the right and transverse colon
    • Goblet cells secrete mucus and are the major cell type of the crypts in the left side of the colon
  • Inflammatory cells are normally present in the lamina propria and include plasma cells, macrophages, eosinophils and lymphocytes
    • Lymphoid aggregates and follicles are also normally present
  • Architectural discipline of colon with parallel test tube-like arrangement of crypts separated by a consistent amount of lamina propria in between is important to identify
    • Expansion of this lamina propria and dropout of crypts can be signs of chronic injury
Terminology
  • Lower gastrointestinal tract, large bowel, large intestine
Physiology
Gross description
  • Colon is approximately 1.5 meters long with a diameter of 6 - 7 cm
  • From terminal ileum to anal canal, the colon is divided into cecum, ascending colon, transverse colon, descending colon, sigmoid colon and rectum
  • Segments of the colon that are completely intraperitoneal include the cecum, transverse colon and sigmoid colon (Amin: AJCC Cancer Staging Manual, 8th Edition, 2017)
  • Ascending colon and descending colon and upper third of the rectum are retroperitoneal, with the anterior and lateral surface covered by peritoneum (Amin: AJCC Cancer Staging Manual, 8th Edition, 2017)
  • Middle third of the rectum only has peritoneum on the anterior surface and the lower third is completely nonperitonealized (Amin: AJCC Cancer Staging Manual, 8th Edition, 2017)
  • These correct anatomic landmarks are very important in evaluation of margins and staging of colorectal cancers (Amin: AJCC Cancer Staging Manual, 8th Edition, 2017)
  • Different from small intestine, the colon has epiploic appendages (pedunculated fat on lateral side) and taeniae coli (discontinuous muscular fibers)
Gross images

Contributed by Reade Quinton, M.D.

Colon

Microscopic (histologic) description
  • Mucosa is composed of the epithelium, lamina propria and muscularis mucosa
    • Epithelium: colonic epithelium is composed of a single layer of the absorptive columnar cells and the goblet cells
      • As the epithelium invaginates into the underlying lamina propria, it forms glandular structures called crypts, arranged in a characteristic parallel test tube-like pattern
      • Crypt is a functional unit of colon and is primarily lined by the goblet cells
      • Crypt also has enteroendocrine cells, Paneth cells and stem cells located at its base
      • During the maturation process, the mature epithelial cells migrate toward the surface of the epithelium (luminal migration), while the immature (stem cells) are at the base of the crypts
      • On maturation, the Paneth cells migrate to the base of the crypt, instead of the luminal migration
      • Enteroendocrine cells also stay in the deeper portion of the crypt and in the middle of the tubule
      • Absorptive cells: predominant cells in the right colon
        • Columnar cells with eosinophilic cytoplasm, basally located nuclei, small apical mucin vacuoles and apical microvilli
        • Primarily line the surface epithelium
      • Goblet cells: predominant cells in the left colon
        • Large cells with intracytoplasmic mucin and basally located hyperchromatic nuclei
        • Mucin composition of goblet cells is different from the mucin in the absorptive cells (Histopathology 2000;37:561)
      • Other cell types of the epithelium:
        • Enteroendocrine cells:
          • Located at the base of the crypts and have eosinophilic secretory granules in the cytoplasm with apically located nuclei
          • Apical location of the nuclei helps to differentiate these cells from the Paneth cells
        • Paneth cells:
          • Paneth cells have a triangular shape with densely eosinophilic cytoplasmic granules
          • Nuclei are basally located, unlike enteroendocrine cells described above, a distinction important in the left colon, since the presence of Paneth cells in this part of colon is abnormal and could be a sign of chronic injury
        • M cells: typically not identified on routine histology
          • Usually associated with lymphoid follicles and seen best with electron microscopy
    • Lamina propria:
      • Loose connective tissue rich in capillaries and lymphatics
      • Supports the crypts and consists of supportive mesenchymal cells and inflammatory cells
      • Mesenchymal cells are divided into 2 types: pericrypt myofibroblasts and subepithelial myofibroblasts
      • Inflammatory cells vary in type and number
        • Presence of eosinophils, macrophages, lymphocytes and mast cells is normal
        • Plasma cells are also normally present and predominantly secrete IgA
        • Presence of neutrophils in large number is abnormal
    • Muscularis mucosa:
      • Muscularis mucosa consists of thin strands of smooth muscle fibers separating the mucosa and submucosa
      • Presence of muscularis mucosae in biopsies is critical to evaluate for architectural distortion
  • Submucosa:
    • Consists of loose connective tissue, thin smooth muscle bundles, nerve plexuses (Meissner plexus and Henle deep submucosal plexus) with ganglion cells, stromal cells, adipose tissue and vasculature
    • Well circumscribed areas of adipose tissue forming a discrete mass within the submucosa may represent lipomas and may present as a mass on endoscopy
    • Mucosal lymphoid aggregates or follicles may extend into submucosa, which may contain the crypts, so called lymphoglandular complex mimicking invasive carcinoma in the submucosa
  • Muscularis propria:
    • Consists of inner circular layer and outer longitudinal layer and Auerbach nerve plexus in between the 2 muscle layers
    • Interstitial cells of Cajal are present within the muscularis propria and play a role in peristalsis
  • Subserosa and serosa: subserosa is composed of fibroadipose tissue and is covered by the serosa lined by cuboidal mesothelial cells
  • Rectum: a continuation of the sigmoid colon with thicker mucosa and longer crypts primarily lined by the goblet cells; the mucosa gradually transitions from columnar to squamous from the rectum into the anal canal
  • References: Westerhoff: Histology for Pathologists, 5th Edition, 2019, Kierszenbaum: Histology and Cell Biology - An Introduction to Pathologists, 5th Edition, 2019
Microscopic (histologic) images

Contributed by Lizhi Zhang, M.D.

Colonic wall

Colonic mucosa

Cross section of crypt and lamina propria

Paneth cells and enteroendocrine cells

Submucosa and submucosal neural plexuses

Muscularis propria and myenteric plexus


CD3

Mast cells - KIT

Calretinin

S100

Interstitial cells of Cajal

Positive stains
Board review style question #1

Which cell type (marked by an arrow) is considered abnormal if present in the left side of the colon?

  1. Enteroendocrine cells
  2. Epithelial cells
  3. Goblet cells
  4. Paneth cells
Board review style answer #1
D. Paneth cells. They have a triangular shape with densely eosinophilic cytoplasmic granules and basally located nuclei, as opposed to enteroendocrine cells (see image above). Their presence in the left side of the colon can be a sign of inflammatory bowel disease.

Comment Here

Reference: Colon - Anatomy & histology
Board review style question #2
Which of the following parts of the colon is entirely intraperitoneal?

  1. Ascending colon
  2. Cecum
  3. Descending colon
  4. Rectum
Board review style answer #2
B. Cecum. It is entirely intraperitoneal. The ascending and descending colon are retroperitoneal. Upper third of the rectum is covered by the peritoneum on the anterior surface.

Comment Here

Reference: Colon - Anatomy & histology

Angiosarcoma (pending)
[Pending]

Anti-PD1 associated colitis
Definition / general
  • Colitis is an immune related adverse event of anti-PD1 (nivolumab, pembrolizumab) and anti-PDL1 medications (atezolizumab, avelumab and durvalumab)
    • PDL1 (programmed death ligand 1) and its receptor PD1 protect host cells from autoreactive T cells; monoclonal antibodies which block this interaction have been approved for treatment of several cancers (see PDL1 topic)
Essential features
  • Colitis is an immune related side effect of anti-PD1 and anti-PDL1 therapy
  • Often mild (diarrhea); very rarely severe (perforation) (Surg Case Rep 2017;3:94)
  • 2 main histologic patterns of injury (Am J Surg Pathol 2017;41:643):
    • Active colitis with neutrophilic crypt microabscesses, prominent crypt epithelial cell apoptosis and crypt atrophy / dropout (the most common pattern)
    • Lymphocytic colitis-like pattern with surface injury
Epidemiology
Sites
Pathophysiology
Clinical features
  • Abdominal pain / cramping, diarrhea, urgency and rectal bleeding in severe form
Diagnosis
  • Diagnosis of anti-PD1 colitis should be considered in patients who present with diarrhea or abdominal pain and have a history of treatment with anti-PD1 medication
  • Further evaluation with endoscopy and biopsy could confirm the diagnosis
Radiology description
Case reports
Treatment
  • Withholding the immune check point inhibitor
  • Most patients respond to corticosteroids
  • In steroid refractory cases, tumor necrosis factor α (TNFα) blocking agents such as infliximab and vedolizumab can be used
  • Reference: Practice Guideline Gastroenterology 2021;160:1384
Clinical images

Images hosted on other servers:
Missing Image

Colonoscopy

Gross images

Images hosted on other servers:
Missing Image

Small bowel perforation

Missing Image

Perforation and colonic mucosa

Microscopic (histologic) description
  • Common patterns of injury (Am J Surg Pathol 2017;41:643, Histopathology 2021;78:532):
    • Focal active colitis: neutrophilic crypt microabscesses or cryptitis with prominent crypt epithelial cell apoptosis, focal crypt atrophy / dropout
    • Lymphocytic colitis-like pattern of injury
  • Other histopathologic patterns include graft versus host disease-like or collagenous colitis patterns of injury (Histol Histopathol 2022;37:699)
  • Anti-PD1 associated colitis rarely causes perforation
Microscopic (histologic) images

Contributed by Maryam Kherad Pezhouh, M.D.
Focal active colitis with crypt drop out

Focal active colitis with crypt drop out

Prominent crypt epithelial cell apoptosis

Prominent crypt epithelial cell apoptosis

Neutrophilic crypt microabscesses

Neutrophilic crypt microabscesses

Lymphocytic colitis-like pattern

Lymphocytic colitis-like pattern

Sample pathology report
  • Colon, biopsy:
    • Colonic mucosa with lymphocytic colitis pattern of injury and mild focal active colitis (see comment)
    • Comment: Patient’s history of advanced melanoma status post treatment with pembrolizumab is noted. The biopsies show a predominantly lymphocytic colitis pattern of injury with focal mild active colitis. No significant chronicity, no granuloma and no viral cytopathic effects are seen. Overall, these findings are most consistent with anti-PD1 associated colitis.
Differential diagnosis
Board review style question #1
A 64 year old man with ulcerative colitis recently completed 3 cycles of pembrolizumab therapy for metastatic carcinoma. He underwent a colonoscopy with a biopsy for diarrhea. Which of the following features is key to differentiating anti-PD1 associated colitis from ulcerative colitis?

  1. Increased crypt epithelial apoptosis, lack of significant chronicity and absence of basal plasmacytosis
  2. Involvement of the rectum
  3. Presence of crypt abscesses
  4. Presence of erosions
Board review style answer #1
A. Increased crypt epithelial apoptosis, lack of significant chronicity and absence of basal plasmacytosis. Increased crypt epithelial apoptosis is most commonly seen in anti-PD1 colitis. On the other hand, presence of significant chronicity and basal plasmacytosis is more in favor of inflammatory bowel disease. Rectum involvement, crypt abscesses and erosions can be seen in both.

Comment Here

Reference: Anti-PD1 associated colitis
Board review style question #2

A 72 year old man with a history of melanoma recently received 3 cycles of nivolumab and developed abdominal pain and diarrhea. A colonoscopy was done which found erythema in the ascending and transverse colon. Histology is shown in the image above. What is the most likely diagnosis?

  1. Anti-PD1 associated colitis
  2. Collagenous colitis
  3. Diverticular associated colitis
  4. Severe ulcerative colitis
Board review style answer #2
A. Anti-PD1 associated colitis. Based on the history and findings of a mild focal active colitis and mild increase in intraepithelial lymphocytes, the most likely diagnosis is anti-PD1 colitis.

Comment Here

Reference: Anti-PD1 associated colitis

APC gene
Definition / general
  • APC is a tumor suppressor "gatekeeper" gene located on 5q that regulates the level of beta-catenin and directs the downstream activity of Wnt / beta catenin pathways (Appl Clin Genet 2015;8:95)
  • APC contributes to orderly migration of intestinal cells within the crypt and beta-catenin plays a crucial role in this function; it also plays a role in intercellular adhesion
  • Germline mutations in APC have various manifestations, often based on the position of the mutation (among other factors):
    • Classic familial adenomatous polyposis (FAP): mutations occur between exon 5 and 5' portion of exon 15; severe polyposis (< 5000 colorectal polyps) associated with mutations between codons 1250 and 1464
    • Attenuated FAP: < 100 colorectal adenomas (average 30), later age of onset (< 40 years old); due to mutations at the extreme 5' or 3' ends or in region of exon 9
    • Gardner syndrome: variant of FAP with benign osteoid tumors and epidermoid skin cysts
    • Hereditary desmoid disease
    • Turcot syndrome: multiple colorectal polyps and medulloblastoma
    • Congenital hypertrophy of retinal pigment epithelium: benign finding, but can identify individuals at risk for FAP (Acta Ophthalmol Scand 1996;74:338)
    • Missense mutation does not lead to classic FAP but increases risk for colorectal carcinoma; found in Ashkenazi Jewish ancestry
Molecular / cytogenetics description
  • 90% of cases of classic FAP are caused by germline mutation of APC (Clin Gastroenterol Hepatol 2014;12:1059)
  • More than 1500 germline mutations exist, the majority being point mutations (Colon Cancer Gene Variant Databases)
  • Exon 15 is the most common target for mutations, at codons 1061 and 1309 (Hum Mol Genet 2001;10:721)
  • Mutations are mostly nonsense or frameshift, resulting in a truncated protein
  • Biallelic inactivation leads to a loss of protein function (Genet Med 2014;16:101), which causes aberrant transcription of c-myc, cyclin D1 and others
  • APC mutations promote the T cell factor-lymphoid enhancer factor (TCF-LEF) pathway and inhibit the cellular adhesion complex, thereby stimulating cell proliferation (Mol Cancer 2003;2:41)
  • The gold standard for mutation detection is direct DNA sequencing of all 15 coding exons
  • Various other methods are available to detect point mutations, including PCR based methods (Tech Coloproctol 2004;8:s305)
Molecular / cytogenetics images

Images hosted on other servers:

Overview of mutations

Differential diagnosis
  • In FAP-like patients without APC mutation, biallelic mutations of MUTY human homologue (MYH) gene may be found (MUTYH associated polyposis)
  • 10% of patients with classic FAP phenotype do not have identifiable mutations in APC or MYH

Apoptotic colopathy
Microscopic (histologic) description
  • Reduced intracellular mucin, epithelial apoptosis and sloughing, increase in mitotic figures (Hum Pathol 1998;29:972, Cesk Patol 2011;47:130)
  • May have erosion of superficial epithelium, consistent with clinical impression of aphthoid ulcer
  • Also increased lymphocytes and neutrophils (Dis Colon Rectum 2006;49:109)
  • Patchy mononuclear infiltration in upper part of lamina propria, increased epithelial cell proliferation of individual crypts, basal cryptitis (Cesk Patol 2010;46:37)
  • Rarely lamina propria edema or extravasated red blood cells
Differential diagnosis

Atresia
Definition / general
  • Imperforate mucosal diaphragm or stringlike segment of bowel
  • Very rare (1 per 20,000 live births)
  • Associated with other congenital anomalies, including Hirschsprung disease
  • May have genetic (J Pediatr Surg 2005;40:390) or vascular cause
  • Type 1: bowel and mesentery are intact, but bowel lumen is interrupted by a complete membrane
  • Type 2: bowel is discontinuous, connected by a fibrous cord
  • Type 3: bowel ends are completely separated and mesentery has a gap
Treatment
Clinical images

Images hosted on other servers:

Type I atresia: mucosal septum

Atresia with hugely distended cecum

Atresia Type I

Gross images

Contributed by Dhiraj B. Nikumbh, M.D.

Congenital pouch colon

Microscopic (histologic) images

Contributed by Dhiraj B. Nikumbh, M.D.

Congenital pouch colon: 10 day old baby boy admitted with abdominal distension and absent anal opening


Balantidiasis
Definition / general
  • Disease caused by the ciliate protozoan Balantidium coli
Epidemiology
  • B. coli is found worldwide, but disease occurs most commonly in parts of the developing world including Latin America, Southeast Asia, Papua New Guinea and parts of the Middle East
  • Estimated prevalence is 1% in Aymara children in northern Bolivia (Am J Trop Med Hyg 1998;59:922);and 0.4% in rural northeast Thailand (Korean J Parasitol 2013;51:727)
  • Very low prevalence in industrialized countries
  • Humans are usually resistant to infection; disease generally occurs in debilitated or poorly nourished patients
  • Pigs are the primary reservoir for human infection and most cases occur in people in close proximity to pigs, although rats and other mammals may also transmit disease
  • Human to human transmission is also described
  • Infection by ingesting fecally contaminated food or water or from ingesting cysts due to other direct contact with pig or rat excrement
Pathophysiology
  • Excystation occurs in the small intestine and trophozoites migrate to the colon
  • Invasion into the intestinal wall occurs, where they multiply and cyst formation occurs
Diagrams / tables

Images hosted on other servers:
Missing Image Missing Image

Life cycle

Clinical features
  • Most infections are asymptomatic
  • Symptomatic patients generally suffer disease similar to amebiasis, with diarrhea, dysentery, abdominal pain and weight loss
  • Chronic disease is most common, although fulminant colitis may occur, including perforation leading to peritonitis
  • Disease in the lung, urinary bladder and bone has been described (see case reports)
  • Similarly to E. histolytica, B. coli causes flask shaped ulcers in the large intestine, most commonly in the cecum and rectosigmoid
  • Neobalantidium coli is the largest protozoan parasite and only pathogenic ciliate to infect humans (Pritt: Creepy Dreadful Wonderful Parasites Blog - Answer to Case 532 [Accessed 23 August 2019])
Diagnosis
  • Diagnosis is usually made by identification of mobile trophozoites in fresh stool or scraped from an ulcer seen during endoscopy
  • Rarely the diagnosis is made by examination of urine or bronchioalveolar lavage fluid, or by identification in biopsy or resection specimens
Case reports
Treatment
  • Tetracycline is the drug of choice
  • Alternative treatments include metronidazole, ampicillin, iodoquinol and nitazoxanide
  • Longer treatment is necessary if immunosuppressed
Microscopic (histologic) description
Microscopic (histologic) images

Contributed by Bobbi Pritt, M.D., Richard Bradbury, Ph.D. and Sarah Sapp, Ph.D.

Cyst and trophozoite of Neobalantidium coli (also known as Balantidioides) obtained from West African baboon



Images hosted on other servers:
Missing Image Missing Image

Various images

Missing Image

Trophic stage (EM)

Missing Image

Intestinal lumen

Missing Image

Encysted ciliate

Electron microscopy description
  • Flattened oval organism covered with cilia with gullet at anterior end

Basidiobolomycosis
Definition / general
  • Basidiobolomycosis is a rare fungal infection caused by environmental saprophyte Basidiobolus ranarum
  • It is a member of the order Entomophthorales of the class zygomycetes
  • The zygomycetes include two fungal orders:
    • Mucorales, involving mainly immunocompromised patients, and
    • Entomophthorales, which causes infection in immunocompetent individuals (J Korean Surg 2012;83:325)
  • Basidiobolomycosis is endemic in tropical and subtropical regions of Africa, Latin America, Middle East and Asia (Curr Pediatr Res 2013;17:1)
  • It mainly involves skin and subcutaneous tissue and rarely infects the gastrointestinal tract
  • First case of visceral basidiobolomycosis (GIB) was reported in 1964 (Trans R Soc Trop Med Hyg 1964;58:242)
Essential features
  • Basidiobolomycosis is a rare fungal infection caused by Basidiobolus ranarum
  • Causes infection in immunocompetent individuals
  • Is endemic in tropical and subtropical regions
  • Mainly involves skin and subcutaneous tissue, visceral involvement is rare
  • Infection is caused by ingestion of contaminated food or use of contaminated toilet paper
  • Abdominal pain and fever are most common presenting features
  • Histology shows broad septate thin walled fungal hyphae with the Splendore-Hoeppli phenomenon
  • Culture is gold standard for diagnosis
  • Treatment: combined surgery and medical therapy with antifungals
  • Prognosis poor in pediatric patients
ICD coding
  • B46.8 - other zygomycoses
Epidemiology
Sites
  • Mostly cause chronic infection in the skin and subcutaneous tissue
  • May cause infection in the stomach, small intestine, colon and liver
Pathophysiology
  • Infection is acquired by ingestion of infected food (e.g., people with pica) and using contaminated papers for cleaning the skin (e.g., toilet papers)
  • Possibly due to ranitidine, which can decrease gastric acidity and allow fungal survival after gastric passing (Iran J Med Sci 2015;40:90)
  • Smoking can decrease mucosal WBC function and facilitate fungal infection by B. ranarum (Clin Infect Dis 2012;54:1685)
Etiology
  • Basidiobolus ranarum
Clinical features
Diagnosis
  • Based on characteristic histopathology findings of fungal hyphae (the Splendore-Hoeppli phenomenon) and culture of B. ranarum from the tissue specimen
  • The gold standard for definite diagnosis is culture
  • Macroscopically, colonies are yellowish gray in color with a waxy appearance and many radial folds (J Med Microbiol 2012;61:1770)
Laboratory
  • Eosinophilia and high ESR (erythrocyte sedimentation rate)
  • Elevated levels of cytokines such as IL4, IL10, TNFα (TH2 type cytokines) (J Clin Microbiol 2001;39:2360)
Radiology description
Radiology images

Images hosted on other servers:

MRI before surgical intervention and treatment

Generalized markedly
thickened wall of
the small bowel with
edematous changes

Poor prognostic factors
  • Pediatric age group at presentation
  • Delayed treatment
Case reports
Treatment
  • Combined surgery and antifungal treatment for a variable period of time (6 months to 2 years) (J Pediatr Surg 2012;47:949)
  • Itraconazole, an antifungal agent is the most common drug used
Gross description
  • Fungus involves the nonmucosal layers such as submucosa and subserosa of the GI tract, hence endoscopic biopsies, which are superficial and small, usually fail to detect the fungal hyphae (Saudi Med J 2013;34:1068)
Gross images

Images hosted on other servers:

Edematous left colon

Adherent small bowel loop

Postmortem liver

Microscopic (histologic) description
  • Marked infiltration of eosinophils
  • Mixed infiltration of PMN leukocytes and granulomatous inflammation
  • Thin wall and broad hyphae surrounded by eosinophilic material, which are easily seen by hematoxylin and eosin staining; however periodic acid-Schiff (PAS) and gomori methenamine silver (GMS) can intensify the fungal wall staining
  • Zygospores which are very similar to trophozoites of amoeba (Clin Infect Dis 2001;32:1448)
Microscopic (histologic) images

Images hosted on other servers:

Staining of the tissue biopsy from left retrocolic mass

Histopathology of surgically resected abdominal mass

Missing Image

Broad hyphae with septation

Missing Image

Splendore-Hoeppli phenomenon


Missing Image

Zygospore

Missing Image

Periodic acid-Schiff stain

Missing Image

Liver biopsy

Missing Image

Spore forms

Missing Image

Various images

Cytology description
  • Broad septate thin walled fungal hyphae along with eosinophils
Positive stains
Differential diagnosis

Behcet's syndrome
Definition / general
  • Multisystem inflammatory disorder of unknown origin, with aphthous stomatitis, genital ulcers, relapsing iritis; also arthritis, cutaneous and vascular disease (eMedicine #1, #2)
  • GI involvement in 10% (ileum, cecum); causes pain, diarrhea, melena; rarely perforation (Surg Today 2003;33:383)
Epidemiology
  • More common in region from Mediterranean to eastern Asia
  • 2 / 3 in males
Case reports
Treatment
  • Steroids
Clinical images

Images hosted on other servers:

Punched-out ulcer

Gross description
  • Numerous punched out ulcers of various sizes, shapes and depths in colon
Gross images

Images hosted on other servers:

Multiple penetrations

Ulcers

Microscopic (histologic) description
  • Ulcers with neutrophils around ulcer bed, lymphocytic vasculitis in submucosal veins
  • Note: diagnosis is clinical, not histologic
Microscopic (histologic) images

Images hosted on other servers:

Loss of normal mucosa

Various images

Differential diagnosis

Brainerd diarrhea
Definition / general
  • Chronic watery diarrhea of unknown etiology with acute onset and long duration (Centers for Disease Control, Wikipedia)
  • First case in 1983 in Brainerd, Minnesota (USA)
  • Similar to epidemic infectious diarrhea, but symptoms usually last 4+ weeks, often years
  • Most patients recover within 3 years or less
Case reports
Microscopic (histologic) description
  • Surface epithelial lymphocytosis (similar to lymphocytic colitis) with no architectural distortion, no degenerative changes, no thickened subepithelial collagen (Am J Surg Pathol 1996;20:1102)

Brown bowel syndrome (pending)
Table of Contents
Definition / general
Definition / general
[Pending]

Campylobacter jejuni
Definition / general
  • Campylobacteriosis refers to disease caused by Campylobacter species and closely related bacteria
  • Campylobacter are comma shaped, gram negative, flagellated, non spore forming, microaerophilic bacteria
  • C. jejuni is by far the most common cause of Campylobacteriosis, and it has >90 serotypes
  • C. fetus Subsp. fetus infection is not rare
  • Other species ("atypical" enteric Campylobacters), including C. coli, C. lari, C. hyointestinalis, Helicobacter cinaedi, C. upsaliensis, Helicobacter fennelliae, and others are rarely isolated and when symptomatic, usually are associated with gastrointestinal illness or periodontal disease
Epidemiology
  • C. jejuni:
    • Causes 1.3 million (CDC) to >2.0 million (Mandell) infections in US annually
    • #1 or #2 most common bacterial cause of diarrheal illness in US
    • Commensal in the wild and in domesticated fowl, cattle, sheep, pigs, goats, dogs, cats, rodents
    • Has a worldwide distribution
    • For human transmission, the most important reservoirs are poultry, sheep, cattle and pigs
    • In the developed world disease is generally sporadic and most commonly occurs after ingestion of undercooked poultry or unpasteurized milk
    • Infection from eating undercooked beef, raw clams and untreated water also occurs
    • Human to human, fecal-oral transmission may occur
    • In commercial slaughterhouses, meat, especially poultry, is commonly contaminated by intestinal contents
  • Campylobacter die quickly in dry and cold conditions limiting environmental spread
  • Pasteurization and chlorine in treated water will kill the organism
  • Outbreaks have occurred following malfunctions of municipal water treatment facilities
  • Infection may occur from direct contact with companion animals, especially puppies or kittens with diarrhea
  • Individuals with occupational exposure to farm animals are at higher risk
  • The bacteria is sensitive to gastric acid and patients on acid reducing drugs such as H2 blockers and proton pump inhibitors are at higher risk
  • In the developing world, infection is mostly human to human and disease is hyperendemic in children under 2 years old; most children have several infections before the age of 2
  • Later in life, most infections are mostly asymptomatic
  • C. fetus subsp. fetus:
    • Has major reservoir in cattle and sheep
    • Reptiles commonly harbor the organism
    • Disease usually occurs in elderly / debilitated patients
    • Disease also reported in men who have sex with men, and rarely in other healthy patients
Sites
  • Disease due to C. jejuni mostly is associated with involvement of the colon; however, the jejunum and ileum may be affected
  • Rarely the bladder, gallbladder or pancreas may be infected through direct extension
  • C. fetus may cause endocarditis, pericarditis, mycotic aneurysms, pneumonitis or CNS infection
  • Rarer enteric Campylobacter species and related organisms usually affect the small and large intestines
Pathophysiology
  • Organisms which survive passage through the stomach multiply in bile and colonize upper small intestine, with subsequent involvement of large bowel
  • Virulence factors include motility (flagella), adherence, invasion and possibly toxin production
  • Adherence and colonization are necessary for invasion
Clinical features
  • Disease occurs year round but at a higher incidence in summer and early fall
  • Affects people of all ages, but is most common in children under 1 year and adolescents / young adults from 15 to 29 years old
  • Disease is more common in males
  • It is an important cause of traveler's diarrhea
  • C. jejuni:
    • Typically causes acute self-limited illness of watery diarrhea, although dysentery may occur
    • May have a flu-like prodrome for 1 day of fever, malaise, headache or myalgias
    • In addition to diarrhea, patients may experience abdominal pain, tenesmus, nausea or fever
    • Symptoms last from one day to a week or longer
    • The presentation may mimic acute appendicitis
    • Relapse of disease may occur
    • Dysentery is more common in children (50%) than adults (15%)
    • Ingestion of as few as 500 microorganisms may cause disease, with an incubation period of up to 8 days
    • Severity of infection is related to the dose of the inoculate, the virulence of the strain, and the host's immune state
    • Ingestion of higher numbers of organisms results in shorter incubation periods
    • More severe and prolonged disease occurs in patients with defects in either cellular or humoral immunity
    • Complications are uncommon but numerous; includes Guillain-Barré syndrome (GBS) in < 0.1%
      • Molecular mimicry between components of C. jejuni and nervous system gangliosides may occur resulting in cross reactivity between antibodies to C. jejuni and nervous system components
      • It is estimated that up to half of patients with GBS have had prior C. jejuni infection
    • Other potential complications include erythema nodosum, Henoch-Schönlein purpura, myopericarditis, hepatitis, septic abortion, hemolytic uremic syndrome, cellulitis and renal disease including IgA nephropathy and interstitial nephritis
    • Toxic megacolon has been reported
    • In patients with HLA-B27, reactive arthritis may occur
    • Campylobacter has been implicated in immunoproliferative small intestinal disease; may also exacerbate preexisting inflammatory bowel disease
    • C. fetus subsp. fetus affects elderly / debilitated patients and causes systemic illness affecting GI tract, heart, blood vessels, lung, nervous system
Diagnosis
  • Microscopic evaluation of fresh (<2hr) stool may enable a presumptive diagnosis to be made due to characteristic "darting motility"
    • Microscopic stool examination shows leukocytes and red blood cells
    • Diagnosis traditionally has been made by microbiologic culture; special techniques are necessary as Campylobacter species grow more slowly than other enteric organisms and will otherwise be overgrown
      • C. jejuni grows best at 42°C but will grow at 37°C
      • Most laboratories incubate cultures at 42° that may make it impossible to isolate other Campylobacter species besides C. jejuni
    • The diagnosis may also be made by PCR of stool or paraffin blocks, but until recently this has been mostly in research settings (Am J Surg Pathol 2006;30:782)
    • Recently a PCR based assay for testing stool has been introduced that in addition to Campylobacter group bacteria, also detects Shigella species, Vibrio group, Yersinia enterocolitica, Shiga toxin 1 and 2, Norovirus G1/GII, Rotavirus A and Aeromonas species
  • Rarely C. jejuni may be diagnosed by blood culture
    • C. fetus subsp fetus is usually detected in blood cultures
    • If Campylobacter species infection is suspected, blood cultures should be incubated for two weeks due to its slow growth
  • Immunoassays are also available
Prognostic factors
  • Usually excellent; worse in immunosuppressed patients
Case reports
Treatment
  • Most cases of C. jejuni do not require antibiotic therapy; if necessary, fluid and electrolyte replacement should be given
  • Antibiotic treatment is recommended in the setting of high fever, bloody diarrhea, patients who have more than 8 bowel movements in a day or disease lasting longer than one week
  • Erythromycin is generally the drug of choice, although tetracyclines, aminoglycosides, chloramphenicol, quinolones, nitrofuran, and clindamycin are also effective
  • Increasing resistance to macrolides (azithromycin, clarithromycin, erythromycin, others) and fluoroquinolones (ciprofloxacin, levofloxacin) is reported
  • In patients with hypogammaglobulinemia, fresh frozen plasma in addition to antibiotics may be useful
Microscopic (histologic) description
  • Biopsy is rarely performed - the non-specific findings are those of acute self-limited colitis
  • Neutrophils are increased within the lamina propria and cryptitis or crypt abscesses often occur
  • Crypt architecture is maintained, although neutrophilic infiltrates may make this hard to appreciate
  • Basal lymphoplasmacytosis is absent
Microscopic (histologic) images

Images hosted on other servers:
Missing Image Missing Image

Cryptitis

Missing Image Missing Image

Edema, cryptitis and crypt abscess


Missing Image

Cystic crypts and cryptitis

Missing Image

Loss of crypts

Missing Image

Marked edema

Missing Image

Neutrophilic aggregates in lamina propria

Differential diagnosis

Candida
Definition / general
  • Candida colonies are normal commensals of the gastrointestinal tracts and rarely pathogenic in gut
  • Candida infections are usually seen in immunocompromised and debilitated patients due to concurrent chronic diseases, malnutrition, use of immunosuppressive drugs or old age (Int J Contemporary Medical Research 2017;4:76)
  • In immunocompetent patients, low pH, regular use of antacids as well as hyperglycemia can cause the yeast to break through the intestinal mucosal barrier and lead to invasive candidiasis and candidemia (Dtsch Arztebl Int 2009;106:837)
Essential features
  • Candida colonies are normal commensals of gastrointestinal tracts
  • Infections are usually seen in immunocompromised and debilitated patients
  • Factors favoring fungal proliferation include: regular antacid use, which lowers pH; use of antibiotics, which diminish useful bacteria; and hyperglycemia, as seen with diabetes
  • Candida usually causes mucosal lesions; rarely perforations
  • Esophagus is the most commonly affected site in GI
  • Gold standard test for diagnosis is culture
  • Antifungals are treatment of choice
ICD coding
  • B37.8 - Candidiasis of other sites
Epidemiology
  • Patients usually middle aged to elderly, with an average age of 65 years
Sites
Pathophysiology
  • Under normal conditions, there is homeostasis between C. albicans and the host; however, if the balance is disrupted by low pH, regular use of antacids or hyperglycemia, the yeast can break through the intestinal mucosal barrier and cause invasive candidiasis and candidemia (Dtsch Arztebl Int 2009;106:837)
Etiology
Diagrams / tables

Images hosted on other servers:

Fungal colonization
in patients with
GI tract disease

Clinical features
  • Invasive Candida infections are characterized by fever and shock along with low blood pressure, abdominal pain, rigors, an elevated heart rate, respiratory distress and multiorgan failure (Emerg Med (Los Angel) 2015;5:264)
Diagnosis
  • Histopathology showing fungal hyphae and spores
  • Gold standard is culture for fungi
Laboratory
Radiology description
  • With intestinal perforation, chest Xray and Xray abdomen in erect view demonstrate gas under right hemidiaphragm
Prognostic factors
  • Prognosis of Candida peritonitis is very poor in critically ill surgical patients, with a mortality rate between 52% and 75% (Curr Opin Crit Care 2007;13:195)
  • Risk factors associated with increased mortality in Candida peritonitis include: extremes of age, upper GI tract infection as well as patient comorbidities, such as cardiac insufficiency, cirrhosis, diabetes mellitus, renal failure and multiorgan failure (Medicine (Baltimore) 2011:90:69)
Case reports
Treatment
  • The antifungal agents for Candida peritonitis include polyenes, azoles, echinocandins and flucytosine
  • Azoles are fungistatic against most Candida species, whereas the polyenes and echinocandins are fungicidal
  • Fluconazole is recommended for patients without prior azole exposure and not at high risk, such as elderly, diabetic and cancer patients (Clin Infect Dis 2009;48:503, Perit Dial Int 2009;29 Suppl 2:S161)
Gross description
  • Candida tends to cause mucosal lesions and produce ulcers of varying configuration, mucosal flecks, sloughed mucous membranes, polypoid masses and segmental lesions (J Clin Pathol 1992;45:806)
  • With perforation, bowel may be covered by a thick, black, serosal exudate (Acta Paediatr 2003;92:258)
Microscopic (histologic) description
  • Ulcer slough with eosinophils and neutrophils
  • Classical budding hyphae and spores of Candida admixed with ulcer slough
Microscopic (histologic) images

Contributed by Nalini Bansal, M.D.

PAS positive fungal hyphae in a case of gastric adenocarcinoma with fungal infection



Images hosted on other servers:

Fungal hyphae with gastric performation

Cytology description
  • Fungal spores and hyphae
Positive stains
Negative stains
Differential diagnosis

Chagas disease (trypanosomiasis)
Definition / general
  • Colonic involvement in the chronic phase of infection by the flagellate protozoan Trypanosoma cruzi, causing chagasic megacolon
Essential features
  • Infection most commonly involves the heart and GI tract (esophagus is most frequent site, followed by colon)
  • Patients with chagasic megacolon suffer from chronic constipation and abdominal pain; in severe cases, there may be several weeks between bowel movements
  • Histology shows ganglionitis with neuronal cell depopulation
Terminology
  • Also known as American trypanosomiasis
ICD coding
  • B57.3 Chagas disease (chronic) with digestive system involvement
  • B57.32 Megacolon in Chagas' disease
Epidemiology
  • Worldwide, an estimated 100 million people are at risk (Am J Trop Med Hyg 2014;90:814); 8 million are chronically infected (CDC - Chagas Disease: Detailed FAQs) with 56,000 new infections a year (Am J Trop Med Hyg 2014;90:814) and 12,000 deaths (PAHO WHO - Chagas disease)
  • Chagas disease is endemic in all of the Western hemisphere south of the United States
  • Vector borne transmission does not occur in the Caribbean
  • Insect vectors are found in the southern United States but vector borne infection is rare there
  • It is estimated that 300,000 residents of the United States are infected, the vast majority are immigrants from countries where the disease is endemic (CDC - Chagas Disease: Epidemiology and Risk Factors)
  • There are very rare reports of disease in tourists
  • Imported cases may occur worldwide
  • Insect vector is more common in outdoor settings, including animal burrows, kennels, dog houses, chicken coops, under porches, between rocky structures and wood piles
  • It is more commonly found in rural settings
  • Deforestation for agriculture has played a role in its spread
  • Indoors, it prefers cracks, nooks, crannies and holes typically found in dilapidated housing, including homes with mud walls and thatched roofs, accounting for its association with poverty
  • Due to migration, disease is also found in urban settings
  • Far less common routes of transmission are congenital, consuming contaminated food or drink, blood transfusion and organ transplants
  • 5% of infants born to infected mothers become infected
  • Public health efforts, including improving housing and public hygiene, as well as bed nets and screening donated blood, have resulted in declining numbers of T. cruzi infections
  • Interestingly, gastrointestinal disease is much more likely to occur in the southern range of endemic Chagas disease
Sites
  • Infection most commonly involves the heart and GI tract (esophagus is most frequent site, followed by colon)
  • Far less often, other parts of the GI tract, the biliary tract and the central nervous system are affected
Etiology
  • Infection is transmitted by blood sucking triatomine bugs, a type of reduviid bug that caries T. cruzi parasites
  • They become infected by biting infected mammals including humans
  • They multiply in the midgut as epimastigotes, a distinct type of flagellate; in the hindgut they transform into infective metacyclic trypomastigotes
  • People are bitten on the face and the insect defecates
  • Infection occurs when feces containing trypomastigotes reaches mucous membrane or skin abrasions, often wiped into the wound or conjunctiva by the host's hand
  • An inflammatory lesion at the site of infection is known as a chagoma
  • Parasites invade several types of host cell, transform into amastigotes and multiply in the cytoplasm
  • Amastigotes differentiate into trypomastigotes and subsequently the cell ruptures
  • Parasites spread hematogenously or invade tissues especially muscle
  • They alternate between intracellular and swimming forms in blood
  • Acute infection sometimes is associated with acute illness, usually with mild nonspecific symptoms, although rarely severe or even fatal disease may occur
  • Reactivation of infection in the setting of immunosuppression may lead to severe disease
  • Chronic Chagas disease occurs in 10 - 30% of infected individuals years or even decades after initial infection, most commonly involving the heart, less likely the gastrointestinal tract
  • Cytotoxic T cells invade ganglia, causing reduced numbers of interstitial cells of Cajal, leading to parasympathetic denervation with reduced and eventual failure of relaxation of anal sphincters during defecation
  • In some cases the inflammatory response may be contained by enteric glial cells (Hum Pathol 2009;40:244)
Diagrams / tables

Images hosted on other servers:

Life cycle

Clinical features
  • Infection may occur from blood transfusion or organ transplantation and there are rare reports of transmission from contaminated food or laboratory accidents
  • In a minority of cases, acute disease occurs that may be lethal due to cardiac or neurologic complications
  • Infection is lifelong without treatment
  • Most infected subjects remain in an asymptomatic indeterminant phase but 10 - 30% develop cardiac or gastrointestinal tract complications
  • Parasite invades the bowel wall and damages the enteric nervous system leading to megacolon
  • Patients with chagasic megacolon suffer from chronic constipation and abdominal pain; in severe cases, there may be several weeks between bowel movements
  • Chronic disease manifests years or even decades after initial infection
    • Fecalith / fecaloma may develop
    • Acute obstruction may occur, possibly with volvulus and lead to perforation
    • Toxic megacolon may occur
Diagnosis
  • Acute disease may be diagnosed by review of a peripheral blood smear; however, other laboratory methods are necessary to diagnosis chronic Chagas disease
Laboratory
  • Serologic diagnosis by enzyme linked immunosorbent, immunofluorescence or chemiluminescence methods
  • Equivocal serologic tests may be confirmed with PCR based testing but this lacks sensitivity for detecting chronic disease
  • False positives may occur; the WHO recommends confirmation with a different format assay
Radiology description
  • Megacolon with contraction of anal sphincter
  • Lengthening of the distal colon
Prognostic factors
  • Immunosuppressed patients may suffer from severe disease
Case reports
Treatment
  • Early / mild cases are treated with high fiber diets, laxatives, stool softeners and enemas
  • With more severe disease, manual disimpaction may be necessary
  • In children, endoscopic emptying may be employed
  • Toxic megacolon and volvulus are treated surgically and surgical intervention may be considered for patients with severe chronic disease
  • Antiparasitic treatment has a high failure rate and potentially toxic side effects
  • Acute infection and children under 18 are usually treated
  • Whether to treat indeterminant or chronic symptomatic infection is controversial
  • Nifurtimox and benznidazole are available from the CDC
Clinical images

Contributed by Bobbi Pritt, M.D., Nicole L. Achee, Ph.D. and John P. Grieco, Ph.D.

Triatomine bugs

Microscopic (histologic) description
  • Ganglionitis with neuronal cell depopulation
  • Fibrosis and mast cell infiltrates are also present
Microscopic (histologic) images

Images hosted on other servers:
Missing Image

Blood smears

Differential diagnosis
Board review style question #1
    Which statement is true?

  1. Chronic Chagas disease most commonly affects the central nervous system
  2. Patients with acute Chagas disease invariably progress to chronic disease
  3. The incidence of Chagas disease is increasing
  4. The majority of individuals infected with T. cruzi spontaneous clear the parasite
  5. Vector borne transmission of Chagas disease only occurs in the Western hemisphere
Board review style answer #1
E. Vector borne transmission only occurs in the Western hemisphere. Chagas disease is endemic in all of the continental Americas south of the United States but it does not occur in the Caribbean. Vector borne transmission in the southern United States is documented but is rare. Imported cases occur worldwide. Infection is life long but only 10 - 30% of individual who are infected will progress to chronic disease that most commonly affects the heart, followed by the esophagus, then the colon. The infection is transmitted by blood sucking triatomine bugs, a type of reduvid bug that caries T. cruzi parasites. Triatomine bugs are more likely to be found in conditions that occur in dilapidated housing; public health campaigns to improve housing as well as screening of donated blood has resulting in decreasing numbers of new infections over time.

Comment Here

Reference: Chagas disease (trypanosomiasis)

Chemotherapy induced colitis (pending)
[Pending]

Chromosomal instability & MUTYH pathways
Chromosomal instability pathway
  • Common pathway in colorectal carcinoma
  • Mutations in oncogenes and tumor suppressor genes, including APC, beta catenin, Kras, BRAF, SMAD4, PTEN, p53 and bax
  • Have altered total DNA content, cytogenetics, aneuploidy and numerous allelic gains and losses and translocations
  • 70% + of colorectal carcinomas (Neoplasia 2008;10:680)
  • Etiology
  • Clinical features:
    • Important in familial adenomatous polyposis and variants (APC gene) and juvenile polyposis (DPC4, PTEN genes)
    • High rate of concordance for Kras status between primaries and metastases (Oncologist. 2008;13:1270)
    • Patterns of Kras mutation vary based on germline mismatch repair defects and hMLH1 methylation status (Hum Mol Genet 2004;13:2303)
    • Updated National Comprehensive Cancer Network (NCCN) Clinical Practice Guidelines in Oncology for Colon Cancer now recommend that tumors from all patients with stage IV disease be tested for the Kras gene
    • Only patients whose tumors have normal (wild type) Kras should receive cetuximab and panitumumab
MUTYH pathway
  • See also MUTYH associated polyposis
  • MUTYH (also called MYH) gene encodes MUTYH glycosylase, involved in oxidative DNA damage repair (Wikipedia)
  • Mutations occur in both copies of MUTYH repair gene
  • Genetic pathway is not well understood (Curr Genomics 2008;9:420)
  • Biallelic patients (mutations in both genes) have multiple adenomas (Hum Mutat 2006;27:1064) and 93× excess risk of colorectal carcinoma compared to normal controls but account for < 1% of all cases (Am J Hum Genet 2005;77:112)
  • Germline mutation of MUTYH gene is a mechanism of development of APC negative familial adenomatous polyposis; these patients may have a family history compatible with recessive inheritance indicating that this may be a different disease; genetic testing may be helpful in screening, diagnosis and management of atypical FAP cases
  • Can be detected by high resolution melting analysis

Chronic granulomatous disease (pending)
[Pending]

Chronic intestinal pseudo-obstruction
Definition / general
  • Syndrome of intestinal obstruction without mechanical obstruction
  • Usually a small bowel disorder but can occur anywhere in GI tract
  • Ogilvie syndrome (acute colonic pseudo-obstruction): abrupt onset of abdominal distension (Radiol Med (Torino) 2005;109:370)
Etiology
  • Gut motility depends on sympathetic (thoracolumbar) and parasympathetic (vagal) innervation to ganglionated plexi; also enteric nervous system, smooth muscle cells and interstitial cells of Cajal
  • Results from autonomic imbalance resulting in sympathetic overactivity affecting some part of colon (Singapore Med J 2009;50:237)
  • Usually congenital in children; in adults due to systemic disease (amyloidosis, Chagas disease, dermato / polymyositis, diabetes, muscular dystrophy, myotonic dystrophy, myxedema, scleroderma), drugs (anti-Parkinson, clonidine, ganglionic blockers, phenothiazines, tricyclic antidepressants) or idiopathic (cathartic colon, ceroidosis, Hirschsprung disease, visceral myopathies, visceral neuropathies)
  • May be due to loss of interstitial cells of Cajal in small and large bowel (Am J Surg Pathol 2003;27:228)
Prognostic factors
Case reports
Treatment
  • Diet, octreotide, surgery, transplant
Microscopic (histologic) description
  • Visceral myopathy (Am J Surg Pathol 1987;11:846):
    • Vacuolar degeneration with swelling and loss of muscle cells, fibrosis of outer longitudinal muscle layer
    • Other cases show cytoplasmic vacuoles, marked nuclear enlargement and irregularity and interstitial fibrosis
  • May have segmental hypoganglionosis at transitional zone (Am Surg 2011;77:736)
Microscopic (histologic) images

Images hosted on other servers:

Familial autonomic visceral myopathy:

Muscularis propria degeneration

Muscularis mucosae with degenerative changes

Differential diagnosis
  • Ischemic colitis:
    • Hemosiderin deposits, fibrous stricture
  • Scleroderma:
    • Patchy bowel involvement, dense fibrosis affecting inner or all muscle layers, no vacuolar change
  • Tuberculosis:
    • Stricture, necrotizing granulomas

Clostridium botulinum
Definition / general
  • Botulism is a disease caused by botulinum toxin ("botox"), a potent neurotoxin produced by Clostridium botulinum or closely related microorganisms such as C. baratii or C. butyricum
Epidemiology
  • Botulism is uncommon in the developed world
  • According to the CDC in the United States, about 145 cases are reported each year, 65% in infants, 20% wound related and 15% foodborne
  • Most cases are associated with eating raw or undercooked food containing botulinum toxin or C. botulinum spores
  • Clusters of disease associated with eating the same food are common
  • Canned (especially home canned alkaline foods), vacuum packed food and fermented or smoked foods are often implicated
  • Infant botulism is related to ingestion of spores which colonize and release toxin in the intestine; may be related to honey ingestion (MMWR 2003;52:21)
  • Wound botulism is caused by toxin released by C. botulinum that has contaminated a wound; may follow heroin injection (Euro Surveill 2013;18:20630, Anaerobe 2014;30:108)
  • Rare cases are associated with inhalation of spores, sometimes in a health care setting
  • Recently, iatrogenic botulism has been reported due to botulinum toxin for cosmetic or therapeutic purposes (Clin Neuropharmacol 2012;35:254, Clin Neuropharmacol 2010;33:158, Clin Neuropharmacol 2007;30:310, JAMA 2006;296:2476)
Etiology
  • Clostridium are toxin producing, spore forming, anaerobic, gram positive bacilli found in soil and marine sediment
  • Botulinum toxin is one of the most dangerous substances known; one millionth of a gram is a lethal dose
    • It is a potent neurotoxin that blocks the presynaptic release of acetylcholine across the neuromuscular junction, leading to a toxic neuropathy
    • Eight toxins have been identified, A-H; A, B, E, and F have been associated with human disease
  • Foodborne botulism is associated with ingesting food containing botulinum toxin while infant botulism is associated with ingesting spores that colonize the intestines and produce toxin; disease in adults via a similar mechanism is rare, but may occur (adult intestinal colonization botulism)
  • Wound, inhalation and iatrogenic botulism are discussed above under Epidemiology
Clinical features
  • The incubation period for infant botulism is 2 to 4 weeks, with a peak incidence from 2 to 4 months
    • Infants usually present with constipation, occasionally hypoventilation, followed by hypotonia, drooling, weak cry, generalized muscular weakness ("floppy baby") and hypoventilation; half have upper airway obstruction and cranial nerve palsies (Pediatr Neurol 2005;32:193)
    • Has been associated with sudden infant death syndrome (Lancet 1985;1:237, Lancet 1978;1:1273)
  • Patients with foodborne botulism generally present with acute, bilateral, symmetric cranial neuropathies
    • Symptoms include lack of coordination of eye muscles, double vision, swallowing difficulties and dizziness
    • Subsequently there is descending progressive weakness of the extremities and respiratory muscles
    • Patients are generally afebrile, alert, and oriented
    • Physical exam demonstrates flaccid muscle weakness of tongue, laryngeal muscles, respiratory muscles and extremities
    • Without rapid intervention, patients may die of respiratory paralysis or cardiac arrest
Diagnosis
  • A high index of suspicion is generally necessary for diagnosis
  • Diagnosis is largely clinical through electromyographic studies
  • PCR based assays for the bacteria in stool and gastric aspirates or culture are suggestive
  • Detection of toxin in stool, blood, contaminated food or the environment is more definitive
  • Testing should be performed in public health laboratories, not Level A Laboratories
  • These laboratories or the CDC should be contacted for information on specimen transport
  • Public health authorities should be notified as epidemiologic investigations are necessary; foodborne botulism is considered a public health emergency
  • Of note, infection does not lead to antibody production, and no serologic testing is available
Prognostic factors
  • With good supportive care, the death rate is currently < 5%; previously was > 50%
Case reports
Treatment
  • Equine derived heptavalent antitoxin (available from CDC) for food borne and wound botulism
  • Debridement with antibiotics for wound botulism
  • Human Botulism Immune Globulin Intravenous (BabyBig) is administered for infant botulism; generally antibiotics are not given for infant botulism
  • Supportive care, especially respiratory support is critical
Diagrams / tables

Images hosted on other servers:
Missing Image

Binary toxin

Microscopic (histologic) images

Images hosted on other servers:
Missing Image

Chitin degradation

Electron microscopy images

Images hosted on other servers:
Missing Image

Electron micrograph

Differential diagnosis
  • Acute motor axonal neuropathy
  • Allergic reaction
  • Guillain-Barré syndrome
  • Miller Fisher syndrome
  • Mosquito borne encephalitis
  • Poliomyelitis
  • Tick paralysis

Colitis cystica profunda
Definition / general
  • Reactive mucosal change characterized by submucosal mucin filled cysts lined by benign epithelium
Essential features
  • Mucosal herniation into submucosa secondary to various forms of injury
  • Bland histology distinguishes from a true invasive process
Terminology
  • Also called colitis cystica polyposa or hamartomatous inverted polyp when presents as polypoid lesion
Epidemiology
  • Uncommon; male predilection
Sites
  • Can occur anywhere in GI tract, including stomach (gastritis cystica profunda) and small intestine (enteritis cystica profunda)
Pathophysiology
  • Secondary to mucosal damage due to ulcerative colitis, Crohn's disease, radiation, diverticulitis or other inflammation / ulceration of bowel; mucus follows granulation tracts to involve large areas of bowel
Clinical features
  • Stools contain blood or mucus; patients may have diarrhea or abdominal pain
Radiology description
Case reports
Treatment
  • Patient education to avoid straining, high fiber diet with bulk laxatives, surgery for rectal prolapse
Gross description
  • May appear as a polypoid or submucosal mass
Microscopic (histologic) description
  • Either localized or diffuse; similar histology to colorectal polyps in Cowden syndrome although inverted
  • Misplaced benign colonic epithelium forming cystic spaces filled with mucin and surrounded by lamina propria; mucin may extravasate into surrounding tissue
Microscopic (histologic) images

Contributed by Raul S. Gonzalez, M.D.
Colitis cystica profunda

Colitis cystica profunda

Sample pathology report
  • Rectum, mass, biopsy:
    • Colitis cystica profunda
    • Negative for dysplasia or malignancy.
Differential diagnosis
Board review style question #1

A patient undergoes routine colonoscopy and is found to have a submucosal mass. A mucosal biopsy is unremarkable and surgery is performed. What is the best diagnosis?

  1. Colitis cystica profunda
  2. Mucinous adenocarcinoma
  3. Pneumatosis cystoides intestinalis
  4. Tubular adenoma with pseudoinvasion
Board review style answer #1
A. Colitis cystica profunda

Comment Here

Reference: Colitis cystica profunda

Collagenous colitis
Definition / general
  • Form of microscopic colitis clinically associated with chronic watery diarrhea and histologically characterized by colonic intraepithelial lymphocytosis, surface mucosal damage, lamina propria inflammation and a subepithelial collagen band
Essential features
  • Subtype of microscopic colitis in which patients present with chronic watery diarrhea and normal or nearly normal endoscopic findings
  • Pathophysiology and etiology are unclear but thought to be multifactorial with a possible luminal antigen triggering inflammation and mucosal damage in a genetically predisposed individual
  • Histology is characterized by a subepithelial collagen band, intraepithelial lymphocytosis, mixed inflammation in the lamina propria and surface mucosal damage with preserved crypt architecture
ICD coding
  • ICD-10: K52.831 - Collagenous colitis
Epidemiology
Sites
Pathophysiology
  • Poorly understood
  • Presumed mechanism includes a dysregulated immune reaction to an unknown luminal antigen (medication, dietary factor, infectious agent, other) in a genetically predisposed individual (Lancet Gastroenterol Hepatol 2019;4:305)
  • Watery diarrhea is a result of inhibited sodium channel upregulation, downregulation of tight junctions and colonocyte aquaporins and bile acid malabsorption (Lancet Gastroenterol Hepatol 2019;4:305)
Etiology
  • Strong association with certain medications
    • NSAIDs, aspirin, proton pump inhibitors, H2 receptor antagonists, selective serotonin reuptake inhibitors, among others (Am J Gastroenterol 2017;112:78)
  • Other specific etiologies are unknown
Clinical features
Diagnosis
Prognostic factors
Case reports
Treatment
  • Discontinuation of any offending medications (Am J Gastroenterol 2017;112:78)
  • Antidiarrheals such as loperamide, diphenoxylate or bismuth subsalicylate
  • For more severe symptoms, corticosteroids such as budesonide can be used
  • Surgical intervention is a last resort
Clinical images

Images hosted on other servers:

Mucosal tears

Endoscopic findings

Microscopic (histologic) description
  • Thickened subepithelial collagen band, usually with capillary, red blood cell and inflammatory cell entrapment (Histopathology 2015;66:613)
    • Collagen band typically > 10 microns
    • Occasional cases can have an irregular collagen band that is < 10 microns; some have labeled such cases as "incomplete collagenous colitis"
    • Trichrome stain can be useful in equivocal cases
  • Surface mucosal damage with loss of mucin, flattening and detachment
  • Increased lamina propria inflammation composed of lymphocytes, plasma cells and eosinophils
    • Neutrophilic inflammation can be seen but is typically not prominent
  • Increased intraepithelial lymphocytes; may be mild (< 20 per 100 epithelial cells)
  • Crypt architecture is preserved or minimally distorted
  • Subepithelial multinucleated giant cells can be seen but have no clinical significance (Pathology 2008;40:671)
  • Pseudomembranous collagenous colitis has been described; unclear whether truly a variant of collagenous colitis or due to superimposed infection or medication injury (Dig Dis Sci 2004;49:1763, Pathol Res Pract 2013;209:735)
Microscopic (histologic) images

Contributed by Catherine E. Hagen, M.D.
Preserved architecture

Preserved architecture

Thickened subepithelial collagen

Thickened subepithelial collagen

Epithelial detachment

Epithelial detachment

Equivocal collagen

Equivocal collagen

Trichrome stain

Trichrome stain

Giant cell

Giant cell


Crypt atrophy

Crypt atrophy

Virtual slides

Images hosted on other servers:

Collagenous colitis biopsy

Positive stains
  • Masson trichrome stain highlights the subepithelial collagen band
  • CD3 highlights intraepithelial lymphocytes
  • Tenascin immunohistochemical stain also highlights the collagen band (Histopathology 2015;66:613)
Negative stains
Sample pathology report
  • Colon, random, biopsies:
    • Colonic mucosa with a thickened subepithelial collagen band, increased intraepithelial lymphocytes and surface epithelial injury, consistent with collagenous colitis
Differential diagnosis
  • Lymphocytic colitis:
    • Lack of subepithelial collagen band and greater number of intraepithelial lymphocytes
  • Inflammatory bowel disease:
    • Lack of subepithelial collagen band
    • Architectural distortion and prominent neutrophilic inflammation
    • Endoscopic evidence of inflammation and clinical history of bloody diarrhea
  • Chronic ischemic colitis:
    • Diffuse lamina propria hyalinization as opposed to a discrete subepithelial collagen band
    • Withered, injured crypts
  • Amyloidosis:
    • Usually surrounds blood vessels, though rarely may create a subepithelial layer mimicking collagenous colitis
    • Congo red stain is positive
    • Lack of surface epithelial injury and inflammation
Board review style question #1

A 55 year old female with a 1 year history of chronic watery diarrhea presents for evaluation. Stool ova / parasites and culture are negative for infectious organisms. Colonoscopy is performed and reveals a grossly normal appearing colon. The biopsy histology is shown. Which of the following diagnoses is correct?

  1. Collagenous colitis
  2. Crohn's colitis
  3. Irritable bowel syndrome
  4. Lymphocytic colitis
Board review style answer #1
A. Collagenous colitis

Comment Here

Reference: Collagenous colitis
Board review style question #2
A 57 year old female was recently diagnosed with collagenous colitis. Which of the following is a characteristic histologic feature of collagenous colitis?

  1. Architectural distortion
  2. Cryptitis and crypt abscess
  3. Granulomas
  4. Mixed lamina propria inflammation
Board review style answer #2
D. Mixed lamina propria inflammation

Comment Here

Reference: Collagenous colitis

Colon carcinoma overview
Definition / general
  • Primary epithelial malignancy arising in the colorectum
Essential features
  • More than 1 million new cases worldwide per year
  • Second or third most common cancer and cause of cancer deaths in men and in women
  • 98% of colonic cancers are adenocarcinomas
Epidemiology
  • Most common gastrointestinal tumor in U.S.; less common in Africa, Asia and parts of South America
  • Affects 5% of the U.S. population during their lifetime
  • More common in men than women
  • Most common cause of cancer related death among nonsmokers
  • Peak age 60 - 79 years
    • < 20% of cases occur before age 50
    • Rare before age 40 except in patients with a predisposition syndrome
Sites
  • May arise anywhere in the colorectum but sigmoid colon and rectum are most common sites
Etiology
  • Risk factors: older age, obesity, physical inactivity, alcohol consumption, inflammatory bowel disease, schistosomiasis, family history of colorectal neoplasia
  • Polyposis syndromes: familial adenomatous polyposis and variants (APC gene), Lynch syndrome and variants (MLH1, MSH2, MSH6 and PMS2 genes), juvenile polyposis (SMAD4, PTEN genes), Peutz-Jeghers syndrome (STK11 gene)
  • Dietary risk factors: low vegetable fiber, high refined carbohydrates, increased beef consumption, decreased vitamins A / C / E
    • Low fiber prolongs transit time (toxic oxidative byproducts are in longer contact with colonic mucosa) and alters bacterial flora
    • Beef consumption enhances synthesis of bile acids by liver, which may be converted into carcinogens by bile acids
Clinical features
  • Screening: colonoscopy, guaiac fecal occult blood test
  • Symptoms:
    • Right sided carcinomas cause anemia (due to blood loss) and vague abdominal pain
    • Left sided carcinomas cause change in bowel habits (diarrhea or constipation) and rectal bleeding
    • Some patients may be asymptomatic, and tumors are detected by screening, especially if lesion is early
  • Metastases:
    • 60% of patients have lymph node or distant metastases at diagnosis
    • Most common metastatic sites are regional lymph nodes, liver, peritoneum, lung, ovaries
    • Metastases may simulate primary tumors of affected organs
  • Prognosis:
    • 5 year survival is 40% - 60%
    • Most recurrences are within 2 years
Laboratory
  • Serum CEA: elevated levels associated with various carcinomas or liver disease
    • Useful for monitoring recurrences but not sensitive for early tumors
Radiology description
  • Imaging allows for clinical staging (depth of invasion, possibility of metastasis)
Prognostic factors
  • Poor prognostic factors: high stage, positive margins (particularly radial margin in rectal carcinoma), poor differentiation, signet ring cells, flat or ulcerative gross configuration, tumor budding, tumor perforation, free tumor cells in peritoneal space, lymphovascular invasion, perineurial invasion (Am J Clin Pathol 2003;119:108)
  • Lack of CDX2 expression and reduced claudin 1 expression portend worse survival in stage II tumors (N Engl J Med 2016;374:211, Mod Pathol 2005;18:511)
  • HER2 / neu and VEGF are not important prognostic markers (BMC Cancer 2011;11:277)
Treatment
  • Resection with endoscopic follow up
  • Local excision (endoscopic mucosal resection) for early rectal carcinoma
  • Surgical excision of isolated distant metastases
  • Radiation therapy or chemotherapy (including neoadjuvant if rectal)
Board review style question #1
What is the most common gastrointestinal malignancy in the U.S.?

  1. Colorectal adenocarcinoma
  2. Esophageal squamous cell carcinoma
  3. Gastric adenocarcinoma
  4. Small bowel adenocarcinoma
Board review style answer #1
A. Colorectal adenocarcinoma

Comment Here

Reference: Colon carcinoma overview
Board review style question #2
The following syndromes confer an increased risk of colorectal carcinoma. Which syndrome is properly linked to its causative germline mutation?

  1. Familial adenomatous polyposis: MLH1 mutation
  2. Juvenile polyposis: SMAD4 mutation
  3. Lynch syndrome: STK11 mutation
  4. Peutz-Jeghers syndrome: APC mutation
Board review style answer #2
B. Juvenile polyposis: SMAD4 mutation

Comment Here

Reference: Colon carcinoma overview

Common variable immunodeficiency (CVID)
Definition / general
  • Common variable immunodeficiency disorders (CVID) are uncommon conditions of primary B cell dysfunction or T cell impairment leading to hypogammaglobulinemia and IgA or IgM deficiency
  • Immunoglobulin deficiency elicits an immunodeficiency state with increased susceptibility to recurrent infections, inflammatory diseases, autoimmune diseases and malignancy (J Allergy Clin Immunol Pract 2016;4:38)
Essential features
  • CVID is due to B cell impairment causing hypogammaglobulinemia with either IgA or IgM deficiency
  • Infectious, inflammatory, autoimmune and malignant gastrointestinal manifestations can occur in approximately 15% of patients with CVID
  • CVID histology in the colon may include one or more of the following: inflammatory bowel disease-like pattern, microscopic colitis pattern, granulomatous inflammation, increased apoptosis, lymphoid aggregates and infections such as cytomegalovirus and herpes simplex virus
  • Presence of plasma cells does not rule out CVID because approximately 33% of CVID cases will have plasma cells
Epidemiology
Sites
Pathophysiology
  • All patients have B cell loss of function due to an intrinsic defect or lack of extrinsic activation by another immune cell (J Allergy Clin Immunol Pract 2016;4:38)
  • B cell impairment leads to reduced or absent IgG, IgA or IgM immunoglobulin levels (J Allergy Clin Immunol Pract 2016;4:38)
  • Reduced immunoglobulin levels leads to increased susceptibility to recurrent infections especially of the sinopulmonary and gastrointestinal tract
  • Gastrointestinal tract involvement is a manifestation of CVID due to its large surface area and presence of numerous immunoglobulin producing plasma cells; if the plasma cells become dysfunctional, immune dysregulation and infection can result
  • Altered immunity can also lead to infection, autoimmune disease, inflammatory disease and malignancy
Etiology
Clinical features
  • Infectious, inflammatory, autoimmune and malignant gastrointestinal tract manifestations can occur in approximately 15% of individuals with CVID (Blood 2012;119:1650)
  • Symptoms of gastrointestinal tract manifestations in CVID patients include: weight loss, anemia, chronic diarrhea, chronic abdominal discomfort, bloody diarrhea (Dig Dis Sci 2007;52:2977, Am J Gastroenterol 2019;114:648)
  • Colonoscopy and biopsy reports from 63 Finnish patients with CVID identified (Am J Gastroenterol 2019;114:648):
    • Nonspecific colitis in 14%
    • Ulcerative colitis in 8%
    • Microscopic colitis in 10%
    • Crohn's disease 2%
    • Colonic polyps (types not specified) in 30%
    • GI malignancies (types not specified) in 3%
    • Cytomegalovirus colitis in 2%
    • Herpes simplex virus colitis in 2%
Diagnosis
  • CVID is diagnosed in the proper clinical situation with the following criteria (J Allergy Clin Immunol Pract 2016;4:38):
    • Presentation with immunodeficiency at greater than 2 years of age
    • Decreased serum IgG (≥ 2 standard deviations below the mean for age) and decreased serum IgA or IgM
    • Absent isohemagglutinins or inability to develop antibodies to vaccines
    • Acquired hypogammaglobulinemia is ruled out
  • Colonoscopy and biopsy are helpful in the assessment of lower gastrointestinal tract involvement by inflammatory, autoimmune, infectious and malignant sequelae of CVID
Laboratory
  • Low serum IgG (≥ 2 standard deviations below the mean for age)
  • Low serum IgA or IgM
  • Circulating B cells may be normal (54% of cases), increased (19% of cases), decreased (12% of cases) or undetectable (12% of cases) (Blood 2008;112:277)
  • Circulating T cells and natural killer cells are usually within normal limits (Clin Immunol 1999;92:34)
  • Important to remember: serology may be falsely negative or falsely positive due to the patient's inability to mount a humoral response and due to intravenous immunoglobulin (IVIG) administration, respectively
Case reports
Treatment
  • IVIG may be insufficient to treat inflammatory and autoimmune conditions associated with CVID (J Allergy Clin Immunol Pract 2016;4:38)
  • Infections, autoimmune diseases, inflammatory diseases and malignancies need to be treated when they are identified
Microscopic (histologic) description
  • Numerous colonic histologic patterns can be seen in CVID including the following (Am J Surg Pathol 2007;31:1800, Am J Gastroenterol 2019;114:648):
    • Ulcerative colitis-like pattern and Crohn's disease-like pattern
      • Some cases may have active colitis but without features of chronicity such as crypt atrophy / loss and crypt branching
    • Lymphocytic colitis pattern and collagenous colitis pattern
      • Unclear whether these patterns of injury are mimicking or are true cases of lymphocytic colitis and collagenous colitis
    • Granulomatous inflammation
      • Etiology may be infectious or inflammatory, fungal and mycobacterial stains are important to rule out infection
    • Viral infection (CMV, HSV, etc.)
      • Important to thoroughly evaluate for viral inclusions as serology may be negative due to hypoglobulinemia associated with CVID
    • Increased apoptosis can occur in colonic epithelium mimicking graft versus host disease, viral infection (CMV) and drug effect (mycophenolate)
    • Lymphoid aggregates are a common finding in the colon of CVID patients
  • Presence of plasma cells does not rule out CVID as reduced plasma cells were identified in 63% of colonic CVID biopsies / resection specimens (Am J Surg Pathol 2007;31:1800)
Microscopic (histologic) images

Contributed by Benjamin J. Van Treeck, M.D and Christopher Hartley, M.D.

Lamina propria neutrophils

Absence of plasma cells

Increased crypt apoptosis

Mild active colitis

Reduced mucosal plasma cells


Increased lamina propria eosinophils

Increased lamina propria neutrophils

Mild to moderate active chronic colitis

CD138

Positive stains
  • CD138 and MUM1 will demonstrate reduced or absent plasma cells in some cases
Sample pathology report
  • Colon, biopsy:
    • Colonic mucosa with mild active chronic colitis, increased apoptosis and absence of plasma cells (evaluated with CD138); negative for dysplasia (see comment)
    • Comment: These features are consistent with the patient’s history of common variable immunodeficiency. Clinicopathologic correlation is recommended.
Differential diagnosis
  • Crohn's colitis:
    • Reduced to absent plasma cells is not a feature of Crohn's colitis
    • Common variable immunodeficiency disorders (CVID) injury pattern can mimic Crohn's disease to the extent that patients may be diagnosed with Crohn's disease prior to being diagnosed with CVID (Am J Surg Pathol 2007;31:1800)
    • Clinical history of CVID is helpful in distinguishing Crohn’s colitis from CVID
  • Ulcerative colitis:
    • Reduced to absent plasma cells is not a feature of ulcerative colitis
    • Clinical history of CVID is helpful in distinguishing ulcerative colitis from CVID
  • Lymphocytic colitis:
    • Reduced to absent plasma cells is not a feature of lymphocytic colitis
    • Clinical history of CVID is helpful in distinguishing lymphocytic colitis from CVID
  • Collagenous colitis:
    • Reduced to absent plasma cells is not a feature of collagenous colitis
    • Clinical history of CVID is helpful in distinguishing collagenous colitis from CVID
  • Cytomegalovirus (CMV) colitis:
    • Evaluate for CMV viral inclusions
    • CMV immunohistochemical stain can be helpful
    • CVID can coexist with CMV infection
  • Herpes simplex virus (HSV) colitis:
    • Evaluate for HSV viral inclusions
    • HSV immunohistochemical stain can be helpful
    • CVID can coexist with HSV infection
  • Acute self limited colitis:
    • Reduced to absent plasma cells is not a feature of acute self limited colitis
    • Clinical history of CVID and microbiology tests are helpful in distinguishing acute self limited colitis from CVID
    • Acute self limited colitis may coexist with CVID
  • Lymphoma such as mantle cell lymphoma, follicular lymphoma and others:
    • Clonal population of lymphocytes is not a feature of CVID
    • CVID may predispose to lymphoma
  • Graft versus host disease (GVHD):
    • Clinical history of transplant is necessary to diagnosis GVHD
    • Reduced to absent plasma cells is not a feature of GVHD
  • Sarcoidosis:
Board review style question #1


A patient with chronic diarrhea, recurrent sinopulmonary infections and reduced serum IgG and IgA levels undergoes colonoscopy. The colonic biopsies demonstrate rare lamina propria plasma cells. Which one of the following statements is correct?

  1. Presence of rare plasma cells does not rule out the possibility of common variable immunodeficiency (CVID) in this patient and should be mentioned in the pathology report
  2. Presence of rare plasma cells is secondary to chronic diarrhea and is not related to common variable immunodeficiency (CVID) in this patient
  3. Presence of rare plasma cells rules out the possibility of common variable immunodeficiency (CVID) in this patient
  4. Presence of rare plasma cells should not be reported and a diagnosis of normal colonic mucosa should be included in the pathology report
Board review style answer #1
A. Presence of rare plasma cells does not rule out the possibility of common variable immunodeficiency (CVID) in this patient and should be mentioned in the pathology report

Comment Here

Reference: Common variable immunodeficiency (CVID)
Board review style question #2
A patient with chronic diarrhea, recurrent sinopulmonary infections and reduced IgG and IgA levels is diagnosed with common variable immunodeficiency (CVID). Which of the following histologic patterns can be seen in colonic biopsies from patients with CVID?

  1. Inflammatory bowel disease-like pattern, increased apoptosis pattern, colonic mucosal prolapse-like pattern
  2. Inflammatory bowel disease-like pattern, increased apoptosis pattern, lymphocytic colitis pattern
  3. Inflammatory bowel disease-like pattern, increased apoptosis pattern, radiation induced colitis-like pattern
  4. Inflammatory bowel disease-like pattern, ischemic colitis-like pattern, lymphocytic colitis pattern
Board review style answer #2
B. Inflammatory bowel disease-like pattern, increased apoptosis pattern, lymphocytic colitis pattern

Comment Here

Reference: Common variable immunodeficiency (CVID)

Congenital absence of muscularis propria (pending)
Table of Contents
Definition / general
Definition / general
[Pending]

COVID-19 associated colitis
Definition / general
  • Colonic mucosal damage associated with severe acute respiratory syndrome coronavirus 2 (SARS-CoV-2) infection
Essential features
ICD coding
  • ICD-10:
    • U07.1 - COVID-19
    • B34.2 - coronavirus infection, unspecified
    • B97.2 - coronavirus as the cause of disease classified elsewhere
Epidemiology
  • WHO declared the COVID-19 outbreak, caused by SARS-CoV-2, a global pandemic
  • Primarily affecting the respiratory system, COVID-19 can also lead to gastrointestinal symptoms
  • Common comorbidities: hypertension, diabetes mellitus, chronic cardiac disease, chronic respiratory disease, chronic renal disease (Clin Gastroenterol Hepatol 2020;18:2378, BMJ Open Gastroenterol 2021;8:e000578)
Pathophysiology
  • SARS-CoV-2 binds to the angiotensin converting enzyme 2 (ACE2) receptor through its surface spike protein for host entry
  • ACE2 receptor is highly expressed in the intestinal epithelial cells
  • Potential mechanisms for COVID-19 associated colitis:
Etiology
  • SARS-CoV-2, an enveloped positive stranded RNA virus related to the severe acute respiratory syndrome (SARS) virus and the Middle East respiratory syndrome (MERS) virus
Clinical features
  • Common clinical presentation:
  • Gastrointestinal symptoms typically develop within 1 week of onset of COVID-19 infection
  • Diffuse abdominal pain / tenderness, metabolic acidosis and hypotension indicate worsening clinical course and potential need for surgical intervention
Diagnosis
Laboratory
  • Elevation of prothrombin time and international normalized ratio (INR)
  • D dimer > 1,850 ng/mL and presence of at least 1 gastrointestinal symptom were independently associated with major endoscopic abnormalities (BMJ Open Gastroenterol 2021;8:e000578)
  • Elevation of liver function test (aspartate aminotransferase [AST], alanine aminotransferase [ALT]) (Am J Gastroenterol 2020;115:766)
Radiology description
Radiology images

Images hosted on other servers:
CT scan

CT scan

Case reports
Treatment
  • Supportive care, bowel rest and close observation
  • Surgical intervention is indicated if there is evidence of perforation, infarction and necrosis (Am J Emerg Med 2020;38:2758.e1)
Clinical images

Images hosted on other servers:
Colonoscopic findings

Colonoscopic findings

Microscopic (histologic) description
Microscopic (histologic) images

Contributed by Feng Yin, M.D., Ph.D.
Ischemic injury with microthrombi

Ischemic injury
with microthrombi

Withered crypts with neutrophils

Withered crypts with neutrophils

Positive stains
Electron microscopy description
Electron microscopy images

Images hosted on other servers:
Virion particles

Virion particles

Sample pathology report
  • Colon, biopsy:
    • Ischemic colitis consistent with clinical history of SARS-CoV-2 infection
Differential diagnosis
  • Inflammatory bowel disease (IBD):
    • Personal or family history of IBD or other autoimmune disorders
    • Onset tends to be subacute to chronic
    • Marked architectural distortion and prominent acute inflammation
  • Ischemic colitis, NOS:
    • Noninfectious colitis
    • Watershed areas most commonly affected
    • Clinical correlation essential
  • Medication associated mucosal damage:
    • History of medication (such as NASID) use
    • May present as chronic mucosal injury pattern
  • Acute infectious colitis (other than COVID-19):
    • Caused by bacteria or virus
    • Acute cryptitis, crypt abscess
    • May present with ischemic injury pattern
    • Bacterial culture, PCR and sequencing diagnostic
Board review style question #1

A 71 year old man with a medical history of diabetes mellites type 2, coronary artery disease and morbid obesity was hospitalized for COVID-19 pneumonia. On day 3 of hospital stay, he developed left sided abdominal pain and hematochezia. Endoscopy showed mucosal erosions and ulceration at the splenic flexure. A representative photomicrograph of the biopsy is shown. Which is the most likely diagnosis?

  1. COVID-19 associated colitis
  2. Diverticulitis
  3. Inflammatory bowel disease
  4. Mesenteric ischemia
  5. Pseudomembranous colitis
Board review style answer #1
A. COVID-19 associated colitis. The biopsy reveals colonic mucosa with focal epithelial mucin depletion, withered crypts, mildly hyalinized stroma, mucosal microthrombi and patchy acute inflammatory infiltrate. In the current clinical setting (patient with COVID-19 pneumonia followed by abdominal pain and hematochezia), this is most compatible with COVID-19 associated ischemic colonic mucosal injury. Clinical correlation is essential.

Comment Here

Reference: COVID-19 associated colitis
Board review style question #2
What is the most common histologic feature of COVID-19 associated colitis?

  1. Basal lymphoplasmacytosis and crypt architectural distortion
  2. Eosinophilic cryptitis and abscess
  3. Intraepithelial lymphocytosis
  4. Pseudomembrane material composed of fibrin, mucin, neutrophils and inflammatory debris
  5. Withered crypts, lamina propria hyalinization and microthrombi
Board review style answer #2
E. Withered crypts, lamina propria hyalinization and microthrombi. The most common histologic feature of COVID-19 associated colitis is ischemic colitis pattern, including withered crypts, lamina propria hyalinization and microthrombi.

Comment Here

Reference: COVID-19 associated colitis

Cowden syndrome
Definition / general
  • Cowden syndrome is one component of the PTEN hamartomatous tumor syndrome, which also includes Bannayan-Riley-Ruvalcaba syndrome, PTEN related Proteus syndrome and Proteus-like syndrome
Essential features
  • Syndrome with autosomal dominant inheritance caused by mutations in PTEN and sometimes other genes
  • Causes benign hamartomatous overgrowths of skin, GI tract and thyroid
  • Also increases risk of malignancy of breast, thyroid, endometrium and colorectum
Sites
Clinical features
  • Gastrointestinal hamartoma found in 35 - 80% of patients; increased risk for colorectal cancer
  • Mucocutaneous lesions include trichilemmoma, acral keratosis and oral papilloma
  • Macrocephaly; dysplastic gangliocytoma of cerebellum (Lhermitte-Duclos disease)
  • Breast fibroadenoma, fibrocysttic disease and carcinoma
  • Thyroid goiter, adenoma and carcinoma
  • Increased risk for endometrial carcinoma
Case reports
Clinical images

Images hosted on other servers:

Feet, face, mouth

Facial papules

Microscopic (histologic) description
  • Colon polyps: nondysplastic epithelium, dilated glands, expanded stroma; histologically similar to juvenile polyps; may also show intramucosal lipomas (Mod Pathol 2018;31:643)
Microscopic (histologic) images

Images hosted on other servers:

Juvenile polyp

Polyps with mildly dilated glands

Molecular / cytogenetics description
Board review style question #1
Cowden syndrome is caused by a germline mutation in what gene?

  1. BMPR1A
  2. KRAS
  3. PTEN
  4. STK11
Board review style answer #1
C. PTEN

Comment Here

Reference: Cowden syndrome
Board review style question #2

In addition to hamartomatous polyps, NOS (as pictured), which of the following colonic lesions suggests the possibility of Cowden syndrome?

  1. Ganglioneuromatosis
  2. Gastric heterotopia
  3. Intramucosal lipoma
  4. Schwann cell hamartoma
Board review style answer #2
C. Intramucosal lipoma

Comment Here

Reference: Cowden syndrome

Crohn's disease
Definition / general
  • Chronic, relapsing, idiopathic inflammatory gastrointestinal disease
  • Involvement of the upper and lower gastrointestinal tract in a discontinuous and transmural manner
Essential features
  • Segmental, patchy inflammatory bowel disease, often involving the ileum, colon and upper GI tract
  • Histologically characterized by chronic active colitis with associated transmural lymphoid aggregates and fissuring ulcers
  • Noncaseating granulomas are characteristic but neither sensitive nor specific
  • Main differential diagnosis: ulcerative colitis, indeterminate colitis and infectious colitis
Terminology
  • Terminal ileitis
  • Granulomatous enterocolitis
  • Regional enteritis
ICD coding
  • ICD-10: K50 - Crohn's disease (regional enteritis)
  • ICD-11: DD70 - Crohn's disease
Epidemiology
  • Incidence: 0.003 - 0.02%
  • Prevalence: 0.3%
  • Slight female predominance in adult onset Crohn's disease
  • Onset of disease at 20 - 40 years of age with a second peak described at the fifth to sixth decade of life
  • More prevalent in Ashkenazi Jewish populations
  • Incidence in Hispanic and Asian populations has increased in recent decades
  • Incidence and prevalence are higher in high income countries and in urban areas, compared to low income countries and rural areas
  • References: Mayo Clin Proc 2017;92:1088, Lancet 2017;389:1741
Sites
  • Ileocolonic involvement: 30 - 40%
  • Exclusive colon involvement: 30 - 40%, 50% rectal sparing
  • Small bowel involvement: 80%
  • Exclusive small bowel involvement: 30 - 40%
  • Of patients with colon disease, 20% will develop ileal involvement in the next 10 years
  • Of patients with ileal disease, 20% will develop colon involvement in the next 10 years
  • 52% have right sided colitis, 40% have left sided colitis and 6% have pancolitis
  • Approximately 25% have perianal complications, including fissures and fistulas
  • Upper gastrointestinal complications are present in 5 - 30%, 50% in children and adolescents
  • < 10% present with isolated perianal, upper gastrointestinal or extraintestinal complications
  • Reference: Mod Pathol 2012;25:295
Pathophysiology
  • Not fully understood
  • Immunobiology: impaired intestinal barrier function and dysregulation of the innate and adaptative immune system responses, with an alteration of gut microbiota
    • Deficient mucus biofilm barrier: decreased expression of mucin secretion genes (MUC1, MUC19 and PTGER4) in the terminal ileum in patients with Crohn's disease
    • Permeability changes in the intestinal epithelium: altered expression of tight junction proteins (claudin)
    • Paneth cell dysfunction
    • Impaired autophagy of invasive microbes
    • Imbalance of effector T cells and naturally regulatory T cells
    • Recruitment and erratic retention of leukocytes
  • Dysbiosis:
    • Continuous alterations in intestinal microbiota resulting in clustering and reduced diversity in Firmicutes and Bacteroidetes phyla
    • Reduction in Faecalibacterium prausnnitzii was associated with an increased recurrence of ileal Crohn's disease in the postoperative setting
  • Genetic risk factors:
    • Increased risk for individuals who have family history
    • Concordance in monozygotic twins is 20 - 50% compared with 10% in dizygotic twins
    • Wide genome association studies identified 200 loci associated to Crohn's disease; however, they contribute only a modest relative risk increase
    • NOD2, ATG16L1, IL23R gene variants are responsible for some of the heritable risk
  • Environmental risk factors:
    • Cigarette smoking doubles the risk of developing Crohn's disease
    • Reduced fiber dietary intake
    • Antibiotic therapy during childhood increases the risk of developing Crohn's disease
    • Other medications including nonsteroidal anti-inflammatory drugs, oral contraceptives and aspirin
    • Breastfeeding appears to be a protective factor against the development of Crohn's disease
  • References: Nat Rev Dis Primers 2020;6:22, Lancet 2012;380:1590
Etiology
  • Idiopathic
Clinical features
  • Signs and symptoms: abdominal pain, diarrhea, fatigue and weight loss
    • Involvement of other GI sites can present as oral painful aphthous ulcers, odynophagia and dysphagia, postprandial vomiting and nausea, and malabsorption (including diarrhea and steatorrhea with associated nutritional deficiencies)
  • 3 phenotypic clinical subtypes:
    • Inflammatory phenotype: inflammation of the GI tract with no fistula or stenosing process; classic symptoms
    • Stricturing phenotype: inflammation can progress to fibrosis and stenosis; symptoms are associated with bowel obstruction (i.e., lack of bowel movements, nausea and vomiting)
    • Fistulizing phenotype: continuous transmural inflammation can cause sinus tract formation resulting in fistulas between bowel and other organs (vagina and bladder)
  • Extraintestinal manifestations:
    • Musculoskeletal: arthritis or arthropathy and bone loss
    • Ocular: uveitis, iritis and episcleritis
    • Cutaneous: erythema nodosum and pyoderma gangrenosum
    • Hepatobiliary: primary sclerosing cholangitis, pyogenic liver abscess
    • Renal: secondary amyloidosis leading to renal disease, calcium oxalate and uric acid renal stones
    • Pulmonary: bronchiectasis, chronic bronchitis, interstitial lung disease, bronchiolitis obliterans with organizing pneumonia, sarcoidosis
  • There are several clinical scoring systems that categorize patients into low and high risk (e.g., Crohn's disease activity index and Harvey-Bradshaw index)
  • Endoscopic findings:
    • Ileocolonoscopy is the gold standard for diagnosis
      • Ulceration: small aphthous ulcers (< 5 mm) and transmural ulcers
      • Cobblestone appearance: linear and serpiginous ulcers with intervening nonulcerated mucosa
      • Skip lesions: discontinuous lesions surrounded by adjacent normal tissue
    • Other (less specific) findings:
      • Normal rectal mucosa
      • Inflammation of the terminal ileum with no colonic inflammation
  • References: Mayo Clin Proc 2017;92:1088, Inflamm Bowel Dis 2011;17:471, Gastroenterol Rep (Oxf) 2018;6:75
Diagnosis
  • Diagnosis consists of a combination of clinical findings and complementary diagnostic tests including blood tests, stool tests, upper and lower endoscopic studies, radiologic imaging and histologic evaluation
  • The aim is to exclude differential diagnoses, establish a diagnosis and classify the severity of the disease
Laboratory
  • C reactive protein: nonspecific marker for acute inflammation; correlates with disease activity
  • ASCA (anti-Saccharomyces cerevisiae antibodies): positive in 60 - 70% of patients with Crohn's disease, 10 - 15% of patients with ulcerative colitis and < 5% of patients without inflammatory bowel disease
  • pANCA (perinuclear antineutrophil cytoplasmic antibodies): positive in 10 - 15% of patients with Crohn's disease, positive in 60 - 70% of patients with ulcerative colitis
    • Patients with Crohn's disease with positive pANCA tend to have an ulcerative colitis resembling phenotype
  • Fecal calprotectin: fecal biomarker used to distinguish between inflammatory bowel disease and functional bowel disease; elevated levels of fecal calprotectin indicate the need for further investigation for inflammatory bowel disease
  • Reference: Lancet 2017;389:1741
Radiology description
  • Frequently used to assess disease in the small bowel: bowel ultrasonography, CT scan and MRI are helpful for the assessment of the extent of the disease and presence of complications
  • Small bowel findings:
    • Asymmetrical segmental mural hyperenhancement: specific to Crohn's disease; other types of segmental mural hyperenhancement are less specific
    • Wall thickening
    • Intramural edema: indicative of bowel inflammation
    • Strictures: more common with fibrosis and inflammation; proximal bowel dilation can correlate with higher fibrotic burden
    • Ulcers: indicative of severe inflammation (Radiology 2018;286:776)
  • Pelvic MRI: used to evaluate and define perianal fistula tracts (Radiology 2017;282:628)
Radiology images

Images hosted on other servers:

CT enterography, small bowel Crohn's disease

MRI of Crohn's disease patient with anovaginal fistula

Prognostic factors
  • Associated with a 1.38 increase in mortality
    • Recurrence in approximately 95% of patients after 10 years of diagnosis, most commonly as ileocolonic disease
    • Postoperative recurrence is dependent on disease location
      • Isolated ileal disease recurrence is proximal to anastomosis
      • Ileocolitis disease recurrence frequently occurs both proximal and distal to the anastomosis (Gut 2012;61:1140)
  • Increased risk for relapse, surgery or complications:
    • Patient features:
      • Young age at diagnosis (< 40 years)
      • Smoking
    • Disease features:
      • High disease burden and prolonged duration of disease
      • Perianal disease
      • Stricturing disease
      • Involvement of the upper GI tract
      • Requiring corticosteroids during first flare
      • Lack of epithelial healing after clinical remission
      • Presence of deep ulcers
      • Granulomas on biopsy specimen
    • Laboratory tests:
      • High C reactive protein and ASCA
      • High fecal calprotectin levels
      • Low serum levels of albumin and hemoglobin
  • Patients with Crohn's disease have an increased risk for:
Case reports
Treatment
  • Mainstay of treatment is medical therapy with a goal to achieve clinical, endoscopic and histologic remission, demonstrated by complete mucosal healing
  • Chosen based on disease stage, severity and location
    • Low risk patients: step up therapy; initially less potent medications with better adverse effect profile are used, while more potent medications are reserved for patients who do not respond to the initial approach
    • High risk patients: top down therapy; potent therapies, including biologic therapy and immunomodulators, are used early in the disease to prevent complications
  • Medical therapies:
    • Corticosteroids
    • Thiopurines
    • Methotrexate
    • Anti-TNF agents
  • Surgical therapy: approximately 50% of patients with Crohn's disease will have at least 1 surgical procedure due to disease complications
    • Stricturing disease: irreversible with medical therapy; if obstructive symptoms are ongoing, surgery may be indicated
    • Fistulizing disease: cases of enterovesicular, enterovaginal and enterocutaneous fistulas or sinus tract and abscess formation
    • Perianal disease: perianal fistula or abscess
  • Reference: Gastroenterology 2014;147:702
Clinical images

Images hosted on other servers:

Endoscopic features of Crohn's disease

Gross description
Gross images

Contributed by Elizabeth Heidi Cheek-Norgan, M.H.S., PA (ASCP) and Catherine E. Hagen, M.D.
Stricture Stricture

Stricture

Microscopic (histologic) description
  • Similar to ulcerative colitis, untreated cases typically show features of active chronic colitis
  • Features of chronicity include:
    • Crypt architectural distortion
    • Inflammatory expansion of the lamina propria with basal lymphoplasmacytosis
    • Paneth cell metaplasia or hyperplasia
    • Pyloric gland metaplasia of small bowel and right colon
  • Features of activity include neutrophilic inflammation with cryptitis, crypt abscess and ulceration
  • Patchy and segmental distribution with skip lesions
  • Aphthous ulcers and deep fissuring ulcers
  • Granulomas are characteristic but only present in approximately 50% of patients (Colorectal Dis 2011;13:1142)
    • Often well formed and sarcoid-like but may be poorly formed or consist of giant cells
    • Should be distinguished from crypt rupture granuloma
  • Transmural inflammation with lymphoid aggregates in the subserosal adipose tissue
  • Sinus tracts and fistula formation
  • Biopsies are limited in determining the depth and distribution of inflammation; therefore, diagnosis is often reliant on clinical correlation or examination of the resection specimen (Histopathology 2014;64:317)
  • Dysplasia may be present in patients with longstanding disease
  • Activity is usually graded similar to ulcerative colitis (Gastroenterology 2007;133:1099):
    • Inactive: absence of neutrophils
    • Mild: activity involving < 50% of the mucosa
    • Moderate: activity involving > 50% of the mucosa; crypt abscesses often seen
    • Severe: presence of surface ulceration or erosion
  • References: Histopathology 2014;64:317, Best Pract Res Clin Gastroenterol 2019;38:101601
Microscopic (histologic) images

Contributed by Catherine E. Hagen, M.D. and Luisa Ricaurte Archila, M.D.
Patchy involvement

Patchy involvement

Active chronic colitis

Active chronic colitis

Pyloric gland metaplasia

Pyloric gland metaplasia


Granuloma Granuloma

Granuloma

Transmural inflammation Transmural inflammation

Transmural inflammation

Negative stains
Sample pathology report
  • Colon, biopsy:
    • Mildly active chronic colitis with multiple nonnecrotizing granulomas (see comment)
    • Comment: The histologic findings are suggestive of idiopathic inflammatory bowel disease. Presence of multiple nonnecrotizing granulomas favors Crohn's disease over ulcerative colitis but clinical correlation with distribution of disease is required.
  • Colon, resection:
    • Severely active chronic colitis with transmural inflammation and fissuring ulcers compatible with Crohn's colitis; no dysplasia or malignancy
Differential diagnosis
  • Ulcerative colitis:
    • Diffuse involvement of the colon and rectum
    • Absence of skip lesions, except in treated cases
    • Absence of granulomas, except when associated with ruptured crypts
    • Backwash ileitis: in the setting of severe cecal disease, frequently without severe activity or chronic changes
  • Indeterminate colitis:
    • Diagnosis of exclusion when it is impossible to distinguish between ulcerative colitis and Crohn's disease
    • This term is only used when a resection specimen is available
  • Infectious colitis:
    • Presence of noncaseating granulomas
    • Yersinia:
      • Central necrosis within granulomas and lack of transmural inflammation
    • Salmonella and Campylobacter:
      • Poorly circumscribed microgranulomas
    • Tuberculosis:
      • Florid coalescent granulomatous inflammation, extensive caseous necrosis and nodal granulomas with no intramural granulomas
  • Segmental colitis associated with diverticulitis:
    • Inflammation limited to segment of colon with diverticulosis
Board review style question #1

A 45 year old woman underwent ileocolectomy for fulminant colitis. Representative sections were submitted for histology (shown above). What is the most likely diagnosis based on the histologic features?

  1. Collagenous colitis
  2. Crohn's disease
  3. Ischemic colitis
  4. Ulcerative colitis
Board review style answer #1
B. Crohn's disease

Comment Here

Reference: Crohn's disease
Board review style question #2

Which of the following features favors Crohn's disease over ulcerative colitis?

  1. Crypt architectural distortion
  2. Diffuse, continuous involvement of colon
  3. Nonnecrotizing granulomas
  4. Superficial, broad based ulceration
Board review style answer #2
C. Nonnecrotizing granulomas

Comment Here

Reference: Crohn's disease

Cronkhite-Canada syndrome
Definition / general
  • Rare, nonhereditary polyposis syndrome of unknown etiology
Essential features
  • Disorder of middle adulthood, with mean age of diagnosis of 59 years
  • Poor prognosis due to associated nutritional complications
  • Up to 50% of patients have remissions
  • Rarely has a fulminant course (Endoscopy 2010;42:E350)
  • Associated carcinomas documented in 15 - 25% of cases (J Clin Pathol 2014;67:891)
Sites
  • Polyps affect entire gastrointestinal tract, except the esophagus
  • Also associated with ectodermal changes including alopecia, nail atrophy and cutaneous hyperpigmentation
Pathophysiology
  • No known genetic link; considered nonhereditary
  • May represent an autoimmune disorder (Digestion 2007;75:96)
Clinical features
  • Patients present with variable symptoms, including diarrhea, weight loss, nausea, GI bleeding, protein losing enteropathy
Case reports
Clinical images

Images hosted on other servers:

Upper endoscopy: gastric polyposis

Microscopic (histologic) description
  • Multiple ill defined polyps with cystically dilated glands and crypts associated with an edematous lamina propria containing mononuclear cells and eosinophils, microscopically indistinguishable from juvenile polyps (Am J Surg Pathol 1989;13:940)
  • Intervening nonpolypoid mucosa characteristically also has dilated glands, edema, congestion and inflammation (Gastroenterol Res Pract 2009;2009:619378)
Microscopic (histologic) images

Contributed by Raul S. Gonzalez, M.D. and Michael Feely, D.O.

Duodenum: elongated, irregular and cystic crypts

Colon: polypoid mucosa with cystically dilated glands



Images hosted on other servers:

Polyp with dilated glands

Duodenum / small intestine

Differential diagnosis
Board review style question #1
Germline mutation in which of the following genes has been linked to Cronkhite-Canada syndrome?

  1. BRAF
  2. KRAS
  3. None (syndrome is nonhereditary)
  4. TP53
Board review style answer #1
C. None (syndrome is nonhereditary)

Comment Here

Reference: Cronkhite-Canada syndrome
Board review style question #2
The risk of developing carcinoma in patients with Cronkhite-Canada syndrome is approximately

  1. 0 - 5%
  2. 15 - 25%
  3. 40 - 50%
  4. 65 - 75%
Board review style answer #2

Cryptosporidium parvum
Definition / general
Epidemiology
  • Cryptosporidium has a worldwide distribution (excepting Antarctica)
  • Infection is usually person to person through the fecal-oral route, via ingestion of infective oocysts
  • In some cases, zoonotic infection from sheep, cows, pigs, rodents, companion animals and other animals may occur
  • The oocysts are hardy and are not killed by chlorination of drinking water
  • Developing world: primarily affects children under age 5 and in most cases persistent diarrhea occurs that may be compounded by malnutrition; uncommon in adults
  • Developed world:
    • More common in children but not to the extent of the developing world
    • Disease due to:
      • Spread of pediatric cases in day care centers
      • Travel to developed countries
      • Spread in mental institutions
      • Contaminated recreational water, including swimming pools, rivers, lakes, fountains
        • Cryptosporidium is the most common cause of waterborne disease in recreational water
      • Animal handlers
      • Food borne spread
      • Breakdowns in municipal water purification systems
    • United States: 300,000 to 750,000 cases each year, more commonly in summer
    • The largest known outbreak occurred in Milwaukee, Wisconsin in 1993, affecting an estimated 400,000 people, although this type of spread is now uncommon in the developed world
    • Sexual transmission in men who have sex with men has been reported
    • Disease is common in immunosuppressed patients especially AIDS patients with CD4 counts under 100

  • There are over 20 species of cryptosporidium:
    • C. hominis, the human genotype that primarily infects people and C. parvum, the bovine genotype, are the most important causes of human disease
    • C. hominis was formerly known as C. parvum anthroponotic genotype 1
    • C. meleagridis, C. Canis, C. felis, C. ubiquitum, C. cuniculus, C. suis, C. muris and other species are known to cause human disease
Sites
  • Infection is most common in terminal ileum and proximal colon
  • Disease also occurs in proximal small intestine, distal colon, gallbladder, bile ducts and pancreas
  • Widespread disease generally occurs with severe immunosuppression and may involve respiratory tract
Pathophysiology
  • Ingested oocysts excyst in stomach and small intestine, releasing 4 infective sporozoites that bind to intestinal epithelial cells
  • The sporozoite becomes embedded in cell membrane in a parasitophorous vacuole
  • Inside the vacuole, the sporozoite undergoes merogony (asexual reproduction) to become trophozoites
  • The trophozoites divide to become type I meronts that mature, causing the parasitophorous vacuole to rupture, releasing motile merozoites that bind to epithelial cells and are engulfed
  • Merogony is repeated or sexual differentiation occurs and merozoites differentiate into micro and macrogamonts
  • The microgamonts release microgametes that penetrate cell walls of cells infected with macrogamonts
  • The macrogamont and microgametes fuse and form zygotes
  • The zygote undergoes meiosis to form an oocyst containing 4 sporozoites
  • There are thin walled and thick walled forms of oocysts
  • The thin walled form excysts in the host causing autoinfection while the thick walled oocyst is shed in the environment
  • Infection causes enterocolitis or malabsorption
Clinical features
  • The incubation period is usually about one week but may be from 1 to 30 days
  • In immunocompetent individuals, acute but self-limited profuse watery diarrhea usually occurs
  • In children in the developing world, persistent diarrhea occurs but most patients recover
  • In immunosuppressed patients, chronic diarrhea occurs that is often debilitating
  • In immunosuppressed patients, acalculous cholecystitis, sclerosing cholangitis, pancreatitis, biliary strictures and respiratory disease may occur
Diagnosis
  • The diagnosis is usually made by visualization of cysts in stool using immunofluorescence or a modified acid fast stain
  • It is important to communicate with the laboratory if there is a suspicion of cryptosporidiosis, as not all laboratories routinely test stool for Cryptosporidiium
  • On occasion, the diagnosis is made on biopsy (see Microscopic (histologic) description)
Prognostic factors
  • In immunocompetent adults, self limited diarrhea lasting one or two days is the rule
  • If the CD4 count is under 100, chronic diarrhea occurs
  • In HIV+ patients with CD4 counts under 50 and other immunosuppressed patients with markedly suppressed T cell function, fulminant diarrhea occurs
  • In HIV+ patients with CD4 counts greater than 150 - 180, self-limited diarrhea usually occurs
  • The ultimate prognosis in chronic and fulminant diarrhea in HIV+ patients is related to the success of therapy to reconstitute the immune system
  • In the developing world, children under 5 usually suffer from persistent diarrhea that is often complicated by malnutrition
Case reports
Treatment
  • Nitazoxanide reduces the severity of disease in immunocompetent patients and has been approved by the FDA for these patients; its effectiveness in immunocompromised patients is unclear although it is used in that setting
  • Reconstitution of the immune system through highly active anti-viral therapy in AIDS patients or reduction in immunosuppression in other settings is important
Microscopic (histologic) description
Microscopic (histologic) images

Contributed by Bobbi Pritt, M.D. and Institute of Tropical Medicine Antwerp

Negative fuchsine staining according to Heine

Ziehl-Neelsen staining



Images hosted on other servers:

Oocysts: auramine-rhodamine stain

Missing Image

Small blue organisms at luminal border

Missing Image

Modified acid-fast oocyst in stool

Immunohistochemistry & special stains
Differential diagnosis
  • Cyclospora:
    • Oocysts are 8 μm, not 2 - 5 microns
  • Isospora:
    • Oocysts are 20 - 30 μm
  • Microsporidium:
    • An intracellular fungus that is not modified acid fast but may coinfect with Cryptosporidium

Cyst of retrorectal space
Definition / general
  • Also called tailgut cyst, retrorectal hamartoma
  • Retrorectal space is loose areolar tissue plane between fascia propria of rectum and presacral space
  • Rare but most common retrorectal cystic lesions in adults, occurring mostly in middle aged women (Radiographics 2001;21:575)
  • Often misdiagnosed clinically (J Am Coll Surg 2003;196:880)
  • Arises from remnants of embryonic postanal gut

Types:
  • Dermoid cyst: unilocular, lined by squamous epithelium and skin adnexae, no smooth muscle
  • Epidermoid cyst: unilocular, lined by squamous epithelium without adnexa
  • Rectal duplication cyst: unilocular, lined by colonic, gastric or respiratory epithelium with organized smooth muscle similar to muscularis propria
  • Cystic hamartoma: multilocular with squamous, transitional or glandular lining, disorganized smooth muscle, occasionally foreign body granulomatous inflammation
Diagrams / tables

Images hosted on other servers:

Retrorectal space

Case reports
Treatment
Gross description
  • Multilocular, variable solid areas
Gross images

Images hosted on other servers:

Retrorectal cyst

Oval shaped, gray-white appearance

Microscopic (histologic) images

Images hosted on other servers:

Squamous epithelium and focal columnar epithelium

Early malignant degeneration


Cytomegalovirus (CMV)
Definition / general
  • Cytomegalovirus (CMV) is a double stranded DNA virus and a member of human herpes virus family
  • Also known as herpes virus type 5
  • 3 patterns of CMV infection:
    • Latent infection
      • Most common, immunocompetent patients
    • Mononucleosis-like syndrome
      • Immunocompetent patients
    • Tissue invasive disease
      • Immunocompromised patients
  • Tissue invasive disease
    • Gastrointestinal tract is most commonly involved (30% of tissue invasive cases) (Virol J 2008;5:47)
Essential features
  • Double stranded DNA virus and a member of human herpes virus family
  • Tissue invasive disease, usually seen in immunocompromised patients
  • Most common sites of infection in gastrointestinal tract:
    • Colon
    • Esophagus
  • Symptoms:
    • Rectal bleeding (most common)
    • Diarrhea
    • Abdominal pain
    • Fever
    • Weight loss
  • Microscopy:
    • Cytomegalic cells with owl's eye intranuclear viral inclusions
    • CMV immunohistochemistry is the gold standard for diagnosis
Terminology
  • CMV infection
    • CMV antigens or antibodies in blood
  • CMV disease
    • Clinical symptoms and end organ damage
  • CMV colitis
    • Identification of characteristic intranuclear / intracytoplasmic inclusions on H&E sections
    • Identification of CMV specific antigens by immunohistochemistry (IHC) and clinical symptoms
ICD coding
  • ICD-10: B25.8 - other cytomegaloviral diseases
Epidemiology
Sites
  • Can involve any part of the gastrointestinal tract
  • Most common sites:
    • Colon
    • Esophagus
Pathophysiology
  • Spread by saliva, urine, respiratory droplets, sexual contact, breast milk and blood transfusions (Clin Microbiol Rev 1989;2:204, Nihon Rinsho 1998;56:179)
  • After initial infection, CMV resides latently in monocytes, fibroblasts, myeloid cells and endothelial cells
  • Tissue invasive disease in colon can lead to ulcerative changes, erosion into blood vessels (causing bloody diarrhea), inflammatory polyps, severe inflammation and vasculitis leading to ischemia and transmural necrosis
Etiology
  • Cytomegalovirus (CMV)
  • Most commonly in immunocompromised patients
    • History of AIDS, organ transplantation, hematologic malignancy, cancer therapy and corticosteroid therapy
  • Risk factors in immunocompetent patients:
    • Renal disease
    • Hemodialysis
    • Neurological disease
    • Rheumatic disease
    • Exposure to antibiotics
    • Antacids
    • Corticosteroid
  • Red blood cell transfusion within 1 month of diagnosis of colitis (Clin Infect Dis 2015;60:e20)
  • Severe ulcerative colitis (patients treated with high dose corticosteroids)
Clinical features
Diagnosis
  • Surgical resection specimen or biopsy: histopathologic diagnosis
  • Clinical symptoms, endoscopic findings, serologic testing, polymerase chain reaction (PCR) and culture
Laboratory
  • Histology:
    • Gold standard test
      • Immunohistochemistry (IHC) for CMV
        • Greater sensitivity than hematoxylin & eosin (H&E)
      • H&E can detect CMV infected cells
        • Cells larger than normal, containing intranuclear or intracytoplasmic inclusions
      • CMV infected cells can be confirmed by IHC staining
  • Serology:
    • Acute infection
      • CMV IgM antibodies
      • 4 times increase in titer of CMV IgG specific antibodies 2 - 4 weeks apart
    • CMV antigenemia
      • Predictor of clinical outcomes
      • Less sensitive for diagnosis of CMV colitis
    • Real time polymerase chain reaction (PCR) / CMV DNA quantification
      • Positive in only 50% of patients with biopsy proven CMV colitis / enteritis
    • CMV culture
      • High sensitivity and specificity for diagnosis of CMV colitis
      • Takes longer to obtain results (1 - 3 weeks)
      • May delay diagnosis and timely treatment (J Clin Microbiol 1993;31:2851)
Radiology description
  • Computed tomography:
    • Nonspecific findings
    • Bowel wall thickening
    • Mural edema
    • Pericolonic fat stranding
    • Free fluid, free air
    • Lymphadenopathy (Radiology 1985;155:585)
  • Endoscopic findings:
    • Easy bleeding, loss of vascular pattern, mucosal edema, erythema, mucinous exudate and wide mucosal defect
    • Mucosal defects, punched out ulcers (most common), longitudinal ulcers, irregular ulceration or cobblestone appearance (Emerg Radiol 2020;27:277)
Radiology images

Images hosted on other servers:

CT findings in a patient with CMV colitis

Prognostic factors
  • Excellent overall prognosis
  • Factors associated with poor prognosis and higher mortality (immunocompetent patients)
    • M > F
    • Age > 55 years
    • Patients requiring surgery
    • CMV colitis reactivation with ulcerative colitis, tends to have poorer prognosis
  • Timely diagnosis and treatment greatly improves clinical outcomes (StatPearls: Cytomegalovirus Colitis [Accessed 12 November 2021])
Case reports
Treatment
  • Intravenous (IV) ganciclovir (5 mg/kg twice daily)
  • After 3 - 5 days of IV ganciclovir, oral valganciclovir (900 mg, twice daily) (Clin Gastroenterol Hepatol 2015;13:949)
  • Foscarnet in ganciclovir resistant or tolerant cases
Clinical images

Contributed by Martha M. Yearsley, M.D.

Diffuse erythema

Pseudomembrane
formation

Irregular ulceration, loss of vascular pattern



Images hosted on other servers:
Missing Image

Diffuse inflammation

Missing Image

Ulcerating mass

Gross description
  • Nonspecific findings
  • Inflamed mucosa, hyperemia, mucosal sloughing
  • Punched out ulcers, aphthous ulcers, exudate
  • Pseudomembrane formation (Arch Pathol Lab Med 2016;140:854)
Gross images

Images hosted on other servers:
Missing Image Missing Image Missing Image

Ulceration and erythema

Missing Image

Ulceration secondary to CMV colitis


Missing Image

Terminal ileum

Missing Image

Cecum

Multiple, small punctate ulcers in the mucosa

Microscopic (histologic) description
  • Most commonly affected cells:
    • Endothelial cells
    • Mesenchymal cells
    • Macrophages
  • Larger (cytomegalic) cells:
    • Usually 25 - 35 micrometers
    • Typically 2 - 4 times larger than normal
  • Owl’s eye:
  • Thickened nuclear membrane
  • Coarse red intracytoplasmic granules (Int Med Case Rep J 2011;4:55)
  • Increased apoptotic bodies
  • Expansion of lamina propria by mixed inflammatory plasma cell rich infiltrate
  • Neutrophilic inflammation
  • Deep fissuring ulcers, cryptitis, crypt abscess, architectural distortion and pseudopolyp formation
  • Submucosal vasculitis and thrombosis of microvessels
Microscopic (histologic) images

Contributed by Martha M. Yearsley, M.D.

Enlarged cells in colonic lamina propria

Cytomegalic endothelial cells

Owl's eye and coarse red granules

Infected endothelial cells


Cytomegalic endothelial cells partially occluding vessel

Immunoreactivity with CMV immunostain

Scattered CMV positive cells

CMV positive endothelial cells



Contributed by @RaulSGonzalezMD on Twitter
Cytomegalovirus (CMV)

Cytomegalovirus (CMV)

Positive stains
  • Immunohistochemistry for CMV
Negative stains
Molecular / cytogenetics description
  • Real time DNA PCR amplification method:
    • Rapid results, high negative predictive value
    • Contradictory reports regarding sensitivity
  • CMV DNA load > 250 copies/mg tissue may predict resistance to steroid treatment in ulcerative colitis (Am J Gastroenterol 2011;106:2001)
Videos

CMV colitis in ulcerative colitis and immunocompromised states

Sample pathology report
  • Colon, random, biopsy:
    • CMV colitis
    • Immunohistochemical stain for CMV is positive

  • Colon, colectomy:
    • CMV colitis in the background of severely active ulcerative pancolitis
    • Immunohistochemical stain for CMV is positive
    • Negative for dysplasia
Differential diagnosis
Board review style question #1


A 71 year old man presents with abdominal pain and diarrhea. Colonoscopy showed diffuse mucosal erythema and irregular ulcerations. The patient undergoes biopsy of the lesion. What is the infected cell and organism causing the histopathologic findings?

  1. Endothelial cell, adenovirus
  2. Endothelial cell, cytomegalovirus
  3. Epithelial cell, adenovirus
  4. Epithelial cell, cytomegalovirus
Board review style answer #1
B. Endothelial cell, cytomegalovirus. The images show cytomegalic endothelial cells with inclusions. Adenovirus typically affects epithelial cells and shows amphophilic nuclear inclusions.

Comment Here

Reference: Cytomegalovirus (CMV)
Board review style question #2
What is the gold standard method for diagnosing CMV colitis?

  1. CMV culture
  2. CMV DNA polymerase chain reaction (PCR) amplification method
  3. CMV serology
  4. Immunohistochemistry for CMV
Board review style answer #2
D. Immunohistochemistry for CMV

Comment Here

Reference: Cytomegalovirus (CMV)

Diarrhea / dysentery
Definition / general
  • Normal intestines receive 9 liters of fluid per day (oral intake: 2 L, saliva: 1L, gastric juices: 2L, pancreatic juices: 2L, intestinal juices: 1L); most is reabsorbed in small intestine and colon
Terminology
Epidemiology
  • 12,000 deaths/year from dehydration in developing countries - 50% of all deaths before age 5
  • Affects 40% of U.S. population - #2 in attack rates in U.S. after common cold
Clinical features
Deranged motility:
  • Improper gut neuromuscular function causes decreased transit time
  • Due to surgical resection of gut, irritable bowel syndrome (neural dysfunction), hyperthyroidism, diabetic neuropathy, carcinoid syndrome
  • Decreased motility due to small bowel diverticula, blind loop, bacterial overgrowth
Exudative disease:
  • Purulent bloody stools, persist with fasting; caused by bacteria (Salmonella, Shigella, Campylobacter), Entamoeba histolytica, idiopathic inflammatory bowel disease, typhlitis (neutropenic colitis in immunosuppressed)
Malabsorption: Osmotic diarrhea:
  • Gut luminal solutes create high stool osmolality, abates with fasting
  • Stool osmolality > electrolyte concentration by 50 mOsm
  • Associated with lactase deficiency, lactulose therapy, gut lavage, antacids, primary bile acid malabsorption
Secretory diarrhea:
  • > 500 mL of fluid stool per day, isotonic with plasma, persists during fasting
  • Infectious (viral damage to epithelium): rotavirus, Norwalk virus, enteric adenoviruses, calicivirus, astrovirus
  • Infectious (enterotoxin): Vibrio cholera, E. coli, Bacillus cereus, Clostridium perfringens
  • Neoplastic: tumor production of peptides, villous adenoma in distal colon
  • Excessive laxatives
Diagrams / tables

Images hosted on other servers:

Chronic diarrhea

Microscopic (histologic) description
  • Patchy lesions with variable villus abnormality, rarely severe
  • Increased chronic and acute inflammatory infiltrate in epithelium and lamina propria
  • Small intestinal bacterial overgrowth: common cause of chronic diarrhea and malabsorption with either villous blunting or normal duodenal biopsy (Arch Pathol Lab Med 2010;134:264)

Diversion colitis
Definition / general
  • Inflammatory condition that affects colon distal to an ileostomy or colostomy
  • It can also occur in sigmoid segments used in colovaginoplasty procedures
Essential features
  • Clinicopathologic condition often arising in colonic segments distal to an ileostomy or colostomy site; it may also occur in colonic pouches, such as those following colovaginoplasty
  • Creation of an ostomy site alters the normal flow of fecal material through the bowel and thereby alters the microbiome of the gut resulting in alteration of the nutrients available for colonic enterocytes
  • Patients may present with nausea, vomiting, abdominal discomfort and bloody bowel movements
  • Histologic features may include lymphoid aggregates with germinal centers, muscularis mucosa hypertrophy, acute inflammation, architectural changes and ulceration
  • When it occurs in patients with underlying inflammatory bowel disease, histologic distinction can be challenging
Terminology
  • Defunctionalized bowel
ICD coding
  • ICD-10: K52.89 - other specified noninfective gastroenteritis and colitis
  • ICD-11: DB33.3 - diversion colitis
Epidemiology
  • Generally reported that no significant associations exist between diversion colitis and age of onset, gender, stoma type or mode of surgery (Int J Colorectal Dis 2017;32:1191)
  • Greatest association with diversion colitis is simply the surgical interruption of the normal flow of fecal material through the bowel
Sites
  • Colonic segments distal to colostomy or ileostomy, as well as segments of colon repurposed for use outside of the alimentary canal
Pathophysiology
  • Pathophysiology of diversion colitis is thought to be multifactorial
  • Disruption in the normal flow of fecal material may lead to changes in the microbiota
    • In a normal healthy colon, luminal bacteria produce short chain fatty acids (SCFAs), such as acetate, propionate and butyrate, with butyrate likely being the most important of the 3 (Nutr Rev 1989;47:257)
    • These SCFAs have been demonstrated to be important nutrients for colonic enterocytes
    • SCFAs may also play a role in vascular smooth muscle relaxation
    • Thus, deficiencies in SCFAs may lead to nutritional deficiencies for colonic enterocytes and increased arterial tone (World J Gastroenterol 2018;24:1734)
  • It has also been hypothesized that patients with diversion colitis may have an increased population of nitrate reducing bacteria and thus higher levels of nitric oxide (NO); while lower levels of nitric oxide appear to play a protective role for the bowel, it has been hypothesized that an excess of nitric oxide may be toxic to colonic tissue (World J Gastroenterol 2018;24:1734)
Etiology
  • Construction of ileostomy or colostomy are common causes for diversion colitis; for instance, this may occur following a Hartmann procedure (proctosigmoidectomy with creation of end colostomy) for management of colorectal cancer, severe diverticulitis or for treatment of refractory fecal incontinence (J Wound Ostomy Continence Nurs 2008;35:231)
  • Diversion colitis has also been reported in patients who underwent sigmoid neovaginal surgery for aplasia vaginae secondary to Mayer-Rokitansky-Küster-Hauser syndrome (Müllerian agenesis), as well as those pursuing gender reassignment surgery (Frontline Gastroenterol 2016;7:227)
Clinical features
  • Clinical features are highly variable but presentation may include
    • Abdominal pain
    • Tenesmus
    • Anal and rectal pain
    • Mucous discharge
    • Rectal bleeding
    • Some patients may present with endoscopic or histologic findings but may remain asymptomatic
  • When evaluating clinical features, it is important to contextualize those signs and symptoms within the appropriate setting; for instance, the above features should raise suspicion for diversion colitis in patients with ileostomies, with some research to suggest that symptoms can arise from months to years after ileostomy construction (World J Gastroenterol 2018;24:1734)
Diagnosis
  • Standardized diagnostic criteria have not been codified; thus, diagnosis must be suspected clinically and supported with imaging and histologic evaluation
Radiology description
  • Findings are largely nonspecific
  • Computed tomography (CT) and magnetic resonance imaging (MRI) scans will demonstrate findings consistent with intestinal discontinuity, such as an ileostomy, colostomy or a colonic pouch
  • Affected colonic segments may show fat stranding, wall thickening and fluid distension
  • Target or water halo sign may be present, which is suggestive of inflamed colonic mucosa and edematous submucosa
  • Reference: Insights Imaging 2019;10:41
Radiology images

Images hosted on other servers:
Postcolectomy CT and MRI

Postcolectomy CT and MRI

Prognostic factors
  • Protracted ostomy reliance generally portends a worse prognosis; additionally, inflammatory states, such as infection or underlying bowel disease, are thought to either precipitate, exacerbate or otherwise contribute to diversion colitis (BMJ Case Rep 2021;14:e243284)
Case reports
  • 12 year old girl with myelomeningocoele with fecal incontinence managed with colostomy construction and subsequent resection of distal defunctionalized bowel (Histopathology 1994;24:395)
  • 19 year old woman with Berdon syndome (megacystis microcolon intestinal hypoperistalsis syndrome [MMIHS]) diagnosed with diversion colitis after ileostomy construction (Dig Dis Sci 2009;54:2338)
  • 42 year old woman with androgen insensitivity syndrome with diversion colitis of sigmoid neovagina (Frontline Gastroenterol 2016;7:227)
  • 51 year old woman with Crohn's disease and microcarcinoids in association with diversion colitis symptoms presenting more than 17 years after subtotal colectomy and ileostomy (J Crohns Colitis 2008;2:246)
Treatment
  • Surgical management with reestablishment of intestinal continuity and stomal closure is generally regarded as the preferred and most successful treatment
  • Medical management may be pursued in place of surgical intervention in patients who are not surgical candidates, at risk for anastomotic leakage or those with severe active bowel inflammation (BMJ Case Rep 2021;14:e243284, World J Gastroenterol 2018;24:1734)
  • Medical management varies and may include
    • Short chain fatty acid (SCFA) enemas
    • 5-aminosalicylic acid
    • Corticosteroids
    • Irrigation with fibers
    • Leukocytapheresis
    • Autologous fecal transplant
    • Dextrose spray
    • TNFα inhibitors, such as infliximab
Microscopic (histologic) description
  • Lymphoid follicular hyperplasia with germinal centers
  • Muscularis mucosa hypertrophy, which is often patchy
  • Architectural changes with distortion of crypts, crypt branching and loss of crypts
  • Degenerative mucosal surface epithelium with cell exfoliation and pyknotic nuclei
  • Loss of goblet cells and mucin
  • Mucosal ulceration with granulation tissue
  • Varying degrees of inflammation, with features of both activity (neutrophil predominant) and chronicity (lymphoplasmacytic)
  • Focal edema
  • Pseudopolyps
  • Paneth cell metaplasia
  • References: Hum Pathol 2022;123:31, World J Gastroenterol 2018;24:1734, Histopathology 1991;19:55
Microscopic (histologic) images

Contributed by Catherine E. Hagen, M.D. and Dustin W. Parsons, M.D.
Prominent lymphoid follicles Prominent lymphoid follicles Prominent lymphoid follicles

Prominent lymphoid follicles

Crypt abscess

Crypt abscess

Lymphoid aggregate and architectural changes

Lymphoid aggregate and architectural changes

Ulceration

Ulceration

Sample pathology report
  • Rectum, stump, biopsy:
    • Diversion colitis (see comment)
    • Comment: Colonic mucosa with prominent lymphoid aggregates and mild architectural changes compatible with diversion colitis.
Differential diagnosis
  • Inflammatory bowel disease (IBD) (ulcerative colitis or Crohn's disease):
    • Histologic distinction can be challenging and diversion colitis can co-occur with IBD
    • More severe inflammatory changes may indicate superimposed IBD (Hum Pathol 2022;123:31)
  • Segmental colitis associated with diverticulosis (SCAD):
    • Occurs in segment of colon with diverticula and rectum is typically spared
  • Radiation colitis:
    • Occurs in patients with history of radiation therapy for malignancy (e.g., of prostate, colon, bladder)
    • Acute radiation may demonstrate a number of histologic findings, including nuclear atypia, reduction in mitotic counts, eosinophilic inflammation (including eosinophilic cryptitis and crypt abscess formation), goblet cell derangements and fibroblastic proliferation
    • Chronic exposure may result in Paneth cell metaplasia, architectural distortion, fibrosis of lamina propria, chronic inflammation, hyalinized vasculature, vascular ectasia and thrombosis
Board review style question #1
A 5 month old infant is seen in the emergency department with bloody bowel movements. She appears uncomfortable and begins crying with abdominal palpation. She is afebrile and her white blood cell count is within normal limits. Medical history is significant for premature birth complicated by necrotizing enterocolitis with bowel resection and ileostomy construction. A biopsy of the colonic stump is likely to demonstrate which of the following findings?

  1. Crypt abscesses, Paneth cell metaplasia, marked architectural distortion, basal lymphoplasmacytosis
  2. Endothelial cells demonstrating owl eye intranuclear inclusions with associated clear halo
  3. Intraepithelial lymphocytosis
  4. Prominent lymphoid hyperplasia, muscularis mucosa hypertrophy and crypt abscesses
Board review style answer #1
D. Prominent lymphoid hyperplasia, muscularis mucosa hypertrophy and crypt abscesses. These findings are characteristic of diversion colitis. The patient’s history of bowel resection with ileostomy should raise suspicion for diversion colitis. Answer A is incorrect because these findings are more demonstrative of ulcerative colitis. It can be difficult to separate out the 2 conditions and reports in the literature exist of concomitant ulcerative colitis and diversion colitis. With overlapping symptoms and the potential for finding crypt abscesses in both conditions, it can certainly pose a diagnostic challenge for pathologists trying to differentiate the 2 conditions on biopsy. The patient’s history of ileostomy construction should be a clue that diversion colitis may be favored here. Without a pre-existing diagnosis of inflammatory bowel disease (IBD), this is unlikely. Answer B is incorrect because the characteristic owl eye inclusions are consistent with cytomegalovirus (CMV) colitis. This consideration of this condition may be greater in patients with a history of immunosuppresion; for instance, in patients with human immunodeficiency virus (HIV) or those receiving immunosuppressive therapy following solid organ transplant. Answer C is incorrect because intraepithelial lymphocytosis is characteristic of lymphocytic colitis, which typically presents with watery, nonbloody diarrhea. Microscopic colitis is uncommon in the pediatric population.

Comment Here

Reference: Diversion colitis
Board review style question #2

A 72 year old man with a history of colon cancer undergoes radiation therapy, followed by partial colon resection with construction of an ileostomy. He denies any history of inflammatory bowel disease. The image shown above demonstrates histologic findings of the resected portion of the colon. These findings are most consistent with which clinical presentation?

  1. Diversion colitis
  2. Infectious agent
  3. Radiation colitis
  4. Ulcerative colitis
Board review style answer #2
A. Diversion colitis. The construction of an ileostomy should always raise suspicion for the possibility of diversion colitis. The prominent lymphoid aggregates are likely the most characteristic finding associated with diversion colitis. Other features appreciated here include patchy hypertrophy of muscularis mucosa, ulceration and loss of colonic crypts. Answer B is incorrect because the clinical vignette does not suggest any fevers or bowel symptoms consistent with an infectious etiology. Infectious colitis typically shows a histologic pattern of acute self limited colitis without architectural distortion. Answer C is incorrect because the presence of lymphoid aggregates should favor diversion colitis over radiation colitis. Radiation colitis is typically pauci-inflammatory with evidence of lamina propria fibrosis and hyalinized ectatic vessels. Answer D is incorrect because while diversion colitis can have overlapping histologic features with inflammatory bowel disease, given that the patient denies any history of underlying inflammatory bowel disease, this is unlikely to be a manifestation of ulcerative colitis.

Comment Here

Reference: Diversion colitis

Diverticular colitis
Definition / general
Pathophysiology
  • Due to mucosal prolapse, fecal stasis and relative mucosal ischaemia; also subserosal peridiverticulitis and suppuration causing mass effect (Pathology 2002;34:568)
Treatment
Microscopic (histologic) description
  • Transmural inflammation with expansion of lamina propria, cryptitis and crypt abscesses, lymphoid aggregates and distorted crypt architecture
Differential diagnosis

Diverticulosis
Definition / general
  • Anatomic change in the colon characterized by outpouchings of mucosa and submucosa through the muscularis (Clin Geriatr Med 2021;37:141)
  • Diverticulosis: presence of diverticula, regardless of symptoms
  • Diverticular disease: clinically relevant symptomatic diverticulosis
Essential features
  • Anatomic change in the colon characterized by outpouchings of mucosa and submucosa through the muscularis
  • Diverticulosis can be asymptomatic or symptomatic
  • Sigmoid colon / left colon, most common site (90%)
  • Mucosa normal to markedly abnormal
  • Expansion of lamina propria by lymphoplasmacytic infiltrate, lymphoglandular complexes, mucin depletion and focal Paneth cell metaplasia may be seen
  • No rationale for treatment of asymptomatic colonic diverticulosis
Terminology
  • Most colonic diverticula are false diverticula
    • Mucosa and submucosa herniate through defect or weakness in muscularis layer and is covered by serosa
  • True diverticula are uncommon
  • Diverticulosis can be asymptomatic or symptomatic
  • Symptomatic diverticulosis:
    • Symptomatic uncomplicated diverticular disease (SUDD)
    • Diverticulitis (acute, chronic or recurrent)
    • Diverticular hemorrhage
    • Segmental colitis associated with diverticulosis (SCAD)
  • Diverticulitis: uncomplicated or complicated
  • Complicated diverticulitis includes abscess, fistula, perforation, obstruction (Clin Geriatr Med 2021;37:141)
ICD coding
  • ICD-10: K57.90 - diverticulosis of intestine, part unspecified, without perforation or abscess without bleeding
Epidemiology
  • Highest incidence in Western world
  • Prevalence increases with age
    • > 75% people above 75 years of age
    • < 50 years of age: more common in males
    • 50 - 70 years of age: slightly more common in females
    • > 70 years of age: significantly increased incidence in females (Expert Rev Gastroenterol Hepatol 2018;12:683)
  • Left sided diverticula: more common in Western world
  • Right sided diverticula: more common in Asia
  • Diverticular bleeding 5 - 15%
    • Usual source is right sided diverticula
    • Possibly due to thinner wall on right side of colon, widened necks and domes of right sided diverticula
  • Diverticulitis: in 4 - 15% of patients with diverticula
Sites
Pathophysiology
  • Occurs in weaker portions of the colonic wall where vasa recta infiltrate the circular muscle layer
  • Abnormal colonic motility → exaggerated segmental contractions → increased intraluminal pressure and separation of colonic lumen into chambers (Expert Rev Gastroenterol Hepatol 2018;12:683)
  • Sigmoid colon, segment with smallest diameter and largest intraluminal pressure
  • In diverticula, vasa recta are separated from intestinal lumen by a layer of mucosa alone
    • Exposed to greater amount of injury and can lead to bleeding
Etiology
  • Host of complex factors are implicated
    • Altered colonic motility
    • Visceral hypersensitivity
    • Inflammation
    • Genetic susceptibility
    • Diet, tobacco use
    • Medication
    • Gut microbiome imbalances (Clin Geriatr Med 2021;37:141)
  • Connective tissue disease, Marfan syndrome, Ehlers-Danlos and autosomal dominant polycystic kidney disease
    • Can cause structural changes in intestinal walls
Diagrams / tables

Images hosted on other servers:

Physiological activity

Clinical features
Diagnosis
Laboratory
  • Fecal calprotectin measurement
    • Calprotectin is a calcium and zinc binding protein; can be considered neutrophil specific
    • Marker of gastrointestinal inflammation
    • Can help in distinguishing IBS from diverticular disease (Expert Rev Gastroenterol Hepatol 2018;12:791)
    • Increased white blood cell (WBC), erythrocyte sedimentation rate (ESR), C reactive protein (CRP) in diverticulitis
Radiology description
  • Barium enema: barium filled outpouchings; can look similar to polyps
  • Ultrasound: gas filled outpouchings
  • CT: diverticula outlined by gas
  • Intramural diverticula seen as a tiny focus of gas or contrast within the colonic wall (J Clin Gastroenterol 2004;38:S11)
  • CT allows grading of severity and detection of complications (Br J Radiol 2020;93:20200670)
Radiology images

Contributed by Bindu Challa, M.D. and Martha M. Yearsley, M.D.
CT of abdomen and pelvis CT of abdomen and pelvis

CT of abdomen and pelvis

Prognostic factors
  • Mortality rate in uncomplicated diverticulitis is negligible (StatPearls: Acute Diverticulitis [Accessed 21 April 2022])
  • Complicated diverticulitis requiring surgery may lead to death in ~5% of patients
  • Perforation of the bowel with resulting peritonitis, risk of death increases to 20%
Case reports
  • 15 year old boy with dysmorphic features and learning disabilities presents with acute onset abdominal pain, nausea and vomiting (ACG Case Rep J 2019;6:1)
  • 50 year old man with longstanding history of ulcerative colitis and severe diverticulosis on colonoscopy (Rev Esp Enferm Dig 2021;113:550)
  • 68 year old man with lower gastrointestinal bleeding and arteriovenous malformations within jejunal diverticulosis (BMC Surg 2019;19:70)
  • 83 year old woman with history of chronic abdominal pain is found to have partial midgut volvulus (Am J Case Rep 2021;22:e933180)
  • 88 year old man presents with hematochezia and perianal pain (Cureus 2021;13:e14900)
Treatment
Clinical images

Contributed by Bindu Challa, M.D. and Martha M. Yearsley, M.D.
Colonoscopy Colonoscopy

Colonoscopy

Colonoscopy

Colonoscopy

Gross description
  • Specimen should be fixed and opened longitudinally for easy sampling of diverticula
  • Usually, multiple flask shaped invaginations of colonic wall
  • Wall thickening may be present (Surg Endosc 2011;25:2586)
  • Narrowing of lumen, shortening of tenia and marked thickening of circular muscle layer may be present (J Clin Gastroenterol 2006;40:S108)
  • If it extends deeply, perforation and serosal adhesions may be seen
Gross images

Images hosted on other servers:

Multiple diverticula

Multiple diverticula

Multiple diverticula


With bowel wall stenosis

Blue-gray diverticula

Perforated single true diverticulum

False diverticulum

Microscopic (histologic) description
  • Mucosa may be normal to markedly abnormal
  • Expansion of lamina propria by lymphoplasmacytic infiltrate (more prominent in basal half)
  • Lymphoglandular complexes, mucin depletion, focal Paneth cell metaplasia (in chronic cases) may be seen (J Clin Gastroenterol 2004;38:S11)
  • Muscularis mucosa extends towards surface between elongated crypts
  • Shortening of the affected bowel and hypertrophy of circular muscle layer (myochosis) leads to exaggerated mucosal folds (Am J Surg Pathol 1991;15:871)
  • Some cases can show lamina propria fibrosis, crypt elongation on the tips of prominent mucosal folds, like mucosal prolapse syndrome in rectum and anus (J Clin Gastroenterol 2008;42:1137)
  • Hemosiderin deposition in submucosa may be seen (StatPearls: Diverticulosis [Accessed 21 April 2022])
  • Can mimic inflammatory bowel disease
  • Hyperplasia of lymphoid aggregates is one of the earliest signs of diverticulitis
  • Cryptitis, crypt abscesses, peridiverticular abscess and fistulas may be superimposed on this background in acute diverticulitis
  • Tracking abscesses can spread longitudinally or circumferentially and can cause diverticular colitis
  • Persistent localized inflammation can lead to phlegmon which is a thickened, firm segment of bowel wall that can lead to strictures and acute or subacute large bowel obstruction (Best Pract Res Clin Gastroenterol 2002;16:543)
Microscopic (histologic) images

Contributed by Bindu Challa, M.D. and Martha M. Yearsley, M.D.
Diverticula

Diverticula

Multiple diverticula

Multiple diverticula

Diverticular abscess

Diverticular abscess

Enterocutaneous fistula

Enterocutaneous fistula


Chronic inflammation in diverticular wall

Chronic inflammation in diverticular wall

Diverticular disease associated chronic colitis

Diverticular disease associated chronic colitis

Lymphoid aggregates in diverticular wall

Lymphoid aggregates in diverticular wall

Diverticulitis with complications

Diverticulitis with complications

Virtual slides

Images hosted on other servers:

Diverticulosis with diverticulitis

Videos

Histopathology of diverticular disease

Whole slide image of case of diverticulosis with diverticulitis

Sample pathology report
  • Colon, sigmoid, segmental resection:
    • Diverticulosis
    • Two lymph nodes, negative for tumor

  • Colon, sigmoid, partial colectomy:
    • Segment of colon with diverticulosis, diverticulitis and abscess formation

  • Colon, total colectomy:
    • Colonic diverticular disease with perforated diverticulum
Differential diagnosis
Board review style question #1

What is the most common site of occurrence for the condition shown in the image?

  1. Appendix
  2. Jejunum
  3. Rectum
  4. Sigmoid colon
Board review style answer #1
D. Sigmoid colon. Diverticulosis occurs most commonly in sigmoid colon in 90% of cases.

Comment Here

Reference: Diverticulosis
Board review style question #2
What is the laboratory test that can help distinguish irritable bowel syndrome from diverticular disease?

  1. Aspartate transaminase
  2. Fecal calprotectin
  3. Fecal lactoferrin
  4. Gamma glutamyl transferase
Board review style answer #2
B. Fecal calprotectin is a marker of gastrointestinal inflammation. Elevated fecal calprotectin favors diverticular disease and helps in distinguishing it from functional gastrointestinal disorders like irritable bowel syndrome.

Comment Here

Reference: Diverticulosis

Drug induced colitis - overview (pending)
Table of Contents
Definition / general
Definition / general
[Pending]

Duplication
Definition / general
  • GI duplications are uncommon congenital abnormalities, 80% recognised before age 2; colorectal duplications occur in 7% of cases
  • Rare in adults, may present with rectal bleeding (J Clin Pathol 2010;63:1080, World J Gastroenterol 2005;11:5072)
  • Partial or complete doubling of a variable length of bowel
  • Saccular to long, cystic structures
  • Often present in mesentery of normal bowel without communication with lumen
  • Associated with complex GU abnormalities
Clinical features
  • Cystic (> 80%) or tubular; often asymptomatic and undiagnosed
  • Symptomatic colonic duplication is rare in adults; present with abdominal pain and intestinal obstruction; also acute abdomen or acute bleeding, necessitating emergency surgery (BMC Surg 2010;10:19)
Case reports
Treatment
Clinical images

Images hosted on other servers:

Fecaloma removed

Sigmoid and colonic duplication

Final attachment of blind end

Colonic pouch and Y shaped duplication

Dilated colon and Y shaped duplication

Gross images

Contributed by Celso Rubens Vieira e Silva, M.D.

Cystic congenital duplication



Images hosted on other servers:

T shaped tubular colonic duplication

Duplicated colon

Various images

Microscopic (histologic) images

Contributed by Celso Rubens Vieira e Silva, M.D.

Cystic congenital duplication



Images hosted on other servers:

Duplicated colon and appendix

Differential diagnosis
  • Enteric cysts: less organized smooth muscle, no nerve plexus
Additional references

Dysplasia
Definition / general
  • Dysplasia of colonic epithelium identified in setting of colonic inflammatory bowel disease (IBD), usually in colonic biopsies from surveillance colonoscopies
  • Precursor of invasive carcinoma
  • Can be endoscopically visible or invisible
  • Aim of surveillance is to reduce morbidity and mortality from colorectal carcinoma by identifying dysplasia (or early invasive carcinoma)
Essential features
  • Precursor of invasive carcinoma in patients with inflammatory bowel disease
  • Dysplasia status should be reported for biopsies from surveillance colonoscopies of patients with inflammatory bowel disease:
    • Negative for dysplasia
    • Indefinite for dysplasia
    • Low grade dysplasia
    • High grade dysplasia
  • Management of dysplasia identified on colonoscopies is dependent on endoscopic appearance (visible [polypoid or nonpolypoid] or invisible) and resectability
Terminology
  • Previously used term dysplasia associated lesion or mass (DALM) and related terms (e.g. adenoma-like or nonadenoma-like DALM) should be avoided due to the historical connotation of a high risk of malignancy (Mod Pathol 2018;31:1180)
  • Endoscopic appearance based on standardized terminology as described by the Surveillance for Colorectal Endoscopic Neoplasia Detection and Management in Inflammatory Bowel Disease Patients: International Consensus Recommendations (SCENIC) is recommended (Gastroenterology 2015;148:639)
ICD coding
  • ICD-10:
    • D12.0 - benign neoplasm of cecum
    • D12.2 - benign neoplasm of ascending colon
    • D12.3 - benign neoplasm of transverse colon
    • D12.4 - benign neoplasm of descending colon
    • D12.5 - benign neoplasm of sigmoid colon
    • D12.6 - benign neoplasm of colon, unspecified
    • D12.7 - benign neoplasm of rectosigmoid junction
    • D12.8 - benign neoplasm of rectum
Sites
  • Colon and rectum in areas of colitis
Pathophysiology
  • Cytotoxic effect of inflammation leads to repeated cycles of epithelial wounding and repair
  • Postulated selective pressure for mutant cells that can survive the inflammatory insult and rapidly repopulate the damaged mucosa
  • Epigenetic and genetic changes accumulate in morphologically nondysplastic colonic mucosa long before development of neoplasia
  • TP53 mutations and aneuploidy are early events in colitis associated neoplasia and have been identified in nondysplastic colonic mucosa (Carcinogenesis 2018;39:11, Nat Rev Gastroenterol Hepatol 2017;14:218, Gastroenterology 2009;136:542)
  • Accumulation of additional mutations, chromosomal abnormalities and epigenetic changes leads to dysplasia and invasive carcinoma
  • In comparison, sporadic colorectal neoplasia tends to have APC mutations early and TP53 mutations later in tumorigenesis
Etiology
Clinical features
Diagnosis
  • Detected with colonoscopies, typically as part of surveillance
  • Targeted biopsies, endoscopic resections or polypectomies of visible lesions
  • Random biopsies may be taken to detect endoscopically invisible dysplasia
Prognostic factors
Case reports
Treatment
  • Depends on endoscopic appearance and resectability
  • Polypoid dysplasia:
    • Endoscopic polypectomy and followup with continued surveillance
  • Nonpolypoid dysplasia:
    • Endoscopically resectable:
      • Endoscopic mucosal resection or submucosal dissection and followup with continued surveillance
    • Not amenable to endoscopic resection:
      • Cannot exclude underlying invasive carcinoma
      • Should be surgically resected
  • Invisible dysplasia:
  • Indefinite for dysplasia:
Clinical images

Images hosted on other servers:

Polypoid dysplasia identified on chromoendoscopy

Nonpolypoid dysplasia

Gross description
  • Can be macroscopically (endoscopically) unapparent
  • Can be polypoid (pedunculated or sessile) or nonpolypoid (superficially elevated, flat or depressed)
Gross images

Contributed by Adela Cimic, M.D.

Descending colon lesion



Images hosted on other servers:

Dysplasia in a Crohn's colitis pseudopolyp

Microscopic (histologic) description
  • Correlate with endoscopic appearance, as this influences management
    • Visible dysplasia (polypoid or nonpolypoid)
    • Invisible dysplasia
  • Low grade dysplasia:
    • Preserved nuclear polarity
    • Pseudostratified, crowded, elongated and hyperchromatic nuclei
    • Lack of surface maturation, i.e. abnormalities persist to surface
  • High grade dysplasia:
    • May show complex architecture, such as cribriform glands
    • Loss of nuclear polarity
    • Nuclear pleomorphism, vesicular nuclei and prominent nucleoli
  • Indefinite for dysplasia:
    • Reserved for cases when distinction between dysplasia and reactive epithelial atypia cannot be made
    • Mucosal erosion or ulceration (precluding assessment of surface maturation) or prominent inflammation may be sources of difficulty
  • Features distinguishing IBD associated polypoid dysplasia and sporadic adenomas have been described but are not reliable and have no therapeutic implications
    • Include lamina propria inflammation, mixture of benign and dysplastic glands at the surface and dysplasia in stalk of polyp (Hum Pathol 1983;14:931, Am J Surg Pathol 1998;22:275)
    • Distinction is also not clinically significant as endoscopic polypectomy suffices either way
  • More recently, Gui et al. reported features that can be seen in IBD associated nonpolypoid dysplasia but only rarely seen in polypoid dysplasia or sporadic adenomas (Hum Pathol 2020;100:24):
    • Dysplastic epithelium associated with a background of inflammatory polyp or granulation tissue, micropapillary or hobnailing surface epithelial cells, nuclear pleomorphism and disarray, and microvesicular or bubbling cytoplasm
  • IBD associated nonconventional dysplasia (i.e. not intestinal type) has been described and is relatively common in IBD patients:
    • 45% of IBD patients with colorectal cancer and of those, 46% did not have associated conventional dysplasia as reported by Choi et al. (Mod Pathol 2020;33:933)
    • 33% of IBD patients with dysplasia as reported by Lee et al. (Histopathology 2021;78:814)
    • 38% of carcinoma related lesions and 41% of nonpolypoid dysplasia compared to 7% of polypoid dysplasia as reported by Gui et al. (Hum Pathol 2020;100:24)
  • Nonconventional dysplasia subtypes (Histopathology 2021;78:814, Hum Pathol 2020;100:24, Mod Pathol 2020;33:933):
    • Hypermucinous (mucinous) dysplasia:
      • Tubulovillous or villous architecture with prominent mucinous differentiation
      • Usually mild nuclear atypia, especially towards the surface
    • Dysplasia with increased Paneth cell differentiation:
      • Tubular architecture with elongated, hyperchromatic nuclei
      • Foci of increased Paneth cell differentiation, defined by Choi et al. as involving at least 2 contiguous crypts in 2 different foci and increased relative to background mucosa (Mod Pathol 2020;33:933)
    • Goblet cell deficient (eosinophilic) dysplasia:
      • Goblet cells are nearly or completely absent
      • Eosinophilic cytoplasm
    • Serrated dysplasia, which includes:
      • Traditional serrated adenoma (TSA)-like
      • Sessile serrated lesion (SSL)-like
      • Serrated lesion not otherwise specified: does not meet criteria for SSL-like or TSA-like dysplasia
    • Dysplasia with terminal epithelial differentiation (differentiated dysplasia) (Hum Pathol 2020;100:24, J Clin Pathol 2020;73:391)
      • Nuclei are enlarged (can be mildly so), round to oval, slightly irregular, hyperchromatic and mostly nonstratified
      • Nucleoli can be present, inconspicuous and occasionally prominent
      • Some authors consider at least a subset of differentiated dysplasia as crypt cell dysplasia (see next heading) (Histopathology 2021;78:814, Hum Pathol 2020;100:24, J Clin Pathol 2020;73:391)
    • Crypt cell dysplasia:
      • Recently proposed to represent a high risk dysplastic lesion but the diagnosis based on histologic features alone is subject to significant interobserver variability and high rates of nondysplastic diagnoses, particularly indefinite for dysplasia (Histopathology 2021;78:814, Histopathology 2019;75:578)
      • Atypia is limited to crypt base and does not involve surface epithelium
      • Nuclei are round or oval, mildly enlarged, hyperchromatic and show slightly irregular contours
  • Dysplasia can show mixed histologic patterns (including mixed nonconventional and conventional dysplasia)
Microscopic (histologic) images

Contributed by Kenrry Chiu, M.D., C.M.

Low grade dysplasia

High grade dysplasia



Contributed by Elaine Alt, M.D.

Low grade dysplasia

Low grade dysplasia - pancolitis

Virtual slides

Images hosted on other servers:

High grade dysplasia in ulcerative colitis

Low grade dysplasia in ulcerative colitis

Cytology images

Images hosted on other servers:

High grade dysplasia

Positive stains
Negative stains
Molecular / cytogenetics description
Molecular / cytogenetics images

Images hosted on other servers:

Model of carcinogenesis in ulcerative colitis

Sample pathology report
  • Presence or absence of dysplasia should be reported for biopsies from surveillance colonoscopies as follows (Hum Pathol 1983;14:931, Gut 2000;47:251):
    • Negative for dysplasia
    • Indefinite for dysplasia
    • Low grade dysplasia
    • High grade dysplasia
  • Polypoid lesion within colitic mucosa, endoscopically resected:
    • Colon (sigmoid), polypectomy:
      • Polypoid low grade dysplasia
  • Nonpolypoid lesion, endoscopically resected piecemeal:
    • Colon (ascending), biopsy:
      • High grade dysplasia
      • Completeness of excision cannot be determined (specimen fragmented)
  • Invisible dysplasia identified on random biopsy:
    • Rectum, biopsy:
      • Low grade dysplasia
  • Note: a comment noting that endoscopically resectable lesions that are completely removed can be managed with continued surveillance rather than colectomy may be helpful
Differential diagnosis
  • Reactive epithelial atypia:
    • May show mild nuclear stratification and crowding, nuclear enlargement and hyperchromasia and nucleoli
    • Surface maturation is present
    • Nuclear polarity is preserved
    • Inflammatory background may be present
    • Nonconventional dysplasia, e.g. increased Paneth cell differentiation or goblet cell deficient variant, can mimic reactive, metaplastic changes (Mod Pathol 2020;33:933)
      • Features such as lack of surface maturation and cytoarchitectural atypia help establish a diagnosis of dysplasia
  • Sporadic adenoma:
    • Occurs in colonic segments outside colitic mucosa
    • Not related to IBD
    • Otherwise similar epithelial changes as those in (conventional) IBD associated dysplasia
Board review style question #1
Which of the following statements regarding surveillance colonoscopies for patients with inflammatory bowel disease is true?

  1. Complete endoscopic resection of colitis associated polypoid high grade dysplasia can be followed up with continued surveillance
  2. Most colorectal dysplasia identified in surveillance colonoscopies is endoscopically invisible
  3. Nonpolypoid dysplasia is never endoscopically resectable and should therefore prompt colectomy
  4. Polyps with dysplasia present in mucosa outside of mucosa affected by chronic colitis are not considered sporadic adenomas
Board review style answer #1
A. Complete endoscopic resection of colitis associated polypoid high grade dysplasia can be followed up with continued surveillance.

Comment Here

Reference: Dysplasia
Board review style question #2

Which of the following statements regarding dysplasia associated with inflammatory bowel disease is true?

  1. Early genetic changes present in colitis associated neoplasia are similar to those in sporadic neoplasia
  2. Indefinite for dysplasia may be diagnosed if the distinction between reactive atypia and dysplasia cannot be established
  3. Loss of nuclear polarity is a feature of low grade dysplasia
  4. Only patients with ulcerative colitis and not Crohn's disease have an increased risk of colorectal carcinoma
Board review style answer #2
B. Indefinite for dysplasia may be diagnosed if the distinction between reactive atypia and dysplasia cannot be established

Comment Here

Reference: Dysplasia

Ehlers Danlos syndrome
Definition / general
  • Inherited heterogenous group of connective tissue disorders characterized by abnormal collagen synthesis (eMedicine, Wikipedia)
  • Six variants of EDS have been described, based on clinical and molecular features (Am J Med Genet 1998;77:31)
Clinical features
  • Clinical symptoms vary by clinical variant, and include skin hyperextensibility and fragility / poor healing; joint hyperextensibility with a propensity to dislocation; eye symptoms with corneal rupture and retinal detachment; kyphoscoliosis and rupture of the colon and large arteries

Vascular type (type IV):
  • Autosomal dominant; caused by a mutation in COL3A1 gene (J Nippon Med Sch 2008;75:254)
  • Three distinct types of mutations, which affect either the rate of synthesis, the secretion of type III procollagen or result in structurally abnormal type III collagen
  • Associated with a reduced median survival of only 48 years
  • Neonates have increased incidence of clubfoot and hip dislocation, and rarely have subarachnoid hemorrhage (Am J Clin Pathol 1990;93;579)
  • In children, inguinal hernia, pneumothorax and recurrent joint dislocation or subluxation are common; skin is translucent with visible veins and is easily bruised; distinctive facial features are often present, including protruding eyes, a thin nose and lips, sunken cheeks and a small chin
  • Blood vessels and intestines are typically rich in type III collagen, and adult patients often present with vascular rupture/dissection or gastrointestinal perforation
  • Uterine rupture may occur during pregnancy, particularly at delivery
Case reports
Treatment
  • No specific treatment
  • Preventative measures are recommended; some patients may improve with high dose Vitamin C
Microscopic (histologic) description
  • Often colonic perforation, with mucosal ulceration and thin bowel wall
  • May have segmental absence of muscularis propria, with replacement by cellular, reactive fibroblastic proliferation
Microscopic (histologic) images

Case #163


9 year old boy with Type IV disease

Additional references

Elastofibromatous change
Definition / general
  • Increase in elastin fibers in colonic submucosa
Essential features
  • Uncommon collection of amorphous elastotic material in colon
  • Seemingly no clinical significance but can mimic amyloid
Terminology
  • Also called elastosis
  • Called elastofibroma if forms a polyp
Sites
  • Occurs throughout gastrointestinal tract
  • In colon, favors left side; can present in rectum
Clinical features
  • Patients typically older than 45
  • No sex predilection
  • May cause nonspecific gastrointestinal symptoms or be asymptomatic
Case reports
Gross description
  • May appear as a polyp or as a linear white lesion
Microscopic (histologic) description
  • Eosinophilic to gray amorphous material, sometimes with fibrous tissue, involving submocusa or muscularis mucosae
  • May be centered around blood vessels (Am J Clin Pathol 2004;122:232)
Microscopic (histologic) images

Contributed by Raul S. Gonzalez, M.D.

Submucosal elastofibromatous change mimicking amyloid

Elastin stain

Positive stains
Negative stains
Sample pathology report
  • Descending colon, polypectomy:
    • Colon with elastofibromatous change (see comment)
    • Comment: The polyp consists of submucosal amorphous material that is negative on a Congo red special stain and positive on an elastin special stain.
Differential diagnosis
Board review style question #1

    The material seen in the photo was seen in a colon polyp. It stained positive with an elastin stain. What is the best diagnosis?

  1. Amyloid
  2. Elastofibromatous change
  3. Lifting agent material
  4. Pulse granuloma
Board review style answer #1
B. Elastofibromatous change

Comment Here

Reference: Elastofibromatous change

Endometriosis
Definition / general
  • Endometriosis arising within colon
Essential features
  • Similar to endometriosis elsewhere: can cause clinical symptoms; histology shows endometrial glands, endometrial stroma and hemosiderin
  • Malignant transformation is rare
Epidemiology
  • Up to 33% with endometriosis have involvement of large or small intestine
Sites
  • Usually rectosigmoid or rectovaginal septum (Med Sci Monit 2000;6:787)
  • Can involve any portion of colonic wall, from mucosa to serosa
Clinical features
  • Typically incidental but can cause pain, obstruction or diarrhea
  • Rarely associated with neoplasms or precancerous changes (Am J Surg Pathol 2000;24:513)
Case reports
Treatment
Gross description
  • Serosal and subserosal nodules < 5 cm; smooth muscle hypertrophy may cause obstruction
  • Gray cut surface with minute areas of hemorrhage; can cause adhesions if subserosal
Gross images

Contributed by @Andrew_Fltv on Twitter
Endometriosis Endometriosis

Endometriosis

Microscopic (histologic) description
  • Endometrial glands and stroma plus hemosiderin in deeper layers of colon; usually surrounded by smooth muscle
  • Epithelium may have inflammation and ulcers, simulating inflammatory bowel disease or solitary rectal ulcer syndrome
  • Often infiltrates along nerves of bowel wall (Hum Reprod 2004;19:996)
  • Mucosa usually normal; bowel wall may be fibrotic; cilia help distinguish endometrial glands from colonic glands
Microscopic (histologic) images

Contributed by Raul S. Gonzalez, M.D. and @Andrew_Fltv on Twitter
Colonic endometriosis Colonic endometriosis Colonic endometriosis

Colonic endometriosis

Endometriosis

Endometriosis



Images hosted on other servers:

Within colonic mucosa

Present in wall of colon

Infiltration along nerves

Positive stains
Negative stains
Differential diagnosis
Sample pathology report
  • Ascending colon, resection:
    • Colon with multiple mural and serosal foci of endometriosis
    • Negative for malignancy.
    • Margins of resection unremarkable.
    • Two benign lymph nodes.
Board review style question #1

Which of the following is true about colonic endometriosis?

  1. It is most common in the ascending colon
  2. It is typically incidental
  3. It never contains hemosiderin histologically
  4. It often contains endometrioid adenocarcinoma
Board review style answer #1
B. It is typically incidental

Comment Here

Reference: Endometriosis

Eosinophilic gastroenterocolitis
Definition / general
  • Eosinophilic gastrointestinal disorders (EGIDs) consist of eosinophilic esophagitis, eosinophilic gastritis, eosinophilic gastroenteritis, eosinophilic enteritis and eosinophilic colitis (J Allergy Clin Immunol 2004;113:11)
    • However, eosinophilic esophagitis differs from eosinophilic gastroenterocolitis (EGEC) in all aspects and should be considered a different entity
  • Eosinophilic gastroenterocolitis is an inflammatory disorder characterized by prominent eosinophilic infiltration of the gastrointestinal tract (stomach, duodenum, small intestine or large intestine) with no known causes of tissue eosinophilia
Essential features
  • Inflammatory process characterized by the abnormal eosinophilic infiltration of different segments of the gastrointestinal tract in the absence of a known cause (drug reaction, parasitic infection, malignancy, etc.)
  • Can occur throughout the gastrointestinal tract; most reported in the stomach, followed by the small intestine and colon
  • Its diagnosis requires the presence of:
    • Gastrointestinal symptoms
    • Demonstration of gastrointestinal eosinophilia by biopsy
    • Exclusion of other known causes of tissue eosinophilia
Terminology
  • Allergic gastroenteritis
  • Eosinophilic gastroenterocolitis
  • Eosinophilic gastrointestinal disorder (includes eosinophilic esophagitis)
ICD coding
  • ICD-10:
    • K52.81 - eosinophilic gastritis or gastroenteritis
    • K52.82 - eosinophilic colitis
    • K20.0 - eosinophilic esophagitis
Epidemiology
Sites
  • Throughout the gastrointestinal tract: most common stomach, followed by small intestine and colon (Scott Med J 1990;35:163)
    • 2 or more gastrointestinal sites of involvement are commonly seen
  • Rare cases of bile duct involvement have been reported (Gut 1990;31:54)
Pathophysiology
  • Although not fully understood, it likely has genetic and environmental factors
  • Leading key players are mast cells and eosinophils
  • Immediate phase of reaction is dependent on the mast cell activation through the high affinity IgE receptor
  • Late phase (non-IgE mediated) of reaction involves Th2 lymphocyte and monocytes producing cytokines and chemokines such as eotaxin 1 and interleukin 5, inducing eosinophil accumulation
    • Eosinophils, when recruited in the intestinal epithelium, become cytotoxic by producing inflammatory mediators, such as eosinophil peroxidase, eosinophil derived neurotoxin, eosinophil cationic protein and major basic protein, causing destruction of the gastrointestinal epithelium and organ dysfunction (Baillieres Clin Gastroenterol 1996;10:443, N Engl J Med 1990;323:645, Clin Rev Allergy Immunol 2016;50:175)
    • Eosinophil's 4 major cationic proteins (Nature 1986;321:613):
      • Eosinophil peroxidase (EPO): cytotoxic effect on the epithelium, by the generation of toxic hydrogen peroxide and halide acid leading to tissue damage (J Allergy Clin Immunol 2004;113:11)
      • Eosinophil derived neurotoxin (EDN): ribonuclease activity
      • Eosinophil cationic protein (ECP): cytotoxic effect on the epithelium, by ion nonselective pore insertion (site of entry to other toxic molecules); also has ribonuclease activity
      • Major basic protein (MBP): cytotoxic effect on the epithelium and also plays a role in the degranulation of mast cells and basophils
Etiology
  • Cause is unknown
  • Some research suggests allergic or immune dysregulation as many patients with eosinophilic gastrointestinal disorders (EGIDs) are also atopic (Adv Immunol 2001;78:291, Pediatr Neonatol 2011;52:272, Paediatr Perinat Epidemiol 2007;21:2)
  • EGIDs can be categorized into primary versus secondary (Clin Rev Allergy Immunol 2016;50:175):
    • Primary EGIDs are defined as an inflammatory process characterized by the abnormal eosinophilic infiltration of different segments of the gastrointestinal tract in the absence of a known cause (drug reaction, parasitic infection, malignancy, etc.) (J Allergy Clin Immunol 2004;113:11)
    • Secondary EGIDs are due to drug reaction, parasitic infection, bacterial infection, hypereosinophilic syndrome, Churg-Strauss syndrome, celiac disease, inflammatory bowel disease, polyarteritis nodosa and other connective disorders
Clinical features
  • Nondisease specific gastrointestinal symptoms:
    • Abdominal pain
    • Vomiting, nausea, diarrhea
    • Malnutrition
    • Poor growth and weight loss
  • Some symptoms may be more frequently observed, depending on the depth of gastrointestinal tract wall involvement (Klein classification) (J Allergy Clin Immunol 2004;113:11, Medicine (Baltimore) 1970;49:299):
    • Mucosal pattern: abdominal pain, nausea, vomiting, diarrhea, bleeding, malabsorption, weight loss
    • Muscular pattern: pyloric or intestinal obstruction
    • Serosal pattern: ascites with eosinophilia
  • Often have a prior or concurrent history of atopy (asthma, allergic rhinitis and elevated serum IgE) (Pediatr Neonatol 2011;52:272)
  • Some may present with gross or occult hematochezia, which may lead to significant anemia (Pediatr Neonatol 2011;52:272)
  • Rare reported cases of EGIDs associated with pancreatitis and duodenal ulcer or perforation (World J Gastroenterol 2009;15:2156, Endoscopy 2011;43:E358)
Diagnosis
  • Combination of clinical history, physical exam, endoscopy, radiology and laboratory studies are required as the diagnosis requires the presence of (Gut 1990;31:54):
    • Gastrointestinal symptoms
    • Demonstration of gastrointestinal eosinophilia by biopsy
    • Exclusion of other known causes of tissue eosinophilia
  • Endoscopy
Laboratory
  • No single laboratory test or procedure is diagnostic for EGIDs
  • Maintaining a high index of suspicion is essential
  • Although not specific, the following laboratory tests / findings may support the diagnosis (Clin Rev Allergy Immunol 2016;50:175):
    • Allergy testing to assess allergies that may trigger the onset of symptoms; its utility remains controversial (J Gastroenterol Hepatol 2013;28:1306)
    • Peripheral blood eosinophilia (seen in 20 - 80% of cases)
    • Elevated erythrocyte sedimentation rate
    • Iron deficiency anemia
    • Elevated IgE levels
    • Low serum albumin
    • Fecal levels of eosinophilic cationic protein
    • Serum eosinophilic cationic protein and eosinophil derived neurotoxin (Scand J Gastroenterol 2011;46:1074)
    • Elevated 2 macroglobulin level (Scand J Gastroenterol 2011;46:1074)
    • Stool examination (to rule out parasites)
Radiology description
  • Its usefulness is limited
    • Ultrasound:
      • To detect the presence of bowel wall thickening, ascites or peritoneal nodules
      • Reported use in follow up setting and determination of response to treatment (Ultraschall Med 2011;32:E57)
    • Computerized tomography:
    • Barium contrast:
      • Identification of variable degree of stenosis, mucosal irregularity and thickened mucosal folds (J Clin Pathol 1986;39:1)
    • Tc-99m hexamethyl propylene amine oxime (HMPAO) white blood cell scintigraphy:
Radiology images

Images hosted on other servers:

CT findings of eosinophilic gastrointestinal disorders

Prognostic factors
  • Unknown due to the rarity of the disease but observed to be dependent on response to treatment and Klein classification of disease (Clin Gastroenterol Hepatol 2011;9:950)
  • Patients have 3 different disease courses (Clin Gastroenterol Hepatol 2011;9:950):
    • Single flare: more associated with serosal pattern
    • Recurring course: more related to muscular pattern
    • Continuous course: more associated with mucosal pattern
Case reports
Treatment
  • No established standard treatment guidelines are available due to the lack of extensive prospective studies; current treatments are based on case reports and case series
  • Treatments are empiric and based on the severity of the clinical manifestation
    • Avoidance of allergen or trigger
      • Common issue: recurrence of symptoms with the reintroduction of the allergen, such as specific food
    • Elemental diet (J Pediatr Gastroenterol Nutr 1996;23:81)
    • Anti-inflammatory medication
    • Systemic and topical steroids are the mainstay of therapy (Curr Allergy Asthma Rep 2005;5:259)
      • Reserved for severe disease presentation or failed dietary restriction
      • Serosal EGIDS often have a better response to steroids (Dig Dis Sci 2003;48:1013)
    • Other steroid sparing treatment modalities: mast cell stabilizers (cromolyn sodium), mast cell secretion inhibitor (ketotifen), leukotriene receptor antagonist (montelukast), selective Th2, cytokine IL4 and IL5 inhibitors (subplatast tosilate)
    • Other specific pathway targeting treatment options: anti-IgE monoclonal antibody (omalizumab), anti-IL5 antibodies (reslizumab and mepolizumab) and anti tumor necrosis factor alpha (infliximab)
    • Fecal microbiota transplantation (World J Gastroenterol 2014;20:16368)
Clinical images

Images hosted on other servers:

Duodenum endoscopy

Colon endoscopy

Stomach endoscopy

Duodenum, antrum, rectum endoscopy

Duodenum endoscopy

Gross description
  • Nonspecific findings on the mucosal surface, including thickened mucosa, erythema, erosion, ulceration, white plaques, edema, telangiectasia or submucosal nodules
Microscopic (histologic) description
  • Diagnosis requires confirmation by histopathological examination of gastric, small intestinal or large intestinal biopsies (Adv Anat Pathol 2011;18:335)
    • Distribution of eosinophils may be patchy (obtaining multiple biopsy samples is helpful)
  • Eosinophils are seen in the entire gastrointestinal tract except in the esophagus in a normal physiologic state; they are an essential part of the innate immune system of the digestive tract
  • Lack of consensus on histological features of EGEC due to the relative rarity of the disease and variation of the numbers of eosinophils but it is characterized by prominent eosinophilic infiltration of the wall (most commonly affected sites: stomach and small intestine) (Case Rep Gastroenterol 2013;7:293)
  • No established cutoff for the number of eosinophils/high power field to diagnose EGECs
  • Pathologists should comment on the findings if eosinophils are more than expected
  • Recently suggested cutoff numbers are shown below but they are not formal diagnostic criteria (Gastroenterol Clin North Am 2014;43:257):
    • Stomach: ≥ 30 eosinophils/high power field in 5 high power fields
    • Duodenum: > 52 eosinophils/high power field
    • Ileum: > 56/high power field in the ileum
    • Right colon: > 100/high power field
    • Transverse and descending colon: > 84/high power field
    • Rectosigmoid colon: > 64/high power field
  • Distribution of eosinophils is also an essential clue for EGEC irrespective of the eosinophil counts
    • Presence of eosinophils within the crypts (intraepithelial eosinophils and eosinophilic crypt abscesses), clusters of eosinophils in lamina propria and eosinophils in muscularis mucosae are abnormal
  • In resection specimens, transmural eosinophilic infiltration may be present
  • May accompany other inflammatory cells, especially mast cells
  • Other nonspecific changes include edema, mucin depletion, erosion, reactive epithelial changes (foveolar hyperplasia and villous atrophy) and mild architectural distortion
  • Areas of increased eosinophilic inflammation may show extracellular deposition of eosinophil granule constituents (Gastroenterology 1992;103:137)
  • Klein et al. classified the disease based on the location of the eosinophilic infiltration in the different layers of the intestinal wall (depth of eosinophilic infiltration) into mucosal, muscular and subserosal patterns of disease (Medicine (Baltimore) 1970;49:299)
    • Mucosal:
    • Muscular:
      • Second most commonly reported
      • Infiltration of eosinophils predominantly in the muscle layer
      • Causes wall thickening, which may lead to gastric outlet obstruction, intestinal obstruction or intussusception without eosinophilic ascites (World J Gastroenterol 2007;13:1758, Am Surg 1997;63:741)
    • Subserosal:
      • Most unusual form; estimated prevalence of up to 9% in Japan and 13% in the U.S. (Intern Med 1996;35:779)
      • Infiltration of eosinophils in the serosa layer
      • Hallmark of this type is ascitic fluid with eosinophilia
      • Often accompanying peripheral eosinophilia, peritonitis (Medicine (Baltimore) 1970;49:299)
      • May present with isolated abdominal ascites or ascites along with symptoms more characteristic of mucosal or muscular EGIDSs
      • Chang et al. observed muscular and serosal types of having concomitant mucosal eosinophilic gastroenteritis, suggesting disease progression from mucosal to serosal (Clin Gastroenterol Hepatol 2010;8:669)
Microscopic (histologic) images

Contributed by Byoung Uk Park, M.D. and Lizhi Zhang, M.D.
Prominent eosinophilic infiltration

Prominent eosinophilic infiltration

Clustering of eosinophils

Clustering of eosinophils

Intraepithelial eosinophils

Intraepithelial eosinophils

Eosinophilic abscesses

Eosinophilic abscesses

Eosinophils in muscularis mucosae

Eosinophils in muscularis mucosae

Eosinophils in mucosa and submucosa

Eosinophils in mucosa and submucosa


Transmural eosinophilic infiltration

Transmural eosinophilic infiltration

Prominent eosinophilic infiltration with reactive epithelial changes

Prominent eosinophilic infiltration

Intraepithelial eosinophils

Intraepithelial eosinophils

Eosinophils in muscularis propria

Eosinophils in muscularis propria

Eosinophils in serosa

Eosinophils in serosa

Positive stains
Sample pathology report
  • Small intestine and stomach, biopsy:
    • Marked eosinophilic infiltration identified in both small bowel and gastric mucosa (see comment)
    • Comment: The biopsy shows diffuse eosinophilic infiltrate (> 50/high power field) involving the mucosa and submucosa with focal intraepithelial eosinophils. No apparent microorganisms, neutrophils, vasculitis or granulomas are identified. In the appropriate clinical setting, the findings may be indicative of eosinophilic gastroenteritis. Clinical and pathologic correlation and further workup are recommended to rule out secondary causes of the observed eosinophilia.
Differential diagnosis
  • Parasitic or bacterial infections:
  • H. pylori infection:
  • Inflammatory bowel disease (Mayo Clin Proc 1997;72:117):
    • May present with peripheral and intestinal eosinophilia
    • Lacks the florid eosinophilia
  • Hypereosinophilic syndrome:
    • Rare and idiopathic condition
    • Persistence (> 6 month) of marked peripheral eosinophilia (often exceeding > 1,500 eosinophils/microliter)
    • Along with the presence of organ damage or dysfunction related to eosinophilic infiltration and release of mediators
    • Most common presenting manifestation: dermatologic and pulmonary (J Allergy Clin Immunol 2009;124:1319)
  • Vasculitis disorders (Churg-Strauss syndrome and polyarteritis nodosa):
    • Eosinophilic infiltrate of the small vessels
    • Peripheral eosinophilia
    • Elevated markers of inflammation and autoantibodies
  • Connective tissue disorders (scleroderma and dermatomyositis):
    • Episodic peripheral eosinophilia and a band-like eosinophil and mast cell infiltrate of the lamina propria
  • Drug hypersensitivity:
    • Gastrointestinal eosinophilia has been associated with a long list of medications, such as carbamazepine, rifampicin, naproxen, NSAIDs, interferon, azathioprine, enalapril and gold compounds
  • Mastocytosis and Langerhans histiocytosis:
    • Gastrointestinal eosinophilia is usually seen in GI involvement by mastocytosis and Langerhans histiocytosis
    • Besides eosinophils, prominent mononuclear cell infiltration with a nodular or tumefactive growth pattern is present
    • Mononuclear cells may show characteristic morphologic features of either mast cells or Langerhans cells which immunohistochemical stains can confirm
Board review style question #1

In the image of a gastric biopsy from a 42 year old woman, what is the most concerning feature in considering a diagnosis of eosinophilic gastritis?

  1. Clustering of eosinophils
  2. Eosinophils in muscularis mucosae
  3. Glandular reactive change
  4. Increased number of eosinophils
  5. Marked congestion
Board review style answer #1
B. Eosinophils in muscularis mucosae. Prominent eosinophilic infiltration is a hallmark of eosinophilic gastroenterocolitis (EGEC), which is sensitive but not specific for diagnosing EGEC. The cutoff numbers of eosinophilic counts have not been well established. On the other hand, the abnormal distribution of eosinophils, such as muscularis mucosae, in this case, is an essential clue for EGEC.

Comment Here

Reference: Eosinophilic gastroenterocolitis
Board review style question #2

Which of the following statements are true regarding the histologic finding of the above gastric biopsy?

  1. Although there are increased eosinophils in the lamina propria, the eosinophil count does not meet the diagnostic criteria for eosinophilic gastritis
  2. Diagnosis of eosinophilic gastritis should be considered but exclusion of other known causes of tissue eosinophilia is needed
  3. Increased eosinophils in the lamina propria are diagnostic for eosinophilic gastritis
  4. Observed changes are diagnostic for H. pylori infection
  5. Similar finding in other sites of the GI tract in this patient is unlikely
Board review style answer #2
B. Diagnosis of eosinophilic gastritis should be considered but exclusion of other known causes of tissue eosinophilia is needed. The picture shows eosinophilia in gastric mucosa, which can be due to various causes, such as eosinophilic gastroenteritis, drug effect, parasites, hypereosinophilic syndrome, etc. The cutoff numbers of eosinophilic counts have not been well established for the diagnosis of eosinophilic gastroenterocolitis. Therefore, it is a diagnosis of exclusion and other known causes of tissue eosinophilia must be ruled out with clinical correlation.

Comment Here

Reference: Eosinophilic gastroenterocolitis

Escherichia coli
Definition / general
  • Escherichia coli (E. coli) is a gram negative, facultative anaerobic, coliform, rod shaped bacterium
  • It is a part of the normal flora of the intestine of humans and many warm blooded animals; it provides benefits such as production of vitamin K2 and a stable environment where more beneficial bacteria can prosper (Elife 2015;4:e05826)
  • Most cases are transmitted through contaminated food and water
Essential features
  • Most cases cause self limiting disease
  • Treatment is supportive
Terminology
  • Enteroaggregative E. coli (EAEC)
  • Shiga toxin producing E. coli (STEC) / enterohemorrhagic E.coli (EHEC)
  • Enteroinvasive E. coli (EIEC)
  • Enteropathogenic E. coli (EPEC)
  • Enterotoxigenic E. coli (ETEC)
  • Diffusely adherent E. coli (DEAC)
  • Adherent invasive E. coli (AIEC)
ICD coding
  • ICD-10:
    • B96.20 - unspecified Escherichia coli (E. coli) as the cause of diseases classified elsewhere, Escherichia coli (E. coli), NOS
    • B96.21 - Shiga toxin producing Escherichia coli (E. coli) (STEC) O157 as the cause of diseases classified elsewhere
    • B96.22 - other specified Shiga toxin producing Escherichia coli (E. coli) (STEC) as the cause of diseases classified elsewhere
    • B96.23 - unspecified Shiga toxin producing Escherichia coli (E. coli) (STEC) as the cause of diseases classified elsewhere
    • B96.29 - other Escherichia coli (E. coli) as the cause of diseases classified elsewhere, non-Shiga toxin producing E. coli
Enteroaggregative E. coli (EAEC)
Shiga toxin producing E. coli (STEC) / enterohemorrhagic E. coli (EHEC)
Enteroinvasive E. coli (EIEC)
Enteropathogenic E. coli (EPEC)
  • Some types of EPEC are known as diffusely adherent E. coli (DAEC)
  • Major cause of infant diarrhea in developing countries
  • Due to contaminated water and meat products
  • Virulence mechanism is unrelated to excretion of typical E. coli enterotoxins; diarrhea and symptoms are caused by invasion of host cells (attachment and effacement) rather than by production of toxins
  • Has pathogenicity island that encodes proteins which modulate the actin microtubule and intermediate filament networks to allow intimate attachment of bacteria to plasma membrane of infected enterocytes, forming attachment and effacing lesions (J Bacteriol 2006;188:3110)
    • Also has large plasmid that contains a cluster of genes encoding bundle forming pili
  • Infectious dose is 106 organisms
  • Atypical cases lack bundle forming pili and are associated with prolonged diarrhea (Emerg Infect Dis 2006;12:597)
  • Clinical features:
    • Infantile diarrhea may lead to dehydration, electrolyte imbalance or death
    • Watery or bloody diarrhea
    • Up to 50% mortality rates in developing countries
  • Treatment: self limited; antibiotics may not improve disease and may increase the risk of developing hemolytic uremic syndrome
  • Micro description: effacement of brush border microvilli of enterocytes; no bacterial invasion but surface adherent organisms may be identifiable
  • References: Food and Drug Administration: Bad Bug Book - Foodborne Pathogenic Microorganisms and Natural Toxins, 2nd Edition, 2012, Todar: Todar's Online Textbook of Bacteriology, 2020, Microbiology (Reading) 2004;150:527, Emerg Infect Dis 2002;8:508
Enterotoxigenic E. coli (ETEC)
Diffusely adherent E. coli (DEAC)
  • Extremely heterogeneous group of potentially pathogenic E. coli, which are characterized by their diffuse adherence to epithelial cells
  • DAEC isolates are detected not only in humans commonly in children of between 2 and 5 years of age but also in various groups of animals (dogs, calves, cattle, poultry, pigs)
  • No diarrheal disease has been reported in adults but the latest data indicates a possible contribution of DAEC strains in the process of the development of colorectal cancer and Crohn's disease (CD) (Clin Microbiol Rev 2014;27:823, Advancements of Microbiology 2019;58:143)
  • On the basis of adhesins, DAEC strains have been divided into 2 basic groups: typical DAEC (which binds to the hDAF receptor) and atypical DAEC (which does not bind the hDAF factor)
  • DAEC are omitted in the routine diagnosis of infections of the digestive tract due to the huge genotypic diversity of these strains
  • Optimum method for diagnosis of DEAC is still under active research; the lack of commonly available diagnostic methods hinders the correct identification of infections caused by DAEC and therefore, the possibility of treatment (BMC Microbiol 2013;13:22, Front Cell Infect Microbiol 2020;10:572951)
Adherent invasive E. coli (AIEC)
  • Adherent invasive Escherichia coli (AIEC) pathobiont bacteria have the ability to adhere to and invade intestinal epithelial cells (IECs) as well as to survive and replicate within macrophages
  • AIEC bacteria have been linked in the etiology of CD; however, active genetic research is ongoing in order to better establish this link and develop possible treatment modalities to prevent and limit AIEC colonization in CD patients (Int J Mol Sci 2020;21:3734)
Epidemiology
  • In the U.S., most outbreaks have been reported in Minnesota, followed by Washington
  • Route of transmission is foodborne; mainly by undercooked ground beef in EHEC
 (Emerg Infect Dis 2005;11:603
  • ETEC organisms are the principal cause of traveler's diarrhea and spread via contaminated food or water
  • EIEC / EAEC organisms are bacteriologically similar to Shigella and are transmitted via food, water or by person to person contact
Sites
  • Colon and rectum
Pathophysiology
  • Depends on the strain of pathogenic E. coli
    • Enteroaggregative E. coli
      • EAEC adhere to epithelial cells
      • EAEC attach to enterocytes via adherence fimbriae and are aided by dispersin, a bacterial surface protein that neutralizes the negative surface charge of lipopolysaccharide
      • They also produce enterotoxins similar to Shigella enterotoxin and ETEC ST toxin (Curr Opin Gastroenterol 2005;21:4)
    • Enterohemorrhagic E. coli / Shiga toxin producing E. coli (STEC)
      • EHEC are classified as E. coli O157:H7 and non-O157:H7 serotypes
      • Both serotypes produce Shiga-like toxins
      • Hemolytic uremic syndrome (HUS) is caused by Shiga-like toxin which is absorbed by inflamed gastrointestinal mucosa into the circulation, where it alters endothelial cell function leading to platelet activation and aggregation
      • Children and the elderly are at highest risk (Med Clin North Am 2013;97:681)
    • Enteroinvasive E. coli
      • EIEC do not produce toxins, they invade epithelial cells and cause acute self limited colitis (J Vet Sci 2022;23:e28)
    • Enterotoxigenic E. coli
      • ETEC produce LT toxin and ST toxin
      • Both induce chloride and water secretion while inhibiting intestinal fluid absorption
      • LT toxin activates adenylate cyclase and stimulates chloride secretion and simultaneously inhibits absorption
      • ST toxins bind to guanylate cyclase and stimulate chloride secretion (Gut Microbes 2022;14:2055943)
Etiology
  • Contaminated food and water
Clinical features
  • EAEC organisms cause nonbloody diarrhea that may be prolonged in AIDS patients (Am J Trop Med Hyg 2010;83:158)
  • EHEC causes symptoms similar to S. dysenteriae
    • Causes large outbreaks of bloody diarrhea and HUS
    • HUS includes microangiopathic hemolytic anemia and thrombocytopenia but no neurologic symptoms and no acute renal failure (Med Clin North Am 2013;97:681)
  • EIEC is common among young children in developing countries
  • ETEC causes traveler's diarrhea commonly in underdeveloped regions and children younger than 2 years of age are particularly susceptible
Diagnosis
  • Typically made by clinical findings and history of ingestion of contaminated food like undercooked meat
Laboratory
  • Stool culture on sorbitol MacConkey agar (SMAC) or the variant cefixime potassium tellurite sorbitol MacConkey agar (CT SMAC)
  • Colonies appear clear on SMAC due to their inability to ferment sorbitol, while sorbitol fermenting colonies of E. coli serotypes appear red
  • E. coli DNA extraction method: DNA probes plus PCR techniques for detecting verocytotoxin (MMWR Recomm Rep 2009;58:1, StatPearls: Escherichia Coli [Accessed 1 June 2023])
Radiology description
  • CT is usually preferred modality for radiological assessment (AJR Am J Roentgenol 2001;177:619)
  • Shows severe diffuse colonic wall thickening, often with a target sign and pericolic stranding
  • Involvement of bowel can be segmental or diffuse
Radiology images

Images hosted on other servers:

CT scan:
thickened colon
with target sign

CECT: pancolitis
with thickened colon
and target sign

Prognostic factors
  • Disease is usually self limiting and patients recover on supportive therapy
Case reports
  • 32 year old woman with infection due to Shiga toxin producing enterohemorrhagic Escherichia coli (EHEC) presenting as ischemic colitis (IDCases 2019;18:e00629)
  • 36 year old woman with severe ulcerative colitis and Coronavirus disease 2019 followed by Escherichia coli 0157:H7 infection (Croat Med J 2021;62:634)
  • 75 year old man with pseudomembranous colitis-like lesions associated with hemolytic uremic syndrome and neurological sequelae (BMJ Case Rep 2017;2017:bcr2016218586)
Treatment
Clinical images

Contributed by Ateeqa Mujeeb Ullah, M.D.
E. coli colonies E. coli colonies

E. coli colonies

Gross description
  • EHEC: colonic mucosa is edematous and erythematous with multiple erosions
  • Erosion may show adherent blood clots (Virulence 2013;4:366)
Microscopic (histologic) description
  • EAEC, EIEC, ETEC
    • No significant microscopic changes
  • EHEC
    • Foci of epithelial cell damage / necrosis with pyknotic nuclei, vacuolated cytoplasm 

    • Areas of hemorrhage within lamina propria
Microscopic (histologic) images

Contributed by Ateeqa Mujeeb Ullah, M.D.
Acute colitis Acute colitis

Acute colitis

Positive stains
  • E. coli are gram negative bacilli
Electron microscopy description
  • Transmission electron microscopy, in cases of EHEC, shows pyknotic nuclei, damaged membranes and cytoplasmic vacuolization of colonic epithelial cells
Sample pathology report
  • Left colon, colectomy:
    • Colonic mucosa with focal active colitis, surface erosions and focal areas of hemorrhage (see comment)
    • Comment: Focal active colitis pattern is a nonspecific histologic finding that may be seen in association with a variety of conditions. The most common clinical diagnoses rendered upon follow up of patients with a focal active colitis pattern include acute self limited / infectious colitis, irritable bowel syndrome, Crohn's disease, ischemic colitis, antibiotic associated colitis, NSAID related colitis, diverticular disease associated colitis, etc.
Differential diagnosis
  • Ischemic colitis:
    • Negative culture for E. coli; no significant history
  • Other infective colitis:
    • Negative culture for E. coli
Board review style question #1
Which E. coli strain has been implicated as a possible precursor to the development of colorectal cancer?

  1. Adherent invasive E. coli (AIEC)
  2. Diffusely adherent E. coli (DEAC)
  3. Enteropathogenic E. coli (EPEC)
  4. Enterotoxigenic E. coli (ETEC)
  5. Shiga toxin producing E. coli (STEC)
Board review style answer #1
B. Diffusely adherent E. coli (DEAC). Diffusely adherent E. coli (DEAC) found in colons of Crohn's disease and colorectal cancer (CRC) patients have been found to harbor the daaC and afaBC genes. The afa1 positive strains isolated from these patients demonstrate adherence and invasion into various epithelial cells and also induce the increased mRNA expression of VEGF (vascular endothelial growth factor), which promotes angiogenesis in tumors. Multiple other genes and the proinflammatory effect of DEAC adhesions on intestinal epithelium has also been associated with the development of Crohn's disease and CRC according to multiple studies (Gastroenterology 2004;127:80, Advancements of Microbiology 2019;58:143, World J Gastrointest Pathophysiol 2014;5:213). No other strain of E. coli has been associated with the development of CRC.

Comment Here

Reference: Escherichia coli
Board review style question #2

A 45 year old woman with chronic kidney disease status post renal transplant presents to the emergency department with vomiting, bloody diarrhea and dehydration. In light of the recent E. coli O157:H7 outbreak, you suspect E. coli colitis. What is the gold standard for diagnosis?

  1. Cefixime potassium tellurite sorbitol MacConkey agar (CT SMAC)
  2. Horse blood agar
  3. Thayer Martin Agar (VPN agar)
  4. Thiosulfate citrate bile salts sucrose agar (TCBS)
Board review style answer #2
A. Cefixime potassium tellurite sorbitol MacConkey agar (CT SMAC). Cefixime potassium tellurite sorbitol MacConkey agar (CT SMAC) is a modified form of MacConkey sorbitol agar (SMAC) that contains selective agents cefixime and tellurite. Cefixime inhibits the growth of other enteric organisms, such as Proteus spp. and potassium tellurite inhibits non-O157 Shiga toxin producing E. coli (STEC). The use of CT SMAC has been associated with increased cultural sensitivity for the isolation of E. coli O157:H7. Therefore, it is recommended by the American Society of Microbiology and Centers for Disease Control and Prevention (CDC) for use as a primary isolation medium for E. coli O157:H7 in community acquired diarrhea (Versalovic: Manual of Clinical Microbiology, 10th Edition, 2011). The mechanism includes the production of pink colonies by E. coli strains other than O157:H7 through the fermentation of D sorbitol, whereas the colonies of CT SMAC are colorless after overnight incubation due to the lack of D sorbitol fermentation (ThermoFisher: MacConkey Sorbitol Agar w/ Cefixime and Tellurite [Accessed 15 March 2023).

Comment Here

Reference: Escherichia coli

Familial adenomatous polyposis, attenuated
Definition / general
  • Subtype of familial adenomatous polyposis (FAP) characterized by fewer than 100 adenomatous colorectal polyps
  • Colonic adenomatous polyps have high risk for progression to colorectal adenocarcinoma (69% cumulative risk by age 80) (Gastroenterology 2004;127:444)
Essential features
  • FAP variant characterized by markedly fewer polyps
  • Due to a defect in APC (5q21)
  • Patients might not require colectomy
Terminology
Epidemiology
  • Phenotypically and genetically heterogeneous (Gut 2006;55:1440)
  • Cancers usually develop at age 50 - 55 years, 15 years later than classic FAP
Sites
  • Patients may develop gastric fundic gland polyps, gastric or small bowel flat adenomas and gastric / duodenal carcinomas (Cancer 1993;71:2709)
  • Extracolonic manifestations of classical FAP are less common in attenuated FAP
Diagnosis
  • Classically reserved for patients with fewer than 100 colonic adenomatous polyps but exact diagnostic criteria have not been firmly established
Case reports
Treatment
  • Patients with fewer than 20 - 30 polyps may not require total colectomy if they have frequent surveillance colonscopies with polypectomies
Gross description
  • Polyps are usually more proximal (i.e. right sided) than in classic FAP
    • Rectum often spared
  • Polyps are flat, slightly raised or plaque-like
  • May have minute central depression or umbilication
Microscopic (histologic) description
  • Adenomatous polyps are microscopically similar to sporadic type adenomas
Molecular / cytogenetics description
  • Associated with pathogenic variants in the 5' and distal 3' end of APC, as well as interstitial deletions of 5q22, which include the APC gene
Videos

Attenuated familial adenomatous polyposis (FAP)

Differential diagnosis
Board review style question #1
How is attenuated familial adenomatous polyposis different from classic familial adenomatous polyposis?

  1. Patients develop carcinomas at a younger age
  2. The adenomas are fewer in number
  3. The adenomas are smaller in size
  4. The adenomas occur in the small intestine, not the colon
Board review style answer #1
B. The adenomas are fewer in number

Comment Here

Reference: Familial adenomatous polyposis, attenuated

Familial adenomatous polyposis, classic
Definition / general
  • Autosomal dominant familial polyposis syndrome due to a defect in the APC gene (5q21) which prototypically results in numerous (> 100) colonic adenomatous polyps
Essential features
Terminology
  • Also known as familial polyposis coli or adenomatous polyposis coli
Epidemiology
  • Incidence of 1 per 7 - 30,000 individuals
  • Males and females are equally affected
  • Colon polyps typically are present by the end of the second decade (mean age: 15.9 years)
  • Essentially 100% of patients develop colonic adenocarcinoma without surgical intervention (average age: 39 years)
  • 25% of patients will be diagnosed with colorectal carcinoma at time of presentation
Sites
  • In addition to colonic adenomatous polyps, patients may develop polyps in the stomach (fundic gland polyps) and small intestine
  • May also develop carcinoma of thyroid gland, gallbladder and adrenal gland
  • Desmoid tumors are a common extraintestinal manifestation (10 - 25%) - they frequently develop in the abdominal wall following local surgery / trauma
  • May develop bone lesions (i.e. osteomas), skin lesions (i.e. epidermal inclusion cysts), dental abnormalities (i.e. supranummary or absent teeth), ocular problems (i.e. congenital hypertrophy of retinal pigment epithelium [CHRPE]) or juvenile nasopharyngeal angiofibromas
  • Gardner syndrome encompasses the subset of FAP patients with extraintestinal tumors
  • Turcot syndrome encompasses the subset of FAP patients with brain tumors, which are typically medulloblastomas
Pathophysiology
  • APC (adenomatous polyposis coli) is a tumor suppressor gene involved in cell cycle control and downregulation of beta catenin through the Wnt signaling pathway
  • APC protein is normally involved in apoptosis of colonic epithelial cells
  • APC mutations may cause expansion of the crypt base cell population, including crypt stem cells
  • When APC is mutated, beta catenin is no longer downregulated and can function to stimulate cell growth
  • As explained by the 2 hit hypothesis, patients with a germline APC mutation develop adenomatous polyps when there is inactivation of the remaining normal APC allele
  • Colorectal adenocarcinomas in FAP develop in a similar fashion to sporadic type colorectal adenocarcinomas (i.e. mutations in KRAS, TP53)
  • Abundance of adenomatous polyps results in the near certain development of colorectal adenocarcinoma
Diagrams / tables

Contributed by @Andrew_Fltv on Twitter
Familial adenomatous polyposis, classic

Familial adenomatous polyposis, classic

Diagnosis
  • Diagnostic criteria:
    • 100 or more colorectal adenomatous polyps or
    • Germline mutation in APC or
    • Family history of FAP with colorectal adenomas (age < 30) or
    • Family history of FAP and presence of at least one epidermoid cyst, osteoma or desmoid tumor
  • Patients with a history of > 10 colorectal adenomas, a family history of adenomatous polyposis syndromes or a history of adenomas with FAP type extracolonic lesions should undergo assessment for adenomatous polyposis syndrome (Am J Gastroenterol 2015;110:223)
Prognostic factors
  • Location of the mutation in APC has an impact on the phenotypic presentation, including number of polyps and presence or absence of desmoid tumors and congenital hypertrophy of retinal pigment epithelium
  • Most common causes of noncolorectal carcinoma deaths in FAP patients are duodenal / ampullary carcinoma and complications of desmoid tumor
Case reports
Treatment
  • Prophylactic colectomy in late second to early third decade
  • Must monitor rectal stump if preserved
  • FAP patients who undergo consistent colorectal screening with subsequent surgery have a significantly decreased rate of mortality from colorectal malignancies
  • Children at risk for FAP should undergo sigmoidoscopy every 1 - 2 years, beginning at age 10 - 12 years
  • Relatives should also be screened
  • NSAIDs and aspirin use have been studied to decrease polyp regrowth following subtotal colectomy
Gross description
  • Most adenomatous polyps are small (< 0.5 cm) and sessile
  • Polyps are more common in the left sided colon, although the entire length of the colon is typically involved
  • Polyps may be flat or depressed (Int J SurgPathol 2006;14:133)
Gross images

Contributed by @Andrew_Fltv on Twitter
Familial adenomatous polyposis, classic

Familial adenomatous polyposis, classic



Images hosted on other servers:
Missing Image Missing Image

Carpet of adenomatous polyps

Missing Image Missing Image

Numerous polyps

Microscopic (histologic) images

Contributed by Jennifer Findeis-Hosey, M.D. and @Andrew_Fltv on Twitter
Missing Image

Adenoma involving few crypts

Missing Image

Tubular adenoma polyp, FAP patient

Familial adenomatous polyposis, classic Familial adenomatous polyposis, classic

Familial adenomatous polyposis, classic



Images hosted on other servers:
Missing Image

Adenoma and adenocarcinoma

Missing Image

FAP versus non-FAP

Missing Image

BCL2

Molecular / cytogenetics description
  • Hereditary colorectal cancer syndromes are usually due to defect in APC gene at 5q21
    • Polyposis patients without APC mutations often have mutations in MYH gene
    • Autosomal dominant trait with high degree of penetrance (> 90%)
    • 20% represent a new de novo APC mutation without family history
Videos

Full colonoscopy in retroflexed maneuver in a familial adenomatous polyposis coli

Sample pathology report
  • Colon, total colectomy:
    • Colon with innumerable tubular adenomas, some with focal high grade dysplasia (see comment)
    • Negative for malignancy.
    • Adenomas focally present at both proximal and distal mucosal resection margins.
    • Twelve benign lymph nodes.
    • Comment: The findings are consistent with the patient's reported history of familial adenomatous polyposis. Numerous grossly identifiable polyps were submitted for microscopy, including all polyps larger than 1 cm.
Differential diagnosis
  • Attenuated FAP:
    • Polyposis syndrome also due to an APC mutation but patients have fewer than 100 colorectal polyps
  • MYH associated polyposis:
    • Polyposis syndrome caused by a mutation in the MYH gene
Board review style question #1
Without surgical intervention, patient with familial adenomatous polyposis have lifetime risk of developing colorectal carcinoma of approximately

  1. 25%
  2. 50%
  3. 75%
  4. 100%
Board review style answer #1
Board review style question #2
Patients with Turcot syndrome develop what tumors in addition to the typical manifestations of familial adenomatous polyposis?

  1. Desmoid tumors (fibromatosis)
  2. Gastrointestinal stromal tumors
  3. Keratoacanthomas
  4. Medulloblastomas
Board review style answer #2

Focal active colitis
Definition / general
Essential features
  • Focal active colitis (FAC) is characterized by a single focus or multiple foci of neutrophilic infiltration of the crypts with an otherwise unremarkable colonic mucosa; the crypt architecture is well preserved with no evidence of chronic injury (Cureus 2020;12:e8140)
  • FAC can be seen in the setting of acute self limited infectious colitis, evolving inflammatory bowel disease (IBD), ischemic colitis, Clostridium difficile colitis, drug induced or chemical injury, irritable bowel syndrome and bowel preparation artifact (Cureus 2020;12:e8140)
  • Incidence of Crohn's disease (CD) in adults presenting with FAC is relatively low and varies between 0 and 13%, whereas the incidence of infectious type colitis has been demonstrated to be nearly 50% (Cureus 2020;12:e8140)
  • In pediatric patients, a 24% rate of IBD is seen with FAC; however, when patients with associated terminal ileal inflammation were excluded, the rate dropped to 11% (similar to the rates of adults) (Hum Pathol 2018;74:164)
  • FAC remains a diagnostic challenge; one must incorporate the patient's presenting history, medications and findings on endoscopy to help narrow the differential diagnosis
ICD coding
  • ICD-11: 1A40.0 - gastroenteritis or colitis without specification of origin
Sites
  • Colon
Clinical features
Diagnosis
  • Diagnosed on colonoscopic biopsy
Prognostic factors
  • Prognosis depends upon the underlying cause whether it is infectious or inflammatory bowel disease
Case reports
  • Teenage girl with orbital myositis associated with focal active colitis (Childs Nerv Syst 2012;28:641)
  • 82 year old woman presented with complaints of diarrhea, nausea, abdominal cramping, increased flatulence, anorexia and weight loss for the past 3 months (Cureus 2020;12:e8140)
Treatment
  • Treatment of underlying etiology, such as inflammatory bowel disease, irritable bowel syndrome or infectious colitis
Microscopic (histologic) description
  • Neutrophil mediated surface epithelial injury, neutrophils within epithelial cells (cryptitis) or within crypt lumina (crypt abscess) (Biomedicines 2023;11:2631)
  • Expansion of lamina propria with neutrophils and eosinophils
  • Well preserved crypt architecture
Microscopic (histologic) images

Contributed by Safina Ahmed, M.B.B.S. and Saroona Haroon, M.B.B.S.
Colonic mucosa with cryptitis Colonic mucosa with cryptitis and crypt abscess Colonic mucosa with cryptitis and crypt abscess Colonic mucosa with cryptitis and crypt abscess

Colonic mucosa with cryptitis and crypt abscess

Colonic mucosa with crypt abscess Colonic mucosa with crypt abscess

Colonic mucosa with crypt abscess

Sample pathology report
  • Colon, biopsy:
    • Focal active colitis (see comment)
    • Comment: The differential diagnosis for focal active colitis includes infectious process, medication associated injury (e.g., nonsteroidal anti-inflammatory drugs) or emerging inflammatory bowel disease. Histologic features of chronic injury are not seen. Negative for viral cytopathic effect and granulomata.
Differential diagnosis
Board review style question #1

A 55 year old man presented with diarrhea, abdominal discomfort and flatulence. Colonoscopy shows patchy erythematous areas in the left colon. Endoscopic biopsy is taken and it shows the histology above. Which of the following statements is the most likely morphological diagnosis?

  1. Focal active colitis
  2. Lymphocytic colitis
  3. Pseudomembranous colitis
  4. Ulcerative colitis
Board review style answer #1
A. Focal active colitis. Focal active colitis is the correct answer as there is focal cryptitis and crypt abscess formation. The crypt architecture is intact. Answer B is incorrect because there are no increased intraepithelial lymphocytes. Answer C is incorrect because pseudomembranous colitis typically has necroinflammatory exudate on the surface. Answer D is incorrect because crypt distortion is typically seen in ulcerative colitis of patients of this age range.

Comment Here

Reference: Focal active colitis
Board review style question #2
A 17 year old boy presented with bloody diarrhea and elevated fecal calprotectin. He had granulomata on upper GI biopsy and colonoscopic biopsy showed focal active colitis. Which of the following is the most likely underlying diagnosis?

  1. Crohn's disease
  2. Lymphocytic colitis
  3. Pseudomembranous colitis
  4. Ulcerative colitis
Board review style answer #2
A. Crohn's disease is the correct answer because in the pediatric population, inflammatory bowel disease can just present by focal active colitis, even though the architecture is intact. Answer B is incorrect because lymphocytic colitis has increased intraepithelial lymphocytes. Answer C is incorrect because pseudomembranous colitis shows inflammatory exudate on the mucosal surface. Answer D is incorrect because ulcerative colitis doesn't show granulomas on upper GI biopsies.

Comment Here

Reference: Focal active colitis

Foreign materials / food
Definition / general
  • Food histology can mimic drugs and parasites in the gastrointestinal tract and lungs
Essential features
  • Foods can be aspirated or found in the gastrointestinal (GI) tract and can mimic bugs or drugs
  • It is essential to be aware of food histology in order to avoid misinterpreting a mimicker
  • Foreign material found in biopsies, surgical specimens and during autopsies
Terminology
N/A
ICD coding
N/A
Epidemiology
N/A
Sites
  • Often found in the GI tract and lungs
  • Can be seen during routine screening, within surgical specimens or found during autopsy
Pathophysiology
N/A
Etiology
N/A
Diagrams / tables
N/A
Clinical features
N/A
Diagnosis
  • Often foreign material is an incidental finding, which can mimic pathological disease and therefore histology of foreign material is essential
Laboratory
N/A
Radiology description
N/A
Radiology images
N/A
Prognostic factors
N/A
Case reports
Treatment
N/A
Clinical images
N/A
Gross description
N/A
Gross images
N/A
Frozen section description
N/A
Frozen section images
N/A
Microscopic (histologic) description
N/A
Microscopic (histologic) images

Contributed by Mona Deerwester, M.D., M.Sc.
Apple seed Apple seed

Apple seed

Asparagus Asparagus

Asparagus

Muscle and adipose tissue of bacon Muscle and adipose tissue of bacon

Muscle and adipose tissue of bacon


Banana Banana

Banana

Beet Beet

Beet

Blackberry Blackberry

Blackberry


Blueberry seeds

Blueberry seeds

Blueberry skin

Blueberry skin

Blueberry ultrastructure

Blueberry ultrastructure

Broccoli parenchymal cells and epidermis Broccoli parenchymal cells and epidermis

Broccoli parenchymal cells and epidermis

Parenchymal cells of broccoli

Parenchymal cells of broccoli


Parenchymal cells of cantaloupe

Parenchymal cells of cantaloupe

Parenchymal, phloem and xylem cells of carrot

Parenchymal, phloem and xylem cells of carrot

Parenchymal cells of celery

Parenchymal cells of celery

Myocytes of chicken Myocytes of chicken

Myocytes of chicken

Parenchymal cell of cinnamon with pigment

Parenchymal cell of cinnamon with pigment


Parenchymal cell of cinnamon with pigment

Parenchymal cell of cinnamon with pigment

Parenchymal cell of coconut Parenchymal cell of coconut

Parenchymal cell of coconut

Cookie Cookie

Cookie

Cucumber

Cucumber


Date fruit Date fruit

Date fruit

Date fruit

Date fruit

Skin of the date fruit

Skin of the date fruit

Eggplant Eggplant

Eggplant


Fig

Fig

Fig fruit, GMS

Fig fruit, GMS

Fig fruit Fig fruit

Fig fruit

Seed of a fig fruit

Seed of a fig fruit

Fig seed

Fig seed


Fish retina (eye)

Fish retina (eye)

Fish eye Fish eye

Fish eye

Garlic

Garlic

Ginger

Ginger

Red grapes

Red grapes


Red grapes

Red grapes

Green bean

Green bean

Hot dog Hot dog

Hot dog

Hot dog

Hot dog

Hot dog calcifications

Hot dog calcification


Hot dog vasculature

Hot dog vasculature

Jalapeño

Jalapeño

Leek

Leek

Lemon skin

Lemon skin

Lettuce

Lettuce

Mango

Mango


Mushroom Mushroom

Mushroom

Oats Oats

Oats

Okra Okra

Okra


Olive Olive

Olive

Olive

Olive

Onion Onion

Onion

Orange

Orange


Pasta noodle Pasta noodle

Pasta noodle

Peanut Peanut

Peanut

Peanut Peanut

Peanut


Peanut

Peanut

Peas Peas

Peas

Peas

Peas

Pepper seeds Pepper seeds

Pepper seeds


Pepper seeds

Pepper seeds

Pepperoni

Pepperoni

Pizza dough

Pizza dough

Potato

Potato

Prune

Prune

Pumpkin seed

Pumpkin seed


Radish Radish

Radish

Raspberry

Raspberry

Raspberry seed Raspberry seed

Raspberry seed

Red bean

Red bean


Red bean

Red bean

Red bell pepper

Red bell pepper

Rice

Rice

Sausage

Sausage

Sesame seed Sesame seed

Sesame seed


Shrimp Shrimp

Shrimp

SPAM SPAM

SPAM

Spinach Spinach

Spinach


Strawberry seed

Strawberry seed

Strawberry

Strawberry

Tomato seed Tomato seed

Tomato seed

Sliced white bread

Sliced white bread

Whole wheat dough

Whole wheat dough

Virtual slides
N/A
Cytology description
N/A
Cytology images
N/A
Immunofluorescence description
N/A
Immunofluorescence images
N/A
Positive stains
N/A
Negative stains
N/A
Electron microscopy description
N/A
Electron microscopy images
N/A
Molecular / cytogenetics description
N/A
Molecular / cytogenetics images
N/A
Videos
N/A
Sample pathology report
  • Skin and soft tissue, right third digit, excision:
    • Foreign body reaction (see comment)
    • Comment: No evidence of malignancy.

  • Mucosal lip, biopsy:
    • Foreign body consistent with hyaluronic acid injection (see comment)
    • Comment: No evidence of malignancy.
Differential diagnosis
  • Epidermal inclusion cyst / epidermoid cyst:
    • Smooth, freely movable on palpation often with dilated punctum
    • Lined by stratified squamous epithelium
    • Lumen contains abundant keratin flakes
    • Granulomatous inflammation contains shards of keratin flakes
  • Pilomatrixoma:
    • Benign tumor arising from hair matrix
    • Solid nests of basaloid cells undergoing abrupt trichilemmal type keratinization
    • Ghost cells, calcification or ossification with extramedullary hematopoiesis and frequent rupture leading to giant cell reaction
  • Sarcoidosis:
    • Exclude foreign material and infection (e.g., with microscopic examination with polarized light and stains for microorganisms)
  • Rheumatoid nodule:
    • Seen on extensor sites, also over the spinous process of vertebrae
    • Subcutaneous
    • Central fibrinoid necrosis, surrounded by histiocytes and perivascular lymphocytes and neutrophils
    • Age of onset: children
  • Granuloma annulare:
    • Intradermal
    • Palisaded or interstitial granulomatous inflammation with mucin
    • Minimal epidermal changes
  • Gouty tophi:
    • Most commonly involving the great toe metatarsophalangeal (MTP) joints
    • Tophi are composed of needle shaped aggregates of urate crystals with surrounding foreign body giant cell reaction
    • Urate crystals dissolve with routine processing, so fix a smear of crystals in absolute alcohol or nonaqueous fixation
    • Under polarized light microscopy, they have a needle-like morphology and strong negative birefringence
  • Calcinosis cutis:
    • Dystrophic calcification at sites of microtrauma or inflammation
    • Lesions are composed of calcium hydroxyapatite
    • Multiple subcutaneous papules or nodules that are yellow-white
    • Irregular deposits of intensely basophilic acellular material in the dermis and subcutaneous tissue
    • Basophilia is so strong that the appearance is of a deep purple
    • Deposits are typically well circumscribed with a thin rim of eosinophilic hyalinization and frequently with a host giant cell reaction
  • Palisaded neutrophilic granulomatous dermatitis (JAAD Case Rep 2017;3:425):
    • Typically associated with underlying disease states, including autoimmune connective tissue disease, lymphoproliferative disorders and infections
    • Most common in patients with rheumatoid arthritis
    • Can be seen in patients with systemic lupus erythematosus and systemic vasculitis
    • Microscopy
      • Early lesions present with neutrophilic infiltrates and leukocytoclastic vasculitis
      • Fully developed lesions feature palisaded granulomas with collagen trapping and neutrophil remnants
  • Necrobiosis lipoidica:
    • Atrophic, yellow depressed plaques, usually on legs of diabetic patients
    • Layered inflammatory process and alternating zones of necrobiosis involving the full thickness of the dermis
    • Changes tend to become more pronounced deeper in the dermis and may extend into the septal panniculus
    • Areas of necrobiosis are poorly defined and run into each other with broad foci of inflammatory infiltrate intervening
    • Variable histiocytic infiltrate with multinucleated giant cells surrounds these areas
    • Accompanying inflammatory infiltrate is predominantly lymphocytic with plasma cells and occasional eosinophils
  • Amyloidosis:
  • Parasites:
    • Infection
Board review style question #1

A 38 year old man presents with small and painful subcutaneous nodules on the left fourth digit. The lesion is biopsied and shown above. What is the best diagnosis?

  1. Epidermal inclusion cyst
  2. Foreign body giant cell reaction
  3. Pilomatrixoma
  4. Osteoma cutis
Board review style answer #1
B. Foreign body giant cell reaction. Answer A is incorrect because a giant cell reaction surrounding shards of keratin should be seen with an inclusion cyst. Answer C is incorrect because pilomatrixomas have islands of basaloid cells and ghost or shadow cells. Answer D is incorrect because bone is not seen.

Comment Here

Reference: Foreign materials / food
Board review style question #2

A 56 year old woman presents with a progressively painful and enlarging bumps on her lips. A lesion is biopsied and shown above. What is the best diagnosis?

  1. Cutaneous mucinosis
  2. Granuloma annulare
  3. Hyaluronic fillers
  4. Metastatic mucinous adenocarcinoma
Board review style answer #2
C. Hyaluronic fillers. Answer A is incorrect because cutaneous mucinosis or focal dermal mucinosis is a collection of mucin between dermal collagen bundles. Answer D is incorrect because metastatic mucinous adenocarcinoma should have atypical glands or nests of epithelial cells which appear to be floating in copious mucin. Answer B is incorrect because granuloma annulare should reveal palisading of histiocytes around a focus of necrobiosis and increased mucin deposition.

Comment Here

Reference: Foreign materials / food

Foreign materials / food (pending)
[Pending]

Ganglioneuroma
Definition / general
  • Benign mesenchymal polyp in the colon
  • Unrelated to lesion of the same name in the adrenal gland
Essential features
  • Uncommon colonic mesenchymal polyp with prominent ganglion cells
  • Associated with Cowden syndrome, especially if multiple polyps
Sites
  • Colorectum is most common site in gastrointestinal tract
Clinical features
Diagnosis
  • Typically found during screening colonoscopy
Case reports
Treatment
  • Polypectomy generally curative
Clinical images

Images hosted on other servers:

Colonoscopy

Gross description
  • Usually small (a few millimeters) but can rarely grow to 2 - 3 cm
Microscopic (histologic) description
  • Expansion of the lamina propria by bland spindled mesenchymal cells (Schwann cells), lacking atypia or mitotic figures
  • Ganglion cells present within the expanded region; may be rare or abundant
Microscopic (histologic) images

Contributed by Christopher Hartley, M.D. and Case #305


Ganglioneuroma

Positive stains
Sample pathology report
  • Transverse colon, polypectomy:
    • Ganglioneuroma
Differential diagnosis
Board review style question #1
A 60 year old patient undergoes colonoscopy and is found to have a solitary ganglioneuroma. What should be the next step in working up this patient?

  1. Colectomy for possible ganglioneuromatosis
  2. Follow up colonoscopy in 1 year
  3. Genetic screening for Cowden syndrome
  4. Imaging to search for ganglioneuromas at other sites
  5. No further workup needed
Board review style answer #1
E. No further workup needed

Comment here

Reference: Ganglioneuroma

Ganglioneuromatosis
Definition / general
  • Benign hamartomatous proliferation involving the colonic muscularis propria
  • Often associated with MEN IIb (RET mutations) and neurofibromatosis type 1 (NF1 mutations) (Am J Surg Pathol 1994;18:250, Gut 1999;45:143)
Essential features
  • Hamartomatous increase of glial and neuronal elements in the muscularis propria
  • Has syndromic associations
  • Must be distinguished from solitary ganglioneuromas and polypoid ganglioneuromatosis
Terminology
  • Also called transmural intestinal ganglioneuromatosis or diffuse intestinal ganglioneuromatosis
Clinical features
  • Patients may have diarrhea or intestinal pseudo-obstruction
Radiology description
Case reports
Clinical images

Images hosted on other servers:

Small bowel follow through showing luminal narrowing


Gross description
  • Poorly demarcated mural thickening, up to 17 cm in length, which can distort surrounding tissue
Microscopic (histologic) description
  • Proliferation of neural cells and ganglion cells in muscularis propria and sometimes in other layers
  • No evidence of malignancy
Microscopic (histologic) images

Contributed by Raul S. Gonzalez, M.D.

Mucosal disease

Mural disease

Mucosal disease

Ganglion and
spindled
proliferation in
muscularis propria

Mural disease with numerous ganglion cells



Case #305

Solitary ganglioneuroma



Images hosted on other servers:

Diffuse ganglioneuromatosis

Positive stains
Molecular / cytogenetics description
Sample pathology report
  • Rectum, resection:
    • Segment of large bowel with extensive ganglioneuromatosis, extending to distal resection margin (see comment)
    • Proximal resection margin unremarkable.
    • Negative for malignancy.
    • Three benign lymph nodes.
    • Comment: Colorectal ganglioneuromatosis is often seen in the setting of a clinical syndrome, including MEN IIb and neurofibromatosis type 1.
Differential diagnosis
  • Crohn's disease:
    • May show neural hyperplasia but mucosal and mural inflammation are also present
  • Intestinal neuronal dysplasia:
    • Controversial entity harboring increased ganglion cells; no mural thickening
  • Polypoid ganglioneuroma:
    • Small, solitary / few, mucosa based, nonsyndromic (Case #305)
  • Ganglioneuromatous polyposis:
    • More than 20 polypoid mucosa based lesions, with variability in ganglionic and neural content; associated with Cowden syndrome / PTEN mutations
Board review style question #1
Colonic ganglioneuromatosis is most common in which of the following syndromes?

  1. Cowden syndrome
  2. Familial schwannomatosis
  3. Lynch syndrome
  4. Neurofibromatosis type 1
Board review style answer #1
D. Neurofibromatosis type 1

Comment Here

Reference: Ganglioneuromatosis

Gardner syndrome
Definition / general
Essential features
  • Features of FAP and extraintestinal lesions, including desmoid tumors, bone osteomas and fibromas
  • Due to a mutation in the APC tumor suppressor gene
Terminology
  • Rarely also called Plenk-Gardner syndrome
Epidemiology
  • Incidence of 1 per million in U.S.
Sites
  • Colonic adenomatous polyps plus extraintestinal features, including multiple osteomas (skull, mandible and long bones), epidermal cysts, desmoid fibromatosis (10%, usually arise after intra-abdominal surgery, aggressive and may cause death), fibromas, lipomas, impacted and supernumerary teeth, dental cysts and congenital hypertrophy of retinal pigment epithelium
Radiology images

Images hosted on other servers:

Abdominal CT showing
desmoid fibromatosis
in Gardner syndrome
patient

Case reports
Gross images

Images hosted on other servers:

Innumerable colon polyps from Gardner patient

Desmoid fibromatosis

Board review style question #1
Gardner syndrome is caused by a germline mutation in what gene?

  1. APC
  2. BRAF
  3. MLH1
  4. STK11
Board review style answer #1
A. APC

Comment Here

Reference: Gardner syndrome

Gastric heterotopia
Definition / general
  • Gastric tissue aberrantly present in colon
  • See also heterotopia topic
Essential features
  • Rare finding that usually consists of oxyntic type gastric tissue
  • Helicobacter pylori organisms may be present
Epidemiology
  • < 50 cases reported
Sites
Clinical features
  • Mean age 18 years but wide range
  • Slight male predominance
  • Associated with other heterotopias, with vertebral and digital anomalies
  • In rectum, usually presents with rectal bleeding and tenesmus (J Clin Gastroenterol 1994;19:41)
  • At colonoscopy, may appear as polyp, ulcer or diverticulum
Case reports
Treatment
  • Excision, H2 blockers
Microscopic (histologic) description
  • Gastric tissue (typically oxyntic type but may be pyloric type) in colon
  • Helicobacter pylori organisms may be present
Microscopic (histologic) images

Contributed by Raul S. Gonzalez, M.D.
Gastric heterotopia

Gastric heterotopia



Images hosted on other servers:

Gastric and colonic mucosa

Gastric and colonic mucosa

Heterotopic gastric mucosa with cystic dilatation

Sample pathology report
  • Rectum, mass, biopsy:
    • Gastric heterotopia (see comment)
    • Comment: Helicobacter organisms are not identified by immunohistochemistry.
Board review style question #1

Which of the following is true about gastric heterotopia in the colorectum?

  1. It can harbor Helicobacter pylori
  2. It is commonly observed endoscopically but rarely biopsied
  3. It is usually pyloric type rather than oxyntic type
  4. It often progresses to malignancy
Board review style answer #1
A. It can harbor Helicobacter pylori

Comment Here

Reference: Gastric heterotopia

Gastrointestinal stromal tumor
Definition / general
  • Tumors that differentiate along lines of interstitial cells of Cajal, the gut's pacemaker cells (Mod Pathol 2003;16:366)
Essential features
  • Mesenchymal spindle cell neoplasm caused by KIT or PDFRGA mutations
  • Risk stratification is based on location, size and mitotic rate
  • Very rare in colon; rectal GISTs account for roughly 5% of all GISTs
Terminology
  • Leiomyoblastoma and gastrintestinal autonomic nerve tumor (GANT) are old terms no longer used
Sites
Clinical features
Prognostic factors
  • Risk of disease progression for rectal GISTs depends on tumor size and mitotic rate (Semin Diagn Pathol 2006;23:70)
    • If mitotic rate is < 5 per 5 square mm, risk of progression is 0% (< 2 cm), 8.5% (> 2 to < 5 cm) or 57% (> 10 cm) (insufficient data for > 5 to < 10 cm)
    • If mitotic rate is > 5 per 5 square mm, risk of progression is 54% (< 2 cm), 52% (> 2 to < 5 cm) or 71% (> 10 cm) (insufficient data for > 5 to < 10 cm)
  • There is insufficient data for progression estimation in colonic GISTs
Case reports
  • Man in mid 40s with GIST of the transverse colon
  • 65 year old man with prostatic stromal sarcoma and rectal GIST (Urology 2006;68:672.e11)
  • 71 year old man with a large rectal adenoma and interstitial cell of Cajal hyperplasia (Case #428)
Treatment
  • Tyrosine kinase inhibitors, including imatinib mesylate (first line) and sunitinib malate
Gross description
  • Often large, bulky, intramural masses
  • Fish flesh or tannish brown parenchyma with hemorrhage, necrosis and cystic softening
Microscopic (histologic) description
  • Mesenchymal tumor usually centered in the muscularis propria
  • Spindle cell GISTs: intersecting fascicles of plump spindled cells with eosinophilic cytoplasm within variably hyalinized or edematous stroma
  • Epithelioid GISTs: rounded epithelioid cells with pseudo compartmental organization
  • Skeinoid fibers (extracellular collagen globules) may be seen
  • Muscle infiltration is common but not predictive of behavior
  • Rarely has osteoclast-like giant cells (Arch Pathol Lab Med 2004;128:440)
Microscopic (histologic) images

Contributed by Raul S. Gonzalez, M.D.

Rectal GIST



Images hosted on other servers:

Epithelioid cells and osteoclast-like giant cells

Spindled cells

KIT+

CD34+

PDGFRA+

Positive stains
Negative stains
Electron microscopy description
  • Processes or cell bodies with intermediate filaments showing solitary focal densities
  • Attachment plaques with incomplete lamina, rare myofilaments and smooth endoplasmic reticulum (Ultrastruct Pathol 2002;26:269)
Molecular / cytogenetics description
  • 80% have mutations in KIT, which encodes CD117; exons affected include 11, 9, 13 and 17 (Nat Rev Cancer 2011;11:865)
  • 10% have mutations in PDGFRA; exons affected include 18, 14 and 12; these GISTs are often gastric and epithelioid
  • Both mutations have been identified in small incidental GISTs, affirming their importance
  • Up to 10% have neither mutation and are associated with syndromes such as neurofibromatosis type 1 and Carney's triad
Sample pathology report
  • Ascending colon, resection:
    • Gastrointestinal stromal tumor (4.3 cm) (see synoptic report and comment)
    • Comment: The tumor is positive for KIT and DOG1 by immunohistochemistry. The mitotic rate is 2 per 5 square mm. Risk stratification for colonic GISTs has not been established but the risk in this case is likely relatively low (an analogous rectal GIST would have a risk of progression of 8.5%).
Differential diagnosis
Board review style question #1
What is the most common site for gastrointestinal stromal tumors in the large intestine?

  1. Appendix
  2. Cecum
  3. Rectum
  4. Transverse colon
Board review style answer #1
Board review style question #2
What 2 factors are used to estimate risk of progression in gastrointestinal stromal tumors?

  1. Ki67 index and necrosis
  2. Ki67 index and size
  3. Mitotic rate and necrosis
  4. Mitotic rate and size
Board review style answer #2
D. Mitotic rate and size

Comment Here

Reference: Gastrointestinal stromal tumor

Graft versus host disease
Definition / general
  • Complication of allogeneic stem cell transplantation in which T cells from the donor recognize the recipient tissue as foreign, leading to organ damage
  • May also occur in some autologous stem cell transplant patients (Mod Pathol 2011;24:117)
Essential features
  • Complication of stem cell transplantation in which T cells from the donor recognize the recipient tissue as foreign, leading to organ damage
  • Histologically characterized by crypt apoptosis, crypt dropout and ulceration
  • Infection (especially cytomegalovirus) and drug induced injury are the most common differential considerations
ICD coding
  • ICD-10: D89.813 - graft versus host disease, unspecified
Epidemiology
Sites
  • Skin > gastrointestinal tract > liver
  • Other sites: mouth, eyes, lungs
Pathophysiology
  • Tissue injury occurs following myeloablative or conditioning chemotherapy regimens in preparation for stem cell transplantation and leads to antigen upregulation on host tissues
    • Donor T cells recognize host tissue antigens as foreign
    • T cell activation and cytokine storm lead to end organ damage (Am J Clin Pathol 2016;145:591)
  • Alteration of the microbiome is likely a contributing factor (Biol Blood Marrow Transplant 2018;24:1322)
  • In autologous stem cell transplant patients, it is hypothesized to result from failure of self tolerance (Mod Pathol 2011;24:117)
Etiology
  • Risk factors: human leukocyte antigen mismatch, age, sex disparity, stem cell source and intensity of the conditioning regimen (N Engl J Med 2017;377:2167)
Clinical features
  • Clinically divided into classic acute graft versus host disease (occurring within 100 days of transplant), persistent / recurrent / late onset acute graft versus host disease (occurring > 100 days posttransplant) and chronic graft versus host disease (defined by presence of diagnostic clinical signs and symptoms; no specific features are diagnostic of chronic colonic GVHD)
  • Gastrointestinal symptoms: dysphagia, anorexia, nausea, vomiting, diarrhea (watery to mucoid / bloody), weight loss and failure to thrive (Nat Rev Gastroenterol Hepatol 2017;14:711)
  • Coexisting skin graft versus host disease is often present, causing diffuse maculopapular rash
  • Endoscopic findings range from edema and mild erythema to mucosal erosions, ulceration and denudation (Nat Rev Gastroenterol Hepatol 2017;14:711)
Diagnosis
  • Combination of clinical, endoscopic and histologic features
  • Biopsy is essential when the clinical presentation is atypical and helps exclude alternative diagnoses
Radiology description
Prognostic factors
Case reports
Treatment
  • Increased immunosuppression
Clinical images

Contributed by Gustavo Moreno, M.D. and Catherine E. Hagen, M.D.

Endoscopic

Microscopic (histologic) description
  • Characteristic histologic features include crypt apoptosis, crypt dropout and ulceration (Biol Blood Marrow Transplant 2015;21:589)
  • Inflammation is generally sparse
  • Endocrine cell nests may be seen (Am J Surg Pathol 2013;37:1319)
  • Fibrosis and crypt architectural changes are markers of longstanding disease but not specific features of chronic graft versus host disease (Am J Clin Pathol 2016;145:591)
  • Apoptosis in the colon is not specific to graft versus host disease and the lower diagnostic threshold is debated
  • Utility of histologic grading is questionable because of poor correlation with patient outcome
    • When grading is performed, the Lerner system is most commonly utilized
      • Grade I: crypt apoptosis without crypt dropout
      • Grade II: single crypt dropout
      • Grade III: contiguous crypt dropout
      • Grade IV: diffuse crypt dropout with ulceration
  • Graft versus host disease in autologous stem cell transplant patients shows more prominent crypt apoptosis but is otherwise histologically identical to allogeneic graft versus host disease (Mod Pathol 2018;31:1619)
  • Histologic examination of at least 8 serial sections is recommended to avoid missing rare apoptotic bodies (Biol Blood Marrow Transplant 2015;21:589)
Microscopic (histologic) images

Contributed by Gustavo Moreno, M.D. and Catherine E. Hagen, M.D.

Apoptosis

Apoptosis, no crypt loss

Ulceration

Crypt dropout


Architectural distortion

Crypt abscess

Indeterminate

Endocrine cell aggregates

Virtual slides

Images hosted on other servers:

GVHD, bone marrow
transplant for acute
myeloid leukemia

GVHD, colon

GVHD, bone marrow transplant with diarrhea

Negative stains
  • Cytomegalovirus IHC, PAS and Giemsa negative for microorganisms (though their presence does not rule out coincident graft versus host disease)
Sample pathology report
  • Colon, biopsy:
    • Colonic mucosa with increased crypt apoptosis and crypt dropout (see comment)
    • Comment: Given the patient's history of allogeneic stem cell transplant, the findings likely represent graft versus host disease.
Differential diagnosis
Board review style question #1

A 55 year old man undergoes peripheral blood stem cell transplant for multiple myeloma. 70 days after the transplant, he complains of mucoid to bloody diarrhea 3 - 5 times per day. Colonoscopy shows multiple ulcerations and biopsies are obtained. What is the most likely diagnosis?

  1. Chemotherapy induced injury
  2. Crohn's disease
  3. Graft versus host disease
  4. Ischemic colitis
  5. NSAID related injury
Board review style answer #1
C. Graft versus host disease

Comment Here

Reference: Graft versus host disease
Board review style question #2
What is the most characteristic histologic feature of colonic graft versus host disease?

  1. Architectural distortion
  2. Crypt apoptosis
  3. Eosinophilic inflammation
  4. Lamina propria hyalinization
  5. Neutrophilic inflammation
Board review style answer #2
B. Crypt apoptosis

Comment Here

Reference: Graft versus host disease

Granulomatous colitis
Etiology
  • Due to Crohn disease, tuberculosis, Yersinia
Case reports
Microscopic (histologic) images

Contributed by Chungja C. Shim, M.D.

Cause unknown

Differential diagnosis
  • Normal germinal centers, tangential sections of blood vessels or pericryptal fibroblastic sheath, inflammatory reaction to extravasated mucin, rarely sarcoidosis (Am J Gastroenterol 1998;93:1949)

Grossing & features to report
Definition / general
  • This topic describes how to gross specimens obtained from polypectomy and colectomy procedures
  • Essential clinical history: clinical diagnosis, procedure performed, prior biopsy or procedure
Grossing - polypectomy
  • Describe: size, color, configuration (sessile or pedunculated), single or multiple pieces (piecemeal polypectomy)
  • Ink the base or stalk margin if it can be identified
    • If stalk present: measure length and diameter of stalk, apply ink to base of stalk
    • If stalk not present: look for pale tissue at base of polyp, apply ink to this area
  • Submit in toto (Am J Clin Pathol 2001;116:336)
    • Small polyp: bisect perpendicular to the stalk or base
    • Large polyp: for polyps with wide heads not able to fit in a cassette, trim the sides away from the stalk; take section through surgical margin of stalk, submit in one cassette and put the sides in other cassette(s)
    • Sectioning protocol based on polyp size (see table 1)
Grossing - endoscopic mucosal resection (EMR) or endoscopic submucosal resection (ESD)
  • Orientation and ink
    • If unoriented, ink the deep and peripheral margins with one color
    • If oriented, ink the deep margin and peripheral margins with multiple colors (similar to inking a skin eclipse)
  • Describe: lesion present or not, size, type, color, shape, borders, distance to all margins
  • Identify the closest peripheral margin and section along that axis at 2 mm intervals, submit in toto
Grossing - colectomy
  • Identify the anatomical subsites of the resected bowel and lesion location (see diagram 1 and table 2)
  • Segmental resection of isolated ascending, descending or sigmoid colon often cannot be distinguished by gross examination
  • Thus, proper documentation by surgeons is the key
Grossing - colectomy with no tumor
  • Identify and record the subsites of colon
    • Open the bowel along the antimesenteric border using scissors
    • Clean the lumen by gently washing out fecal material
    • Fix in formalin overnight (ensure the specimen is immersed in an adequate volume of clean formalin)
    • Measure and record the dimensions (length and circumferences), length of mesentery and size of omentum if present
    • Describe and record any abnormal findings
  • Sections to submit:
    • Sections to submit depend on the pathological findings (IBD, ischemia, volvulus, diverticular disease, arteriovenous malformation, etc.)
    • Abnormal areas by taking sections perpendicular to mucosal folds (through bowel wall)
    • Resection margins
    • Representative sections of other organs present (appendix, terminal ileum, etc.)
    • At least one representative section from grossly uninvolved colon
    • Lymph nodes
      • Representative lymph nodes (usually one cassette) suffices
Grossing - colectomy with tumor
  • Identify and record the subsites of colon
  • Identify the location of the lesion(s) by palpation
  • Open the bowel along the uninvolved colonic wall using scissors
  • Clean the lumen by gently washing out fecal material
  • Ideally, pin the colon on a flat board; immerse in clean formalin, at least overnight
  • Measure and record the dimensions of the specimen (length and circumferences), length of mesentery and size of omentum if present
  • Measure and describe the lesion(s)
  • Measure and record the distance of lesion(s) from margins
  • Identify and ink the closest mesenteric margin (see Arch Pathol Lab Med 2009;133:1539 and diagram 3)
  • Identify and ink the colonic radial resection margin if present
    • Radial (retroperitoneal) margins are present in the proximal ascending colon and the descending colon
  • Identify where tumor is close to or extending through a serosal surface and ink any suspicious serosal area with a different color
    • Suspicious areas are those areas of serosa that are roughened, granular or appear hypervascular
  • Inspect the entire bowel for polyps, other lesions or any abnormality
  • All lymph nodes must be found and submitted in their entirety (must record whether nodes are bisected or trisected and submitted in one block or in multiple blocks)
    • Best achieved by removing the fat close to the bowel wall and then using inspection and palpation to identify nodes
    • Important to examine the fat that remains adhered to the bowel wall, as this is often a location for small nodes
    • At least 12 lymph nodes are needed for accurate staging (Am J Surg Pathol 2002;26:179)
    • For more tips of lymph node searching, see Grossing & features to report - lymph nodes
Grossing - total mesorectal excision (TME) specimens
  • Identify anatomical landmarks and location of tumor by palpation
    • Peritoneal reflection is low on the anterior aspect but high on the posterior aspect; the nonperitonealized tissue distal to the reflection is the radial resection margin
    • Note that tumors in the upper / proximal rectum will have a serosal covering anteriorly and laterally and a radial margin posteriorly; mid to low / distal rectal tumors have a circumferential radial resection margin beneath the anterior peritoneal reflection
  • Photograph in fresh state (useful for correlation with imaging and documentation of completeness of mesorectum)
    • Anterior aspect
    • Posterior aspect
    • Other findings (e.g. significant defects)
  • Open the specimen along the anterior aspect from the top and the bottom, leaving the bowel intact at a level just above and just below the tumor
  • Place loose gauze (soaked in formalin) into the unopened ends of the bowel
  • Fix all rectal cancer specimens for 72 - 96 hours in adequate volume of formalin
  • Measure and record the dimensions of the specimen (length and circumferences)
  • Evaluation of mesorectum completeness
    • Bulk: good, moderate, little
    • Surface: smooth or irregular
    • Defect (record the depth of defect and extension)
      • None > 5 mm
      • > 5 mm but no visible muscularis propria
      • Down to muscularis propria
    • Coning: present or absent
    • Completeness of the mesorectum is scored according to the worst area
      • Complete (Cancer 2009;115:3400, J Clin Pathol 2007;60:849) (see gross images)
        • Intact bulky mesorectum with a smooth surface
        • Only minor irregularities of the mesorectal surface
        • No surface defects greater than 5 mm in depth
        • No coning towards the distal margin of the specimen
        • After transverse sectioning, the circumferential margin appears smooth
      • Nearly complete (Cancer 2009;115:3400) (see gross images)
        • Moderate bulk to the mesorectum
        • Irregularity of the mesorectal surface with defects greater than 5 mm but none extending to the muscularis propria
        • No areas of visibility of the muscularis propria except at the insertion site of the levator ani muscles
      • Incomplete (Cancer 2009;115:3400, J Clin Pathol 2007;60:849) (see gross images)
        • Little bulk to the mesorectum
        • Defects in the mesorectum down to the muscularis propria
        • After transverse sectioning, the circumferential margin appears very irregular
  • Paint the bare area below the peritoneal reflection with ink
  • Describe any other organs / tissues present as appropriate (e.g. vagina, prostate, bladder, etc.)
  • Slice through the unopened bowel at 3 - 5 mm intervals
  • Photograph (see above):
    • All rings laid out sequentially from proximal to distal
    • Close up photographs of any ring(s) to show significant findings (e.g. defects, tumor close to radial margin, etc.)
  • Inspect slices to record:
    • CRM (circumferential radial margin): smooth, regular, moderately irregular, very irregular
    • Extent of tumor
    • Closest distance of tumor to the CRM and whether it is anterior, posterior or lateral
    • Obviously positive nodes and the distance of any positive node to the CRM
    • Examine fat away from tumor for lymph nodes
    • Inspect the entire bowel for polyps, other lesions or any abnormality
  • Sections to submit:
    • Tumor (entire tumor if 5 sections or less or 1 section per cm diameter)
      • Deepest point of invasion (at least 2 sections)
      • Relationship to radial margin (at least 2 sections; may be the same as sections showing deepest invasion)
      • Relationship to serosa (at least 2 sections)
      • If no obvious tumor following neoadjuvant treatment (Biomed Res Int 2015;2015:574540):
        • Take at least 5 blocks from scarred area, including deepest point, relationship to radial margin and relationship to serosa
        • If no viable tumour cells are identified microscopically, submit the entire scarred area
        • If no viable tumour cells are found after submitting the entire scarred area, cut through at least three levels for each block
        • If no viable tumour cells are present after the above steps, report as complete pathologic response or ypT0
    • Longitudinal resection margins
    • Interface with uninvolved bowel
    • Relationship with other organs (if present)
    • Uninvolved bowel
    • Appendix, terminal ileum, cecum and ileocecal value, if present
    • Any other abnormalities
    • All the lymph nodes submitted in their entirety (must record whether nodes are bisected or trisected in 1 block or in multiple blocks)
Features to report for adenocarcinoma
Colonic biopsy - mandatory to report
  • Biopsy site
  • Histologic type
Colonic biopsy - recommended but not required to report
  • Tumor size
  • Histologic grade
  • Depth of invasion (if identifiable)
  • Lymphovascular invasion
  • Tumor budding
  • Type of polyp it arises from (if applicable)
Polypectomy - mandatory to report
  • Tumor site
  • Specimen integrity (intact or fragmented)
  • Polyp size
    • Maximum dimensions of intact specimen and intact polyp are mandatory
  • Type of polyp
  • Histologic type
  • Histologic grade
  • Maximum size of invasive component
  • Tumor thickness - vertical dimension
  • Tumor extension
  • Distance of invasive carcinoma from margin / polyp base
  • Involvement of mucosal / lateral margin by invasive carcinoma or dysplasia
  • Lymphovascular invasion
    • Small vessel
    • Large vessel: intramural or extramural
  • Perineural invasion
  • Ancillary studies performed (depending on local protocols)
    • Microsatellite instability, immunohistochemistry for mismatch repair proteins, mutational analysis (required in some places in patients younger than 70, others in all patients)
Polypectomy - recommended but not required to report
  • Aggregated dimensions of fragmented polyp or maximum dimensions of largest piece for fragmented polyp
  • Maximum width of invasion
  • Tumor budding: low, intermediate or high
  • Additional findings
Colorectal resection for tumor - mandatory to report
  • Specimen type
  • Number of tumors (for multiple primary tumors, each tumor requires a complete report)
  • Tumor site(s)
  • Tumor location relative to anterior peritoneal reflection for rectal tumors: entirely above, below or straddling
  • Tumor size
  • Macroscopic tumor perforation
  • Macroscopic intactness of mesorectum (required only for rectal tumors)
  • Histologic type
  • Histologic grade
  • Tumor extension
  • Margins
    • Involved by invasive carcinoma or not
    • Involved by low or high grade dysplasia or not
    • If all margins are negative, specify closest margin and distance of invasive carcinoma from this margin
    • Distance of tumor from CRM (required only for rectal tumors)
  • Lymphovascular invasion: present or not identified
    • Small vessel
    • Large vessel: intramural or extramural
  • Perineural invasion
  • Treatment effect
    • No known presurgical therapy
    • Present
      • Complete response: no viable cancer cells (can have acellular mucin present)
      • Near complete response: single cells or rare small groups of cancer cells
      • Partial response: more than single cells or rare small groups of residual cancer with tumor regression
      • Poor or no response: extensive cancer with no evident tumor regression
  • Tumor deposits (absence / presence and numbers)
  • Regional lymph node: number of examined and number of involved
  • Ancillary studies performed (depending on local protocols)
    • Microsatellite instability, immunohistochemistry for mismatch repair proteins, mutational analysis (required in some places in patients younger than 70, others in all patients)
Colorectal resection for tumor - recommended but not required to report
  • Margin:
    • Rectum cancer: distance of tumor from negative distal margin
  • Tumor budding: number of buds in 1 hotspot field (0.785 mm2)
    • Low score: 0 - 4
    • Intermediate score: 5 - 9
    • High score: 10 or more
  • Type of polyp in which invasive carcinoma arose
  • Additional pathologic findings (e.g. adenoma, ulcerative colitis, Crohn’s disease, diverticulosis, etc.)
  • Checklists: Michigan Cancer Consortium
Gross images

Images hosted on other servers:

Complete and incomplete mesorectum

Sample gross description report

Polypectomy:
  • The specimen is received in a properly labeled container with the patient's identifiers and accession number, designated as sigmoid polyp
  • Specimen: brown polypoid piece(s)
    • Number: 1
    • Size: 1.2 cm
    • Stalk(s): not identified
    • Site(s) of attachment: inked
  • Section code (entirely submitted):
    • (A1) trisected


Colectomy:

Sample 1:
  • The specimen is received in a properly labeled container with the patient's identifiers and accession number, designated as right hemicolectomy
  • Specimen:
    • Terminal ileum: 5.5 x 4.7 cm (length x circumference proximal margin)
    • Appendix: 5.5 x 0.6 cm (length x greatest diameter)
    • Colon: 13.5 x 8.4 cm (length x circumference distal margin)
    • Proximal margin received stapled
    • Distal margin received stapled
    • Pericolic soft tissue: 8.0 cm in depth
  • Number of tumors: 1
  • Location(s): within the cecum at the appendiceal orifice
  • Configuration: polypoid
  • Size: 2.2 x 1.2 x 0.4 cm (length x width x thickness)
  • Distance from longitudinal margins: 4.5 cm; 12.5 cm (proximal; distal)
  • Extension into pericolic / rectal tissue: absent
  • Extension into other organs or tissues: absent
  • Tumor perforation: absent
  • Serosa underlying tumor:
    • Unremarkable
    • Distance of tumor to closest serosa: 0.5 cm
  • Radial margin:
    • Closest distance of direct tumor extension to radial margin: 4.2 cm
  • Lesions in nontumoral mucosa: diverticula
  • Lymph nodes: within the pericolic soft tissue are multiple possible lymph nodes (0.2 - 0.8 cm)
  • Ink code:
    • Colon radial margin: black
    • Serosal surface: blue
    • Proximal and distal resection margins: black
  • Section code (representative sections):
    • (A1 - A5) entire polypoid lesion with closest serosa
    • (A6) closest radial margin
    • (A7) proximal margin
    • (A8) distal margin
    • (A9) appendix
    • (A10) diverticulum
    • (A11 - A13) multiple intact lymph nodes in each
    • (A14 - A16) 1 lymph node in each


Sample 2:
  • The specimen is received in a properly labeled container with the patient's identifiers and accession number, designated as rectosigmoid resection
  • Specimen:
    • Sigmoid colon and rectum 24.5 x 6.0 x 7.5 cm (length x circumference proximal margin x circumference distal margin)
    • Proximal margin received stapled
    • Distal margin received stapled
    • Pericolic and perirectal soft tissue up to 4.5 cm in depth
  • Number of tumors: 1
  • Location(s): sigmoid colon
  • Configuration: annular / circumferential
  • Size: 4.1 x 2.2 x 1.5 cm (length x width x thickness)
  • Distance from longitudinal margins: 13.0 cm; 6.5 cm (proximal; distal)
  • Extension into pericolic / rectal tissue: present
  • Extension into other organs or tissues: absent
  • Tumor perforation: absent
  • Serosa underlying tumor:
    • Puckered with fibrous adhesions
    • Distance of tumor to closest serosa: 0.0 cm
  • Radial margin:
    • Closest distance of direct tumor extension to radial margin: 1.2 cm
    • Closest distance of involved lymph node or tumor nodule to radial margin: 0.0 cm
    • Closest margin is mesenteric (vascular tie)
  • Lesions in nontumoral mucosa:
    • Polyp(s): 2 sessile polyps (0.3 and 0.6 cm) immediately distal to the mass
    • Diverticula
  • Lymph nodes: within the pericolic fat are multiple possible lymph nodes (0.2 - 2.8 cm)
  • Ink code:
    • Colon radial margin: black
    • Serosal surface: blue
    • Proximal and distal resection margins: black
  • Section code (representative sections):
    • (A1 - A3) mass and closest radial margin
    • (A4 - A6) mass and deepest point of invasion
    • (A7 - A8) mass and closest overlying serosa
    • (A8) proximal margin
    • (A9) distal margin
    • (A10) 1 polyp
    • (A11 - A12) 1 polyp, bisected
    • (A13) lymph node abutting radial margin
    • (A14 - A16) multiple intact lymph nodes in each
    • (A17 - A20) 1 lymph node in each
    • (A20 - A21) 1 lymph node
    • (A22 - A25) 1 lymph node


Sample 3:
  • The specimen is received in a properly labeled container with the patient's identifiers and accession number, designated as rectosigmoid resection
  • Specimen:
    • Sigmoid colon and rectum 13.0 x 2.8 x 7.0 cm (length x circumference proximal margin x circumference distal margin)
    • Proximal margin received stapled
    • Distal margin received stapled
    • Pericolic and perirectal soft tissue up to 6.0 cm in depth
    • Mesorectum: good bulk, intact with smooth surface
    • Defects: none > 5 mm
    • Coning: none
  • Number of tumors: 1
  • Location(s):
    • Rectum
      • Tumor at anterior peritoneal reflection
      • Tumor 6.5 cm distal to posterior peritoneal reflection
  • Configuration: annular / circumferential
  • Size: 3.5 x 3.5 x 0.7 cm (length x width x thickness)
  • Distance from longitudinal margins: 8.0 cm; 1.2 cm (proximal; distal)
  • Extension into pericolic / rectal tissue: present
  • Extension into other organs or tissues: absent
  • Tumor perforation: absent
  • Serosa underlying tumor:
    • Unremarkable
    • Distance of tumor to closest serosa: 0.2 cm
  • Radial margin:
    • Closest distance of direct tumor extension to radial margin: 0.4 cm
    • Closest radial margin is left lateral
    • Radial margin on cross section is smooth
  • Lesions in nontumoral mucosa: diverticulum
  • Lymph nodes: multiple possible tan lymph nodes (0.5 to 2.0 cm) identified
  • Ink code:
    • Colon radial margin: black
    • Rectal posterior radial margin: black
    • Rectal anterior radial margin: green
    • Serosal surface: blue
    • Proximal and distal resection margins: black
  • Section code (representative sections):
    • (A1) proximal margin
    • (A2) distal margin with tumor
    • (A3 - A4) tumor, closest to serosa
    • (A5) tumor, closest to radial margin
    • (A6 - A8) tumor, deepest point of invasion
    • (A9) diverticulum
    • (A10 - A13) 3 possible lymph nodes in each
    • (A14) 2 possible lymph nodes
    • (A15 - A17) 1 possible lymph node in each
    • (A18 - A20) multiple possible lymph nodes in each
Diagrams / tables

Table 1: sectioning protocol based on polyp size
Polyp Size
Sectioning Protocol
Smaller than 0.4 cm No sectioning
0.4 - 0.8 cm Bisect; place in 1 cassette
0.9 - 1.2 cm Trisect by shaving two sides off central section with stalk;
place central section in separate cassette
Larger than 1.2 cm More than 3 sections as appropriate



Images hosted on other servers:

Anatomic subsites of colon and rectum (table 2)

Anatomic subsites of the colon

Mesenteric / radial margin


T category staging

Regional lymph nodes

Board review style question #1
You are grossing a subtotal colectomy specimen for a biopsy proven colon adenocarcinoma. You find 50 lymph nodes with 15 of them larger than 1 cm in size. Which of the following will be the most appropriate sections to submit?

  1. All lymph nodes submitted in their entirety
  2. Lymph nodes larger than 1 cm submitted in their entirety
  3. Representative sections of all lymph nodes
  4. Representative sections of lymph nodes larger than 1 cm and in toto of lymph nodes smaller than 1 cm
Board review style answer #1
A. All lymph nodes submitted in their entirety

Comment Here

Reference: Colon - Grossing & features to report
Board review style question #2
You are grossing a TME specimen and found a defect on the mesorectum under the peritoneal reflection down to the muscularis propria. How would you report on the macroscopic evaluation of mesorectum?

  1. Cannot be determined
  2. Complete
  3. Incomplete
  4. Near complete
Board review style answer #2

Hemangioma
Definition / general
  • Uncommon benign vascular proliferation of the colon
Essential features
  • Benign vascular proliferation
  • Often related to a syndrome
  • Typically a cavernous hemangioma (majority) or capillary hemangioma
Sites
  • Most common site in large intestine is rectum
  • More common in small intestine than large intestine
Clinical features
  • Causes bleeding, melena, anemia, rarely intussusception or obstruction
  • Patients are often young and may have a syndrome (blue rubber bleb nevus syndrome, Klippel-Trénaunay syndrome, etc.) (Clin Colon Rectal Surg 2011;24:193)
Case reports
Gross description
  • Well circumscribed reddish lesion
Microscopic (histologic) description
  • Cavernous hemangioma: localized or diffuse; blood filled sinus-like spaces with scant connective tissue, variable smooth muscle; may be infiltrative but no other concerning features
  • Capillary hemangioma: small, closely packed capillaries, rarely multifocal; no features suggestive of malignancy (Dtsch Med Wochenschr 2004;129:1970)
  • Other rare variants with distinctive morphology can occur, such as anastomosing hemangioma (Am J Surg Pathol 2013;37:1761)
Microscopic (histologic) images

Contributed by Ed Uthman, M.D. and Raul S. Gonzalez, M.D.

Incidental submucosal
hemangioma in
sigmoid colectomy

Colonic hemangioma Colonic hemangioma

Colonic hemangioma

Positive stains
Sample pathology report
  • Sigmoid colon, resection:
    • Segment of colon with submucosal cavernous hemangioma (1.4 cm)
    • Margins of resection unremarkable.
    • Six benign lymph nodes.
Differential diagnosis
Board review style question #1
What is the most common subtype of hemangioma encountered in the colon?

  1. Anastomosing hemangioma
  2. Capillary hemangioma
  3. Cavernous hemangioma
  4. Glomeruloid hemangioma
Board review style answer #1
C. Cavernous hemangioma

Comment Here

Reference: Hemangioma

Hepatoid carcinoma
Definition / general
  • Rare, aggressive subtype of adenocarcinoma that morphologically resembles hepatocellular carcinoma and produces alpha fetoprotein
Essential features
  • Extremely rare subtype of colorectal carcinoma
  • Resembles hepatocellular carcinoma morphologically and sometimes immunohistochemically
  • Serum alpha fetoprotein may be elevated
Terminology
  • Originally termed alpha fetoprotein producing tumor
Epidemiology
  • More common in men
  • Fewer than 20 colonic cases reported
Sites
Clinical features
Case reports
Treatment
Microscopic (histologic) description
  • Resembles hepatocellular carcinoma, with trabecular or solid pattern of large polygonal cells with abundant eosinophilic or clear cytoplasm
  • Component of traditional adenocarcinoma also often present
  • Lymphovascular invasion is common (Histopathology 1997;31:47)
Microscopic (histologic) images

Images hosted on other servers:

Collision with liposarcoma

Associated with ulcerative colitis

Gastric tumor: H&E, AFP+ HepPar1- glypican3+

Sample pathology report
  • Sigmoid colon, resection:
    • Hepatoid carcinoma of the colon, moderately differentiated (see synoptic report)
Differential diagnosis
Board review style question #1
What serum marker is often elevated in patients with hepatoid colon carcinoma?

  1. Alpha fetoprotein
  2. CA125
  3. CA19-9
  4. HER2
Board review style answer #1
A. Alpha fetoprotein

Comment Here

Reference: Hepatoid carcinoma

Hirschsprung disease
Definition / general
Essential features
  • Diagnostic: Absent ganglion cells, submucosal nerve hypertrophy, absent calretinin immunoreactive mucosal innervation and increased cholinergic mucosal innervation (AChE histochemistry or ChT immunohistochemistry)
  • Intraoperative:
    • Levelling biopsy: identification of ganglion cells
    • Frozen section of proximal resection margin: exclude features of transition zone (partial circumferential aganglionosis, myenteric hypoganglionosis, submucosal nerve hypertrophy)
Terminology
  • Also called congenital aganglionic megacolon
  • Short segment (~80% of patients): aganglionic portion limited to rectosigmoid colon; M:F ≈ 4:1
  • Long segment (~20% of patients): aganglionic segment extends proximal to the sigmoid colon, may extend into small bowel; M:F approaches 1:1 as length of aganglionic segment increases; some regard long segment as proximal to splenic flexure and “colonic” as aganglionosis extending to somewhere between sigmoid colon and splenic flexure
  • Very (ultra) short segment: aganglionic portion restricted to < 2 cm of the distal most rectum; more common in boys; potentially more difficult to document because this portion of rectum is normally hypoganglionic
  • Total colonic aganglionosis (TCA): uncommon; M:F ≈ 1:1; clinical and genetic differences; rectal biopsy may lack submucosal nerve hypertrophy; more difficult to diagnose and manage (Pediatr Surg Int 2016;32:221)
  • Pan-intestinal aganglionosis: very rare; aganglionosis of entire small and large intestines
  • Skip area (or skip lesion): aganglionic segment is interrupted by a focus or segment of ganglionic bowel; proximal aganglionic portion frequently includes appendix + / - adjacent cecum and distal ileum
  • Zonal colonic aganglionosis: interstitial aganglionic segment flanked proximally and distally by ganglionic bowel; distal rectum innervated normally; pathogenesis (likely secondary disruption as opposed to malformation) considered different from classic Hirschsprung disease
  • Transition zone: ganglionic but neuroanatomically abnormal, bowel located proximal to aganglionic segment; typically < 5 cm in short segment aganglionosis; may be longer in long segment aganglionosis; may cause persistent obstructive symptoms if not resected with aganglionic segment (Am J Surg Pathol 2016;40:1637)
ICD coding
  • ICD-10: Q43.1 - Hirschsprung disease
  • ICD-11: LB16 - Hirschsprung disease
Epidemiology
  • Average: 1 in 5,000 live births (0.75 in Caucasians; 2.8 in Asians) (Clin Genet 2020;97:114)
  • Male predominance for short segment aganglionosis (3 - 4.5:1)
  • ~80% sporadic
  • ~18% have other congenital anomalies; some with specific syndromes
  • ~12% have major chromosomal variants; ~10% associated with Down syndrome
  • About 5% have neurologic abnormalities that may be serious
  • Occasionally associated with intestinal atresia and malrotation, anorectal malformations and other abnormalities of the GI tract
  • 5% have a sibling with disease; variation in risk depending on sex, segment length and familiality
  • Patients usually present as neonates; 80% present before their first birthday but up to 10% of cases are diagnosed in adulthood
Sites
  • Classic Hirschsprung disease invariably includes aganglionosis of the distal rectum and variable length of contiguous bowel
  • Short segment, long segment, total colonic and pan-intestinal forms as defined above
Pathophysiology
  • Absence of enteric ganglia causes a loss of intrinsic excitatory and inhibitory innervation; net effect is spastic contraction of aganglionic segment, including internal anal sphincter, with impaired peristalsis and defecation
  • Bowel proximal to the aganglionic segment undergoes muscular hypertrophy and dilation
  • Abnormal extrinsic innervation (enlarged cholinergic nerves) in aganglionic rectum + / - more proximal aganglionic left colon may contribute to spasticity
  • Numerous alterations in expression of various receptors, channels, cytoskeletal proteins, neurotrophic factors and other molecules described in ganglionic bowel but functional significance unclear (Pediatr Dev Pathol 2020;23:40)
Etiology
  • Proximate cause is failure of neural crest derived neuroglial progenitors to colonize the entire length of the intestinal tract (Dev Biol 2016;417:158)
  • Complex genetic basis with at least 24 susceptibility genes; variable penetrance for most genetic alterations; combinatorial effects of low penetrance gene variants (N Engl J Med 2019;380:1421)
  • Coding variants and large copy number variants associated with highest risk but less frequently implicated than noncoding variants
  • Coding or noncoding mutations RET proto-oncogene appear to play some role in the majority cases
Diagrams / tables

Images hosted on other servers:

Proximal extent of aganglionic segment and corresponding frequency

Clinical features
  • Most common presentation: failure to pass meconium within 48 hours of birth
  • Other findings may include abdominal distension, bilious emesis, explosive defecation in response to manual anal dilatation, bowel perforation and enterocolitis
  • Older children may have chronic constipation and failure to thrive
  • Down syndrome patients more commonly have associated enterocolitis
  • 80% male; usually sporadic (1 per 5,000 live births); 5% have affected sibling
  • 10% have Down syndrome; another 5% have other serious neurologic impairment
  • Complications: proximal innervated colon may become massively distended (15 cm in diameter) with muscular wall hypertrophy and rupture / perforation, usually near cecum or appendix; also acute intestinal obstruction, enterocolitis with fluid and electrolyte imbalance
  • Mortality: currently 5 - 10%
  • Eur J Pediatr Surg 2008;18:140
Diagnosis
  • Screening tests
    • Anorectal manometry - absent rectoanal inhibitory reflex (RAIR, relaxation of internal anal sphincter in response to balloon dilatation of the rectum)
    • Contrast enema - ratio of rectal diameter to sigmoid diameter is < 1
  • Rectal biopsy, either suction or incisional
    • Suction rectal biopsy: most common, does not require sedation in neonate or young infant, samples mucosa and submucosa; no direct visualization of rectum or anal canal; recommend multiple biopsies at different sites (e.g., 2, 3 and 4 cm above the anal verge)
    • Incisional biopsy: requires anesthesia, some prefer this for patients > 1 year of age, may sample myenteric plexus in addition to mucosa and submucosa; direct visualization of dentate line
  • Pediatr Dev Pathol 2020;23:8
Case reports
Treatment
  • Surgical: excision of aganglionic segment and transition zone (ganglionic but neuroanatomically abnormal bowel immediately proximal to the aganglionic segment) with a pullthrough procedure to anastomose ganglionic bowel just superior to the anal canal
  • One stage (resection and anastomosis) or two stage (ostomy then later resection and anastomosis)
  • Levelling biopsy with intraoperative frozen section should be performed during either procedure to insure ganglionic bowel proximal to aganglionic segment is used for anastomosis
    • Biopsy is seromuscular or full thickness
    • Identification of myenteric ganglion cells is the key; rebiopsy of more proximal bowel is necessary until ganglion cells are located
    • 5 - 10 well oriented sections is usually sufficient
    • H&E is satisfactory; some prefer Diff-Quik, Giemsa or toluidine blue
    • Use of Giemsa, Diff-Quik or toluidine blue stains in addition to routine H&E has been advocated to improve diagnostic accuracy at frozen section
    • Presence of ganglion cells in seromuscular biopsy does not exclude transition zone
  • To reduce likelihood of transition zone pullthrough or ostomy, intraoperative frozen section of a full circumference (donut) section from the proximal resection margin (or ostomy site) is recommended (Pediatr Dev Pathol 2020;23:23)
    • Histological features of transition zone include:
      • Partial circumferential aganglionosis (⅛ or more of circumference devoid of ganglion cells)
      • Myenteric hypoganglionosis (⅛ or more of circumference predominantly tiny ganglia with 1 or 2 ganglion cells and minimal neuropil)
      • Submucosal nerve hypertrophy (e.g., > 2 submucosal nerves > 40 µm diameter in a 400x field)
  • Surgery successful in most patients
  • Treatment failures may be due to transition zone pullthrough, strictures at anastomosis site, twists proximal to the anastomosis, excessive retained aganglionic distal rectum, post-surgical loss of ganglion cells proximal to anastomosis or poorly understood physiological abnormalities in the ganglionic bowel
Gross description
  • Aganglionic segment typically narrow with dilated proximal ganglionic bowel and funnel shaped transition near interface
  • Resection specimen from Soave pullthrough procedure lacks muscularis propria distally; sleeve of submucosa and mucosa at distal end
  • Resection specimen from Swenson or Duhamel pullthrough procedure is full thickness throughout
  • Histological sampling of resection should include:
    • Full circumference adjacent to proximal margin (confirm normal neuroanatomy)
    • Full circumference at distal margin (confirm aganglionosis)
    • One or more longitudinal strip of entire resection or closely spaced transverse sections along the length of the resection (establish length of aganglionic segment)
  • J Pediatr Surg 2019;54:2017
Gross images

Contributed by Raj P. Kapur, M.D., Ph.D.

Resection specimen

Histological sampling of resection



Images hosted on other servers:

Dilated bowel

Microscopic (histologic) description
  • Aganglionic segment
    • Complete absence of submucosal and myenteric ganglion cells
    • Submucosal and myenteric nerve hypertrophy in aganglionic rectum; gradually lost in more proximal aganglionic colon; may not be prominent in total colonic aganglionosis; usually assessed subjectively but in neonate nerves > 40 µm in diameter are abnormal; enlarged nerves have extrinsic features including Glut1 positive perineurium but occasional Glut1+ nerves are also present in normal rectum
  • Transition zone (any of the following)
    • Partial circumferential aganglionosis (⅛ or more of circumference devoid of ganglion cells)
    • Myenteric hypoganglionosis (⅛ or more of circumference predominantly tiny ganglia with 1 or 2 ganglion cells and minimal neuropil)
    • Submucosal nerve hypertrophy (e.g., > 2 submucosal nerves > 40 µm diameter in a 400x field)
  • Ganglia in newborns are smaller than in older children and nucleoli and Nissl substance may be lacking, a situation even more pronounced in premature infants
  • Myenteric ganglia in the transition zone may be ectopically situated within the muscularis externa or serosa
  • Associated nerve hypertrophy is a helpful clue but is not diagnostic
  • Active mucosal inflammation (enterocolitis)
  • Stercoral ulcers (sharply demarcated shallow ulcers with mucosal inflammation due to pressure of feces on obstructed colon)
  • Fibromuscular dysplasia of submucosal or seromuscular arteries in aganglionic segment or transition zone (Pediatr Dev Pathol 2018;21:363)
Microscopic (histologic) images

Contributed by Raj P. Kapur, M.D., Ph.D.

Ganglionic biopsy

Aganglionic biopsy

Calretinin: ganglionic biopsy / aganglionic biopsy


Choline transporter IHC: ganglionic biopsy / aganglionic biopsy

Acetylcholinesterase histochemistry: ganglionic biopsy /
aganglionic biopsy

Leveling frozen section biopsy: ganglion cells / aganglionic segment


Myenteric hypoganglionosis

Positive stains
  • Most diagnostically useful are acetylcholinesterase histochemistry (AChE) or choline transporter immunohistochemistry; both demonstrate increased cholinergic innervation in lamina propria and muscularis mucosa
  • AChE requires frozen sections and special methodology (Fetal Pediatr Pathol 2016;35:399)
  • Choline transporter immunohistochemistry: formalin fixation and paraffin embedding; relatively new and has not been validated in many laboratories; may be more challenging to interpret than AChE (Pediatr Dev Pathol 2017;20:308)
Negative stains
  • Calretinin immunohistochemistry
    • Most widely used ancillary method
    • Run routinely with all rectal biopsies to exclude aganglionosis
    • Absence of calretinin staining in muscularis mucosae and lamina propria of aganglionic segment
    • Negative result pertains only to mucosa; submucosal nerves retain calretinin positive neurites are retained in submucosal nerves
    • Immunoreactive mucosal nerves may be present in rectal biopsy from patient with very short segment aganglionosis due to descending fibers from transition zone (Pediatr Dev Pathol 2014;17:28)
  • Many antibodies available to identify neuronal cell bodies (e.g., Hu C/D, Phox2B); seldom used in practice as H&E is sufficient
Electron microscopy description
  • Not clinically relevant
  • Hypertrophic nerves in aganglionic segment resemble extrinsic peripheral nerves (perineurium, endoneurial collagen, non-myelinating Schwann cells)
Molecular / cytogenetics description
  • Complex genetics as described above; see Etiology
  • Formal testing generally not indicated but karyotype, genomic array or mutational analysis may be pursued for syndromic cases (e.g., Waardenburg-Shah syndrome, Down syndrome, multiple endocrine neoplasia type 2A)
Sample pathology report
  • Rectal biopsy
    • Rectum, suction biopsies: findings diagnostic of Hirschsprung disease
      • No identifiable ganglion cells
      • Submucosal nerve hypertrophy
      • Absent calretinin immunoreactive mucosal innervation
  • Pullthrough resection
    • Rectosigmoid colon, Soave resection (12.5 cm): Hirschsprung disease with the following features:
      • Aganglionosis (distal 4 cm)
      • No significant neuromuscular pathology at the proximal surgical margin
  • Resection synoptic report
Differential diagnosis
  • Clinical
    • Chronic idiopathic pseudo-obstruction
      • Clinical condition with heterogeneous etiologies, most affect small and large intestine, some primary (e.g., visceral myopathy; myenteric hypoganglionosis), some secondary (e.g., hypothyroidism)
    • Megacystis microcolon intestinal hypoperistalsis syndrome (MMIHS)
      • Bladder and intestinal dysmotility
      • Typically presents pre- or perinatally
      • Mutations in genes for smooth muscle contractile proteins
    • X-linked intestinal pseudo obstruction
    • Meconium ileus
      • Especially with perforation, consider cystic fibrosis
    • Anatomic intestinal obstruction (e.g., atresia, volvulus, stenosis)
    • Anal achalasia
      • Spasticity of internal anal sphincter with absent RAIR and reduced nitrergic innervation of sphincter muscle (Pediatr Surg Int 2013;29:855)
  • Histological (clinical mimics with different diagnostic findings in rectal biopsy)
    • Intestinal neuronal dysplasia type B
    • Neuroglial hamartoma syndromes (multiple endocrine neoplasia type 2B, neurofibromatosis, PTEN-hamartoma syndromes)
      • Ganglioneuromatous hyperplasia of submucosal and/or myenteric plexuses, + / - mucosal ganglioneuromas
      • Investigate clinical history and extra enteric findings
      • Confirm with molecular genetic testing
    • Primary visceral myopathy
      • Changes rarely diagnostic in muscularis propria of rectal biopsy
      • Key features include muscle atrophy and fibrosis as well as myocyte vacuolar degeneration
    • Mitochondriopathy
      • Changes rarely diagnostic in muscularis propria of rectal biopsy
      • Key features include muscle atrophy and fibrosis as well as eosinophilic megamitochondria in enteric ganglia
Additional references
Board review style question #1

A neonate with Hirschsprung disease, diagnosed by suction rectal biopsy, undergoes primary endorectal pullthrough surgery. During the operation, a leveling seromuscular biopsy from the proximal sigmoid colon is sent for frozen section diagnosis. Which of the following diagnoses and recommendations is appropriate, based on the microscopic field shown in the above image?

  1. No definitive ganglion cell, obtain another biopsy from an upstream location
  2. Ganglion cells present, recommend pullthrough resection at least 5 cm proximal to site of this biopsy
  3. Ganglion cells present, insist on rebiopsy of rectum to confirm Hirschsprung disease before resection
  4. Probably intestinal neuronal dysplasia, recommend additional upstream biopsies to define extent of dysplastic ganglia
  5. Inadequate sample to assess for ganglion cells, rebiopsy
Board review style answer #1
B. Ganglion cells present, recommend pullthrough resection at least 5 cm proximal to site of this biopsy

Comment Here

Reference: Hirschsprung disease
Board review style question #2
A rectal biopsy to exclude Hirschsprung disease may be considered inadequate if it demonstrates

  1. Submucosal nerve hypertrophy
  2. Absent calretinin immunoreactive mucosal innervation
  3. Crowded, coarse, acetylcholinesterase positive mucosal
  4. Transitional mucosa
  5. Absent submucosal ganglion cells
Board review style answer #2
D. Transitional mucosa indicative of distalmost rectum

Comment Here

Reference: Hirschsprung disease

Histoplasmosis
Definition / general
Essential features
  • Invasive mycosis caused by inhalation of the spores of dimorphic fungi Histoplasma capsulatum
  • Distal ileum and cecum are most commonly affected, followed by the rectum and descending colon
  • Tissue culture and histopathology remain the gold standard for the diagnosis of colonic histoplasmosis
  • Small, round to ovoid, narrow based budding yeast cells measuring 2 - 5 micrometers
  • Stains positive for Gomori methenamine silver (GMS) stain and periodic acid-Schiff (PAS); negative for mucicarmine and Fontana-Masson silver stain
Terminology
ICD coding
  • ICD-10:
    • B39.4 - histoplasmosis capsulati, unspecified
    • B39.9 - histoplasmosis, unspecified
Epidemiology
  • Occurs widely but is highly endemic in Ohio, Mississippi river valleys and Latin America
  • Also endemic in Asia, Southeast Asia and India
  • Centers for Disease Control records estimate 50,000 infections yearly (Infect Dis Clin North Am 2016;30:207)
  • Gastrointestinal (GI) infection by Histoplasma is common and a manifestation of disseminated disease
Sites
  • Distal ileum and cecum are most commonly affected, followed by the rectum and descending colon
  • Colon is among the most frequently affected sites in GI (Gastrointest Endosc 2020;91:951)
Pathophysiology
  • Dimorphic fungus exists as a mold in the environment
  • Once inhaled, it enters the lung and transforms into yeast form at 35 - 37 °C
  • Alveolar macrophages phagocytose these yeast forms, which multiply intracellularly until the advent of immune response
  • Primary infection is usually asymptomatic and becomes reactivated in immunocompromised states
  • Usually manifests as acute or chronic pulmonary disease
  • Severe cases: disseminated to the skin, adrenal gland, central nervous system, lymph nodes, spleen, bone marrow and GI tract (Int J Surg Pathol 2017;25:592)
  • In most healthy individuals, nonviable organisms persist in granulomas
Etiology
  • Risk factors: HIV / AIDS, autoimmune disease, old age, organ transplant, hepatitis C virus infection, hematologic malignancies, corticosteroid use, tumor necrosis factor antagonists, primary immunodeficiency syndromes (ACG Case Rep J 2021;8:e00598, BMC Infect Dis 2013;13:143)
  • Most commonly linked to farming, exposure to chicken coops or caves, remodeling or demolition of old buildings and cutting down trees or clearing brush from sites in which blackbirds have roosted (Infect Dis Clin North Am 2016;30:207)
Clinical features
Diagnosis
  • Multifaceted approach: history, clinical features, radiographic / endoscopic findings, blood, stool, tissue culture, antigen and antibody testing and molecular testing
  • The European Organization for Research and Treatment of Cancer / Invasive Fungal Infections Cooperative Group and the National Institute of Allergy and Infectious Diseases Mycoses Study Group (EORTC / MSG) criteria for the diagnosis of invasive fungal infections:
    • Proven diagnosis: confirmation by either histopathology or culture
    • Probable diagnosis: the presence of an appropriate clinical presentation, a predisposing condition and mycological evidence, such as the presence of antigenuria (J Clin Microbiol 2017;55:1612)
  • Tissue culture and histopathology remain the gold standard for the diagnosis of colonic histoplasmosis (Case Rep Med 2018;2018:2723489, J Fungi (Basel) 2020;7:12)
    • Slow, takes 4 - 6 weeks
Laboratory
  • Culture and microbiology stains:
    • Histoplasma capsulatum stains poorly for Gram stain
    • Calcuflour white binds to chitin in fungal cell walls and helps in identification
    • Colonies show septate hyphae followed by smooth or less commonly spiny microconidia (2 - 5 micrometers in size) and characteristic tuberculate macroconidia (7 - 15 micrometers)
    • Small, round to oval, narrow based budding yeast forms at 37 °C
  • Antigen detection:
    • Galactomannan antigen testing in body fluids: rapid test (Infect Dis Clin North Am 2016;30:207)
    • Sensitivity of enzyme immunoassay (EIA) is high in patients with disseminated disease (95%)
    • Urine antigen detection: more sensitive than serum
    • EIA's quantitative nature also allows for sequential monitoring of antigen clearance
    • Crossreactivity can be seen with Blastomyces dermatitidis, Paracoccidioides brasiliensis, T. marneffei and less commonly, Coccidioides immitis and Coccidioides posadasii
  • Antibody detection:
    • Antibodies detectable after 4 - 8 weeks
      • More suitable for diagnosis of subacute or chronic histoplasmosis
    • A titer of ≥ 1:32 or 4 fold rise in antibody titers from acute to convalescent phase is strongly suggestive of acute infection
  • Molecular methods: polymerase chain reaction (PCR) assays
Radiology description
  • Bowel wall thickening, mass lesions, signs suggesting bowel obstruction, free peritoneal air (J Med Microbiol 2010;59:610)
  • Other findings of disseminated disease: hepatosplenomegaly, pulmonary infiltrates, generalized lymphadenopathy
Radiology images

Images hosted on other servers:

CT findings in colonic histoplasmosis

Prognostic factors
  • Prognosis is good after early and aggressive treatment; the disease is fatal if remains untreated (Int J Surg Pathol 2017;25:592)
  • Improvements in both antiretroviral and antifungal therapies appear to have contributed to a lower prevalence and may favorably impact the prognosis of this disease (Diagn Microbiol Infect Dis 2006;55:195)
Case reports
  • 30 year old cachectic woman with a history of HIV and poor compliance with highly active antiretroviral therapy (HAART) presents with diffuse abdominal pain and fever (Cureus 2020;12:e7775)
  • 39 year old man with HIV presents with severe pain in the abdomen (Case Rep Med 2018;2018:8923972)
  • 57 year old asymptomatic man with a history of orthotopic liver transplant was found to have sessile polyps on elective surveillance colonoscopy (ACG Case Rep J 2021;8:e00598)
  • 60 year old woman presents with recurrent diarrhea and abdominal discomfort (Cureus 2018;10:e2951)
  • 60 year old man presents with 75 pound weight loss and dysphagia, and was found to have idiopathic Cd4 T lymphocytopenia (Cureus 2021;13:e19748)
Treatment
  • According to the Infectious Diseases Society of America, clinical guidelines for the treatment of GI histoplasmosis without CNS involvement: amphotericin B (300 mg/kg daily) for 1 - 2 weeks followed by oral itraconazole (200 mg 3 times daily for 3 days and then 200 mg twice a day for a total of at least 12 months) (Infect Dis Clin North Am 2016;30:207)
Clinical images

Contributed by Wei Chen, M.D., Ph.D.
Missing Image

Colonoscopy showing GI histoplasmosis



Images hosted on other servers:
Missing Image

Ulcer in the sigmoid colon

Gross description
  • Focal / multiple mucosal ulcerations, polypoid lesions, strictures, obstruction due to circumferential thickening (Gig Sanit 1986:39)
Microscopic (histologic) description
  • Colonic tissue shows diffuse expansion of lamina propria and submucosa by foamy histiocytes associated with lymphocytes and eosinophils; mucosal ulceration can be seen frequently (Clin Chest Med 2009;30:217)
  • Granulomas are less frequently observed
  • Organisms are usually intracellular within histiocytes, rarely extracellular mixed with ulcer debris (Am J Clin Pathol 2000;113:64)
  • Protoplasm of the organisms retracts during fixation, resulting in a pseudocapsule on routine H&E (Pathologe 2022;43:16)
  • GMS and PAS special stains highlight the yeast cell wall and are helpful for characterization of the organisms
  • Romanowsky type stains, such as the Wright-Giemsa stain, impart a dark blue hue to the fungal nuclear compartment
  • Yeast forms can also pick up Ziehl-Neelsen stain (J Pathol Transl Med 2017;51:482)
  • H. capsulatum var. capsulatum: small, round to ovoid, narrow based budding yeast forms measuring 2 - 5 micrometers
  • H. capsulatum var. duboisii are larger and measure 6 - 12 micrometers (Infect Dis Clin North Am 2016;30:207)
Microscopic (histologic) images

Contributed by Wei Chen, M.D., Ph.D.
Missing Image

Colon with mucosal and submucosal inflammation

Missing Image

Submucosa with numerous macrophages

Missing Image

Macrophage containing yeast

Missing Image

GMS stain


Missing Image

PAS stain

Missing Image

Mucicarmine stain

Missing Image

Fontana-Masson silver stain

Virtual slides

Images hosted on other servers:

Small bowel histoplasmosis

GMS stain for histoplasmosis

Positive stains
Videos

Histoplasmosis colitis

Colonoscopy in histoplasmosis

Sample pathology report
  • Colon, cecum, ulcer, biopsy:
    • Mucosal and submucosal histiocytic inflammation containing numerous yeasts, morphologically compatible with Histoplasmosis capsulatum (positive GMS and PAS stains, negative mucicarmine and Fontana-Masson stains)
Differential diagnosis
Board review style question #1

A 45 year old man with a history of HIV presents with abdominal pain and diarrhea. Colonoscopy showed erythematous mucosal ulceration in the cecum. Biopsy from the ulcer showed numerous foamy histiocytes. GMS stain highlights numerous intracellular small, round to oval organisms as shown in the above image. Organisms stain positive with PAS and negative with mucicarmine. What is the diagnosis?

  1. Candidiasis
  2. Cryptococcosis
  3. Histoplasmosis
  4. Leishmaniasis
Board review style answer #1
C. Histoplasmosis. GMS stain highlights numerous intracellular round to oval yeast forms showing narrow based budding, morphologically consistent with histoplasmosis.

Comment Here

Reference: Histoplasmosis
Board review style question #2

The organisms shown in this image stain positive for GMS and PAS and negative for mucicarmine. What is the gold standard for diagnosis of the tissue invasive disease caused by these organisms?

  1. Clinical history and imaging findings
  2. Imaging findings and serology testing
  3. Serology testing and histopathological examination
  4. Tissue culture and histopathological examination
Board review style answer #2
D. Tissue culture and histopathological diagnosis. The diagnosis is histoplasmosis. Clinical history and imaging findings can be nonspecific in histoplasmosis, while serology testing can be false negative or show crossreactivity with other organisms. Tissue culture isolates the organisms and histopathological examination with H&E and special stains help in definitive diagnosis.

Comment Here

Reference: Histoplasmosis

HIV/AIDS associated
Definition / general
Essential features
  • Gastrointestinal tract is a major site for HIV replication; virus localizes in gastric mucosal tissue and depletes CD4+ T cells
Terminology
  • AIDS colitis
ICD coding
  • AIDS (related complex) B20
Epidemiology
Sites
  • Can occur anywhere in gastrointestinal tract
Pathophysiology
  • Gastrointestinal tract is a major site of HIV replication
  • HIV virus localizes and replicates in gut associated lymphoid tissues (GALT) and specifically targets mucosal CD4+ lymphocytes
  • Constant viral replication result in depletion of CD4+ lymphocyte count, leading to secondary opportunistic infections (Immunol Rev 2013;254:54)
  • With advanced HIV, there are markedly reduced numbers of CD4+ cells along with a parallel increase in CD8+ T cells (Gastroenterology 1986;91:651)
  • There are also significant increases in levels of inflammatory cytokines IL1β and IFN ϒ but a decrease in the levels of IL10 (AIDS 1994;8:461)
Etiology
  • HIV virus
Diagrams / tables

Images hosted on other servers:

HIV associated damage to the GI tract

Clinical features
  • Chronic diarrhea, abdominal pain, rectal bleeding
Diagnosis
  • Noninfective, HIV related colitis is easily recognized; this diagnosis may be made once infective pathologies have been excluded and the disease remains refractory to standard therapies for inflammatory bowel disease (Clin Infect Dis 2008;47:133)
Laboratory
  • Stool culture, CD4+ T cell count
Radiology description
  • Diffuse proctocolitis, consisting of contact bleeding, superficial ulcerations, exudates or loss of vascular pattern
Prognostic factors
  • Very low CD4+ T cell count
Case reports
  • Chronic colitis associated with HIV infection can be related to intraepithelial infiltration of the colon by CD8+ T lymphocytes (Int J STD AIDS 2008;19:524)
Treatment
  • For HIV related diarrhea, maintain adequate hydration and good nutrition
  • Antiviral reteroviral therapy ART (HAART) or combination ART
  • Thalidomide has been tried for refractory HIV (Clin Infect Dis 2008;47:133)
Clinical images

Images hosted on other servers:

HIV associated - various images

Colonic ulceration

Healing colonic ulceration

Gross description
  • Superficial ulceration and hemorrhage
Microscopic (histologic) description
  • Histologic changes can be minimal or absent in patients with well-controlled HIV

Esophagus
  • Mid esophagus commonly effected by HIV
  • Causes well circumscribed / linear ulcers with irregular margins and overhanging edematous edges
  • Histologically show granulation tissue with mixed acute and chronic inflammatory infiltrate often with eosinophils

AIDS enteropathy
  • HIV enteropathy causes villous blunting and atrophy, crypt hypertrophy, increased intraepithelial lymphocytes, variably increased mononuclear cell in lamina propria. Increased mitosis in glandular epithelial cells and increased number of apoptotic enterocytes at the surface

AIDS colonopathy
Microscopic (histologic) images

Contributed by Nalini Bansal, M.D.

Cryptosporidiosis

Strongyloides

Amoeboma

Herpes



Images hosted on other servers:
Missing Image

Candida (esophagus)

Missing Image

Candidal pseudohyphae and spores (PASD stain)

Missing Image

CMV gastritis

Missing Image

CMV vasculitis

Missing Image

Cryptococcosis (GMS stain)


Missing Image Missing Image

Cryptosporidiosis

Missing Image

Giardiasis

Missing Image

Histoplasmosis (GMS stain)

Missing Image

HIV enteropathy


Missing Image

HSV esophagitis

Missing Image

HSV cytopathic effect

Missing Image Missing Image

Kaposi sarcoma

Missing Image

Mycobacterium avium-intercellulare (acid fast stain)

Molecular / cytogenetics description
  • HIV nucleic acid identified by in situ hybridization in colonic biopsies
  • HIV DNA confirmed by Southern blot analysis
Board review style question #1
What is the most common site of localization and replication of HIV virus?

  1. Brain
  2. Gastrointestinal tract
  3. Liver
  4. Lungs
Board review style answer #1
B. Gastrointestinal tract

Comment Here

Reference: HIV/AIDS associated
Board review style question #2
What is the most common opportunistic infection in AIDS colitis?

  1. CMV
  2. Cryptosporidiosis
  3. Giardia
  4. Herpes
Board review style answer #2
Board review style question #3
What is the most opportunistic infection occur when CD4+ T cell count is below?

  1. 200/mm3
  2. 400/mm3
  3. 500/mm3
  4. 700/mm3
Board review style answer #3
A. 200/mm3

Comment Here

Reference: HIV/AIDS associated

HSV
Definition / general
  • Rare in the gastrointestinal tract; more common in the esophagus and in immunocompromised patients
  • Painful, discrete ulcers, vesicles or pustular lesions in distal rectum or perianal skin
  • Associated with immunocompromise and idiopathic inflammatory bowel disease (IBD)
  • Diagnose with viral culture
Essential features
  • HSV1 tends to cause predominantly oral and esophageal herpetic lesions
  • Commonly acquired during childhood, though it has been associated with proctitis in a minority of cases
  • HSV1 is primarily transmitted via oral secretions and has a higher seroprevalence in lower socioeconomic communities
  • HSV2 is most often sexually transmitted; it primarily causes genital and rectal infections
Terminology
  • Herpetic colitis
  • Herpes simplex colitis
ICD coding
  • ICD-10:
    • B00.8 - other forms of herpesviral infections
    • B00.9 - herpesviral infection, unspecified
Epidemiology
  • Estimates suggest that HSV1 has a worldwide seroprevalence of 80% to 90%, whereas HSV2 has a 22% seroprevalence rate in the U.S. (Infect Dis 1998;178:1616)
  • Among patients infected with HIV, HSV is the most common viral coinfection and 95% of men who have sex with men (MSM) are seropositive for HSV1, HSV2 or both
Sites
  • After primary infection, viral particles are transported retrograde from the peripheral site of infection to dorsal root ganglia (Lancet Neurol 2007;6:1015)
  • Virus may become latent for an unpredictable period
  • At any time, the virus may be reactivated
    • Asymptomatic reactivation may cause viral shedding; symptomatic reactivation frequently causes symptoms that are less severe and shorter in duration than the symptoms of the primary infection but that have a similar distribution (Lancet Neurol 2007;6:1015)
Pathophysiology
  • Primary HSV infection causes mild / asymptomatic oral labial (usually HSV1) or genital infections (usually HSV2) in immunocompetent patients
  • Latent HSV persists in sensory nerve ganglia
  • In immunocompromised patients, HSV infection can lead to systemic infections
  • Patients on immunosuppressive agents (i.e., corticosteroids, azathioprine, cyclosporine, tacrolimus or methotrexate) are at increased risk
  • Almost 25% of patients have preceding cough or sore throat; 27% have oral lesions and 13% have herpetic skin rashes (Dis Esophagus 2005;18:340)
Etiology
  • Most cases of gastrointestinal HSV infection are in immunocompromised hosts
  • HSV does not commonly affect the small and large intestine but there are scattered case reports of HSV colitis and jejunitis after bone marrow and solid organ transplantation (Bone Marrow Transplant 1997;20:989)
  • Most HSV proctitis cases (70%) are related to HSV2
  • Typical endoscopic findings in HSV proctitis include friable mucosa, diffuse distal ulcerations and vesicular lesions (Clin Infect Dis 1999;28:S84)
Diagrams / tables
No information provided
Clinical features
  • Symptoms are similar to colitis of other causes and include watery or bloody diarrhea, fever, tenesmus, abdominal pain, nausea, neurological symptoms, fatigue and weight loss
  • Symptoms of inflammatory bowel disease may also be present if there is concomitant disease
Diagnosis
  • Biopsies and serologic testing
  • Immunohistochemical stain with anti-HSV is available for tissue diagnosis
  • Immunocytochemistry also has an excellent diagnostic yield, being positive in 87.5% of patients
  • Characteristic viral pathology includes multinucleated giant cells and Cowdry type A inclusion
Laboratory
  • Serology uses a blood test, which looks for antibodies to the herpes simplex virus (HSV), including HSV1 and HSV2
  • Serum HSV IgG and IgM immunoglobulin assays
  • Detection of HSV DNA by PCR in colonic biopsies is more reliable due to high seropositivity of HSV worldwide
  • Viral cultures have slow turn around time
Radiology description
  • Typically normal findings, unless associated with inflammatory bowel disease, which can show toxic megacolon or strictures related to inflammatory bowel disease
Radiology images
No information provided
Prognostic factors
  • Overall prognosis for HSV colitis or esophagitis without treatment is poor
  • Patients do tend to respond to antiviral medication, including acyclovir (Gut 1989;30:1033)
  • Disseminated disease may be fatal
Case reports
Treatment
  • No vaccination is available for HSV
  • Antiviral therapy with acyclovir
Clinical images

Images hosted on other servers:
Colonoscopy showing deep ulcers in the sigmoid colon

Colonoscopy showing deep ulcers in the sigmoid colon

Gross description
  • Perianal vesicles and pustules
  • Ulceration
  • Mucosal friability
Gross images
No information provided
Frozen section description
No information provided
Frozen section images
No information provided
Microscopic (histologic) description
  • Exudate with sloughed epithelial cells
  • Neutrophilic infiltrate
  • Aggregates of macrophages
  • Perivascular lymphocyte cuffing
  • Characteristic inclusions in sloughed mucosa or mucosa at edge of ulcers
  • Inclusions are exclusively nuclear
  • Characteristic cytoplasmic inclusions
    • Cowdry type A (acidophilic inclusion with surrounding clear halo)
  • Some patients, especially HIV+, can have multiple infectious etiologies (Curr Gastroenterol Rep 2008;10:417)
Microscopic (histologic) images

Contributed by Zarrin Hossein-Zadeh, M.D. and Raul S. Gonzalez, M.D.
Pathognomonic Cowdry type A and type B inclusions Pathognomonic Cowdry type A and type B inclusions Pathognomonic Cowdry type A and type B inclusions

Pathognomonic Cowdry type A and type B inclusions

Positive HSV antibodies

Positive HSV antibodies

Virtual slides
No information provided
Cytology description
No information provided
Cytology images
No information provided
Immunofluorescence description
No information provided
Immunofluorescence images
No information provided
Positive stains
No information provided
Negative stains
No information provided
Electron microscopy description
No information provided
Electron microscopy images
No information provided
Molecular / cytogenetics description
No information provided
Molecular / cytogenetics images
No information provided
Videos
No information provided
Sample pathology report
  • Colon, random, biopsy:
    • Herpes simplex virus (HSV) colitis (see comment)
    • Comment: Immunohistochemical stain for HSV1 is positive.

  • Colon, colectomy:
    • Herpes simplex virus (HSV) colitis (see comment)
    • Comment: In the background of severely active ulcerative pancolitis. Immunohistochemical stain for HSV1 is positive. Negative for dysplasia.
Differential diagnosis
  • Acute self limited / infectious colitis:
    • Inflammation of lamina propria (active much more than chronic), edema, hemorrhage
    • Intranuclear inclusion, epithelial vacuolization and nuclear disarray can be seen in viral etiologies
  • Other viral colitis / CMV colitis:
    • Large, ovoid or pleomorphic nucleus with basophilic intranuclear inclusions (owl eyes) surrounded by a clear halo
  • Graft versus host disease:
    • Crypt apoptosis
    • Crypt dropout
    • Ulceration
    • No microscopic features of margination, multinucleation and molding
    • Lack of Cowdry type A and B inclusions
  • Inflammatory bowel disease:
    • No inclusions
    • Microscopic features of chronicity, ulceration, cryptitis or crypt abscess formation
Board review style question #1

A 69 year old man presents with abdominal pain and diarrhea. Colonoscopy showed diffuse mucosal erythema. The patient underwent a biopsy of the lesion and histology is shown above. Which organism is causing the histopathologic findings?

  1. Adenovirus
  2. Cytomegalovirus
  3. Hepatitis B virus
  4. Herpes simplex virus
Board review style answer #1
D. Herpes simplex virus. The image shows Cowdry type A inclusions and the characteristic changes associated with HSV infection, including molding, margination and multinucleation (the 3M phenomenon).

Comment Here

Reference: Colon - HSV
Board review style question #2
What is the gold standard method for diagnosing HSV colitis?

  1. Histology and immunohistochemistry for HSV
  2. HSV culture
  3. HSV DNA polymerase chain reaction (PCR) amplification method
  4. HSV serology
Board review style answer #2
A. Histology and immunohistochemistry for HSV. The gold standard diagnostic test for tissue invasive HSV disease, including colitis and hepatitis, is histopathologic evidence of herpes simplex virus infection, which can include characteristic eosinophilic intracytoplasmic inclusions as well as positive immunohistochemical staining using labeled HSV1 and HSV2 specific monoclonal antibodies.

Comment Here

Reference: Colon - HSV

HSV
Definition / general
  • Painful discrete ulcers, vesicles or pustular lesions in distal rectum or perianal skin
  • Diagnose with viral culture
Epidemiology
  • > 90% of worldwide population is seropositive for HSV by age 30, but HSV colitis is rare (JAMA 2006;296:964)
  • Associated with immunocompromise and idiopathic inflammatory bowel disease
Pathophysiology
  • Primary HSV infection causes mild/asymptomatic oral labial (usually HSV1) or genital infections (usually HSV2) in immunocompetent patients
  • Latent HSV persists in sensory nerve ganglia
  • In immunocompromised patients, HSV infection can lead to systemic infections
  • Patients on immunosuppressive agents (ie. corticosteroids, azathioprine, cyclosporine, tacrolimus, or methotrexate) are at increased risk
Clinical features
  • Symptoms are similar to colitis of other causes and include watery or bloody diarrhea, fever, abdominal pain, nausea, fatigue, and weight loss
  • Symptoms of inflammatory bowel disease may also be present if there is concomitant disease
Diagnosis
  • Immunohistochemical stain with anti-HSV is available for tissue diagnosis
Laboratory
  • Serum HSV IgG and IgM immunoglobulin assays
  • Detection of HSV DNA by PCR in colonic biopsies is more reliable due to high seropositivity of HSV worldwide
  • Viral cultures have slow turn around time
Case reports
Treatment
  • No vaccination is available for HSV
  • Antiviral therapy with acyclovir
Gross description
  • Painful discrete ulcers, vesicles or pustular lesions in distal rectum or perianal skin
Microscopic (histologic) description
  • Ulceration with neutrophils in lamina propria, cryptitis, crypt abscess, multinucleated giant cells, inclusions in anal transition zone epithelium and perianal skin

Hyperplastic polyp
Definition / general
  • Very common type of polyp in the colon and rectum
  • Often numerous, particularly within the rectum
  • Usually small (5 mm or less)
  • Associated with no significant risk of malignant progression
Essential features
  • Can occur throughout the large intestine but is most common in the sigmoid colon and rectum
  • Associated with no significant risk of malignant progression
  • Serrated architecture with a sawtooth appearance, with serrations generally limited to the upper half of the crypts
  • Main differential diagnosis is with sessile serrated lesions (SSLs)
  • Microvesicular variant not uncommonly contains a BRAF gene mutation
Terminology
  • These lesions used to be termed metaplastic polyps
  • Microvesicular and goblet cell rich variants exist
  • Goblet cell poor variant no longer recognized
ICD coding
  • ICD-10: K63.5 - polyp of colon
Epidemiology
Sites
  • Large intestine
  • Can occur throughout the large intestine but is most common in the sigmoid colon and rectum
  • Commonly multiple within the rectum
  • Reference: Histopathology 2021;78:780
Pathophysiology
  • Unknown
Etiology
  • Shares lifestyle risk factors with colorectal adenoma and colorectal cancer; e.g. low dietary fiber content, raised body mass index, increased alcohol consumption and cigarette smoking are all associated with an increased risk of hyperplastic polyp development (Gastroenterology 1997;113:423)
Clinical features
  • Almost always asymptomatic
  • May be present as part of a spectrum of lesions that also includes sessile serrated lesions, in serrated polyposis (Am J Gastroenterol 2012;107:1315)
Diagnosis
  • Diagnosis is made on histological examination
Case reports
Treatment
  • Endoscopic biopsy or removal is often performed to confirm the diagnosis
  • No further treatment is required unless the patient is thought to have serrated polyposis, in which case endoscopic surveillance may be warranted
Clinical images

Images hosted on other servers:
Hyperplastic polyp at endoscopy

Hyperplastic polyp at endoscopy

Gross description
  • These are small mucosal lesions, typically less than 5 mm in size
  • Appear as slightly elevated and pale lesions at endoscopy but lack the mucin cap that is commonly seen with sessile serrated lesions (World J Gastroenterol 2018;24:3250)
Microscopic (histologic) description
  • Serrated architecture with a sawtooth appearance, with serrations generally limited to the upper half of the crypts
  • Mild nuclear enlargement, stratification and hyperchromasia, limited to the crypt bases
  • No dilated, branched or horizontally spreading crypts
  • No evidence of conventional dysplasia, e.g. as is seen in colorectal adenomas
  • Microvesicular variant is more common and comprises epithelial cells with small mucin vacuoles
  • Goblet cell rich variant is less common and contains prominent goblet cells
  • Comparison with adjacent nonlesional large intestinal mucosa may facilitate the recognition of goblet cell rich variant and reduce the risk of underdiagnosis, since the serrations are more subtle than in the microvesicular type (Histopathology 2021;78:634)
Microscopic (histologic) images

Contributed by Adrian C. Bateman, M.B.B.S., M.D.
Microvesicular variant

Microvesicular variant

Sawtooth glands

Sawtooth glands

Surface tufting

Surface tufting

Nuclear changes at crypt bases

Nuclear changes at crypt bases

Goblet cell rich variant

Goblet cell rich variant

Tapered crypts at base

Tapered crypts at base

Positive stains
  • Ki67 immunohistochemistry shows proliferation confined to the lower portions of the polyp, mirroring the morphological evidence of maturation towards the surface
Molecular / cytogenetics description
Sample pathology report
  • Rectum, cold biopsy excision of 4 mm polyp:
    • Hyperplastic polyp
Differential diagnosis
Board review style question #1
Which of the following is characteristic of hyperplastic polyps of the large intestine?

  1. Are rare lesions
  2. Different risk factors for development compared with those for adenomas
  3. Little or no association with an increased risk of colorectal cancer
  4. Not often multiple within the rectum
  5. Usually require treatment by bowel resection
Board review style answer #1
C. Hyperplastic polyps that occur outside of the context of polyposis syndrome are not associated with a significant risk of colorectal cancer, in contrast to sessile serrated lesions, which are associated with increased cancer risk.

Comment Here

Reference: Hyperplastic polyp
Board review style question #2

Which of the following histological features is characteristic of a hyperplastic polyp of the large intestine?

  1. Branched crypts
  2. Conventional dysplasia
  3. Dilated glands with little or no intervening lamina propria
  4. Prominent glandular serration extending down to the crypt bases
  5. Sawtooth pattern glands within the superficial aspect of the lesion
Board review style answer #2
E. Hyperplastic polyps characteristically contain sawtooth pattern glands within the superficial aspect of the lesion. In contrast to sessile serrated lesions, hyperplastic polyps do not show serration to the crypt bases, branched crypts or dilated crypts.

Comment Here

Reference: Hyperplastic polyp

IBD associated
Definition / general
  • Colitis associated colorectal cancer (CAC) is a malignant epithelial tumor originating in the large bowel associated with longstanding inflammatory bowel disease (IBD)
Essential features
  • Associated with longstanding inflammatory bowel disease
  • Other risk factors: disease extent, longstanding anorectal fistula, pseudopolyposis and coexisting primary sclerosing cholangitis (PSC)
  • Evenly distributed (left sided and right sided)
  • Mucinous adenocarcinoma and signet ring cell adenocarcinoma are common in this setting
ICD coding
  • No official coding established
Epidemiology
  • Longstanding inflammatory bowel disease patients are at a higher risk of developing colitis associated colorectal cancer compared with the general population; the risks are higher along with longer duration of inflammatory bowel disease
  • An inflammatory bowel disease patient with a first degree relative having colorectal adenocarcinoma has an eightfold increase in risk of developing colitis associated colorectal cancer (Clin Gastroenterol Hepatol 2019;17:1807)
  • Among inflammatory bowel disease patients, the disease extent (> 50% of the colon), longstanding anorectal fistula, presence of pseudopolyposis and coexisting primary sclerosing cholangitis (PSC) determine higher risk of developing colitis associated colorectal cancer (Clin Colon Rectal Surg 2018;31:168)
Sites
  • Colitis associated colorectal cancer is more evenly distributed across the colon, compared with sporadic colorectal adenocarcinoma which is located mainly distal to the splenic flexure (Clin Gastroenterol Hepatol 2006;4:335)
Pathophysiology
  • Cytokines such as TNFα, IL1 and IL6 are involved in the promotion of colitis associated colorectal cancer in inflammatory bowel disease patients through chronic inflammation induced accumulation of reactive oxygen species, mutagenic DNA damage and activation of transcription factors such as NFκB and STAT3 (Curr Cancer Drug Targets 2011;11:451, Gastroenterology 2011;140:1807)
Etiology
Clinical features
  • Similar to sporadic colorectal adenocarcinoma; hematochezia or rectal bleeding may be present
Diagnosis
  • Clinical history of longstanding inflammatory bowel disease; endoscopic, histopathologic and radiographic findings
Radiology description
  • Two different CT patterns are described:
    • Type 1 tumor (47%): clearly visible soft tissue mass
    • Type 2 tumor (53%): circumferential thickening of the colorectal wall with stenosis; significant association with the presence of signet ring cells (Abdom Imaging 2013;38:421)
Prognostic factors
  • Risk factors: long duration of inflammatory bowel disease, degree of inflammation, disease extent (pancolitis or left sided colitis), longstanding anorectal fistula, pseudopolyposis, coexistenting primary sclerosing cholangitis
  • Role of chemoprevention with 5-aminosalicylic acid derivatives (5-ASA) as a protecting factor has been controversial (Dig Dis 2012;30 Suppl 2:55)
Case reports
  • 16 year old girl with synchronous colorectal neoplasms in ulcerative pancolitis of 6 years duration (BMJ Case Rep 2013;2013:bcr2013200172)
  • 36 year old man with synchronous multifocal colorectal carcinoma after delayed diagnosis of ulcerative pancolitis (Pathol Res Pract 2008;204:905)
  • 36 year old man with primary sclerosing cholangitis and inflammatory bowel disease developed two synchronous colitis associated colorectal cancers (World J Gastroenterol 2014;20:12657)
  • 41 year old man with colitis associated colorectal cancer developing only 2 years after the first symptoms of ulcerative colitis (J Gastroenterol 2007;42:854)
  • 51 year old man with Crohn’s disease developed a mucinous adenocarcinoma with signet ring cell carcinoma 3 years post resection of a sporadic adenocarcinoma (World J Surg Oncol 2013;11:295)
Treatment
  • Prophylactic proctocolectomy in patients with primary sclerosing cholangitis and inflammatory bowel disease is still controversial (Gastrointest Cancer Res 2011;4:53)
Microscopic (histologic) description
Microscopic (histologic) images

Contributed by Lei Sun, M.D. and Feng Yin, M.D., Ph.D.

CAC, conventional type

CAC, signet ring type

CAC, mucinous type

Positive stains
Negative stains
  • CK7 (similar to sporadic colorectal adenocarcinoma)
Molecular / cytogenetics description
  • Mutation of TP53 occurs at earlier stage in inflammatory bowel disease associated dysplasia and is observed in 50 - 85% of colitis associated colorectal cancers
  • KRAS and APC mutations are less frequently seen and occur at later stage in colitis associated colorectal cancer (Oncotarget 2017;8:22175)
  • Mutations such as MLH1, RNF43, RPL22 and BRAF have been reported (Pathol Res Pract 2019;215:730)
  • IDH1 R132 mutations more common in patients with Crohn’s disease but not ulcerative colitis
  • MYC amplification more common in colitis associated colorectal cancer (Gastroenterology 2016;151:278)
Sample pathology report
  • Colon, mass, biopsy:
    • Adenocarcinoma, well differentiated, arising in a background of chronic active colitis with high grade dysplasia, consistent with colitis associated colorectal adenocarcinoma
Differential diagnosis
  • Sporadic colorectal adenocarcinoma:
    • Usually affects elderly population, single lesion
    • Less likely to have mucinous or signet ring features
    • No clinical history of long standing chronic colitis
Board review style question #1



A 32 year old woman with Crohn’s disease presented with abdominal pain and hematochezia. The abdominal CT scan showed a segment of circumferential bowel wall thickening. Colonoscopy showed a stricture in the descending colon. A representative photomicrograph of the biopsy is shown. Which is the most likely diagnosis?

  1. Colorectal adenocarcinoma, NOS
  2. Metastatic breast lobular carcinoma
  3. Metastatic gastric signet ring cell carcinoma
  4. Signet ring cell carcinoma, colitis associated
Board review style answer #1
D. Signet ring cell carcinoma, colitis associated. The biopsy reveals signet ring cell carcinoma in a young patient with longstanding Crohn’s disease. This is a classic presentation of colitis associated colorectal cancer. Signet ring cell carcinoma and mucinous adenocarcinoma are more common in colitis associated colorectal cancer as compared with sporadic cases.

Comment Here

Reference: IBD associated carcinoma
Board review style question #2
Which mutation often occurs at an early stage of colitis associated colorectal cancer?

  1. APC
  2. BRAF
  3. KRAS
  4. MLH1
  5. TP53
Board review style answer #2
E. TP53. TP53 mutation occurs early in colitis associated colorectal cancer but is a late event in sporadic colorectal adenocarcinoma. In contrast, APC mutation occurs early in sporadic colorectal adenocarcinoma but is very rare and usually a late event in colitis associated colorectal cancer.

Comment Here

Reference: IBD associated carcinoma

Idelalisib associated
Definition / general
  • Colitis caused by the use of idelalisib, a highly potent small molecule phosphoinositide 3 kinase δ (PI3Kδ) inhibitor approved for the treatment of chronic lymphocytic leukemia / small lymphocytic lymphoma (CLL / SLL) and follicular lymphoma
Essential features
  • A triad of intraepithelial lymphocytosis, epithelial cell apoptosis and neutrophilic cryptitis in colonic biopsies with moderate to severe diarrheal symptoms along with clinical history of idelalisib therapy (Am J Surg Pathol 2015;39:1661)
  • Many patients develop gastrointestinal symptoms during idelalisib therapy
  • Diarrhea with enterocolitis is a common adverse effect of long term idelalisib therapy (Am J Surg Pathol 2015;39:1661)
Epidemiology
  • Most of the patients with severe gastrointestinal toxicity are older adults (mean age is 65 years)
  • M > F
  • History of drug intake for an average of 7 months before severe symptom development (Future Oncol 2018;14:2265)
Sites
Pathophysiology
  • Diarrhea with enterocolitis is a common adverse effect of long term idelalisib therapy
  • Clinically present with loose, watery and nonbloody stools
  • Majority of patients who undergo a colonoscopy have endoscopically apparent colitis with erythema, congestion and granularity
  • More severe cases can exhibit erosions or ulceration (Am J Surg Pathol 2015;39:1653)
Etiology
  • Idelalisib (Zydelig) is a small molecule inhibitor of phosphoinositide 3 kinase δ (PI3Kδ) that was recently approved for treatment of relapsed B cell malignancies, including chronic lymphocytic leukemia and follicular lymphoma
  • Median time for onset of severe grade 3 or 4 diarrhea is ~7 months (Am J Surg Pathol 2015;39:1653)
Clinical features
  • Diarrhea, nausea, fatigue, cough, pyrexia, chills
  • Rarely associated with transaminitis, pneumonitis
Diagnosis
  • Physical examination including assessment of patient for fever, dizziness, abdominal pain / cramping and weakness (i.e., rule out risk for sepsis, bowel obstruction, dehydration)
  • History of onset and duration of diarrhea
  • Travel history to exclude infectious etiologies
  • Description of number of stools and stool composition (e.g., watery, bloody, nocturnal)
  • Medication profile to identify diarrheagenic agents
  • Dietary profile to identify diarrheagenic foods (Leuk Lymphoma 2015;56:2779)
Laboratory
  • Stool workup
  • Occult blood determination
  • Culture for enteric pathogens (Salmonella, Escherichia coli, Campylobacter, Clostridium difficile)
  • Complete blood count, electrolytes and blood urea nitrogen / creatine (BUN:Cr) (grade 3: neutropenia, thrombocytopenia, anemia and alanine transaminase [ALT] or aspartate transferase [AST] elevation)
  • Consider colonoscopy in selected cases (Leuk Lymphoma 2015;56:2779)
Radiology description
  • Majority of patients who undergo a colonoscopy have endoscopically apparent colitis with erythema, congestion and granularity or ulceration
Prognostic factors
  • Poor prognostic factors include fatal or severe diarrhea or colitis, hepatotoxicity, pneumonitis and intestinal perforation
  • Adverse idelalisib events include pneumonia, pyrexia and febrile neutropenia (Leuk Lymphoma 2015;56:2779)
Case reports
  • 40 year old woman with relapsed grade 2 follicular lymphoma presented with 3 days of vomiting, cramping abdominal pain and diarrhea (JAMA Oncol 2016;2:1361)
  • 56 year old man with a history of relapsed follicular lymphoma presented with severe diarrhea and a skin eruption mimicking graft versus host disease 6 months after starting idelalisib (Clin J Gastroenterol 2017;10:142)
  • 72 year old woman presented with abdominal pain, nonbloody diarrhea for 2 months and 15 lbs of weight loss (Ann Gastroenterol 2016;29:233)
Treatment
  • Discontinue idelalisib
  • Rule out infection
  • Intravenous or oral rehydration if grade 3 diarrhea or colitis
  • Budenoside or oral steroids (intravenous if orals are not tolerated)
  • Diet modification recommendations such as plenty of fluids, 8 - 12 glasses a day of oral rehydration drinks, 5 - 6 small meals each day and low fat, high protein foods such as lean meat and eggs (Leuk Lymphoma 2015;56:2779)
Clinical images

Contributed by Ashwini Kumar Esnakula, M.D., M.S.
Colonoscopic findings

Colonoscopic findings

Microscopic (histologic) description
  • Presence of intraepithelial lymphocytosis in both the crypts and surface epithelium
  • Apoptotic crypt epithelial cells (sometimes numerous with large exploding apoptotic cells)
  • Neutrophilic infiltration of the crypt epithelium with occasional crypt abscesses
  • Minority show crypt architectural distortion in those patients that had a prolonged interval between symptom onset and colonoscopy
  • Histologic patterns can be divided into 4 distinct morphological categories:
    • Predominantly normal
    • Apoptotic pattern without intensive inflammatory component
    • Infectious colitis-like pattern characterized predominantly by inflammation with varying degrees of attenuated glands (apoptotic cells not a prominent feature)
    • Mixed pattern with varying degrees of apoptotic cells, mixed inflammation or attenuated glands, with the most striking features of architectural distortion, mixed inflammation (including intraepithelial lymphocytes and neutrophils and increased eosinophils), crypt abscess / destruction and apoptosis
  • References: Am J Surg Pathol 2015;39:1661, Am J Surg Pathol 2015;39:1653
Microscopic (histologic) images

Contributed by Ashwini Kumar Esnakula, M.D., M.S.
Prominent crypt epithelial apoptosis and apoptotic bodies

Prominent crypt epithelial apoptosis and apoptotic bodies

Prominent crypt epithelial apoptosis

Prominent crypt epithelial apoptosis

Cryptitis and crypt epithelial apoptosis

Cryptitis and crypt epithelial apoptosis

Cryptitis and crypt abscess

Cryptitis and crypt abscess

Cryptitis and lamina propria mixed inflammation

Cryptitis and lamina propria mixed inflammation

Positive stains
Negative stains
Sample pathology report
  • Left colon, biopsy:
    • Colonic mucosa with mild acute cryptitis, mucosal eosinophilia, intraepithelial lymphocytes and prominent crypt apoptosis (see comment)
    • Comment: The patient's clinical history of refractory follicular lymphoma and treatment with idelalisib are noted. The histologic findings are likely consistent with idelalisib associated colitis.
Differential diagnosis
Board review style question #1

A 68 year old man was admitted for evaluation of chronic dehydration, diarrhea, abdominal pain and weight loss for > 1 month (~20 lbs). He has a history of relapsed follicular lymphoma and has been on Zydelig (idelalisib) for ~11 months. Which of the following favors idelalisib associated colitis over inflammatory bowel disease?

  1. Basal plasmacytosis
  2. Epithelial cell apoptosis
  3. Paneth cell metaplasia
  4. Splaying of muscularis mucosae
Board review style answer #1
B. Epithelial cell apoptosis. Crypt apoptosis is a common feature associated with idelalisib associated colitis and is less commonly seen in patients with inflammatory bowel disease.

Comment Here

Reference: Idelalisib associated colitis
Board review style question #2

A 56 year old man with a history of relapsed follicular lymphoma status postallogenic bone marrow transplant who developed severe diarrhea with a skin eruption mimicking graft versus host disease (GVHD) 6 months after starting idelalisib. Which of the following histological findings is indicative of idelalisib associated colitis?

  1. Basal plasmacytosis
  2. Increased eosinophils within the lamina propria and crypt abscesses
  3. Lack of apoptosis
  4. Severe chronic injury
Board review style answer #2
B. Increased eosinophils within the lamina propria and crypt abscesses. Idelalisib associated colitis closely mimics graft versus host disease and can be challenging to distinguish. The presence of prominent neutrophilic cryptitis and crypt abscesses can be helpful to favor idelalisib associated colitis.

Comment Here

Reference: Idelalisib associated colitis

Idiopathic myointimal hyperplasia of mesenteric veins
Definition / general
  • Unusual thickening of mesenteric veins, resulting in gastrointestinal symptoms and ischemic colitis
Essential features
  • Rare condition that clinically mimics inflammatory bowel disease in young men
  • Thought to result from arteriovenous fistulization
  • Histology: hyperplastic veins mimic arteries (distinguish with elastin stain); thick walled vascular proliferation in mucosa
Epidemiology
Sites
  • Usually (but not always) involves the left colon
Etiology
Clinical features
  • Diarrhea / constipation, abdominal pain, rectal bleeding
Case reports
Treatment
  • Surgery usually required
Microscopic (histologic) description
  • Biopsy findings: ischemic mucosal change accompanied by proliferation of hyperplastic, thick walled, hyalinized blood vessels (Am J Surg Pathol 1996;20:1271)
  • Resection findings: mesenteric and mural veins show myointimal hyperplasia, making them somewhat resemble arteries; no vasculitis present; arteries are unaffected (Gastroenterology 1991;101:533)
Microscopic (histologic) images

Contributed by Raul S. Gonzalez, M.D.

Biopsy: mucosal
proliferation
of thick walled
vessels

Resection:
thickened
mural veins

2 thickened veins and 1 unaffected artery

Elastin stain
highlights
internal elastic
lamina of artery

Positive stains
  • Elastin stain confirms that blood vessels with myointimal hyperplasia are veins, not arteries
Sample pathology report
  • Descending colon, resection:
    • Segment of colon with prominent vascular reactive changes and myointimal hyperplasia of mesenteric veins (see comment)
    • Margins of resection unremarkable.
    • Seven benign lymph nodes.
    • Comment: There is no definitive evidence of chronic colitis. An elastic special stain highlights mesenteric veins with myointimal hyperplasia, distinguishing them from arteries.
Differential diagnosis
  • Inflammatory bowel disease:
    • Similar clinical picture but histology shows a classic chronic colitis, without vascular abnormalities
  • Mesenteric phlebosclerosis:
    • Unusual vascular disease seen only in Japanese patients; mesenteric and colonic mucosal vessels are calcified and sclerotic (Dis Colon Rectum 2003;46:209)
Board review style question #1

Which of the following is true about idiopathic myointimal hyperplasia of mesenteric veins?

  1. An elastin stain can help distinguish arteries from abnormal veins
  2. Histologic changes are seen in resection but not biopsy specimens
  3. It classically occurs along the right colon in older female patients
  4. It is asymptomatic and incidentally observed
Board review style answer #1
A. An elastin stain can help distinguish arteries from abnormal veins

Comment Here

Reference: Idiopathic myointimal hyperplasia of mesenteric veins

Idiopathic retroperitoneal fibrosis
Definition / general
  • Nonneoplastic fibrosis that develops in retroperitoneum and may encroach upon colon
Essential features
  • Benign fibrosing process of the retroperitoneum that can rarely involve the gastrointestinal tract secondarily
  • Some cases belong to the spectrum of IgG4 related disease
Terminology
  • Also called Ormond disease, idiopathic retroperitoneal fibrosis
Sites
  • Arises in retroperitoneum, often at aortic bifurcation
  • Gastrointestinal tract involvement is rare
Etiology
  • IgG4 positive plasma cells implicated as a causative factor (Am J Surg Pathol 2009;33:1833)
  • Can also be caused by methysergide or lymphoma or may be idiopathic
Clinical features
Laboratory
  • Increased serum IgG and IgG4 in patients with IgG4 related disease
Radiology images

Images hosted on other servers:
Missing Image

Retroperitoneal mass enveloping aorta

Case reports
Treatment
Gross description
  • Poorly circumscribed fibrotic mass
Microscopic (histologic) description
  • Storiform fibrotic process infiltrated by eosinophils and IgG4 positive plasma cells (which may be sparse)
  • May show obliterative phlebitis, fat necrosis
  • Similar findings may be observed in lymph nodes (Am J Clin Pathol 1996;105:430)
Microscopic (histologic) images

Contributed by Raul S. Gonzalez, M.D.
Missing Image

Fibrosis and inflammation



Images hosted on other servers:
Missing Image

Fibrosis involving ureter

Positive stains
  • IgG4 (in plasma cells)
Sample pathology report
  • Transverse colon and adjacent soft tissue mass, excision:
    • Prominent bland fibrotic process involving soft tissue and focally extending into colon wall (see comment)
    • Negative for malignancy.
    • Margins of resection unremarkable.
    • Comment: The soft tissue fibrosis shows a storiform pattern and contains abundant chronic inflammation. An immunostain for IgG4 highlights numerous plasma cells. The overall findings are most consistent with idiopathic retroperitoneal fibrosis.
Differential diagnosis
Board review style question #1
Which of the following is true about idiopathic retroperitoneal fibrosis?

  1. Colon is often secondarily involved
  2. Most patients present with lower gastrointestinal bleeding
  3. Numerous IgG4 positive plasma cells are seen histologically
  4. Obliterative arteritis can be seen
Board review style answer #1
C. Numerous IgG4 positive plasma cells are seen histologically

Comment Here

Reference: Idiopathic retroperitoneal fibrosis

Infarct
Definition / general
  • Death of colonic tissue due to any marked loss of blood supply to colon, including vascular insufficiency and trauma
  • See also ischemic colitis
Essential features
  • Necrosis of colon due to loss of blood supply
  • Cause may not be evident histologically
  • Surgical resection necessary
Clinical features
  • Depends on cause but abdominal pain and hematochezia are common
Case reports
Treatment
  • Surgical resection of necrotic colon
Gross description
  • Intestine is dusky and necrotic
Gross images

Images hosted on other servers:

Necrotic segments of colon

Microscopic (histologic) description
  • Typically transmural colonic necrosis, with nonviable mucosa and edema, hemorrhage and congestion of wall
Microscopic (histologic) images

Contributed by Raul S. Gonzalez, M.D.
Colon infarct Colon infarct

Colon infarct

Videos

Histopathology colon - hemorrhagic infarct

Sample pathology report
  • Descending colon, resection:
    • Segment of colon with diffuse transmural necrosis and hemorrhage, extending to both resection margins
Board review style question #1

Which of the following is true about colonic infarction?

  1. Cause can always be observed on histologic examination
  2. Colon paradoxically appears normal grossly
  3. Only the mucosa is affected
  4. Surgery is necessary for treatment
Board review style answer #1
D. Surgery is necessary for treatment

Comment Here

Reference: Infarct

Infarcted epiploic appendages
Definition / general
  • Infarction and subsequent fat necrosis of epiploic appendages (fat containing pouches of colonic peritoneum) that may remain attached or autoamputate and lie loose in the peritoneum
Essential features
  • Fat necrosis of epiploic appendage, which may detach and lie loose in the peritoneum
  • May cause pain or be discovered as an incidental curiosity
Terminology
  • Epiploic appendagitis: inflammation but not infarction of appendages
  • Unattached infarcted appendages are known as peritoneal loose bodies or peritoneal mice (J Clin Gastroenterol 2006;40:427)
Sites
  • Epiploic appendages are chiefly on transverse and sigmoid colon
  • Can occur on appendix (S D Med 2006;59:511)
Etiology
Clinical features
Diagnosis
  • Laparoscopy
Radiology description
Radiology images

Images hosted on other servers:

Central calcified oval mass in the pelvis

Pelvic mass

Case reports
Gross description
  • Firm, gray-white nodules that may resemble metastatic tumor
  • Loose bodies can resemble an egg
Gross images

Images hosted on other servers:

Egg shaped mass

Giant loose peritoneal body

Giant loose body attached to omentum

Round pelvic mass

Central calcifications and a distinct fat plane

Macrograph of giant loose body

Microscopic (histologic) description
  • Central infarcted adipose tissue with peripheral fat necrosis and calcification, surrounded by thick, inflamed fibrotic tissue
Microscopic (histologic) images

Contributed by Raul S. Gonzalez, M.D.

Rounded contour

Central fat necrosis

Circumferential fibrous tissue

Sample pathology report
  • Abdominal cavity, loose body, removal:
    • Fat necrosis with fibrotic rim, consistent with infarcted epiploic appendage
Differential diagnosis
  • Fat necrosis from other cause:
    • Not rounded / encapsulated or free floating, may show more inflammation
Board review style question #1
Infarcted epiploic appendages Infarcted epiploic appendages


A loose egg-like object is found in a patient's abdomen during planned abdominal surgery. It has a microscopic appearance as shown above. What is the best diagnosis?

  1. Calcifying fibrous pseudotumor
  2. Fecal material
  3. Infarcted epiploic appendage
  4. Well differentiated liposarcoma
Board review style answer #1
C. Infarcted epiploic appendage

Comment Here

Reference: Infarcted epiploic appendages

Inflammatory bowel disease, indeterminate type
Definition / general
  • Inflammatory bowel disease (IBD), indeterminate type is a provisional term used to describe cases of IBD in which a definitive diagnosis of ulcerative colitis or Crohn's disease cannot be established on the basis of histopathological and clinical (including radiologic and endoscopic) data
  • IBD, indeterminate type is not a specific disease entity and has no established diagnostic criteria (Mod Pathol 2015;28:S30, J Clin Pathol 2004;57:1233)
  • Comprises 5 - 15% of IBD cases but most will eventually be declared as 1 of the 2 subtypes (only up to 15% thought to be Crohn's disease)
Essential features
  • IBD, indeterminate type is a diagnosis of exclusion and should only be used in resection specimens when it is impossible to distinguish ulcerative colitis from Crohn's disease
  • Term should be avoided in the following circumstances:
    • Recognized IBD variants: appendiceal or cecal patch in left sided ulcerative colitis, patchy / segmental disease (including rectal sparing) in treated or longstanding ("burned out") ulcerative colitis, fulminant (severe or toxic) ulcerative colitis, ulcerative colitis with backwash ileitis, ulcerative colitis with cryptolytic granulomas, superficial (ulcerative colitis-like) Crohn's disease
    • In the presence of convincing criteria for Crohn disease: nonnecrotizing, noninfectious epithelioid granulomas, true segmental colonic or small bowel disease, extensive transmural lymphoid aggregates underlying intact mucosa, definitive perianal or fistulizing disease
    • Secondary or superimposed non-IBD colitides, e.g. infectious colitis, ischemic colitis, diverticular disease associated colitis, diversion colitis, microscopic colitis (i.e., lymphocytic or collagenous colitis), pseudomembranous colitis, colitis due to medication related injury (e.g. nonsteroidal anti-inflammatory drugs (NSAIDs), mycophenolate, immune checkpoint inhibitors), etc.
    • In the setting of insufficient clinical, pathologic, radiologic or endoscopic data (e.g. on biopsy material in the absence of granulomas): use the term "IBD, unclassified" instead
  • Even though up to 15% will ultimately receive a diagnosis of Crohn's disease, patients with IBD, indeterminate type are generally thought to have similar ileal pouch anal anastomosis (IPAA) failure rates as ulcerative colitis; therefore, IPAA surgery is probably warranted in most cases
  • Higher rates of IBD, indeterminate type diagnosis are seen in the pediatric population, given the patchy disease distribution often seen in childhood ulcerative colitis
Terminology
  • Indeterminate colitis (IC):
    • Inexact and confusing term; not limited to IBD per se
    • Used in some instances when the etiology of chronic colitis cannot be ascertained from clinicopathologic findings; in these cases, nonspecific colitis or chronic active colitis, type unknown is preferred, accompanied by a note as to the reason for the uncertainty and possible differential diagnoses
  • Inflammatory bowel disease, unclassified (IBD-U):
    • Primarily a temporary term used during colonoscopy when the pathologist or gastroenterologist favors a diagnosis of IBD but the histologic or endoscopic impression do not permit distinction between ulcerative colitis and Crohn's disease
    • In most cases, a diagnosis of ulcerative colitis or Crohn's disease can be yielded upon evaluation of follow up resection specimens or additional clinical data
  • Other terms:
    • Colitis of uncertain type or etiology (CUTE) (Inflamm Bowel Dis 2008;14:850)
    • IBD, NOS (not otherwise specified)
    • IBD, unknown etiology
    • Uncertain colitis
ICD coding
  • ICD-10: K52.3 - indeterminate colitis
  • ICD-10: K52.9 - noninfective gastroenteritis and colitis, unspecified
Sites
  • Colon
Diagnosis
  • IBD, indeterminate type is a diagnosis of exclusion and requires conflicting histopathological findings in a resection specimen or in the setting of contradictory clinical or radiological data
Laboratory
  • Limited clinical utility of serological markers ASCA and pANCA
  • ~50% of patients with IBD, indeterminate type are ASCA- / pANCA- (Gastroenterology 2002;122:1242)
  • Only in a minority of patients do serologic markers help establish a definitive diagnosis of Crohn's disease (ASCA+ / pANCA-) or ulcerative colitis (ASCA- / pANCA+) (Gastroenterology 2002;122:1242)
Prognostic factors
Case reports
Treatment
Gross description
  • Ulcerative colitis-like colectomy specimen showing predominantly continuous, mucosa based inflammation but with 1 or more Crohn's disease-like features, such as:
    • Rectal sparing (except when treated or longstanding colitis)
    • Proximal skip lesions (except in pediatric population)
    • Ulceration in small bowel (even in the setting of backwash ileitis)
    • Deep fissuring ulceration (in the absence of fulminant colitis or history thereof)
    • Focal wall thickening
    • "Cobblestone" mucosa (deep linear ulcers with intervening normal mucosa)
  • Left sided colitis with discontinuous cecal or appendiceal orifice involvement (so called “cecal patch” or “periappendiceal red patch”) is a well recognized variant of ulcerative colitis and should not be designated as IBD, indeterminate type
  • References: Mod Pathol 2015;28:S30, J Clin Pathol 2004;57:1233, Histopathology 2007;50:83, J Clin Pathol 2009;62:201
Gross images

Contributed by John D. Paulsen, M.D. and Alexandros D. Polydorides, M.D., Ph.D.
IBD, indeterminate type

IBD, indeterminate type

IBD, indeterminate type, favor Crohn's disease

IBD, indeterminate type, favor Crohn's disease

Ulcerative colitis with cecal patch

Mimic: ulcerative colitis with cecal patch

Superficial (ulcerative colitis-like) Crohn's disease

Mimic: superficial
(ulcerative
colitis-like)
Crohn's disease

Microscopic (histologic) description
  • Active mucosal inflammation (same as IBD) (J Clin Pathol 2009;62:201):
    • Cryptitis (neutrophilic infiltration of crypt epithelium)
    • Crypt abscess (crypt with intraluminal aggregates of neutrophils)
    • Erosions
    • Ulceration
  • Chronic mucosal inflammation (same as IBD) (J Clin Pathol 2009;62:201):
    • Increased lymphoplasmacytic inflammation in the lamina propria with loss of gradient (i.e. lack of increased infiltrates near lumen)
    • Basal plasmacytosis (so called crypt shortfall)
  • Evidence of chronic injury (same as IBD) (J Clin Pathol 2009;62:201):
    • Paneth cell metaplasia (Paneth cells are normally absent distal to the mid transverse colon)
    • Crypt architectural distortion (branched, dilated and disorganized crypts)
  • Microscopic features that are still compatible with ulcerative colitis and do not warrant a diagnosis of Crohn's disease or IBD, indeterminate type (Mod Pathol 2015;28:S30, J Clin Pathol 2004;57:1233):
    • Cryptolytic granulomas
    • Backwash ileitis: active inflammation in the terminal ileum in the setting of at least similar in intensity active inflammation in the cecum; usually there is minimal or no chronicity in the ileum
  • Microscopic features considered major diagnostic criteria for Crohn's disease (Mod Pathol 2015;28:S30, J Clin Pathol 2004;57:1233):
    • True segmental disease of the colon (in untreated patients)
    • Absence of rectal involvement (in untreated patients)
    • Transmural lymphoid aggregates (away from deep mucosal ulcers)
    • Penetrating sinus tracts or fistulas (unrelated to other causes, e.g. diverticulitis)
    • Nonnecrotizing epithelioid cell granuloma (unrelated to infection or crypt rupture)
    • True small intestinal involvement unrelated to backwash ileitis
    • Perianal disease unrelated to other causes
  • Superficial (ulcerative colitis-like) Crohn's disease: nonnecrotizing epithelioid cell granulomas (unassociated with infection or crypt rupture) in the mucosa or lymph node of a specimen that is otherwise suggestive of ulcerative colitis (i.e. mucosa based chronic active colitis); a diagnosis of IBD, indeterminate type is preferred by some authors in these cases but an explanation as to the reason of the uncertainty should be included
  • Dysplasia / carcinoma: same as IBD (Inflamm Bowel Dis 2009;15:1076)
Microscopic (histologic) images

Contributed by John D. Paulsen, M.D. and Alexandros D. Polydorides, M.D., Ph.D.
Transmural chronic inflammation

Transmural chronic inflammation

Granuloma, pericolonic lymph node

Granuloma, pericolonic lymph node

Cryptolytic granuloma, ulcerative colitis

Mimic: cryptolytic granuloma, ulcerative colitis

Superficial (ulcerative colitis-like) Crohn's disease Superficial (ulcerative colitis-like) Crohn's disease

Mimic: superficial (ulcerative colitis-like) Crohn's disease

Fulminant ulcerative colitis

Mimic: fulminant ulcerative colitis

Sample pathology report
  • Terminal ileum, cecum, appendix and colon, total abdominal colectomy:
    • Severely active chronic colitis with continuous retrograde extension to the hepatic flexure (see comment)
    • Comment: The inflammation is generally mucosa based with extensive ulceration and associated transmural disease but diagnostic characteristics of Crohn's disease are lacking. Rectal sparing has been described clinically but should be confirmed histologically, if possible. The ileal segment and appendix are unremarkable.
    • The findings are best classified as inflammatory bowel disease, indeterminate type.
Differential diagnosis
  • Ulcerative colitis:
    • Diffuse mucosa based / superficial involvement of the colorectum
    • Well recognized patterns of ulcerative colitis with Crohn's-like features: cecal patch, cryptolytic granulomas, fulminant ulcerative colitis with transmural inflammation in areas of ulceration
  • Crohn's disease:
    • Transmural lymphoid aggregates in areas of nonulcerated mucosa, nonnecrotizing epithelioid cell granuloma (not associated with ruptured crypts), fissuring ulcers
  • Ischemic colitis:
    • Involvement of watershed areas (e.g. splenic flexure) mostly
    • Surface active inflammation / necrosis
    • Hyalinized lamina propria
    • Withered / shrunken crypts
  • Diversion colitis:
    • Prominent mucosa / submucosal follicular lymphoid hyperplasia
    • History of fecal stream diversion
  • Segmental colitis associated with diverticular disease (SCAD):
    • Limited to areas of the colon with known diverticulosis
Board review style question #1

A 26 year old woman with no significant past medical history is found to have severe colitis and undergoes total abdominal colectomy. The gross examination of the resection specimen reveals a predominantly left sided colitis with discontinuous involvement of the cecal and periappendiceal mucosa, as shown in the image above. Microscopic evaluation demonstrates a mucosa based, active chronic colitis without evidence of transmural inflammation. Several nonnecrotizing epithelioid cell granuloma associated with ruptured crypts are identified. Which of the following is true regarding the patient’s pathologic diagnosis?

  1. A diagnosis of Crohn's disease is warranted due to the presence of discontinuous colitis
  2. The findings are best classified as inflammatory bowel disease (IBD), type indeterminate because the specimen shows overlapping features of Crohn's disease and ulcerative colitis
  3. The findings are consistent with a recognized variant of ulcerative colitis
  4. The presence of nonnecrotizing epithelioid cell granuloma precludes a diagnosis of ulcerative colitis in this case
Board review style answer #1
C. The findings are consistent with a recognized variant of ulcerative colitis

The photograph shows a total abdominal colectomy specimen with a predominantly left sided colitis and discontinuous involvement of the cecal or periappendiceal mucosa. The gross and microscopic findings are convincing for a diagnosis of ulcerative colitis. It is well documented that patients with left sided ulcerative colitis may have discontinuous involvement of the periappendiceal or cecal mucosa, known as a cecal patch. A diagnosis of IBD, indeterminate type should be avoided when dealing with well recognized variants of IBD, such as ulcerative colitis with cecal patch and superficial (ulcerative colitis-like) Crohn's disease. Nonnecrotizing epithelioid cell granulomas associated with ruptured crypts (cryptolytic granulomas) are known to occur in cases of ulcerative colitis and often result in diagnostic difficulties; however, this finding should not preclude a diagnosis of ulcerative colitis.

Comment Here

Reference: Inflammatory bowel disease, indeterminate type

Inflammatory cap polyp
Definition / general
  • First described in an abstract in 1985 (Br J Surg 1985; 72 (Suppl) S133)
  • Rare
  • Etiology unknown, but may be associated with mucosal prolapse (Gut 1993;34:562)
  • Often presents with mucoid or bloody diarrhea and hypoproteinemia
  • Endoscopy: multiple sessile rectosigmoid polyps with adherent white flecks with normal intervening mucosa (Endoscopy 2001;33:262)
Treatment
Gross description
  • Multiple, distinctive, inflammatory rectosigmoid polyps
Gross images

Images hosted on other servers:

Multiple colonic polyps

Microscopic (histologic) description
  • Inflamed mucosa with tortuous, elongated crypts attenuated towards the mucosal surface with abundant inflammation in the lamina propria and characteristic "cap" of inflamed and ulcerated granulation tissue on the mucosal surface
Microscopic (histologic) images

Contributed by Andrea L. Wiens, D.O. and Janet E. Roepke, M.D., Ph.D.

Inflammatory polyp of colon secondary to mucosal prolapse

Differential diagnosis
  • Amebic colitis
  • Antibiotic associated colitis
  • Inflammatory bowel disease

Inflammatory fibroid polyp
Definition / general
  • Rare mesenchymal polypoid neoplasms that arise anywhere throughout the GI tract but most commonly in the stomach and small intestine
Essential features
  • Benign mesenchymal polypoid neoplasm centered in the submucosa
  • Rare in the large bowel
  • Proliferation of bland spindle cells with a variable myxoid background and varying degree of inflammatory infiltration
  • Association with activating PDGFRA gene mutation
Terminology
  • Formerly known as Vanek tumor (no longer recommended)
  • Terminological confusion in older literature (e.g., granuloblastoma, gastric fibroma with eosinophilic infiltration, eosinophilic granuloma)
  • Inflammatory fibroid polyp (IFP) is the recommended term
ICD coding
  • ICD-10: K51.419 - inflammatory polyps of colon with unspecified complications
Epidemiology
Sites
  • Most frequent location is the stomach (> 65% in a recent meta analysis of 417 cases), mainly the antrum (In Vivo 2021;35:81)
  • < 10% of cases arise in the large bowel
Pathophysiology
  • For a long time, it was considered reactive in nature (autoimmune, allergic)
  • Discovery of overexpression and mutations in platelet derived growth factor receptor alpha (PDGFRA) has led to considering inflammatory fibroid polyp a neoplastic process
Etiology
Clinical features
  • Symptoms largely depend on location
  • For large bowel lesions, abdominal pain and acute intestinal obstruction are the most frequent presenting symptoms
  • Nonmalignant potential
  • Some familial cases and occasional association with GI malignancies (Trop Gastroenterol 2014;35:196, Ann Diagn Pathol 2012;16:148)
  • Not usually associated with peripheral eosinophilia
  • No endoscopic surveillance is required after the histological diagnosis is confirmed
Diagnosis
  • Histologic
Prognostic factors
  • Good prognosis with no risk of malignization
Case reports
  • 23 year old woman with iron deficiency anemia and a 4.5 cm polypoid lesion in the descending colon removed with a needle knife (Yonsei Med J 2008;49:680)
  • 50 year old woman with bloating and abdominal pain with a diagnosis of intussusception undergoes an emergency right hemicolectomy (Medicina (Kaunas) 2022;58:310)
  • 53 year old man with diabetes and hypercholesterolemia undergoes a routine colonoscopy, which reveals a 4 cm lobular mass in the proximal rectum removed by transanal resection (J Surg Case Rep 2019;2019:rjz164)
  • 65 year old woman with mild abdominal pain for 3 years and a large elongated mass in the cecum (Chin Med J (Engl) 2017;130:2130)
Treatment
  • Resection is the preferred therapy
  • Endoscopic resection is the preferred option, either polypectomy or endoscopic submucosal or radical resection
  • Need of surgical resection in some complicated cases
  • No further therapy is necessary
Clinical images

Contributed by M.J. Fernández-Aceñero, M.D., Ph.D.

Endoscopic lesion

Gross description
  • Solitary intraluminal, usually sessile polypoid lesion
  • Frequently ulcerated
  • Can reach large size (giant inflammatory fibroid polyp measuring > 4 cm)
  • As opposed to usual gastric inflammatory fibroid polyp, large bowel lesions can infiltrate through the wall, reaching neighboring structures and mimicking malignancy (Kaohsiung J Med Sci 2013;29:460)
Gross images

Contributed by Taofic Mounajjed, M.D.

Pedunculated cecal polyp

Cut surface with white color

Microscopic (histologic) description
  • Mostly centered in the submucosa but can infiltrate lamina propria
  • Well demarcated but encapsulated
  • Proliferation of bland mesenchymal cells
  • Background with small vessel proliferation, little collagen, inflammatory infiltrates and variable degrees of edema
  • Distinctive onion skin arrangements of spindled cells around vessels
  • Vessels can be ectatic
  • Inflammatory infiltrate rich in eosinophils
  • 5 histopathological patterns proposed, probably representing a spectrum of disease over time:
    • Classical fibrovascular
    • Nodular
    • Sclerotic
    • Edematous
    • Atypical (very rare)
  • Scarce mitosis, never atypical
Microscopic (histologic) images

Contributed by M.J. Fernández-Aceñero, M.D., Ph.D.

Typical features

Large bowel lesion

Typical growth pattern

Edema

Myxoid change


Stain for collagen

Mitotic activity

CD34

SMA

Virtual slides

Images hosted on other servers:

Inflammatory fibroid polyp

Positive stains
Negative stains
Electron microscopy description
  • No recent reports regarding electron microscopy features of this lesion
  • Older reports indicate a combination of fibroblastic and histiocytic features in the tumor cells
  • Putative origin in primitive submucosal stromal cells
Molecular / cytogenetics description
  • Activating mutations in exons 12, 14 and 18 of PDGFRA in > 50% of inflammatory fibroid polyps, with described differences according to location (Mod Pathol 2009;22:1049)
  • PDGFRA mutations seem less frequent in large bowel lesions (Pathol Int 2018;68:205)
  • Descriptions of germline mutations in PDGFRA leading to familial cases of inflammatory fibroid polyp or gastrointestinal stromal tumor (GIST) (Cancer Genet 2021;256:106)
Sample pathology report
  • Large bowel, endoscopic polypectomy
    • Inflammatory fibroid polyp (1.3 cm) (see comment)
    • Comment: The lesion is centered in the submucosa and is composed of bland proliferation of spindled cells with prominent eosinophils. The lesional cells express CD34 and are negative for CD117 and DOG1.
Differential diagnosis
Board review style question #1

What is the large bowel lesion in the picture above characterized by?

  1. CD34 is always positive
  2. Eosinophils are always abundant in the stroma
  3. It is always sporadic
  4. It is polypoid and originates in the submucosa
  5. Its most frequent location is the transverse colon
Board review style answer #1
D. It is polypoid and originates in the submucosa. Inflammatory fibroid polyps are usually polypoid and originate in the submucosa, as opposed to other kind of stromal lesions, like gastrointestinal stromal tumors or schwannoma.

Comment Here

Reference: Inflammatory fibroid polyp
Board review style question #2
Which statement is true regarding IHC for PDGFRA in inflammatory fibroid polyps?

  1. Allows differential diagnosis with GIST
  2. Always positive
  3. Always negative
  4. Confirms reactive nature of the process
  5. Does not always reflect a mutation in PDGFRA gene
Board review style answer #2
E. Does not always reflect a mutation in PDGFRA gene. IHC for PDFGRA can be positive in cases with no shown mutation of PDGFRA gene.

Comment Here

Reference: Inflammatory fibroid polyp

Inflammatory myofibroblastic tumor
Definition / general
  • Low grade mesenchymal neoplasm that can arise in the colonic wall or mesentery, usually of children
  • See also topic in Soft tissue chapter
Essential features
  • Mesenchymal neoplasm, often pediatric, with spindled cells admixed with inflammation
  • t(2;5) (TPM3-ALK) translocation
  • Rare aggressive variant called epithelioid inflammatory fibroblastic sarcoma
Terminology
  • Older terms include called inflammatory pseudotumor, inflammatory fibrosarcoma and plasma cell granuloma
  • Epithelioid inflammatory fibroblastic sarcoma refers to a rare, aggressive variant with unique morphologic and molecular features (Am J Surg Pathol 2011;35:135)
Sites
  • Can arise in the colonic wall or the mesentery
  • Most common site in gastrointestinal tract is stomach
Clinical features
  • Patients usually in the pediatric age range (Am J Surg Pathol 2007;31:509)
  • Symptoms include anemia, abdominal pain, fever, weight loss and high sedimentation rate (J Pediatr Surg 2001;36:169)
  • May recur but metastases rare; these outcomes may be more common in epithelioid inflammatory fibroblastic sarcoma
Radiology images

Images hosted on other servers:
Missing Image

Polypoid lesion, on imaging

Case reports
Clinical images

Images hosted on other servers:
Missing Image

Endoscopic view of polypoid lesion

Gross description
  • Circumscribed, often polypoid
Gross images

Images hosted on other servers:
Missing Image

2 cm polypoid lesion with central depression

Missing Image

3.9 cm fungating yellow mass

Missing Image

2 year old boy with colonic mass causing anemia

Microscopic (histologic) description
  • Bland spindle cells with abundant amphophilic cytoplasm and variably prominent nucleoli
  • Lymphoplasmacytic infiltrate with polyclonal plasma cells
  • Background may show myxoid change or laminated / whorled fibrosis
  • Epithelioid inflammatory myofibroblastic sarcoma: similar, except cells are more epithelioid, with large nucleoli
Microscopic (histologic) images

Contributed by Raul S. Gonzalez, M.D.
Missing Image Missing Image Missing Image

Not necessarily colon



Contributed by Michael Feely, D.O.
Missing Image Missing Image Missing Image Missing Image Missing Image

Epithelioid inflammatory myofibroblastic sarcoma



Images hosted on other servers:
Missing Image

Various images (H&E and IHC)

Missing Image

Spindle cells
in hyaline
stroma, with
inflammation

Missing Image

Positive
for vimentin
and smooth
muscle actin

Missing Image

Inflammatory cells and spindle cells

Positive stains
Negative stains
Molecular / cytogenetics description
  • Abnormalities of 2p23, usually t(2;5) (TPM3-ALK) translocation, in approximately half of cases; more common in children (Mod Pathol 2001;14:569)
  • Epithelioid inflammatory myofibroblastic sarcoma: t(2;2) RANBP2-ALK translocation
Sample pathology report
  • Ascending colon, resection:
    • Inflammatory myofibroblastic tumor (3.1 cm) (see comment)
    • Margins of resection unremarkable.
    • Two benign lymph nodes.
    • Comment: An immunostain for ALK1 is positive in the tumor.
Differential diagnosis
Board review style question #1

A young male patient presents with abdominal pain. Imaging reveals a large mesenteric mass, which is resected. An image of the tumor’s histology is seen above. Immunohistochemistry shows ALK1 staining of the nuclear membranes. What molecular abnormality is present in this tumor?

  1. ALK1 deletion
  2. ALK1 inversion
  3. t(2;2) RANBP2-ALK translocation
  4. t(2;5) TPM3-ALK translocation
Board review style answer #1
C. t(2;2) RANBP2-ALK translocation. This is an epithelioid inflammatory myofibroblastic sarcoma.

Comment Here

Reference: Inflammatory myofibroblastic tumor

Inflammatory polyp
Definition / general
Essential features
  • Nonneoplastic colon polyp composed of inflamed mucosa
  • Typically shows surface erosion with surrounding granulation tissue and epithelial distortion
Terminology
  • Inflammatory polyp as a diagnosis is generally used to describe small foci of nonspecifically inflamed colonic mucosa or inflammatory pseudopolyps
  • Inflammatory polyp as a category includes several subtypes, including:
ICD coding
  • ICD-10: K51.4 - inflammatory polyps of colon
Epidemiology
  • Typically second and third decades for inflammatory bowel disease; incidence range of 10 - 20% in ulcerative colitis patients (World J Gastroenterol 2017;23:1541)
  • May occur in older patients with peripheral vascular disease
Sites
  • Can arise anywhere in the colon, especially at the ileocecal region in Crohn’s disease
  • May form at anastomotic sites
Pathophysiology
  • Believed to be secondary to repeated bouts of intense inflammation
  • Formation of inflammatory polyps may be related to increases in C reactive protein, C4 and procollagen III peptide (World J Gastroenterol 2003;9:619)
Clinical features
  • Sporadic inflammatory polyps are usually incidental at colonoscopy
  • May present with intussusception or obstructive symptoms
  • Presence of pseudopolyps in inflammatory bowel disease may represent recent flare, although lesions are found in active or dormant disease (World J Gastroenterol 2017;23:1541)
    • Also may be related to arthropathy or other extracolonic symptoms (Lancet 1969;2:555)
Case reports
Treatment
  • Typically treated endoscopically via polypectomy
  • Examples related to inflammatory bowel disease may improve with infliximab (J Crohns Colitis 2010;4:707)
  • Argon plasma coagulation or ablation for bleeding control
  • Surgical resection if profuse bleeding, obstruction or intussusception
Gross description
  • Usually sessile and less than 3 cm
  • May be pedunculated or filiform
Gross images

Images hosted on other servers:

Inflammatory pseudopolyps in ulcerative colitis

Microscopic (histologic) description
  • Often consists of normal colonic mucosa in a polypoid configuration, with increased inflammation (expanded lamina propria and crypt abscesses or cryptitis)
  • Epithelium can show various degrees of surface erosion, crypt distortion / dilation or hyperplasia, along with reactive nuclear features within the mucosal epithelial cells
  • May consist entirely of granulation tissue (abundant thin walled and dilated vessels surrounded by mixed neutrophilic and lymphoplasmacytic inflammation)
  • Reactive stromal cells may be markedly pleomorphic and mimic sarcoma
  • Cases associated with inflammatory bowel disease may rarely show epithelial dysplasia
Microscopic (histologic) images

Contributed by Andrew L.J. Dunn, M.D.

Dense inflammation in lamina propria

Surface mucosal ulceration

Surface erosion with granulation tissue

Cryptitis

Negative stains
Molecular / cytogenetics description
  • Usually no abnormalities
Videos

Inflammatory polyp on colonoscopy

Sample pathology report
  • Sigmoid colon, polypectomy:
    • Inflammatory polyp
Differential diagnosis
Board review style question #1
Which of the following is not a typical feature of colonic inflammatory polyps?

  1. Crypt distortion / branching
  2. Granulation tissue changes
  3. Microsatellite instability
  4. Surface mucosal erosion
Board review style answer #1
C. Microsatellite instability. Inflammatory polyps are a benign process with various degrees of mucosal erosion, increased vascular density similar to granulation tissue and architectural changes.

Comment here

Reference: Inflammatory polyp

Intestinal neuronal dysplasia
Definition / general
  • Also known as neuronal colonic dysplasia, hyperganglionosis, pseudo or variant Hirschsprung disease
  • Type A: hypoplastic or aplastic sympathetic innervation
  • Type B: hypoplastic or plastic parasympathetic innervation (Virchows Arch A Pathol Anat Histopathol 1992;420:171); has disturbed submucous plexus development (Semin Pediatr Surg 2009;18:206)
  • A controversial entity; should diagnose only if no other diagnosis, no obstruction and multiple adequate biopsies (30 sections) of submucosa and muscularis propria available; may be due to delayed maturity of enteric nervous system as 95% have normal gut motility within 1 year
  • Associated with neurofibromatosis 1 and MEN2b syndromes
  • Associated with hypoganglionosis of myenteric plexus or aganglionosis of rectum
Diagnosis
Diagnostic criteria:
  • Biopsies 8 - 10 cm above dentate line, sufficient submucosa, 15 - 20% of ganglia are giant ganglia with more than 8 nerve cells in submucosa of 30 serial sections (Eur J Pediatr Surg 2004;14:384)
  • Hyperganglionosis of submucous plexus, giant ganglia and rectal biopsies show either ectopic ganglia, increased acetylcholinesterase activity in lamina propria or increased acetylcholinesterase in nerve fibers around submucosal blood vessels (J Pediatr Surg 2001;36:777, Indian J Pathol Microbiol 2004;47:4)
Case reports
Microscopic (histologic) description
  • Resembles Hirschsprung disease with hyperplasia of myenteric nerves and increased acetylcholinesterase staining but with occasional (15 - 20%) submucosal giant ganglia (containing 7 - 10 neurons vs. 3 - 5 normally) and isolated ganglion cells in submucosa
  • Note: giant ganglia by themselves are not specific (Virchows Arch 1998;432:103)
Microscopic (histologic) images

Images hosted on other servers:
Missing Image

Various images

Positive stains

Intestinal spirochetosis
Definition / general
  • Colonization of the colonic epithelium by the filamentous nontreponemal anaerobic spirochetes, Brachyspira aalborgi or Brachyspira pilosicoli
Essential features
  • Colonization of the colonic epithelium by spirochetes of the Brachyspira species
  • Most commonly asymptomatic but may present with diarrhea or abdominal pain / cramps
  • Diagnosed by histologic examination of colorectal mucosal biopsies that demonstrate a basophilic fringe on the surface epithelium
  • May be confirmed with silver stains (Warthin-Starry, Steiner) or Treponema pallidum immunohistochemistry
  • Typically treated with metronidazole, if symptomatic; otherwise, may be observed
ICD coding
  • ICD-10:
    • A08.8 - other specified intestinal infections
    • A69.9 - spirochetal infection, unspecified
Epidemiology
  • Human intestinal spirochetosis (HIS) was originally recognized in 1967; however, the pathogenicity is still unclear
  • HIS has a global distribution and Brachyspira species have been detected on nearly every continent, with the exception of Antarctica
  • Prevalence rate is highly variable and is not entirely clear from the current literature; however, there appears to be lower prevalence rates (ranging from 0.2% to 3.2%) among healthy people residing in developed countries and higher prevalence rates (ranging from 10.8% to 64.8%) in developing countries
  • In developed nations, the prevalence rates are thought to be highest among men who have sex with men (MSM) and patients with HIV
  • More common in men and patients in the fourth or fifth decades of life
  • Although rare, it may occur in pediatric patients
  • No confirmed seasonality
  • References: Br Med J 1967;3:718, J Gastroenterol Hepatol 2022;37:1222, J Bacteriol 2019;201:e00465, Ann Clin Lab Sci 2020;50:386, Annals Diagn Pathol 2015;19:414, Pediatr Gastroenterol Hepatol Nutr 2019;22:193
Sites
Pathophysiology
Clinical features
  • In most children and adults, human intestinal spirochetosis is asymptomatic and often an incidental histologic finding on endoscopic biopsy material
  • In symptomatic patients, the 2 most common symptoms are diarrhea (often chronic and watery) (51%) and abdominal cramps or pain (46%)
    • Other symptoms may include bloating, weight loss, constipation, nausea, alternating constipation / diarrhea or irritable bowel syndrome-like symptoms and rectal or perirectal bleeding
    • Pediatric patients may develop severe symptoms
  • Brachyspira pilosicoli has a broader host range including swine, birds, humans and nonhuman primates
    • Known cause of diarrhea in pigs and chickens
    • Usually asymptomatic in humans but may cause diarrhea, abdominal pain or perirectal bleeding
    • Critically ill patients may have spirochetemia
  • Brachyspira aalborgi has only been detected in humans, nonhuman primates and opossums
    • Typically, not associated with histologic findings or gastrointestinal symptoms
    • Some believe this is simply a commensal organism
  • Some studies have found an association between human intestinal spirochetosis and juvenile polyps in children
  • References: Ann Clin Lab Sci 2020;50:386, J Gastroenterol Hepatol 2022;37:1222, Pediatr Gastroenterol Hepatol Nutr 2019;22:193, J Bacteriol 2019;201:e00465
Diagnosis
  • Gold standard for diagnosis is histologic examination of an endoscopic colon or rectal biopsy
  • Noninvasive diagnostic methods are not currently routine
  • Reference: J Gastroenterol Hepatol 2022;37:1222
Case reports
Treatment
Gross description
Microscopic (histologic) description
Microscopic (histologic) images

Contributed by Aaron R. Huber, D.O. and @RaulSGonzalezMD on Twitter

Basophilic fringe

Silver stain

Treponema IHC

Intestinal spirochetosis Intestinal spirochetosis Intestinal spirochetosis

Intestinal spirochetosis

Virtual slides

Images hosted on other servers:
Missing Image

54 year old man with intestinal spirochetosis

Positive stains
  • Organisms may be highlighted by silver stains (Steiner, Warthin-Starry) or immunohistochemical stains for Treponema pallidum, which crossreacts with Brachyspira species
Electron microscopy description
  • Nontreponemal spirochetes are attached perpendicularly to the luminal border of enterocytes
    • The head of the spirochetes anchors between microvilli, while the tail is directed towards the colonic lumen
    • Creates a confluent forest of spirochetes on the surface epithelium, while the microvilli are reduced
  • References: J Gastroenterol Hepatol 2022;37:1222, J Bacteriol 2019;201:e00465
Electron microscopy images

Images hosted on other servers:
Missing Image

Spirochetes attach end on to the colonic epithelium

Sample pathology report
  • Colon, random, biopsy:
    • Intestinal spirochetosis (see comment)
    • Comment: H&E stained sections demonstrate a basophilic fringe on the surface of the colonic epithelium that is highlighted by a Warthin-Starry stain for spirochetes. The findings are compatible with intestinal spirochetosis.
Differential diagnosis
Board review style question #1

A 50 year old man with chronic watery diarrhea had random colon biopsies taken during a colonoscopy, which demonstrated macroscopically normal mucosa throughout the colon. A representative H&E image of his colon biopsies is shown above, as well as a Warthin-Starry stain performed to confirm the morphologic impression. What is the most likely diagnosis?

  1. Enteroadherent Escherichia coli
  2. Intestinal spirochetosis
  3. Normal colonic mucosa
  4. Normal colonic mucosa with prominent glycocalyx
Board review style answer #1
B. Intestinal spirochetosis. The H&E image demonstrates the classic basophilic fringe on the surface epithelium, which is characteristic of intestinal spirochetosis. This is confirmed with the Warthin-Starry stain that highlights the organisms. The findings of intestinal spirochetosis may be subtle and appear like normal colonic mucosa. It is important to evaluate the surface epithelium for the presence of the spirochetes in random colon biopsies performed for diarrhea. Enteroadherent E. coli would be strongly gram negative, if performed.

Comment Here

Reference: Intestinal spirochetosis
Board review style question #2
Which of the following statements is true regarding human intestinal spirochetosis (HIS)?

  1. All patients diagnosed histologically with human intestinal spirochetosis should be treated with antibiotics
  2. A majority of patients with human intestinal spirochetosis are symptomatic
  3. The most common symptoms in symptomatic patients are diarrhea and abdominal pain / cramps
  4. The most commonly utilized treatment for symptomatic patients is clarithromycin
  5. The pathogenesis, transmission and risk factors for human intestinal spirochetosis are clearly understood and delineated
Board review style answer #2
C. The most common symptoms in symptomatic patients are diarrhea (which is commonly chronic and watery) and abdominal pain / cramps. Most patients with human intestinal spirochetosis diagnosed histologically are asymptomatic and do not require treatment but may be followed or observed. Metronidazole is the most commonly used antibiotic therapy in symptomatic patients. The pathogenesis, transmission and risk factors for human intestinal spirochetosis are still unclear.

Comment Here

Reference: Intestinal spirochetosis

Intramucosal carcinoma
Definition / general
  • Malignant change confined to mucosa (considered pTis in AJCC staging)
  • Proper cutoff between high grade dysplasia and intramucosal carcinoma is not entirely clear
Essential features
  • Adenocarcinoma solely within the mucosal layer of the colon
  • Usually within a polyp but may occur in flat mucosa in inflammatory bowel disease
  • Term typically not used by Western pathologists, who prefer high grade dysplasia
Terminology
  • Controversial term that reflects differing thresholds for malignancy between the West (higher threshold) and Japan (lower threshold), as Japanese pathologists often diagnose as malignant what Western pathologists might call high grade dysplasia
  • United States Preventive Services Task Force specifically recommends against using the term intramucosal carcinoma for lesions confined to the mucosa (Gastrointest Endosc 2020;92:997)
Clinical features
Case reports
Treatment
Clinical images

Images hosted on other servers:
Missing Image

Mass in colonoscopy

Missing Image

Subpeduculated polyp

Gross description
  • May appear as a flat / ulcerated lesion or arise in a polyp
Gross images

Images hosted on other servers:
Missing Image

Resected intramucosal carcinoma

Microscopic (histologic) description
  • Glandular proliferation showing high grade nuclei and architectural complexity, extending no deeper than the muscularis mucosae
  • Necrosis, desmoplasia and single malignant cells may suggest intramucosal carcinoma over high grade dysplasia but this remains subjective
  • Often arises in an adenoma but may occur in flat colonic mucosa (such as in patients with inflammatory bowel disease)
  • May rarely show signet ring cells (Virchows Arch 2009;454:115)
Microscopic (histologic) images

Contributed by Raul S. Gonzalez, M.D.
Intramucosal carcinoma Intramucosal carcinoma

Intramucosal carcinoma



Images hosted on other servers:
Missing Image

Intramucosal adenocarcinoma

Sample pathology report
  • Rectum, mass, biopsy:
    • Tubulovillous adenoma with extensive high grade dysplasia (see comment)
    • Comment: The findings are compatible with an interpretation as intramucosal carcinoma, though this concept is rarely applied to colorectal lesions. The sample is relatively superficial and an unsampled invasive process cannot be excluded.
Differential diagnosis
Board review style question #1
Which of the following is true about intramucosal carcinoma of the colorectum?

  1. It only arises within traditional serrated adenomas
  2. Roughly 10% of cases metastasize to lymph nodes despite early stage
  3. There are well defined histologic criteria to distinguish it from high grade dysplasia
  4. Usage of the term differs between Eastern and Western pathologists
Board review style answer #1
D. Usage of the term differs between Eastern and Western pathologists

Comment Here

Reference: Intramucosal carcinoma

Ipilimumab associated colitis
Definition / general
  • Colitis caused by use of the anti-CTLA4 medication, ipilimumab
Essential features
  • Colitis is a somewhat common side effect of ipilimumab
  • Can be mild (diarrhea) or severe (perforation, death)
  • Histology: increased intraepithelial lymphocytes, increased apoptosis, cryptitis
Epidemiology
  • Occurs in up to 29% of patients taking ipilimumab, an immunologic agent used for melanoma and other malignancies (Melanoma Res 2015;25:321)
Sites
Pathophysiology
  • Causes dysregulation of gastrointestinal mucosal immunity
Clinical features
  • Diarrhea, colitis
Radiology description
Prognostic factors
Case reports
Treatment
  • Steroids are usually effective but anti-TNF agents or surgery may be required for severe cases (Dig Dis Sci 2016;61:2132)
Clinical images

Image hosted on other servers:
Missing Image

Ipilimumab associated colitis
on colonoscopy

Gross images

Image hosted on other servers:
Missing Image

Cecal perforation due to severe colitis

Microscopic (histologic) description
  • Lymphoplasmacytic expansion of lamina propria; increased apoptosis and intraepithelial lymphocytes; cryptitis and crypt elongation (Am J Surg Pathol 2008;32:1130)
  • Severe cases may show ulceration/perforation, pseudopolyp formation and crypt dilation, without lymphocytosis or apoptosis (J Clin Gastroenterol 2013;47:781)
Microscopic (histologic) images

Contributed by Raul S. Gonzalez, M.D. and @SueEPig on Twitter
Missing Image

Acute inflammation with focal cryptitis

Missing Image

Broad mucosal ulceration

Missing Image

Increased apoptosis and lymphocytosis

Missing Image

Lymphoplasmacytic expansion of lamina propria


Ipilimumab associated colitis Ipilimumab associated colitis

Ipilimumab associated colitis

Ipilimumab associated colitis Ipilimumab associated colitis

Ipilimumab associated colitis

Positive stains
  • Immunohistochemistry (CD3, CD4, CD8) shows increase in T cell subsets
Sample pathology report
  • Sigmoid colon, biopsy:
    • Colonic mucosa with mild acute inflammation, expanded lamina propria, increased intraepithelial lymphocytes and focally increased apoptosis (see comment)
    • Comment: The patient's use of ipilimumab is noted. The findings are most consistent with ipilimumab associated colitis.
Differential diagnosis
Board review style question #1
Which of the following is true about gastrointestinal tract injury secondary to ipilimumab use?

  1. It can cause ulceration and perforation
  2. It is an extremely rare side effect
  3. It only occurs in the colon
  4. The main histologic findings are Paneth cell metaplasia and a thickened basement membrane
Board review style answer #1
A. It can cause ulceration and perforation

Comment Here

Reference: Ipilimumab associated colitis
Board review style question #2
Ipilimumab associated colitis Ipilimumab associated colitis


A 55 year old man with a history of metastatic melanoma presents with diarrhea and undergoes colonoscopy, which finds musical erythema. A biopsy is taken, with histology shown above. What is the most likely diagnosis?

  1. Ipilimumab induced colitis
  2. Lymphocytic colitis
  3. Segmental colitis associated with diverticulosis
  4. Ulcerative colitis
Board review style answer #2
A. Ipilimumab induced colitis

Comment Here

Reference: Ipilimumab associated colitis

Ischemic colitis
Definition / general
  • Ischemic colitis is a leading cause of acute abdomen in elderly patients
  • Gross and microscopic findings depend on the severity and duration of ischemia
  • Classic findings include superficial mucosa necrosis / regeneration, withered crypts and lamina propria hyalinization
Essential features
  • Gross and microscopic spectrum of ischemic colitis varies by the duration and extent of ischemia
  • Classic ischemic type mucosal injury pattern is described as superficial mucosa necrosis / regeneration, withered crypts and lamina propria hyalinization; this histology is not specific for ischemia and may be seen with infections (C. difficile, E. coli O157:H7), medications and inflammatory bowel disease
  • In cases with severe sudden ischemia, the affected segment of bowel is infarcted with red, congested or grayish dusky discoloration with transmural necrosis
Terminology
  • Ischemic colitis
  • Noninfectious colitis
ICD coding
  • ICD-10: K55.9 - Vascular disorder of intestine, unspecified
Epidemiology
Sites
  • Watershed areas most commonly affected (splenic flexure and rectosigmoid colon); the right colon is involved in 20 - 25% of cases (WMJ 2016;115:196)
Pathophysiology
  • 2 categories: vascular factors and bowel factors, both leading to inadequate blood flow to the colonic wall and causing ischemic injury (WMJ 2016;115:196)
  • Vascular factors:
    • Transient hypoperfusion is the most common cause
    • Vasospasm of the colonic vessels is another mechanism, either due to systemic hypoperfusion which shunts blood away from intestine or due to vasoconstrictive effects of certain drugs and substances
    • Hypercoagulable states, such as coagulation factor deficiencies
    • Vasculitis, such as systemic lupus erythematous and antiphospholipid syndrome
  • Bowel factors:
    • Constipation, irritable bowel syndrome (IBS), fecal impaction, colonic obstruction and any other condition that increases the intraluminal pressure, which may compromise the blood flow the colonic wall, leading to ischemic injury
  • Extent of ischemic changes in the intestines depends on the severity, rate of onset and duration of ischemia
  • Transient, reversible vascular obstruction often leads to mild superficial changes, whereas sudden, complete obstruction results in complete infarction of the affected segment with transmural necrosis (see colon infarct)
Etiology
  • Most cases represent nonocclusive ischemia caused by a sudden decrease in blood flow in the small arterioles of the colon, resulting from a low volume state (Front Surg 2017;4:47)
  • Risk factors include atherosclerotic disease, aortic surgery, coronary artery bypass graft, atrial fibrillation, oral contraceptives, hereditary coagulopathies, cocaine abuse, strenuous physical activity, bacterial pathogens (cytomegalovirus and E. coli), hypertension, diabetes mellitus and hypoalbuminemia
  • Constipation, hyperuricemia, smoking may contribute to disease development in young adults
  • Other risk factors in younger patients: collagen vascular diseases, amyloidosis or cocaine use
  • Idiopathic myointimal hyperplasia of mesenteric veins (Am J Surg Pathol 2017;41:1657)
Clinical features
  • Classic symptoms in elderly are abdominal pain, hematochezia and leukocytosis
  • Left sided ischemic colitis presents with acute abdominal pain and lower gastrointestinal bleeding
  • Right sided ischemic colitis presents with severe abdominal pain but rarely bloody diarrhea
  • Full thickness involvement of the colonic wall can lead to perforation with peritonitis and sepsis
  • Reference: Front Surg 2017;4:47
Diagnosis
  • CT scan is the investigation of choice (BMJ 2016;355:i6600)
  • CT angiogram finds abnormalities in the vasculature (Front Surg 2017;4:47)
  • Xrays can help in detecting perforation by demonstrating gas under the diaphragm
  • Colonoscopy should be performed within 48 hours to visualize mucosa; it can show edematous and fragile mucosa, segmental erythema, erosions, hemorrhages and ulceration
  • An endoscopic finding, the so called colon single stripe sign represents a single line of erythema with erosion or ulceration placed along the longitudinal axis of the large bowel; this sign is highly suggestive of ischemic injury and indicates a milder disease
  • Pseudomembranes can be seen, characterized by elevated yellow-white nodules or plaques on the mucosal surfaces of the colon biopsy (Dis Mon 2015;61:181)
Laboratory
  • Complete blood count, metabolic panel and liver function tests are required to assess the physiologic status of the patient (Front Surg 2017;4:47)
  • C reactive protein and neutrophil count are likely to be raised
  • Serum lactate levels may be raised as a result of systemic dysfunction and hypoperfusion (BMJ 2016;355:i6600)
Radiology description
  • CT may appear normal in mild disease (Front Surg 2017;4:47)
  • In more advanced but nongangrenous forms of the disease, CT will often show colonic wall thickening, thumbprinting, colonic dilatation and pericolic fat stranding, with or without the presence of ascitic fluid
  • Double halo or target sign may be present
  • Emboli or thrombus occluding the inferior mesenteric artery is occasionally seen while the corresponding colonic wall appears thin due to the lack of reperfusion
  • Air within the mesenteric or the portal venous system or pneumatosis coli is a serious finding indicating bowel infarction
Prognostic factors
  • If the ischemia is limited to the mucosa, the disease will be transient and complete recovery is expected
  • Ischemia involving the muscularis propria can lead to scarring and stricture formation
  • If ischemia involves full thickness of the bowel wall, it may lead to gangrene and perforation of the colonic wall with peritonitis and sepsis
  • Reference: Front Surg 2017;4:47
Case reports
Treatment
  • Transient ischemia is managed conservatively and has a relatively good prognosis (Front Surg 2017;4:47)
  • Conservative therapy includes fluid resuscitation, optimizing cardiac output, bowel rest and discontinuing the etiologic agent
  • Antibiotic therapy can be started
  • For severe disease, urgent laparotomy and removal of the necrotic part of the colon is recommended
Clinical images

Images hosted on other servers:

Colonoscopic findings

Endoscopic images

Gross description
  • Ulcers, pseudopolyps
  • Frank blood or dark mucus in lumen
  • Segmental thinning in areas of full thickness infarction or gangrene
  • Tan-brown dusky areas of the bowel mucosa
  • Pseudomembranous appearance in exudative cases
  • Perforation can be identified in severe cases
  • Fibrosis and stricture formation in late cases
  • When grossing specimens, carefully dissect blood vessels and submit numerous sections to detect vascular lesions
  • Rarely, ischemic colitis can produce mass forming lesions (Am J Surg Pathol 2015;39:1275)
  • References: Dig Dis Sci 2009;54:2009, World J Gastroenterol 2008;14:7302
Gross images

Contributed by Gagandeep Kaur, M.D. and Monika Vyas, M.D.

Ischemic bowel

Ischemic bowel with pseudomembranes

Gangrenous bowel

Microscopic (histologic) description
  • Early transient ischemia may result in mild changes, such as mucosal edema, denudation and congestion
  • With more advanced ischemia, superficial mucosal necrosis ensues, which results in crypt damage (withering) or loss and hyalinization of the lamina propria
  • A reduced number and size of crypts and fibrosis in lamina propria suggests a more severe injury
  • Necrosis, ulceration and granulation tissue extend into submucosa and surrounding smooth muscle fibers of muscularis mucosae in more severe cases
  • Acute cases may exhibit acute inflammatory exudate (pseudomembranes) on the mucosa, even when noninfectious
  • Atypical reactive or degenerative changes may be seen, mimicking dysplasia or malignancy (e.g. pseudosignet ring cells)
  • Hemosiderin / hemorrhage and edema in lamina propria
  • May see crypt abscesses (especially in cases with reperfusion); however, fissures, lymphoid follicles and granulomas are absent
  • In patients with chronic ischemia, histologic findings may represent a mixture of ischemic features described above, along with foci of chronic mucosal injury, such as crypt architectural distortion, Paneth cell metaplasia, basal lymphoplasmcytosis and active inflammation, mimicking inflammatory bowel disease
  • Vessels may show necrotizing phlebitis, hyaline thrombi or capillary microthrombi, cholesterol crystals (atheroembolism) or myointimal hyperplasia
  • Cholesterol crystals, when present in vascular lumens, support a diagnosis of atheroembolism
  • In some cases, underlying etiology can be identified
  • References: Goldblum: Rosai and Ackerman's Surgical Pathology, 11th Edition, 2017, Arch Pathol Lab Med 2001;125:224, Am J Surg Pathol 1997;21:706
Microscopic (histologic) images

Contributed by Gagandeep Kaur, M.D. and Monika Vyas, M.D.

Acute ischemic changes

Ischemic changes

Sample pathology report
  • Colon, biopsy:
    • Colonic mucosa with changes consistent with ischemic type mucosal injury / colitis (see comment)
    • Comment: Possible etiologies for this ischemic type mucosal injury include true ischemic colitis, infection (such as E. coli 0157:H7) and a drug reaction (including oral contraceptives, NSAIDs, digoxin and ergotamine derivatives). Clinical and endoscopic correlation is recommended.
  • Colon, colectomy:
    • Ischemic colitis with transmural necrosis / hemorrhage / perforation
    • The resection margins are viable.
Differential diagnosis
  • Drug / medication related injury:
    • Ischemic pattern of injury in young adults and involvement of nonwatershed areas should raise the possibility of drug induced injury
    • Drugs / medication related ischemic type injury (Arch Pathol Lab Med 2016;140:748)
    • Characterized by eosinophilic, lymphocytic and granulocytic infiltration (World J Gastroenterol 2019;25:967)
    • Increased intraepithelial apoptosis (mycophenolate mofetil)
    • Fragments of resins in cases associated with kayexalate or bile acid squestrants
    • Common drugs: oral contraceptives, cocaine, ergotamine derivatives, sumatriptan, kayexalate, cholestyramine
  • Crohn's disease:
    • Often younger patients
    • Transmural inflammation, lymphoid aggregates
    • No necrosis
    • Granulomas
  • Infectious colitis:
    • Occurs in epidemiological clusters
    • Younger patients
    • Children, elderly and immunocompromised are susceptible
    • Bacterial colitis is the most common cause
    • Pseudomembranous colitis is caused by Clostridium difficile (Am J Surg Pathol 1997;21:706):
      • Typically, history of use of antibiotics such as vancomycin is present and useful in raising a clinical suspicion of C. difficile colitis; stool antigen testing for presence of C. difficile toxin is confirmatory
      • Presence of pseudomembranes is usually more widespread in the colon in infectious etiologies and more segmental in ischemia
      • Hyalinization of lamina propria and specific vascular pathology are usually absent in C. difficile colitis
      • Crypt atrophy, lamina propria hemorrhage and full thickness mucosal atrophy favor a diagnosis of ischemia over C. difficile (Am J Surg Pathol 1997;21:706)
  • Ulcerative colitis:
    • Often younger patients
    • Cryptitis and crypt abscesses
    • Basal plasmacytosis
    • No fibrosis of lamina propria
    • No hemosiderin deposition
Board review style question #1
A 62 year old woman, status post coronary artery bypass graft, complains of abdominal pain and bloody stools. A CT scan is performed, which shows significant thickening of the colon wall. Biopsy is performed, which shows changes consistent with ischemic colitis. What is the most common site of ischemic colitis?

  1. Ascending colon
  2. Cecum
  3. Hepatic flexure
  4. Rectum
  5. Splenic flexure
Board review style answer #1
E. Splenic flexure is the most common site

Comment Here

Reference: Ischemic colitis
Board review style question #2

A 73 year old woman presents with severe abdominal pain and a recent history of passing bloody stools. The pain began about 3 days ago and is more concentrated at the left side of the abdomen. Physical examination shows decreased to absent bowel sounds, abdominal distention and diffuse tenderness to palpation over her abdomen. An Xray is performed, which shows thumbprinting of the colon. What is the most likely histologic change seen in biopsy of the colon in patients with ischemic colitis?

  1. Crypt abscess with granulomas
  2. Increased lamina propria eosinophils
  3. Necrotizing vasculitis, fibrin thrombi and lamina propria hyalinization
  4. Pseudomembrane formation
Board review style answer #2
C. Necrotizing vasculitis, fibrin thrombi and lamina propria hyalinization

Comment Here

Reference: Ischemic colitis

Juvenile (retention) polyp
Definition / general
Essential features
  • Most common type of pediatric intestinal polyp, with prominent, cystically dilated glands and inflammatory stroma
  • Solitary juvenile polyps have minimal malignant potential; multiple juvenile polyps may indicate a premalignant condition (juvenile polyposis coli or juvenile polyposis syndrome [JPS])
  • Juvenile polyposis syndrome is autosomal dominant and caused by a germline defect in SMAD4 or BMPR1A
Terminology
  • Retention polyps and juvenile hamartomatous polyps (Can J Gastroenterol 2007;21:233)
  • Juvenile polyposis syndrome (WHO 5th edition):
    • > 3 - 5 juvenile polyps in the colorectum or
    • Juvenile polyps throughout the gastrointestinal tract or
    • Any number of juvenile polyps with a positive family history of juvenile polyposis (World J Gastroenterol 2011;17:4839)
    • Other syndromes involving hamartomatous gastrointestinal polyps should be ruled out clinically or by pathological examination
ICD coding
  • ICD-10:
    • K63.5 - polyp of colon
    • D12.6 - benign neoplasm of colon, unspecified
Epidemiology
Sites
  • Majority in distal colon / rectosigmoid
  • Less frequently seen in stomach and small bowel
Pathophysiology
  • Earliest morphologic lesions are focal areas of mucosal hyperplasia, which seem to evolve into minute hyperplastic polyps
  • These become inflamed, frequently with ulceration and scarring of their surface, sealing over some of the crypts, with subsequent cystic dilatation of the crypts, accumulation of mucus within the cysts, enlargement of the structure to form a typical juvenile polyp and further inflammation and scarring
  • In some of the larger polyps, focal epithelial atypia develops
  • Second hypothesis: landscape defect, in which an abnormal stromal environment leads to neoplastic transformation of adjacent epithelium (World J Gastroenterol 2011;17:4839)
  • Juvenile polyposis syndrome is caused by a germline defect in SMAD4 or BMPR1A (Am J Surg Pathol 2011;35:530)
Etiology
  • Juvenile polyposis syndrome is autosomal dominant
Clinical features
Diagnosis
Radiology description
  • Filling defect on barium enema
  • Rounded hypoechoic nodule within the lumen of gastrointestinal tract with a peripheral hyperechoic layer (Ital J Pediatr 2020;46:66)
Radiology images

Images hosted on other servers:

Ultrasonography and color Doppler sonography

Prognostic factors
Case reports
Treatment
  • Many of these polyps outgrow their blood supply, become ischemic and autoamputate
  • For solitary juvenile polyp:
    • Colonoscopy / sigmoidoscopy and polypectomy
    • No further evaluation or followup is needed after polypectomy
    • Adenomatous changes in a juvenile polyp: followup and screening endoscopy is recommended (Clin Colon Rectal Surg 2008;21:280)
  • For juvenile polyposis syndrome:
    • High suspicion of JPS: endoscopic screening of the colon and upper gastrointestinal tract at age 15 or at the time of first symptoms
    • Followup: recommended every 2 to 3 years in patients with juvenile polyposis syndrome
    • Surgery: considered in patients with colorectal polyposis unmanageable by endoscopy (> 50 - 100 polyps), those with severe gastrointestinal bleeding or diarrhea, juvenile polyps with dysplasia and patients with a strong family history of colorectal cancer (World J Gastroenterol 2011;17:4839)
Clinical images

Images hosted on other servers:

Endoscopic images

Gross description
  • Sessile or pedunculated hamartomatous polyps
  • 5 mm to 50 mm, typically spherical, lobulated and pedunculated with surface erosion
  • Reference: World J Gastroenterol 2011;17:4839
Gross images

Contributed by Nalini Bansal, M.D.
Multiple polyps Multiple polyps

Multiple polyps



Images hosted on other servers:
Macroscopic appearance

Macroscopic appearance

Microscopic (histologic) description
  • Characterized by an abundance of edematous lamina propria with inflammatory cells and cystically dilated glands lined by cuboidal to columnar epithelium with reactive changes (nonneoplastic, hamartomatous, epithelial retentions)
  • Dilated glands filled with mucus and inspissated inflammatory debris
  • Juvenile polyposis syndrome:
    • Polyps appear similar to sporadic solitary juvenile polyps, although syndromic polyps often have a frond-like growth pattern with less stroma, fewer dilated glands and more proliferative smaller glands
    • JPS polyps frequently show neoplastic changes to the epithelium not common in sporadic solitary juvenile polyps
  • Reference: World J Gastroenterol 2011;17:4839
Microscopic (histologic) images

Contributed by Lewis Hassell, M.D., Nalini Bansal, M.D. and Christopher Hartley, M.D.
Juvenile polyp from stomach

Juvenile polyp from stomach

No dysplasia

No dysplasia

Ulcerated

Ulcerated

Dilated colonic glands Dilated colonic glands

Dilated colonic glands

Dysplasia

Dysplasia


Mild frond-like growth pattern Mild frond-like growth pattern

Mild frond-like growth pattern

Virtual slides

Images hosted on other servers:
Pediatric patient

Pediatric patient

Ulcerated

Ulcerated

Juvenile polyposis syndrome histology

Juvenile polyposis syndrome histology

Juvenile polyp from stomach

Positive stains
  • TP53 mutations (p53 mutated pattern staining) can be seen with high grade dysplasia in polyps of juvenile polyposis syndrome
Negative stains
  • p53 shows wild type pattern staining in benign solitary juvenile polyp, as there is no dysplastic change
Molecular / cytogenetics description
  • Germline mutation in the SMAD4 (18q21.1) or BMPR1A (10q23.2) gene is found in 40 - 60% of patients with juvenile polyposis syndrome
  • JPS patients with SMAD4 mutations are associated with more aggressive gastric polyps and appear to have a higher risk for gastric cancer (J Pediatr Gastroenterol Nutr 2019;68:453)
Videos

Histopathology of juvenile polyps

Endoscopy of juvenile polyps

Sample pathology report
  • Colon, polypectomy:
    • Solitary juvenile polyp (see comment)
    • Comment: Negative for dysplasia or malignancy
Differential diagnosis
Board review style question #1

A 5 year old boy presented with rectal bleeding for 1 month. Endoscopic examination revealed a polyp in the sigmoid colon. Histologic details are shown in the image above. What type of polyp is it?

  1. Adenomatous polyp
  2. Hamartomatous polyp
  3. Hyperplastic polyp
  4. Neoplastic polyp
Board review style answer #1
B. Hamartomatous polyp. The picture shows a juvenile polyp with cystically dilated glands in an edematous and inflammatory stroma with ulceration. Juvenile polyp is a hamartomatous polyp.

Comment Here

Reference: Juvenile (retention) polyp
Board review style question #2
Which of the following is true regarding juvenile polyp?

  1. Most common clinical presentation is intussusception
  2. Mostly seen in proximal stomach
  3. Solitary juvenile polyp has minimal risk of malignant transformation
  4. Uncommon type of pediatric polyp
Board review style answer #2
C. Solitary juvenile polyp has minimal risk of malignant transformation. Juvenile polyp is the most common type of pediatric intestinal polyp, mostly seen in distal colon. Painless rectal bleed is the most common clinical presentation. Solitary juvenile polyps have minimal malignant potential; however, multiple juvenile polyps may indicate a premalignant condition (juvenile polyposis coli or juvenile polyposis syndrome [JPS]).

Comment Here

Reference: Juvenile (retention) polyp

Juvenile polyposis syndrome
Definition / general
  • Autosomal dominant syndrome leading to growth of numerous juvenile polyps, predominantly in the colon
Essential features
  • Syndrome that encompasses 3 particular phenotypic groups, including juvenile polyposis of infancy, juvenile polyposis coli (colonic involvement only) and generalized juvenile polyposis
  • Diagnostic criteria:
    • 3 - 10 juvenile polyps, depending on criterion used
    • Juvenile polyps throughout entire GI tract
    • Any number of polyps in a patient with a family history of juvenile polyposis (Histopathology 1988;13:619)
  • Associated risk of upper or lower GI malignancies is approximately 55% (Ann Surg Oncol 1998;5:751)
  • Juvenile polyposis of infancy is a rare form that usually manifests before age 2, is not hereditable and is associated with a poor prognosis
Sites
  • Polyps found throughout the GI tract, with the exception of the juvenile polyposis coli form, which is restricted to the colon
Pathophysiology
Clinical features
  • If symptomatic, children typically present with painless rectal bleeding or prolapsing of polyps through rectum
Case reports
Treatment
  • Annual upper and lower endoscopic examinations with polypectomies
  • Surgery reserved for diffuse polyposis not amenable to polypectomy or for associated malignancies
  • Screening of family members recommended
Gross images

Images hosted on other servers:
Missing Image

Bowel resection

Microscopic (histologic) description
  • Numerous cystic and dilated crypts or glands with inspissated mucin and occasional intraluminal neutrophils
  • Lamina propria typically edematous with associated lymphocytes, plasma cells and occasional eosinophils and neutrophils
  • Filiform, multilobated forms with increased glandular to stroma ratio may be encountered and have been variably termed nonclassic or atypical polyps
  • May contain areas of conventional dysplasia (Arch Pathol Lab Med 1996;120:1032)
Microscopic (histologic) images

Images hosted on other servers:
Missing Image

Prominent lamina
propria with
edema and
inflammatory cells

Missing Image

Dense stroma

Videos

Colonoscopy of juvenile polyposis

Sample pathology report
  • Colon, total colectomy:
    • Colon with numerous juvenile polyps, some with focal low grade dysplasia (see comment)
    • Negative for high grade dysplasia or malignancy.
    • Eight benign lymph nodes.
    • Comment: The findings are consistent with the patient’s reported history of juvenile polyposis. Numerous grossly identifiable polyps were submitted for microscopy, including the largest.
Differential diagnosis
Board review style question #1
Which of the following is a criterion for diagnosis of juvenile polyposis syndrome?

  1. 1 juvenile polyp measuring > 5 cm
  2. At least 5 juvenile polyps in a patient with a family history of juvenile polyposis
  3. Juvenile polyps throughout entire GI tract
  4. More than 25 juvenile polyps
Board review style answer #1
C. Juvenile polyps throughout entire GI tract

Comment Here

Reference: Juvenile polyposis syndrome

Kaposi sarcoma
Definition / general
  • Vascular malignancy caused by human herpes virus 8 (see topic in Soft tissue chapter)
Essential features
  • Vascular malignancy most often seen in AIDS patients with HHV8 infection
  • Rare in gastrointestinal tract
Epidemiology
  • Most common gastrointestinal malignancy attributable to AIDS (Am J Gastroenterol 1991;86:715)
  • Currently rare; incidence decreased from use of HAART for AIDS treatment
Etiology
  • HHV8 infection
Clinical features
Prognostic factors
Case reports
Treatment
  • Chemotherapy and radiation preferred over surgery
Clinical images

Images hosted on other servers:

Kaposi sarcoma

Gross description
  • Reddish brown to purple macules or polyps, typically 0.5 - 1.5 cm
Microscopic (histologic) description
  • Lamina propria based lesion composed of spindle cells with mild to moderate atypia, arranged in vague fascicles and separated by slit-like vessels
  • Extravasated red blood cells common
  • PAS positive, diastase resistant hyaline globules sometimes seen
Microscopic (histologic) images

Contributed by Raul S. Gonzalez, M.D.

H&E, intermediate power

H&E, high power



Images hosted on other servers:

Low power

High power

CD34

Positive stains
Negative stains
Sample pathology report
  • Rectum, mass, biopsy:
    • Vascular proliferation most consistent with Kaposi sarcoma (see comment)
    • Comment: An immunostain for HHV8 is positive in the lesional cells.
Differential diagnosis
Board review style question #1

Which of the following is considered a risk factor for developing Kaposi sarcoma?

  1. Female sex
  2. HIV / AIDS
  3. Prior bariatric surgery
  4. Young age
Board review style answer #1
B. HIV / AIDS

Comment Here

Reference: Kaposi sarcoma

Leiomyoma
Definition / general
  • Benign smooth muscle proliferation of colon
Essential features
  • Benign incidental tumor usually arising in muscularis mucosae
Terminology
  • Symplastic leiomyoma refers to benign tumor with bizarre nuclear atypia and possibly increased mitotic activity
Sites
  • Most gastrointestinal tract leiomyomas arise in colon
  • Majority arise in rectosigmoid (Mod Pathol 2001;14:950)
Clinical features
  • Benign
  • More common in men
  • Median age is 62 years
Diagnosis
  • Generally discovered incidentally during colonoscopy
Case reports
Treatment
  • Total excision is adequate treatment
Clinical images

Images hosted on other servers:

Leiomyoma on colonoscopy

Endoscopic removal of a leiomyoma

Gross description
  • Firm, white, well delineated nodule or polyp, generally < 1 cm
Microscopic (histologic) description
  • Well differentiated smooth muscle cells with red fibrillar cytoplasm, arranged in fascicles and arising from muscularis mucosae (Arch Pathol Lab Med 2011;135:1311)
  • Usually no mitotic activity or necrosis
  • Symplastic leiomyomas demonstrate significant atypia and possibly increased mitotic activity
Microscopic (histologic) images

Contributed by Raul S. Gonzalez, M.D., @Andrew_Fltv and @ThatGlassTho on Twitter

Leiomyoma

Symplastic leiomyoma


Leiomyoma Leiomyoma Leiomyoma Leiomyoma

Leiomyoma

Leiomyoma Leiomyoma

Leiomyoma


Leiomyoma Leiomyoma Leiomyoma

Leiomyoma

Positive stains
Negative stains
Sample pathology report
  • Sigmoid colon, polypectomy:
    • Leiomyoma
Differential diagnosis
Board review style question #1
What is the most common location of origin for leiomyomas of the colon?

  1. Muscularis mucosae
  2. Muscularis propria
  3. Submucosal blood vessels
  4. Subserosal blood vessels
Board review style answer #1
A. Muscularis mucosae

Comment Here

Reference: Leiomyoma

Leiomyosarcoma
Definition / general
  • Malignant smooth muscle tumor of colon
Essential features
Terminology
  • Most older reports of colonic leiomyosarcomas were probably GIST
Epidemiology
Sites
  • Small intestine and colon most common GI sites for leiomyosarcoma
Etiology
Clinical features
Prognostic factors
Treatment
Gross description
  • Intraluminal, bulging / polypoid mass measuring up to 25 cm
Microscopic (histologic) description
  • Resembles leiomyosarcoma at other sites: malignant smooth muscle cells with prominent nuclear atypia, coagulative necrosis and at least 1 mitosis per 10 high power fields
  • Rare examples have osteoclast-like giant cells (Arch Pathol Lab Med 2004;128:440)
Microscopic (histologic) images

Contributed by Raul S. Gonzalez, M.D.

Colonic leiomyosarcoma

Nuclear atypia

Numerous mitotic figures



Images hosted on other servers:

Well differentiated tumor

Moderately differentiated

Poorly differentiated

Positive stains
Negative stains
Molecular / cytogenetics description
  • Negative for KIT mutations
Sample pathology report
  • Transverse colon, resection:
    • Leiomyosarcoma (4.7 cm) (see comment)
    • Margins of resection unremarkable.
    • Three benign lymph nodes.
    • Comment: The tumor shows prominent nuclear atypia, numerous mitotic figures and regions of necrosis. Immunohistochemical stains for SMA and desmin are positive in the tumor. Leiomyosarcomas are rare in the colon and risk factors for progression in this location remain poorly defined.
Differential diagnosis
Board review style question #1

Which of the following properties is known to confer worse prognosis for colonic leiomyosarcomas?

  1. Mitotic rate > 1 per 10 high power fields
  2. Necrosis
  3. Patient age > 65 years
  4. Size > 5 cm
Board review style answer #1
D. Size > 5 cm

Comment Here

Reference: Leiomyosarcoma

Lifting agent granuloma
Definition / general
  • Submucosal granulomatous reactive process due to the use of lifting agents during colonoscopy
Essential features
  • Reactive process in the submucosa arising from reaction to lifting agents injected during colonoscopy
  • Appearance changes over time
  • Often forms prominent, hard, hyalinized, homogenously eosinophilic amorphous ribbons and globules with foreign body giant cells
  • Should be differentiated from mimickers such as amyloid and pulse granuloma
ICD coding
  • ICD-10: M60.28 - foreign body granuloma of soft tissue, not elsewhere classified, other site
Sites
  • Luminal gastrointestinal tract
Pathophysiology
  • Lifting agents such as Eleview (Cosmo Technologies, 2015, also known as SIC-8000) and ORISE (Boston Scientific, 2018) are injected in the submucosa underneath a lesion to create a fluid cushion between muscularis mucosae and muscularis propria in order to visualize that lesion for endoscopic resection
  • Immediately after injection, the material has an acellular mucin-like quality (basophilic, bubble-like, amorphous)
  • Over time (~ 2 - 3 months) it becomes prominent, hard, hyalinized, homogenously eosinophilic, forming amorphous ribbons and globules with foreign body giant cells, scattered eosinophils and fibrosis
  • Median time interval for an Eleview or ORISE lifting agent granuloma to form is 16 weeks between injection and resection; however, the material can be seen as long as 31 weeks post injection (Am J Surg Pathol 2020;44:793)
Diagnosis
  • Histology
Prognostic factors
  • Outcomes of lifting agent granulomas are unknown as long term case studies are unavailable
  • Portion of the gel not resected during polypectomy likely remains in the submucosa
  • Hyalinized ribbons represent degrading agent or tissue reaction to the foreign agent (Am J Surg Pathol 2020;44:793)
Case reports
Gross description
  • Often forms a well circumscribed submucosal nodule
Microscopic (histologic) description
  • Morphology of the lifting agents changes with time (Am J Surg Pathol 2020;44:793, Am J Clin Pathol 2020;153:630, Mod Pathol 2020;33:1581):
    • Immediately after injection: lifting agents demonstrate acellular mucin-like quality with basophilic, amorphous and bubbly extracellular material
    • 1 day after injection: infiltrating neutrophils and a more solid and less bubbly texture
    • 2 - 3 months after injection: prominent hard, hyalinized, homogenously eosinophilic, amorphous ribbons and globules with foreign body giant cells, scattered eosinophils and fibrosis
  • ORISE and Eleview are neither refractile nor polarizable (Am J Surg Pathol 2020;44:793)
Microscopic (histologic) images

Contributed by Maryam Kherad Pezhouh, M.D., M.Sc.
Fresh ORISE in the submucosa

Fresh ORISE in the submucosa

Fresh ORISE in a background of red blood cells

Fresh ORISE in a background of red blood cells

Several months after injection of lifting agent

Several months after injection of lifting agent

Foreign body reaction to lifting agent

Foreign body reaction to lifting agent

Lifting agent granuloma

Lifting agent granuloma

Lifting agent is negative for Congo red

Lifting agent is negative for Congo red




Contributed by Catherine E. Hagen, M.D. (Case #507)
Fat forming SFT Fat forming SFT Fat forming SFT

Lifting agent granuloma

Positive stains
Negative stains
Sample pathology report
  • Colon, left, polyp, polypectomy:
    • Tubulovillous adenoma (see comment)
    • Comment: The submucosa adjacent to tubular adenoma contains prominent hyalinized, homogenously eosinophilic, amorphous ribbons and globules with foreign body giant cells, scattered eosinophils and fibrosis consistent with lifting agent granuloma. Congo red stain is negative for amyloid. (Am J Surg Pathol 2020;44:793)
Differential diagnosis
  • Amyloid:
    • Prominent intramucosal and perivascular distribution
    • Positive for Congo red (negative in lifting agent granuloma or pulse granuloma)
  • Pulse granuloma:
    • Benign entity occurring within the gastrointestinal tract
    • Foreign body reaction to legume material embedded in the bowel wall or peritoneum due to ulceration or perforation (Arch Pathol Lab Med 2001;125:822)
    • Round in shape (well circumscribed), contains ribbons of glassy material and calcifications
    • In contrast, lifting agent granulomas do not contain entrapped calcifications but show irregular, infiltrative pattern (border) that occurs secondary to forceful injection of gel into tissue (Mod Pathol 2020;33:1581)
Board review style question #1

Which stain is most helpful to differentiate between lifting agent granuloma and amyloidosis?

  1. Congo red because amyloid stains positive with Congo red
  2. Congo red because lifting agent stains positive with Congo red
  3. Congo red because lifting agent granulomas are birefringent as they have two different refractile indices
  4. Thioflavin T stain because lifting agent granulomas are nonpolarizable
Board review style answer #1
A. Congo red because amyloid stains positive with Congo red

Comment Here

Reference: Lifting agent granuloma
Board review style question #2

Histologically, pulse granuloma can be differentiated from lifting agent granuloma as they

  1. Are round in shape (well circumscribed), contain ribbons of glassy material and calcifications and are more limited in size
  2. Demonstrate acellular mucin-like quality with basophilic, amorphous and bubbly extracellular material with prominent hemorrhage
  3. Demonstrate prominent intramucosal and perivascular distributions
  4. Do not contain entrapped calcifications but show irregular, infiltrative pattern (border)
Board review style answer #2
A. Are round in shape (well circumscribed), contain ribbons of glassy material and calcifications and are more limited in size

Comment Here

Reference: Lifting agent granuloma
Board review style question #3
Which of the following features is characteristic of submucosal lifting agents (Eleview or ORISE)?

  1. Association with ruptured diverticula
  2. Involvement of vascular walls
  3. Positive for Congo red stain
  4. Surrounding giant cell reaction
Board review style answer #3
D. Surrounding giant cell reaction. Submucosal lifting agents frequently show a surrounding giant cell reaction. Amyloid will stain positive for Congo red and frequently is seen within vessels walls. Pulse granulomas are frequently seen in association with ruptured diverticula.

Comment Here

Reference: Lifting agent granuloma

Lipoma
Definition / general
  • Benign colonic neoplasm composed of mature adipose tissue
Essential features
  • Most common submucosal mesenchymal lesion of colon
  • Typically small and benign but can cause symptoms if large
Epidemiology
Sites
  • Usually submucosal in right colon
Clinical features
Case reports
Treatment
Gross description
  • Soft, yellow cut surface
  • Size can vary; large lesions may have overlying ulceration
Gross images

Images hosted on other servers:

Large pedunculated submucosal tumor

Cut section of tumor

Microscopic (histologic) description
Microscopic (histologic) images

Contributed by Raul S. Gonzalez, M.D.

Intramucosal lipoma



Images hosted on other servers:

Lipoma with overlying sessile serrated adenoma

Sample pathology report
  • Ascending colon, polypectomy:
    • Colonic mucosa with focus of benign submucosal fat, compatible with lipoma
  • Descending colon, polypectomy:
    • Intramucosal lipoma (see comment)
    • Comment: Intramucosal lipomas have been associated with Cowden syndrome in some patients. Additional clinical evaluation may be warranted.
Differential diagnosis
Board review style question #1
Benign fatty lesions of the colon usually arise in what layer of the wall?

  1. Lamina propria
  2. Muscularis mucosae
  3. Muscularis propria
  4. Submucosa
  5. Subserosa
Board review style answer #1
D. Submucosa

Comment here

Reference: Lipoma

Low grade tubuloglandular adenocarcinoma
Definition / general
Essential features
  • Arises in patients with ulcerative colitis or Crohn's disease
  • Low grade histology may be difficult to distinguish from benign glands
  • Can arise directly from low grade dysplasia
Epidemiology
  • ~10% of inflammatory bowel disease associated colon carcinomas are low grade tubuloglandular subtype
Clinical features
  • Median age 41.5 years
  • More common in ulcerative colitis than Crohn's disease
  • Patients may have synchronous conventional colorectal adenocarcinoma (see IBD associated carcinoma)
Prognostic factors
  • Patients generally do well unless a synchronous conventional colorectal carcinoma is also present
Gross description
  • May not be identifiable grossly; may be only identified on random sections
Microscopic (histologic) description
  • Small or medium sized glands with round or tubular shape
  • Malignant glands are extremely well differentiated and might only be identifiable as malignant based on location deep within the bowel wall
  • Desmoplasia is rare or absent
  • Overlying mucosa may show only low grade dysplasia
  • Areas of conventional adenocarcinoma may be visible
Microscopic (histologic) images

Contributed by Raul S. Gonzalez, M.D.
Missing Image Missing Image Missing Image Missing Image

Low grade tubuloglandular adenocarcinoma in ulcerative colitis patient

Missing Image

Background colitis

Positive stains
Negative stains
Molecular / cytogenetics description
Sample pathology report
  • Ascending colon, resection:
    • Low grade tubuloglandular adenocarcinoma (see synoptic report)
    • Background severely active chronic colitis, consistent with patient’s reported history of ulcerative colitis.
Board review style question #1

Low grade tubuloglandular adenocarcinoma of the colon arises in patients with what risk factor?

  1. Familial adenomatous polyposis
  2. Inflammatory bowel disease
  3. Lynch syndrome
  4. Prior personal history of colon cancer
Board review style answer #1
B. Inflammatory bowel disease

Comment Here

Reference: Low grade tubuloglandular adenocarcinoma

Lymphocytic colitis
Definition / general
  • Chronic nonulcerating colitis; subtype of microscopic colitis
  • Common cause of chronic nonbloody diarrhea in older adults with normal or near normal colonoscopy and increased intraepithelial lymphocytes as the histologic hallmark
Essential features
  • Cause of chronic watery diarrhea, often in older females
  • Normal, edematous or mildly erythematous mucosa on endoscopy
  • Colonic intraepithelial lymphocytosis (> 20 per 100 enterocytes) with diffuse increase in lamina propria inflammatory cells
ICD coding
  • ICD-10: K52.832 - lymphocytic colitis
Epidemiology
Sites
Pathophysiology
Etiology
  • Not clearly determined yet
Clinical features
  • Classic symptom is chronic nonbloody watery diarrhea
  • Other symptoms include urgency, fecal incontinence, abdominal pain, weight loss
  • Some asymptomatic
  • Associated autoimmune disorders (Gut 2004;53:536)
    • Thyroiditis
    • Celiac disease
    • Diabetes mellitus
    • Psoriasis
    • Rheumatoid arthritis
Diagnosis
  • Normal colonoscopy or mild nonspecific erythema or edema
  • Biopsies from all segments of the colon, proximal to rectosigmoid
    • Proximal to rectosigmoid
      • Rectal biopsy alone cannot rule it out
Laboratory
  • Mild anemia, elevated erythrocyte sedimentation
  • Autoantibodies (ANA, ANCA, antithyroid peroxidase antibodies)
  • Fecal leukocytes may be present
  • Elevated fecal calprotectin
  • Fecal eosinophil derived proteins
  • Negative stool cultures, ova and parasites
  • Negative lactose malabsorption test (Am J Gastroenterol 2017;112:78)
Radiology description
  • Normal barium enema
Prognostic factors
Case reports
Treatment
Gross description
  • Normal, edematous or mildly erythematous mucosa on endoscopy
Microscopic (histologic) description
  • Increased intraepithelial lymphocytes (Hum Pathol 1989;20:18)
    • > 20 IELs per 100 epithelial cells, away from lymphoid aggregates
  • Increased lamina propria inflammatory cells
    • Lymphocytes, plasma cells, eosinophils, occasional neutrophils
    • Predominantly upper half of the mucosa
    • Less prominent in left colon
  • Preserved / intact crypt architecture (Hum Pathol 1989;20:18)
  • Surface epithelial damage (Hum Pathol 1989;20:18)
    • Flattening, mucin depletion, vacuolization, nuclear irregularities
  • Unremarkable subepithelial collagen (< 10 μm)
  • Rare acute cryptitis, crypt abscess (30 - 38% of cases) (Am J Surg Pathol 2002;26:1414)
    • Focal and mild, not predominant
  • Paneth cell metaplasia, rarely seen
  • Less frequent findings:
    • Subepithelial giant cells
    • Ruptured crypt granulomas
Microscopic (histologic) images

Contributed by Martha M. Yearsley, M.D.
Intact crypt architecture

Intact crypt architecture

Lamina propria expansion

Lamina propria expansion

Mixed lamina propria inflammatory infiltrate

Mixed lamina propria inflammatory infiltrate

Increased surface intraepithelial lymphocytes

Increased surface intraepithelial lymphocytes


Crypt intraepithelial lymphocytes

Crypt intraepithelial lymphocytes

Surface epithelial damage

Surface epithelial damage

Lymphocytic colitis and celiac disease

Lymphocytic colitis and celiac disease

Positive stains
Negative stains
Videos

Microscopic colitis

Sample pathology report
  • Colon, random biopsies:
    • Colonic mucosa with increased intraepithelial lymphocytes, consistent with lymphocytic colitis
Differential diagnosis
  • Collagenous colitis:
    • Thickening and qualitative abnormalities of subepithelial collagen band with mild increase in intraepithelial lymphocytes
  • Inflammatory bowel disease (IBD):
    • Architectural distortion
    • Basal lymphoplasmacytosis
    • Cryptitis, crypt abscesses
    • Paneth cell metaplasia
  • Acute infectious colitis:
    • Edema
    • Luminal, lamina propria, surface neutrophils and cryptitis
    • No mononuclear inflammation or increased intraepithelial lymphocytes
  • Checkpoint inhibitor induced colitis (Cancer 2019;125:1768):
    • Scattered crypt apoptosis
    • Basal lymphoplasmacytosis
    • May otherwise mimic lymphocytic colitis; clinical history essential
  • Autoimmune enteropathy:
    • Loss of Paneth and goblet cells
    • Basal crypt intraepithelial lymphocytes and apoptosis
    • Crypt distortion and crypt abscesses
    • No surface intraepithelial lymphocytosis
Board review style question #1

Which of the following histologic findings best describes colonic mucosa in lymphocytic colitis?

  1. Crypt architectural distortion, basal lymphoplasmacytosis and crypt abscesses
  2. Depleted lamina propria, increased crypt apoptosis and crypt distortion
  3. Intraepithelial lymphocytes, surface epithelial damage, intact crypt architecture and lamina propria expansion
  4. Intraepithelial lymphocytes, thick subepithelial collagen band with entrapped inflammatory cells
  5. Lamina propria hyalinization, hemorrhage and withering crypts
Board review style answer #1
C. Intraepithelial lymphocytes, surface epithelial damage, intact crypt architecture and lamina propria expansion. Answer A best fits inflammatory bowel disease. Answer B is an apoptotic colopathy pattern of injury that can be seen in graft versus host disease and mycophenolate induced injury, for example. Answer D describes collagenous colitis and answer E corresponds to ischemic colitis.

Comment Here

Reference: Lymphocytic colitis
Board review style question #2

A 53 year old woman presents with a history of intermittent diarrhea of unexplained origin. The colonoscopy shows mild erythema throughout the colon. Based on representative images of random colon biopsies, what is the most likely diagnosis?

  1. Idiopathic inflammatory bowel disease
  2. Infectious colitis
  3. Ischemic colitis
  4. Lymphocytic colitis
  5. Normal histology
Board review style answer #2
E. Normal histology. The left image shows normal architecture. The intraepithelial lymphocytes on the right image are overlying a lymphoid aggregate. Intraepithelial lymphocytes should be evaluated away from the mucosal lymphoid aggregates.

Comment Here

Reference: Lymphocytic colitis

Lymphoid polyp
Definition / general
  • Benign lesion that may endoscopically resemble a mucosal polyp
  • Predominantly asymptomatic and incidental
Essential features
  • Endoscopically, these lesions may appear as small nodules (Case Rep Gastrointest Med 2018;2018:5758689)
  • Frequently sampled as a polyp biopsy or polypectomy
  • Consideration of a neoplastic hematolymphoid process is suggested when the lymphoid aggregate appears monotonous and without an obvious germinal center; very small nonneoplastic lymphoid follicles may not develop germinal centers
Terminology
  • Also called nodular lymphoid hyperplasia, lymphoid aggregate, rectal tonsil
  • The term pseudolymphoma is not recommended
  • Various terminologies may be used to report the findings of lymphoid polyp on a pathology report
  • May be reported as reactive lymphoid aggregate if there is a prominent germinal center with polymorphous population of cells and tingible body macrophages
  • Other reporting terminology may be colonic mucosa with prominent lymphoid aggregate; otherwise with no significant histopathological changes
    • The latter may be used when there is no other associated lesion in addition to the colonic lymphoid polyp
  • Note: there are instances when certain lesions, such as hyperplastic polyp or sessile serrated lesions / polyp, may occur alongside lymphoid polyp (see Microscopic (histologic) images)
ICD coding
  • ICD-10
Epidemiology
  • More frequent in younger population and in children
  • Nodular lymphoid hyperplasia, normally between 2 and 10 mm, may occur as a normal physiology (World J Gastrointest Endosc 2014;6:534)
  • Certain immunodeficiency disorders and infections are associated with prominent lymphoid aggregates and may appear polypoid on endoscopic examination
    • Examples include common variable immunodeficiency (CVID), selective IgA deficiency, Giardia infection, human immunodeficiency virus infection (uncommon), celiac disease and Helicobacter pylori infection
Sites
  • Widely distributed throughout the stomach, small intestine and large bowel
  • Polypoid endoscopic findings are more frequent in the colon and particularly in the ileocecal region because of prominent Peyer patches
Pathophysiology
  • Lymphoid follicles may occur as the normal component of intestinal or gut associated mucosal lymphoid tissue (Ann N Y Acad Sci 2010;1207:E86)
  • Gut associated lymphoid tissue (GALT) contains a significant percentage of immune cells (up to 70%)
  • Mucosa overlying the lymphoid follicles is specialized follicle associated epithelium that is rich in macrophages and specialized antigen presenting cells (Biomedicines 2022;10:226)
  • Average diameter of lymphoid aggregates ranges from 0.1 to 0.7 mm
  • Reactive hyperplasia of GALT due to various infections can occur and lead to the appearance of polypoid or granular mucosa
  • Associated with adenovirus infection, immunodeficiency (low IgA and IgM)
  • EBV associated with immunosuppression may cause progression of the benign lymphoid follicle to lymphoma (Am J Surg Pathol 2019;43:1253)
Etiology
  • No specific etiology has been associated and such lesions are frequently seen
  • Certain specific conditions have an association; before considering a strong association be reported on pathology diagnosis, it is recommended to have more clinical information
    • ~20% of adults with CVID may have nodular lymphoid hyperplasia or have more frequent lymphoid polyps; diffuse nodular hyperplasia may occur more frequently in the proximal small bowel (Indian J Pathol Microbiol 2009;52:530)
Diagrams / tables

Contributed by Arvind Rishi, M.D., M.B.B.S.
Histological algorithmic approach

Histological algorithmic approach

Clinical features
  • Usually localized but may show diffuse involvement of mucosa when the portion of colon is diverted from the fecal stream (see Colon-Diversion colitis)
  • Rectal lymphoid polyps are usually asymptomatic but colonic lymphoid polyps may present with gastrointestinal bleeding
  • Children: multiple lesions; larger lymphoid polyps may cause intestinal obstruction and intussusception
  • Adults: usually an isolated lesion
  • Subtypes: idiopathic (most common), reactive, hypogammaglobulinemia associated
  • Awareness of associated conditions such as CVID or IgA deficiency can be helpful to correlate clinical findings with histology
  • Such systemic associations are more common with lymphoid aggregates in the small bowel and with increased numbers of lymphoid aggregates at an unusual mucosal site
    • Frequent physiological lymphoid aggregates normally occur as Peyer patches in the ileocecal region and should generally not raise concern
  • Nodular lymphoid hyperplasia is associated with some other conditions, such as common variable immunodeficiency, selective IgA deficiency, Giardia infection, human immunodeficiency virus infection, celiac disease and Helicobacter pylori infection, etc. (Arch Pathol Lab Med 2013;137:83)
    • Associated conditions are due to the antigenic stimulation of lymphoglandular complexes
    • A study done on lymphoid aggregates suggests prediction of chronic diarrhea in a subset of patients
  • References: Case Rep Surg 2019;2019:9017863, Int J Colorectal Dis 1991;6:165, Indian J Pathol Microbiol 2009;52:530, Endoscopy 2013;45:320, Toxicol Pathol 2006;34:599
Diagnosis
  • Diagnosis is made on light microscopy on routine H&E stain
  • Special stains are usually not required except when the lymphoid aggregate appears neoplastic on the H&E stain
Prognostic factors
  • Overall favorable outcome with no follow up required for benign lymphoid aggregates
Case reports
Treatment
  • Polypectomy
Clinical images

Images hosted on other servers:
Endoscopic appearance

Endoscopic appearance

Gross description
  • Small polypoid or raised lesions, which generally are sessile or may have broad stalk
  • Surface erosions with acute inflammation over the lymphoid polyp / follicle clinically and endoscopically may appear as an aphthoid ulceration, which may indicate possibility of Crohn's colitis but is not diagnostic
Microscopic (histologic) description
  • Intestinal mucosal associated lymphoid tissue (MALT) is distributed throughout the large intestinal tract as lymphoglandular complexes (World J Gastrointest Oncol 2019;11:59)
    • They may be located within the lamina propria or extend to the deeper layers within muscularis mucosae or submucosa
    • Antigenic stimulation of lymphoglandular complex leads to prominent lymphoid aggregates, thereby forming lymphoid polyp
  • Benign lymphoid follicles covered by columnar (colonic mucosa) or transitional epithelium (anorectal mucosa)
    • Surface epithelium is usually flat but may have reactive epithelial changes in the form of surface hyperplasia in the distal rectum
  • Germinal centers may be seen in larger lymphoid polyps
  • Reactive lymphoid follicles may also be present in the lamina propria (more commonly) or submucosa
  • Intraepithelial lymphocytes are commonly present over a lymphoid polyp and should not be considered pathologic
  • Intraepithelial neutrophils over a lymphoid follicle is a pathologic finding and should be reported in the final diagnosis as focal acute inflammation
    • It is discouraged to use the term colitis if the acute inflammation is identified in focal areas; this is to prevent the patient from falsely being clinically labeled with chronic inflammatory bowel disease
  • Presence or absence of dysplasia in the surface epithelium of the lymphoid polyp is not dependent on the nature of underlying lymphoid follicle
  • Herniation of normal colonic epithelium or dysplastic epithelium (tubular adenoma) may occur and could pose a diagnostic challenge to evaluate for invasion
Microscopic (histologic) images

Contributed by Arvind Rishi, M.D., M.B.B.S.
Lymphoid aggregate with germinal center

Lymphoid aggregate with germinal center

Prominent Peyer patches

Prominent Peyer patches

Polarized germinal center

Polarized germinal center

Tingible body macrophages

Tingible body macrophages

Superficial lymphoid aggregate

Superficial lymphoid aggregate


Small bowel polyps

Small bowel polyps

Surface reactive changes, rectum

Surface reactive changes, rectum

Tingible body macrophages

Tingible body macrophages

Incidental hyperplastic polyp with lymphoid aggregate

Incidental
hyperplastic polyp
with lymphoid
aggregate

Positive stains
Negative stains
  • BCL2 is negative in germinal centers (only scattered T cells in germinal centers stain positive)
Sample pathology report
  • Colon, polyp, polypectomy:
    • Colonic mucosa with lymphoid aggregate; otherwise, with no significant histopathologic findings (see comment)
    • Comment: The mucosal lymphoid tissue shows polymorphous population of lymphoid tissue. There is an admixture of CD20 positive B cells and CD3 positive T cells. CD45 stain shows no aberrant expression in the regions stained by CD20.
  • Note: Immunohistochemical stains are not required for the majority of cases. Comment may be added for cases where there are immunohistochemical stains performed based on morphologic (H&E) concern. In routine cases without immunohistochemistry, comment is not required.
Differential diagnosis
  • MALT lymphoma and follicular lymphoma:
    • More uniform appearance of small to medium sized type B lymphocytes with histological overlap with MALT lymphoma
    • Translocation t(14;18)(q32;q21) is a characteristic finding for follicular lymphoma
    • Lymphoma immunohistochemical panel workup is essential for the subclassification of neoplastic hematolymphoid process
  • Inverted lymphoglandular polyp (Case Rep Pathol 2015:2015:646270):
    • In some instances, the dysplastic colonic epithelium may herniate or extend into the prominent mucosal or submucosal lymphoid aggregates
    • Although the location of dysplastic epithelium may raise a concern for an invasive malignancy, the herniated tissue still retains the mucosal morphology with intact lamina propria and without any desmoplastic reaction
    • Smooth muscle markers such as desmin immunohistochemistry may be misleading in cases where herniation of dysplastic epithelium extends into submucosal lymphoid aggregates
    • On the contrary, invasive adenocarcinoma extending to the lymphoid aggregate will lack lamina propria and may have angulated pattern of arrangement of malignant glands
    • Frequently, fibrous stroma or desmoplastic reaction will accompany invasive malignancy extending to the lymphoid aggregates
Board review style question #1

A 14 year old boy presented with small sessile polypoid areas in the terminal ileum. A wide mucosal biopsy was performed with concern for inflammatory bowel disease. There was no clinical history of failure to thrive or frequent infections. Which of the following is the most likely diagnosis?

  1. IgA deficiency
  2. MALT lymphoma
  3. Mucosa associated lymphoid tissue (Peyer patches)
  4. Small bowel follicular lymphoma
Board review style answer #1
C. Mucosa associated lymphoid tissue (Peyer patches). Mucosa associated lymphoid tissue is the most likely diagnosis because of the age of the patient and presence of reactive germinal centers and lymphoid follicles, which do not show a fused pattern. Answer A is incorrect because IgA deficiency disease will present with numerous lymphoid follicles in the upper gastrointestinal tract and higher propensity to infections. Answers B and D are incorrect because lymphomas will appear to have monotonous small to medium sized lymphocytes and generally lack lymphoid aggregate.

Comment Here

Reference: Lymphoid polyp
Board review style question #2

Which of the following suggests mucosal lymphoid aggregate to be benign?

  1. Absence of tingible body macrophages
  2. High Ki67 proliferation index
  3. Polymorphous population of lymphocytes in the germinal centers
  4. Strong BCL2 immunoreactivity in the germinal center
Board review style answer #2
C. Polymorphous population of lymphocytes in the germinal centers. Nonneoplastic germinal center will have admixture of cells of variable sizes. Answer B is incorrect because a neoplastic hematolymphoid process will have a monotonous appearance of small to medium sized lymphocytes with high Ki67 proliferation index. Answer A is incorrect because benign follicles frequently show germinal center and presence of tingible body macrophages. Answer D is incorrect because BCL2 should be negative in germinal centers of benign mucosal lymphoid aggregate.

Comment Here

Reference: Lymphoid polyp

Lynch syndrome
Definition / general
  • Autosomal dominant hereditary disorder due to mutation in a mismatch repair (MMR) gene
Essential features
  • Most common hereditary colorectal carcinoma syndrome (accounts for 2 - 5% of all colorectal carcinomas)
  • 80% of patients develop colorectal carcinoma; also increased risk of endometrial carcinoma (33%), ovarian carcinoma (5%) and cancers of small bowel, stomach, upper urinary tract and brain (Fam Cancer 2005;4:245)
  • Muir-Torre syndrome is a clinical variant
  • See also MSI testing for Lynch syndrome, MLH1, MSH2
Terminology
Epidemiology
  • Patients with Lynch syndrome tend to develop carcinomas at an earlier age than the general population (average age: 44 years old)
Sites
  • Colon cancers more likely to occur in the proximal colon
Pathophysiology / etiology
  • Mutation in MMR gene results in defective repair of DNA sequence mismatches, which most frequently occur in long, repetitive DNA sequences (such as seen in microsatellite regions, hence the term microsatellite instability [MSI])
  • Accumulation of DNA mismatches lead to increased risk of developing malignant neoplasms
Etiology
  • Typically due to mutation in an MMR gene: most commonly due to a germline mutation in MSH2 or MLH1; less frequently due to germline mutations in MSH6 or PMS2
  • Mutations in EPCAM (epithelial cellular adhesion molecule) / TACSTD1 (tumor associated calcium signal transducer 1) gene may result in Lynch syndrome: EPCAM gene is adjacent to MSH2 and mutations in EPCAM may result in MSH2 promoter hypermethylation and subsequent inactivation of MSH2
  • Rarely due to inactivation through germline promoter hypermethylation of MLH1; differs from the somatic MLH1 hypermethylation, which may be seen in sporadic colon cancers
  • Rarely the result of inactivation of CHEK2 (cell cycle checkpoint kinase 2)
Diagnosis
  • Amsterdam I screening criteria (all 4 must be met):
    • 3 or more family members with a confirmed diagnosis of colorectal cancer, one of whom is a first degree (parent, child, sibling) relative of the other two
    • 2 successive affected generations
    • 1 or more colon cancers diagnosed before age 50
    • Familial adenomatous polyposis has been excluded (Dis Colon Rectum 1991;34:424)
    • These criteria are relatively sensitive but not specific for Lynch syndrome, since these patients may lack MMR gene mutations
  • Amsterdam II criteria (all 4 must be met):
    • 3 or more family members with HNPCC related cancers, one of whom is a first degree relative of the other two
    • 2 successive affected generations
    • 1 or more HNPCC related cancers diagnosed before age 50
    • Familial adenomatous polyposis has been excluded (Gastroenterology 1999;116:1453)
  • Revised Bethesda criteria (any criteria must be met; these are guidelines for when tumors should be tested for MSI):
    • Colorectal cancer diagnosed before age 50 years
    • Presence of synchronous or metachronous colorectal cancers or other HNPCC associated tumors, regardless of age
    • Colorectal cancer with MSI-H histology (see below) diagnosed before age 60 years
    • Colorectal cancer diagnosed in one or more first degree relatives with an HNPCC related tumor, with one of the cancers being diagnosed before age 50 years
    • Colorectal cancer diagnosed in 2 or more first or second degree relatives with HNPCC related tumors, regardless of age (J Natl Cancer Inst 2004;96:261)
  • Recommended screening for patients with Lynch syndrome:
    • Full colonoscopy every 1 - 2 years beginning at age 20 - 25 years
    • Annual screening for endometrial cancer beginning at age 25 - 35
    • Annual urinalysis and cytologic examination beginning at age 25
    • Annual skin surveillance and upper GI endoscopy beginning at age 35 when gastric cancer is part of the family spectrum (JAMA 2006;296:1507)
    • At colonscopic screening, patients have similar rate of adenomas as nonsyndrome patients but much higher incidence of carcinoma (Gastroenterology 2006;130:1995)
Laboratory
  • Germline testing for mutations can be performed
  • Microsatellite Instability testing of tumor specimens via PCR is widely utilized; this typically consists of a panel of 5 mono / dinucleotide repeats which are analyzed, and a shift in PCR product size of tumor versus normal indicates instability; designation of MSI-H requires instability in at least 30% of examined loci
  • Immunohistochemical testing panel for 4 MMR proteins (MLH1, MSH2, PMS2, MSH6) is widely utilized
  • BRAF V600E mutation analysis may be performed on cases with loss of MLH1 and PMS2 IHC staining: if mutation is present, then Lynch syndrome is virtually excluded
  • MLH1 gene promoter hypermethylation may be utilized to determine sporadic versus Lynch syndrome related colon cancers
Prognostic factors
  • Syndromic cancers have better survival than nonsyndromic
Case reports
Treatment
Gross description
  • Usually proximal colon; 18% multiple and 40% metachronous (multiple separate occurrences)
Microscopic (histologic) description
  • Tumoral features suggestive of MSI-H etiology include: tumor infiltrating lymphocytes and peritumoral lymphocytes, Crohn's-like lymphoid reaction, mucinous features, medullary features, tumoral heterogeneity and absence of dirty necrosis
Negative stains
  • Defective mismatch repair genes can be reliably detected (negative staining) by immunohistochemistry (Am J Clin Pathol 2004;122:389, N Engl J Med 2005;352:1851)
    • Loss of only PMS2 staining is suggestive of PMS2 mutation
    • Loss of only MSH6 staining is suggestive of MSH6 mutation
    • Loss of MLH1 and PMS2 staining may be seen in MLH1 mutation (although sporadic colon cancers may also have this profile; if this profile is observed, follow up with BRAF V600E mutation analysis or MLH1 hypermethylation studies is warranted)
    • Loss of MSH2 and MSH6 staining may be seen in MSH2 mutation
Molecular / cytogenetics description
  • Defect in mismatch repair genes (caretaker genes that proofread DNA replication): typically MLH1 or MSH2, while MSH6 and PMS2 are less common (Gastroenterology 2006;130:312)
  • Rarely due to EPCAM mutation, germline promoter hypermethylation of MLH1, inactivation of CHEK2 (cell cycle checkpoint kinase 2)
  • Associated with microsatellite instability (MSI; microsatellites are dinucleotide repeat sequences, such as (CA)n, normally present in human genome), although MSI is not specific (World J Gastroenterol 2006;12:4745)
Board review style question #1
What is the most common hereditary colorectal cancer syndrome?

  1. Familial adenomatous polyposis
  2. Juvenile polyposis
  3. Lynch syndrome
  4. MUTYH associated polyposis
Board review style answer #1
C. Lynch syndrome

Comment Here

Reference: Lynch syndrome
Board review style question #2
Lynch syndrome usually arises from a germline mutation in a gene coding for a mismatch repair protein. A germline mutation in which of the following genes could also cause Lynch syndrome?

  1. BRAF
  2. CDH1
  3. EPCAM
  4. MUTYH
Board review style answer #2
C. EPCAM

Comment Here

Reference: Lynch syndrome

Malakoplakia
Definition / general
  • Marked histiocytic infiltrate, more common in bladder, that may diffusely involve colon
  • Usually adults, associated with immunosuppression
  • Due to defective inflammatory response to gram negative bacterial infection
  • Rarely associated with colorectal carcinoma in elderly men (Pathology 2002;34:332)
Case reports
Gross description
  • Diffuse mucosal involvement and submucosal thickening; may cause mass
Microscopic (histologic) description
  • Marked histiocytic infiltrate containing Michaelis-Gutmann bodies (calcospherites) that may resemble signet ring cells at frozen section
Microscopic (histologic) images

Images hosted on other servers:
Missing Image Missing Image Missing Image

Michaelis-Gutmann bodies

Missing Image

Liver transplant recipient

Missing Image

Adjacent to adenocarcinoma

Virtual slides

Images hosted on other servers:
Missing Image Missing Image

Large bowel


Mantle cell lymphoma
Definition / general
Case reports
Gross description
  • Nodular, sessile or polypoid lesions, widely spaced and confluent studded or cobblestone appearance
  • Each polyp 2 mm to 2 cm
  • May be dominant tumor mass in ileocecum
  • Endoscopically normal mucosa may have small tumor infiltrates also
Gross images

Images hosted on other servers:

Lymphomatoid polyposis

Microscopic (histologic) description
  • Mantle cells (small lymphocytes with pale cytoplasm and cleaved nuclei), often invasion of submucosa, proliferation around germinal centers and sparing of mucosa
  • Late epithelial invasion and ulceration
  • Note: may also be minute lymphoid infiltrates in known mantle cell patients
Microscopic (histologic) images

Images hosted on other servers:

80 year old woman

Positive stains
Negative stains
Differential diagnosis

Mast cell disorders
Definition / general
  • Rare condition, described in 2006, occurring in cases of chronic intractable diarrhea (Arch Pathol Lab Med 2006;130:362)
  • Not intended as a specific diagnosis (Ann Allergy Asthma Immunol 2008;101:645)
  • Helps select patients amenable to specific mast cell targeted therapies
  • No uniform or standardized criteria for diagnosis
  • Must be distinguished from GI involvement by systemic mastocytosis, a clonal proliferation of mast cells with specific WHO diagnostic criteria
Essential features
  • Chronic intractable diarrhea
  • Normal endoscopy
  • Increase in the number of mast cells in the gastrointestinal biopsies with sheet and nest formation
Terminology
ICD coding
  • ICD-10: K52.9 - noninfective gastroenteritis and colitis, unspecified
Epidemiology
  • Chronic diarrhea is a frequent health problem, affecting 5 - 10% of the adult population (Dig Dis 2018;36:409)
  • Despite normal colonoscopy, biopsy can reveal abnormalities in almost 30% of cases (20% corresponding to microscopic colitis)
  • No gender differences
  • Any age
Sites
  • Mast cells can be increased anywhere along the gastrointestinal tract
  • The most important differences from controls and other disorders are found in the left colon
Pathophysiology
  • Mast cells are immunological cells derived from a CD34+ progenitor in the bone marrow
  • They are granulated and measure ~20 microns
  • They can be activated through IgE dependent (allergic) and IgE independent pathways
  • 2 types, according to granule contents: those containing only tryptase and those that also contain chymase and carboxypeptidase
  • In the normal mucosa, the tryptase containing mast cells are located in the mucosa (reactive), while the chymase containing ones are mainly submucosal (constitutive)
  • They act on the regulation of intestinal permeability, secretion, peristalsis, immunity and angiogenesis
  • Upon activation, mast cells are involved in allergies, anaphylaxis, gastrointestinal disorders, malignancies and cardiovascular diseases (Cancers (Basel) 2021;13:3316)
Etiology
  • Unclear
  • Some patients have a history of allergies
Clinical features
  • Chronic intractable diarrhea with normal endoscopy
  • Can also be associated with abdominal pain or bloating
Diagnosis
  • Based on the clinical history and confirmed by histopathology
Case reports
Treatment
  • Antihistamines (H1 and H2)
  • Mast cell stabilizers, including cromolyn
  • Steroids (e.g., prednisone, budesonide) (J Gastrointestin Liver Dis 2018;27:327)
  • Inhibitors of mast cell mediators, inhibitors of tryptase and chymase
  • Leukotriene antagonists (e.g., montelukast, zileuton)
  • Kinase inhibitor (e.g., imatinib, nilotinib, dasatinib)
Gross description
  • Normal mucosa on endoscopy
Microscopic (histologic) description
  • Normal mucosal architecture
  • Increased number of mast cells in the lamina propria
  • Might be associated with an increase in eosinophils
  • Scattered mast cells but also nests and sheets
  • Sometimes presence of a subepithelial band of mast cells
  • Hard to recognize in routine H&E stained slides
  • Controversy regarding the cutoff value for mast cells
  • Most reports recommend more than 20 mast cells/high power field (HPF), although cutoff can be different for the left and right colon and other gastrointestinal locations
  • Lack of standardization for counting
  • Routine mast cell counting is not recommended at this time
  • Significant overlap exists between mast cell counts in asymptomatic control patients and patients with suspected mastocytic enterocolitis, arguing against the clinical utility of routinely counting mast cells in colonic biopsies (Am J Surg Pathol 2014;38:832)
  • Patients with gastrointestinal disorders, other than mastocytic enterocolitis showing increases in mast cells, can also benefit from mast cell stabilizing therapies
Microscopic (histologic) images

Contributed by M.J. Fernández-Aceñero, M.D., Ph.D.
Slight inflammatory infiltration

Slight inflammatory infiltration

Eosinophils in the infiltrate

Eosinophils in the infiltrate

KIT expression

KIT expression

Increased mast cells

Increased mast cells

Positive stains
Sample pathology report
  • Large bowel, endoscopic biopsy:
    • Large bowel mucosa with slight increase in lamina propria mast cells (see comment)
    • Comment: Large bowel mucosa with a slight expansion of the lamina propria related to an increase in the number of mast cells in the immunohistochemical stains for KIT. The mast cell count is 36 per high power field.
Differential diagnosis
Board review style question #1

The picture shows KIT immunostaining in a large bowel biopsy of a patient with chronic diarrhea. Which of the following statements regarding KIT immunostaining is true?

  1. Mast cell counts under 20/HPF should never be reported
  2. Should be performed in all large bowel biopsies with normal endoscopy
  3. The presence of a subepithelial band of mast cells is diagnostic of mastocytic enterocolitis
  4. There are clear differences between normal population and patients with different diseases in the number of mast cells
  5. There are no uniform criteria for the diagnosis of mastocytic enterocolitis
Board review style answer #1
E. There are no uniform criteria for the diagnosis of mastocytic enterocolitis. To date, the criteria for the diagnosis of mastocytic enterocolitis are not standardized and although most authors recommend 20/HPF, the most recent guidelines and meta analysis emphasize the large overlap in the mast cell numbers between controls and patients with diarrhea and other gastrointestinal pathologies. On the other hand, patients with lower mast cell counts can benefit from therapy, so it is recommended to report the absolute number of mast cells per HPF.

Comment Here

Reference: Mastocytic enterocolitis
Board review style question #2
Mastocytic enterocolitis is a(n)

  1. Clearly defined clinicopathological disease
  2. Diagnosis that can only be made on imaging
  3. Disease of young allergic people with a clear clinical phenotype
  4. Orientative diagnosis to help select patients that are candidates for targeted therapies
  5. Outdated entity that no longer exists
Board review style answer #2
D. Mastocytic enterocolitis is an orientative diagnosis to help select patients that are candidates for targeted therapies. This diagnosis is almost never certain on pure histopathological criteria and is only useful to select candidates to mast cell stabilizing therapies, as a significant number can respond to them.

Comment Here

Reference: Mastocytic enterocolitis

Medullary carcinoma
Definition / general
Essential features
  • Strongly associated with high degree of microsatellite instability (MSI), indicative of loss of normal DNA repair gene function
  • Often better clinical outcome independent of stage than microsatellite stable tumors or tumors with low levels of microsatellite instability
  • Usually no / few nodal metastases
Terminology
  • Older names: undifferentiated carcinoma, solid type poorly differentiated carcinoma, large cell minimally differentiated carcinoma
Epidemiology
  • Often elderly women (average age ~70 years)
Sites
  • Usually right sided
Case reports
Treatment
  • Surgery and chemotherapy (although 5-FU not effective, as with other MSI high tumors)
Gross description
  • Large, ulcerating colon mass
Gross images

Images hosted on other servers:
Missing Image

Fleshy, ulcerated lesion

Microscopic (histologic) description
  • Malignant, well circumscribed neoplasm with solid growth pattern (no gland formation) and pushing border
  • Can form nested, organoid or trabecular patterns
  • Cells are uniform, polygonal to round and medium sized with amphophilic cytoplasm, vesicular nuclei with prominent nucleoli and frequent mitotic activity
  • Mucin production absent or very focal
  • Prominent lymphocytic infiltration within and around tumor
Microscopic (histologic) images

Contributed by Raul S. Gonzalez, M.D.

Medullary carcinoma

Positive stains
Negative stains
Molecular / cytogenetics description
Sample pathology report
  • Cecum, biopsy:
    • Poorly differentiated carcinoma, consistent with medullary carcinoma (see comment)
    • Comment: Immunohistochemical stains for MLH1 and PMS2 show aberrant loss of nuclear expression in the tumor. Staining for MSH2 and MSH6 is unremarkable (i.e. retained).
Differential diagnosis
Board review style question #1

Which of the following is typically considered a main pathogenetic aspect of medullary carcinoma of the colon?

  1. EBV infection
  2. Microsatellite instability
  3. POLD1 or POLE mutations
  4. SWI / SNF complex mutations
Board review style answer #1
B. Microsatellite instability

Comment Here

Reference: Medullary carcinoma
Board review style question #2

Which of the following is true about medullary carcinoma of the colon?

  1. It accounts for 20% of colorectal carcinomas
  2. It is poorly differentiated, with a poor prognosis
  3. It most often arises in elderly women
  4. It usually arises in the rectosigmoid
Board review style answer #2
C. It most often arises in elderly women

Comment Here

Reference: Medullary carcinoma

Melanosis coli
Definition / general
  • Deposition of dark melanin-like pigment in colonic macrophages
Essential features
  • Pigment deposition in colon with striking gross and macroscopic features but minimal direct clinical consequences
  • Linked to use of anthraquinone laxatives
  • Common endoscopic finding
Sites
  • Can involve all parts of colon and rectum but typically spares mucosal regions with lymphoid nodules, polyps or carcinomas
  • Should biopsy nonpigmented regions in these patients
Etiology
  • Linked to use of anthraquinone laxatives but may be secondary to any increase in colonic epithelium apoptosis (Histopathology 1997;30:160)
Clinical features
Diagnosis
  • Colonoscopy
Case reports
Gross description
  • Diffuse brown-black discoloration of colonic mucosa
Microscopic (histologic) description
  • Diffuse deposition of melanized ceroid in macrophages of lamina propria (Gastrointest Endosc 1997;46:131)
  • Silver stains may show abnormalities of myenteric plexus
Microscopic (histologic) images

Contributed by Raul S. Gonzalez, M.D. and Yuri Tachibana, M.D.

Melanosis coli

Pigment in lamina propria

Positive stains
Negative stains
Videos

Melanosis coli

Sample pathology report
  • Random colon, biopsy:
    • Colonic mucosa with melanosis coli, otherwise within normal limits
Differential diagnosis
  • Blue / green / red bowel:
    • Coloration different
    • No histologic changes
  • Brown bowel / ceroidosis:
    • Pigment in smooth muscle cells
  • Hemochromatosis:
    • Pigment in epithelial cells
  • Pseudomelanosis:
    • Black pigment in macrophages of duodenum
Board review style question #1

This finding in the colon is typically secondary to

  1. Chronic granulomatous disease
  2. Laxative use
  3. Metastatic melanoma
  4. Pica
Board review style answer #1
B. Laxative use

Comment Here

Reference: Melanosis coli

Metastases
Definition / general
  • Secondary involvement of colon by a neoplasm of extracolonic origin
  • Uncommon (for example, occurs in 2% of breast and lung carcinomas)
Essential features
  • Metastases more common in small intestine than in colon
  • Primary lesion is often but not always identifiable
  • Most common primaries are stomach, lung, prostate, breast and ovarian carcinomas and melanoma
Clinical features
  • Endometrioid adenocarcinoma: if associated with colonic endometriosis, then represents an enigmatic primary rather than a metastasis
  • Leukemia: usually involves right colon and terminal ileum, as polypoid or transmural infiltrates
  • Melanoma: may have long interval between primary lesion and metastases (Dis Colon Rectum 2003;46:441)
  • Prostate: more common to be direct spread / invasion than a metastasis
    • Resections for rectal cancer may rarely contain lymph nodes with metastatic prostate carcinoma
Clinical images

Images hosted on other servers:
Missing Image

Soft tissue mass
due to squamous
cell carcinoma
primary of lung

Case reports
Gross description
  • Central ulceration extending toward mucosa but often sparing it
Microscopic (histologic) description
  • Disease usually resembles primary tumor microscopically
  • Metastases may be present along serosal surface or within the wall but mucosal involvement is uncommon
  • Adjacent / overlying colonic epithelium is not dysplastic
Microscopic (histologic) images

Contributed by Raul S. Gonzalez, M.D.

Metastatic breast carcinoma

Metastatic ovarian clear cell carcinoma



Images hosted on other servers:
Missing Image

Metastatic melaonoma to sigmoid colon

Missing Image

Poorly differentiated
squamous cell
carcinoma metastatic
from lung

Sample pathology report
  • Descending colon, resection:
    • Segment of colon with multiple serosal foci of metastatic high grade serous carcinoma, consistent with known ovarian primary
    • Margins of resection unremarkable.
    • Eight lymph nodes, negative for malignancy (0/8).
Differential diagnosis
Board review style question #1
While metastases to the colon are rare, some diseases metastasize there more often than others. Which of the following malignancies metastasizes to the colon relatively frequently?

  1. Melanoma
  2. Salivary gland carcinoma
  3. Synovial sarcoma
  4. Urothelial carcinoma
Board review style answer #1
A. Melanoma

Comment Here

Reference: Metastases

Micropapillary carcinoma
Definition / general
  • Uncommon subtype of colonic adenocarcinoma with distinctive behavior
  • Officially recognized in WHO classification
Essential features
  • Histology: small clusters of malignant cells with abundant eosinophilic cytoplasm and pleomorphic nuclei
  • High rate of lymph node metastasis (even when early stage) and poor prognosis
Epidemiology
Pathophysiology
Case reports
Microscopic (histologic) description
  • Small clusters of malignant cells with abundant eosinophilic cytoplasm and pleomorphic nuclei
  • Micropapillae inhabit lacunar-like spaces and demonstrate a reverse polarity configuration, with apical surfaces facing the periphery rather than the center
  • Lymphovascular invasion is commonly present
  • Morphology is similar to micropapillary carcinomas of other organs (Adv Anat Pathol 2004;11:297)
  • Coexisting regions of conventional adenocarcinoma also usually present
Microscopic (histologic) images

Contributed by Raul S. Gonzalez, M.D.

Micropapillary carcinoma

Positive stains
Molecular / cytogenetics description
Sample pathology report
  • Ascending colon, mass, biopsy:
    • Carcinoma with prominent micropapillary features (see comment)
    • Comment: Micropapillary carcinoma of the colon has a reported propensity to metastasize to lymph nodes even when early stage.
Differential diagnosis
Board review style question #1

Which of the following is true about micropapillary carcinoma of the colon?

  1. It is characterized by high microsatellite instability
  2. It metastasizes to lymph nodes often
  3. It represents < 1% of colon cancers
  4. Micropapillae are the same thing as tumor budding
Board review style answer #1
B. It metastasizes to lymph nodes often

Comment Here

Reference: Micropapillary carcinoma

Microsatellite instability pathway
Definition / general
  • Microsatellites (short tandem repeats) are repetitive DNA sequences composed of repeating motifs of 2 - 10 nucleotides
  • When there are defects in mismatch repair (MMR) genes (MLH1, PMS2, MSH2, MSH6), mutations introduced into microsatellite regions during DNA synthesis are not repaired, which results in microsatellite instability (MSI)
  • Microsatellite instability (MSI) is one of the pathways implicated in colorectal adenocarcinoma (CRC) carcinogenesis; colorectal cancer with MSI is referred to as MSI / microsatellite unstable or MMR deficient (dMMR) colorectal cancer
Essential features
  • Microsatellite instability (MSI) results from abnormal function of one or more mismatch repair genes (MLH1, PMS2, MSH2, MSH6)
  • 10 - 15% of colorectal adenocarcinomas (CRC) are MSI / MMR deficient (dMMR)
  • dMMR colorectal cancer arises due to sporadic hypermethylation of the MLH1 promoter or due to germline mutations in MMR genes (Lynch syndrome)
  • In pathology, colorectal cancer is initially screened for MSI by assessing the presence of MMR proteins using immunohistochemistry
  • Loss of one or more MMR proteins is followed by either MLH1 hypermethylation testing or BRAF mutation testing; the absence of MLH1 hypermethylation or a BRAF mutation prompts germline mutation to assess for Lynch syndrome
  • Identifying dMMR colorectal cancer is significant as these tumors have a better stage adjusted survival compared to microsatellite stable (MSS) tumors
Terminology
  • Lynch syndrome was formerly known as hereditary nonpolyposis colorectal cancer (HNPCC) syndrome
  • Biallelic germline mutation in an MMR related gene is termed constitutional mismatch repair deficiency
Epidemiology
  • 10 - 15% colorectal cancer is dMMR (Clin Cancer Res 2012;18:1506)
    • 80% of dMMR colorectal cancer is sporadic
      • Hypermethylation of MLH1 promoter
      • ≥ 60 years old
      • F > M
    • 20% of dMMR colorectal cancer is familial
      • Autosomal dominant, germline mutation (Lynch syndrome)
      • ~ 40 - 50 years old
      • M = F
Sites
Pathophysiology
  • MMR pathway (see diagram 1) (J Gastroenterol 2020;55:15)
    • Corrects DNA base substitution mismatch, insertions, deletions or slippage made during DNA replication
    • Errors are corrected through the action of two protein complexes:
      • MutSα: heterodimer of mutS homologue 2 (MSH2) and mutS homologue 6 (MSH6) proteins
      • MutSβ: heterodimer of MSH2 and mutS homologue 3 (MSH3) proteins
    • MutSα or MutSβ bind to the area with the defect and recruit MutLα (heterodimer of mutL homologue 1 [MLH1] and postmeiotic segregation increased 2 [PMS2] proteins)
    • PMS2 endonuclease makes a nick at 5’ to the mismatch (upstream)
    • Exonuclease 1 is recruited and activated by MSH2 or MLH1
      • Catalyzes excision of the nascent DNA strand up to and slightly beyond the mismatch
    • DNA excision gap is resynthesized by polymerase δ and nick is sealed by DNA ligase I
  • MLH1 is required for the function of PMS2 (when MLH1 is lost, PMS2 must also be lost)
  • MSH2 is required for the function of MSH6 (when MSH2 is lost, MSH6 must also be lost)
Etiology
  • Also see pathophysiology
  • Colorectal cancer is MSS / MMR proficient (pMMR) or MSI / MMR deficient (dMMR) (see diagram 2)
    • pMMR colorectal cancer: intact MMR genes
    • dMMR colorectal cancer: abnormality affecting one or more MMR genes
      • Sporadic
        • MLH1 promoter hypermethylation
        • 50% BRAF V600E mutation
        • ARID1A may have a role in carcinogenesis (Hum Pathol 2014;45:2430)
      • Familial - Lynch syndrome
        • Germline mutation followed by somatic mutation in wild type allele
        • Mutations in MLH1, PMS2, MSH2, MSH6, EPCAM (adjacent to MSH2)
      • Other:
Diagrams / tables

Images hosted on other servers:

DNA mismatch repair pathway

Intact versus defective mismatch repair mechanism

Algorithm: screening
colorectal cancer
for Lynch syndrome

Clinical features
  • Lynch syndrome is associated with increased risk of carcinoma at other sites
    • Gastrointestinal: colon, small intestine, stomach, hepatobiliary, pancreas
    • Genitourinary: kidney, bladder, prostate
    • Gynecological: endometrium, ovary
Diagnosis
  • Clinical assessment
    • Patients with dMMR colorectal cancer may be asymptomatic and have an incidentally discovered mass during surveillance colonoscopy, while others may present with signs of MSS colorectal cancer (anemia, fatigue, weight loss, change in bowel habits or hematochezia) - prompting colonoscopy
    • If there is clinical suspicion for Lynch syndrome, screening may be done using the Amsterdam Criteria or Bethesda System:
      • Amsterdam criteria (Dis Colon Rectum 1991;34:424)
        • At least 3 relatives with colorectal cancer (1 is a first degree relative of the other 2)
        • At least 2 consecutive generations involved
        • At least 1 person with colorectal cancer diagnosed < 50 years old
        • Familial adenomatous polyposis (FAP) excluded
      • Bethesda system (revised) (J Natl Cancer Inst 1997;89:1758)
        • Colorectal cancer diagnosed < 50 years old
        • Presence of other colorectal cancer or other Lynch syndrome associated tumors
        • Colorectal cancer with MSI-H phenotype diagnosed < 60 years old (MSI-H versus MSI-L discussed under Molecular / cytogenetics section)
        • Patient with colorectal cancer with 2+ first or second degree relatives with a Lynch syndrome associated tumor
  • Pathologic assessment (see diagram 3)
    • Most institutions test all colorectal cancer cases for MMR deficiency
      • Immunohistochemistry (IHC): IHC markers for MLH1, PMS2, MSH2, MSH6
        • Loss of one or more markers prompts additional testing
          • BRAF mutation status (IHC or PCR)
          • MLH1 methylation status
        • Absence of methylation or BRAF mutation prompts germline testing
          • Next generation sequencing
      • Specific staining patterns
        • MLH1 / PMS2 loss: favor sporadic
        • MSH2 / MSH6 loss: favor familial
        • Isolated PMS2 loss: favor familial
        • Isolated MSH6 loss: favor familial
Laboratory
  • Similar to findings as MSS colorectal cancer: anemia with or without elevated CEA
Radiology description
  • Similar to findings as MSS colorectal cancer (Radiographics 2000;20:419)
    • Luminal obstruction with or without soft tissue mass
    • Fat stranding
    • Lymphadenopathy
Prognostic factors
Case reports
  • 57 year old white woman with a history of colon cancer status post hemicolectomy and diagnosis of Lynch syndrome (BMJ Case Rep 2018;2018)
  • 61 year old man had a colonoscopy that showed metachronous colorectal carcinoma with massive submucosal invasion (World J Surg Oncol 2017;15:140)
  • 62 year old man with mismatch repair deficient metastatic colorectal adenocarcinoma, urothelial carcinoma and a history of sebaceous carcinomas (Cancer Biol Ther 2017;18:651)
Treatment
  • Surgery
    • pMMR colorectal cancer
      • Node negative disease = surgical excision
      • Higher stage (T3, T4) or LN positive = consider chemotherapy
    • dMMR colorectal cancer
      • Surgical excision
      • Lynch: with or without prophylactic hysterectomy
  • Immunotherapy
  • Other
Gross description
  • Larger masses with a polypoid / exophytic appearance
  • Classic colorectal cancer appearance: ulcerated lesion with heaped up / rolled edges
  • Reference: Dis Colon Rectum 1977;20:661
Gross images

Contributed by Phoenix D. Bell, M.D.

MMR deficient adenocarcinoma

MMR deficient mucinous adenocarcinoma

MMR deficient medullary carcinoma

MMR deficient mucinous adenocarcinoma

Microscopic (histologic) description
  • dMMR colorectal cancer histologic characteristics (Cancer 2001;91:2417, Anticancer Res 1994;14:1631):
    • Mucinous, poorly differentiated, signet ring or medullary subtypes
    • Increased intratumoral / peritumoral lymphocytes
    • Crohn-like reaction
    • Relative lack of intraluminal (dirty) necrosis
    • Tumoral heterogeneity
    • Decreased tumor budding (Hum Pathol 2011;42:1833)
Microscopic (histologic) images

Contributed by Phoenix D. Bell, M.D.

Colon adenocarcinoma biopsy (low power)

MMR deficient colorectal cancer (low power)

Colon adenocarcinoma resection

MMR deficient colorectal cancer (medium power)

MMR deficient medullary carcinoma

MMR deficient mucinous adenocarcinoma

Positive stains
  • MSS / pMMR colorectal cancer: all MMR proteins intact (MLH1, PMS2, MSH2, MSH6)
Negative stains
Molecular / cytogenetics description
  • BRAF somatic mutation testing
    • DNA change C.1799T>A
    • Amino acid change: p.V600E (Val600Glu)
  • BRAF mutation absent: send for somatic tests to evaluate tumor for Lynch syndrome
  • Germline mutation testing
    • PCR amplification of seven markers including five mononucleotide repeat markers (BAT-25, BAT-26, NR-21, NR-24 and MONO-27) and two pentanucleotide repeat markers (Penta C and Penta D) followed by separation of the PCR products by capillary electrophoresis
      • ≥ 2 unstable loci = MSI-H, < 1 unstable loci = MSI-L, 0 unstable loci = MSS
  • Reference: Cancer Res 1998;58:5248
Molecular / cytogenetics images

Contributed by Phoenix D. Bell, M.D.

Microsatellite instability analysis

Sample pathology report
  • Interpretations for MSI immunohistochemistry, BRAF mutation or hypermethylation status should be included in a colorectal adenocarcinoma pathology report. Examples are as follows:
    • MSI (by IHC) interpretation:
      • pMMR colorectal cancer:
        • Immunohistochemical stains for the mismatch repair proteins MLH1, MSH2, MSH6 and PMS2 were performed. The tumor cells demonstrate retained nuclear staining for all four proteins, indicating the tumor is MMR proficient (pMMR) and likely microsatellite stable.
      • dMMR colorectal cancer:
        • Immunohistochemical stains demonstrate the tumor is mismatch repair protein deficient (dMMR) with loss of expression of MLH1 and PMS2, while nuclear expression of MSH2 and MSH6 is retained. BRAF mutational analysis (or methylation status) is being performed to determine if this represents a sporadic type carcinoma.
    • BRAF mutation interpretation:
      • Absence of mutation:
        • BRAF mutational analysis was performed and the V600E mutation was not identified. These are somatic tests being performed to evaluate the tumor phenotype. dMMR tumors may be sporadic or associated with Lynch syndrome / hereditary nonpolyposis colorectal cancer. Referral to a hereditary cancer screening program is recommended.
      • Presence of mutation:
        • Molecular testing demonstrated the presence of BRAF V600E mutation, which correlates with sporadic hypermethylation of the MLH1 promoter. However, if there is a strong family history, caution should be exercised in excluding patients from germline screening on the basis of BRAF V600E mutations.
Differential diagnosis
Board review style question #1

A 64 year old woman presents for routine colonoscopy and a mass is identified in the ascending colon. She undergoes a partial hemicolectomy. Representative histologic sections of the mass are shown. Which of the following features is associated with MSI-H colorectal carcinoma?

  1. Extensive intraluminal (dirty) necrosis
  2. Lack of a Crohn-like reaction
  3. Micropapillary subtype
  4. Mucinous features
Board review style answer #1
D. Mucinous features. Histologically, traditional MSS colorectal adenocarcinomas are seen as invasive irregular glands surrounded by a desmoplastic stroma. The cells are composed of elongated hyerchromatic nuclei, surrounding luminal (dirty) necrosis. In contrast, MSI / dMMR colorectal carcinomas are associated with mucinous / poorly differentiated / medullary / signet ring cell subtypes, a Crohn-like reaction (dense lymphoid aggregates), increased intratumor / peritumoral lymphocytes, relative lack of intraluminal necrosis and tumoral heterogeneity. Although many institutions regularly test colorectal carcinoma cases for MSI, these features in particular should signal one to consider an MSI / dMMR tumor.

Comment Here

Reference: Microsatellite instability pathway
Board review style question #2
Sporadic MMR deficient colorectal carcinoma is due to promoter hypermethylation of which of the following genes?

  1. EPCAM
  2. MLH1
  3. MSH2
  4. MSH6
  5. PMS2
Board review style answer #2
B. Hypermethylation of the MLH1 (heterodimer of mutL homologue 1) promoter is the most common cause of sporadic microsatellite unstable (MSI) CRC. MLH1 is part of a heterodimer, which also includes post-meiotic segregation increased 2 (PMS). MLH1/PMS2 are members of the DNA mismatch repair (MMR) pathway, which fixes mutations in microsatellite regions made during DNA replication. Defects in any of the MMR genes (including MLH1) prevent these mutations from being repaired, resulting in MSI/MMR deficient (dMMR) CRC. In contrast to promoter hypermethylation that leads to sporadic MMR deficient colorectal carcinoma, Lynch syndrome is due to germline mutations in any of the MMR genes.

Comment Here

Reference: Microsatellite instability pathway

Mixed neuroendocrine nonneuroendocrine neoplasm
Definition / general
  • Extremely rare and aggressive with a high grade neuroendocrine component (in the majority of cases)
  • Composed of neuroendocrine cells, nonneuroendocrine cells or amphicrine cells that show both neuroendocrine and nonneuroendocrine cell differentiation (Int J Surg Case Rep 2020;76:125)
Essential features
  • Mix of neuroendocrine and nonneuroendocrine components, each constituting at least 30% of the tumor; both components should be histologically and immunohistochemically proven
  • Biological behavior is predominantly driven by the endocrine component, which is mostly poorly differentiated (World J Gastroenterol 2020;26:5181)
  • Prognosis of mixed neuroendocrine nonneuroendocrine neoplasm (MiNEN) is intermediate between that of pure adenocarcinoma and pure poorly differentiated neuroendocrine carcinoma (Neuroendocrinology 2017;105:412)
  • When considering treatment, the more aggressive component of MiNEN should be considered first; MiNEN containing a well differentiated neuroendocrine tumor (NET) (grades 1 or 2) component and an adenocarcinoma component should be treated as adenocarcinoma (J Chin Med Assoc 2015;78:454)
Terminology
ICD coding
  • ICD-10:
    • C7A.1 - malignant poorly differentiated neuroendocrine tumors
    • C7A.029 - malignant carcinoid tumor of the large intestine
    • C7A.020 - malignant carcinoid tumor of the appendix
    • C7A.8 - other malignant neuroendocrine tumors
Epidemiology
Sites
Pathophysiology
  • 3 main hypotheses:
    • First hypothesis: 2 components (endocrine and nonendocrine) arise independently from distinct precursor cells and unite
    • Second hypothesis: 2 components originate from a common pluripotent stem cell that undergoes biphenotypic differentiation during tumorigenesis
    • Third hypothesis: 2 components arise from a common precursor but postulates that the neuroendocrine component develops from an originally nonneuroendocrine cell, via progressive accumulation of genetic aberrations (World J Gastroenterol 2019;25:5991)
Etiology
Diagrams / tables

Images hosted on other servers:

Treatment modalities in retrospective study

Clinical features
  • Mass lesion, gastrointestinal bleeding, bowel obstruction, intussusception or paraneoplastic syndromes, rarely carcinoid syndrome
Diagnosis
  • Discovered during colonoscopy, some are diagnosed on evaluation of distant metastasis
  • Imaging features are nonspecific and necessitate histopathologic confirmation
  • CT is helpful in identifying and staging the disease
Radiology description
Radiology images

Images hosted on other servers:

CT with colon tumor

CT: liver metastasis, intraperitoneal fluid

Prognostic factors
Case reports
Treatment
  • First consideration is the more aggressive component of MiNEN
  • MiNEN containing a well differentiated NET (grades 1 or 2) component and an adenocarcinoma component, should be treated as adenocarcinoma (J Chin Med Assoc 2015;78:454)
  • Surgery preferred for nearly all potentially curable cases
  • Chemotherapy and radiotherapy considered on case by case basis (World J Gastroenterol 2020;26:5181)
  • Antiprogrammed death 1 receptor monoclonal antibody (pembrolizumab) therapy is effective where 5 - 10% of tumor cells express the programmed cell death receptor ligand 1 (J Immunother 2019;42:274)
Clinical images

Images hosted on other servers:

Tumor causing intussusception

Colonoscopy

Gross description
  • Polypoid or ulcerated tumor with raised edges or large fungating mass (World J Gastroenterol 2020;26:5181)
  • Cut surface of the tumor often shows white tan, poorly circumscribed lesions with an infiltrating border
  • Focal necrosis and foci of hemorrhage may be seen
Gross images

Images hosted on other servers:

Multiple polypoid masses

Microscopic (histologic) description
  • 2 components include epithelial / nonneuroendocrine component (mostly adenocarcinoma; less commonly, squamous cell carcinoma, acinar cell carcinoma, adenoma and other tumors) and neuroendocrine component, each constituting at least 30% of the tumor
    • This cut off value only applies to resected specimens after histological examination of the entire neoplasm within the specimen
  • Previously, these mixed neoplasms were classified under the category of mixed adenoneuroendocrine carcinoma; however, it is now recognized that the nonneuroendocrine component is not necessarily adenocarcinoma and either one or both components may not be carcinomas
  • Both components should be histologically and immunohistochemically proven
  • Per WHO, evaluate and grade each histologic component separately
  • All neuroendocrine components are classified according to the WHO criteria based on mitotic and Ki67 index, into well differentiated neuroendocrine tumors (G1, G2 and G3) and poorly differentiated neuroendocrine carcinomas (small or large cell neuroendocrine tumors)
  • Neuroendocrine component is mostly high grade and demonstrate a high mitotic index
    • Small cell type: nested or diffuse proliferation of tumor cells with round to oval nuclei, finely dispersed chromatin, inconspicuous nucleoli and scant cytoplasm
    • Large cell type: cells with abundant cytoplasm, vesicular nuclei and prominent nucleoli (Endocr Pathol 2016;27:284)
    • Prominent oncocytic differentiation represented by large cells with marked eosinophilia, has also been reported in a mixed tumor of the transverse colon (Endocr Pathol 2013;24:54)
  • Amphicrine tumors are distinguished by a divergent immunophenotype in which both exocrine and neuroendocrine traits are expressed in the same cell (Cancer Cell Int 2019;19:310)
  • About 5% of colorectal MiNEN show low grade neoplasms, combining adenomas with G1 / G2 NET (Neuroendocrinology 2017;105:412)
  • Because of possible response to platinum based treatments, the presence of even a minor component of small cell neuroendocrine carcinoma (SCNEC) should be mentioned in the diagnosis
    • Presence of a focal (< 30%) neuroendocrine component does not change the diagnostic categorization but it may be mentioned in the report
Microscopic (histologic) images

Contributed by Lewis A. Hassell, M.D. and Catherine E. Hagen, M.D.
Mixed acinar cell neuroendocrine carcinoma

Mixed acinar cell neuroendocrine carcinoma

Acinar cell carcinoma component

Acinar cell carcinoma component

Neuroendocrine carcinoma component

Neuroendocrine carcinoma component

Mixed adenocarcinoma neuroendocrine carcinoma

Mixed adenocarcinoma neuroendocrine carcinoma

Small cell carcinoma

Small cell carcinoma

Adenocarcinoma

Adenocarcinoma


Chromogranin immunohistochemistry

Chromogranin IHC

Synaptophysin immunohistochemistry Synaptophysin

Synaptophysin IHC

Trypsin immunohistochemistry

Trypsin IHC

Virtual slides

Images hosted on other servers:

Multiple polypoid masses

Positive stains
Negative stains
Molecular / cytogenetics description
Videos

Mixed neuroendocrine and nonneuroendocrine carcinoma of the esophagus

Mixed neuroendocrine nonneuroendocrine neoplasms

Sample pathology report
  • Sigmoid colon, resection:
    • Mixed neuroendocrine nonneuroendocrine neoplasm with well differentiated adenocarcinoma (G1, 30%) and poorly differentiated small cell neuroendocrine carcinoma (70%)
    • See synoptic report for more details

      Brief synoptic report
      Tumor site Sigmoid colon
      Histologic type Mixed neuroendocrine nonneuroendocrine carcinoma
      Size 4.5 x 2.0 x 1.2 cm
      Depth of invasion Subserosal adipose tissue
      Lymphovascular invasion Present
      Perineural invasion Not identified
      Margins Uninvolved by invasive carcinoma, intramucosal
      adenocarcinoma, high grade dysplasia and adenoma
      Lymph nodes with metastatic tumor 2
      Lymph nodes examined 15
      Tumor deposits Not identified
      Distant metastasis Not applicable
      AJCC staging pT3 N1
Differential diagnosis
Board review style question #1


An 8 mm polypoid mass located in the sigmoid colon was resected. The microscopic examination revealed a biphasic tumor that was positive for epithelial and neuroendocrine markers, each component accounting for > 30% of the tumor (see images above: chromogranin, trypsin). Which statement is true regarding this lesion?

  1. It is a common colorectal neoplasm
  2. It is mostly an aggressive tumor
  3. More common in females
  4. Recognized as part of the multiple endocrine neoplasia type 1 group of tumors
  5. Treatment is determined by the predominant (volume wise) component
Board review style answer #1
B. It is mostly an aggressive tumor

Comment Here

Reference: Mixed neuroendocrine nonneuroendocrine neoplasm

Mucinous adenocarcinoma
Definition / general
  • WHO recognized subtype of colorectal carcinoma with mucin lakes comprising at least 50% of tumor mass, though utility of 50% cutoff is unclear (Histopathology 2019;74:406)
Essential features
  • Most common subtype of colorectal carcinoma
  • Mucin production easily observed on histology
  • Some cases are microsatellite instability (MSI) high
  • Unclear whether worse prognosis than conventional colorectal carcinoma
Terminology
  • Variably called colloid carcinoma
Epidemiology
Sites
Pathophysiology
Clinical features
Prognostic factors
Case reports
Gross description
  • Exophytic, gelatinous mass
Gross images

Images hosted on other servers:
Missing Image

Causing intussusception

Missing Image

Cecal tumor

Missing Image

Arising in villous adenoma

Microscopic (histologic) description
  • Adenocarcinoma with strips of tumor cells floating in large extracellular mucin lakes comprising at least half of tumor mass
  • Signet ring cells (with intracellular mucin) may also be present but should comprise less than half of tumor
Microscopic (histologic) images

Contributed by Raul S. Gonzalez, M.D.

Abundant extracellular mucin

Epithelial cells within mucin

Positive stains
Molecular / cytogenetics description
Sample pathology report
  • Ascending colon, resection:
    • Mucinous adenocarcinoma, well differentiated (see synoptic report)
Differential diagnosis
Board review style question #1

Which of the following is true about mucinous adenocarcinoma of the colorectum?

  1. It is the most common subtype of colorectal carcinoma
  2. It unequivocally has a better prognosis than standard colorectal carcinoma
  3. It usually affects pediatric patients
  4. Signet ring cells are always at least a minor component
Board review style answer #1
A. It is the most common subtype of colorectal carcinoma

Comment Here

Reference: Mucinous adenocarcinoma

Mucosal prolapse polyp (pending)
Table of Contents
Definition / general
Definition / general
[Pending]

Mucosal Schwann cell hamartoma
Definition / general
  • Benign mucosal proliferation composed of Schwann cells
Essential features
  • Benign, incidental spindle cell lesion
  • Not associated with any clinical syndrome
  • Positive for S100, negative for EMA
Terminology
  • Has been called neuroma or neurofibroma in past
Sites
  • Can arise anywhere in the colorectum but more common distally
Clinical features
  • Average patient age is 62 years, with a female predominance (Am J Surg Pathol 2009;33:781)
  • Not associated with any particular syndrome
  • Lesions do not recur
Diagnosis
  • Discovered incidentally during colonoscopy
Case reports
Clinical images

Images hosted on other servers:

Rectal polyps on colonoscopy

Gross description
  • Small (6 mm or less) sessile polyp
Microscopic (histologic) description
  • Poorly circumscribed mucosal proliferation of spindle cells, with no whorling, no palisading, no fasciculation
  • Nuclei are generally small, bland and elongated
Microscopic (histologic) images

Contributed by Raul S. Gonzalez, M.D.

Lesion confined to mucosa

Bland nuclei with occasional enlargements

S100+



Images hosted on other servers:

Low power view of hematoxylin and eosin

S100 positivity

Histologic findings
of mucosal
Schwann cell
hamartomas

Positive stains
Sample pathology report
  • Transverse colon, polypectomy:
    • Mucosal Schwann cell hamartoma (see comment)
    • Comment: An immunohistochemical stain for S100 is positive.
Differential diagnosis
Board review style question #1

Which of the following is true about mucosal Schwann cell hamartoma of the colon?

  1. It demonstrates loss of nuclear H3K27me3 staining
  2. It is associated with neurofibromatosis
  3. It is usually > 1 cm
  4. It is usually discovered incidentally
Board review style answer #1
D. It is usually discovered incidentally

Comment Here

Reference: Mucosal Schwann cell hamartoma

Muir-Torre syndrome
Definition / general
  • Clinical variant of Lynch syndrome with cutaneous sebaceous neoplasms (including sebaceous adenoma, sebaceoma and sebaceous carcinoma) or keratoacanthomas in addition to visceral malignancies of Lynch syndrome
  • See also Lynch syndrome, sebaceous adenoma: skin, sebaceous carcinoma: skin, breast
Essential features
  • Variant of Lynch syndrome with skin manifestations
  • More common in men
Terminology
  • Also called Torre-Muir syndrome
Epidemiology
  • Sebaceous neoplasms present at mean age of 53 years
  • M:F = 3:2
Sites
  • Sebaceous tumors are most common below the neck
  • Colorectal tumors are more commonly located in proximal colon
Etiology
  • Typically due to germline mutations in a DNA mismatch repair gene: most commonly MSH2 (60 - 90%); may be due to mutation in MLH1 or MSH6
  • May also be due to inactivation through hypermethylation of a DNA mismatch repair gene
Clinical features
  • Skin neoplasm may be presenting lesion 22% of time (J Am Acad Dermatol 1999;41:681)
  • Colorectal adenocarcinoma is prototypical tumor of Lynch syndrome (typically proximal colon); however, patients may develop other low grade carcinomas, including endometrial carcinomas (15%) and genitourinary carcinomas
Diagnosis
  • Clinical algorithms include Amsterdam criteria and revised Bethesda criteria
  • Mayo Muir-Torre syndrome risk scoring system assesses whether patients with sebaceous neoplasms are in need of further evaluation for Lynch Syndrome (Genet Med 2014;16:711)
  • Can screen using immunohistochemical testing for loss of mismatch repair protein expression (MLH1, MSH2, MSH6, PMS2) or microsatellite instability testing via PCR
  • Germline genetic testing
Prognostic factors
  • Muir-Torre associated neoplasms tend to be less aggressive than the same tumors in nonsyndromic patients
Case reports
  • 58 year old man with retroperitoneal undifferentiated pleomorphic sarcoma associated with Muir-Torre syndrome (J Cutan Pathol 2013;40:730)
  • Unusual case of hepatocellular carcinoma in noncirrhotic background in Muir-Torre patient (Fam Cancer 2012;11:7)
  • Sebaceous adenoma arising within an ovarian mature cystic teratoma in Muir-Torre syndrome (Ann Diagn Pathol 2012;16:485)
Microscopic (histologic) description
  • Cutaneous tumors demonstrate sebaceous origin or differentiation, including coarsely vacuolated cytoplasm and starry nuclei (mulberry cells) (Arch Pathol Lab Med 2014;138:1685)
  • Muir-Torre syndrome associated sebaceous tumors are often nodulocystic
  • Cystic sebaceous neoplasms have only been reported in patients with Muir-Torre syndrome
  • Colorectal adenocarcinomas tend to have mucinous features, with tumor infiltrating lymphocytes and Crohn-like reaction
Microscopic (histologic) images

Images hosted on other servers:
Missing Image

MSH2+

Missing Image

Significant loss of MSH6

Missing Image

Colon adenoma and colonic mucosa

Positive stains
  • EMA in sebaceous neoplasms
Negative stains
Molecular / cytogenetics description
  • Autosomal dominant inheritance
  • Mutations most commonly in MLH1 or MSH2 mismatch repair genes (3p22.2 and 2p21, respectively)
  • Microsatellite instability is present in approximately 70% of Muir-Torre syndrome associated tumors
  • Germline testing confirms the diagnosis
Differential diagnosis
  • Lynch syndrome:
    • Similar presentation and genetics, except no sebaceous neoplasms
Board review style question #1
Muir-Torre syndrome is considered a clinical variant of what syndrome?

  1. Cowden syndrome
  2. Juvenile polyposis syndrome
  3. Li-Fraumeni syndrome
  4. Lynch syndrome
Board review style answer #1
D. Lynch syndrome

Comment Here

Reference: Muir-Torre syndrome
Board review style question #2
Which of the following is a typical manifestation of Muir-Torre syndrome?

  1. Cylindroma
  2. Desmoid fibromatosis
  3. Sebaceous adenoma
  4. Trichilemmoma
Board review style answer #2
C. Sebaceous adenoma

Comment Here

Reference: Muir-Torre syndrome

MUTYH associated polyposis
Definition / general
  • Autosomal recessive polyposis syndrome characterized by development of numerous colon adenomas but usually fewer than in classic familial adenomatous polyposis
Essential features
  • Attenuated polyposis syndrome with biallelic MUTYH mutations and no FAP mutation
  • Increased risk for colorectal carcinoma
Terminology
  • Outdated terms include:
    • MYH associated polyposis
    • Familial adenomatous polyposis 2
    • Autosomal recessive adenomatous polyposis
Epidemiology
Sites
Clinical features
  • Mean age of diagnosis is about 50 years
  • Patients often have between 10 and 100 polyps (Int J Cancer 2004;109:680)
  • Biallelic mutations confer a 50 fold risk of colorectal carcinoma and monoallelic mutation a 3 fold risk (Cancer Epidemiol Biomarkers Prev 2006;15:312)
  • Patients can also develop ovarian, bladder and breast carcinoma, as well as sebaceous gland tumors
Diagnosis
  • Molecular testing to confirm biallelic MUTYH mutations
Case reports
Microscopic (histologic) description
  • Patients most commonly develop colonic tubular adenomas that resemble sporadic adenomas
  • Sessile serrated polyps can also occur
Microscopic (histologic) images

Contributed by Raul S. Gonzalez, M.D.

Adenoma associated with MUTYH mutation

Molecular / cytogenetics description
  • MUTYH is located at 1p34 and repairs oxidative DNA damage that otherwise can lead to carcinoma (Mod Pathol 2013;26:1371)
  • Most common mutations are Y165C and G382D
Differential diagnosis
Board review style question #1
How does MUTYH associated polyposis differ from familial adenomatous polyposis?

  1. It is autosomal recessive
  2. Molecular testing cannot confirm the diagnosis
  3. Patients never develop upper gastrointestinal tract adenomas
  4. The polyps are all serrated
Board review style answer #1
A. It is autosomal recessive

Comment Here

Reference: MUTYH associated polyposis

Mycobacterium avium-intercellulare (pending)
[Pending]

Mycophenolate mofetil associated colitis
Definition / general
  • Gastrointestinal tract toxicity related to mycophenolate therapy
  • Mycophenolate mofetil (CellCept) is a therapeutic immunosuppressant agent that inhibits the proliferation of B and T cells through noncompetitive, reversible inhibition of inosine monophosphate dehydrogenase and is used for (Pharmacotherapy 1997;17:1178):
    • Prevention of acute rejection of solid organ transplants
    • Treatment of various inflammatory and autoimmune disorders
    • Prophylaxis of acute graft versus host disease (GVHD) and treatment of chronic graft versus host disease
Essential features
  • Gastrointestinal tract toxicity commonly seen in transplant patients taking mycophenolate, particularly renal transplant recipients
  • Symptoms resolve with medication withdrawal or dose reduction
  • Histologically characterized by crypt apoptosis, crypt injury, lamina propria eosinophils and architectural distortion
  • Lack of neuroendocrine cell aggregates, presence of eosinophils and paucity of apoptotic microabscesses can help distinguish from graft versus host disease
Terminology
  • Also called CellCept colitis
ICD coding
  • ICD-10: Z79.899 - other long term (current) drug therapy
Epidemiology
Sites
  • Anywhere in gastrointestinal tract can be involved:
Pathophysiology
Etiology
Clinical features
Diagnosis
  • Endoscopy with tissue biopsy
  • Symptom resolution following drug withdrawal / dose reduction
  • Exclusion of other etiologies (e.g. infection, other drugs, etc.)
Prognostic factors
  • Patients universally do well following withdrawal or dose reduction
Case reports
Treatment
  • Dose reduction or discontinuation of mycophenolate
Clinical images

Images hosted on other servers:

Endoscopic appearance

Microscopic (histologic) description
  • Crypt apoptosis with no or few apoptotic microabscesses
  • Increased eosinophils (> 15/10 high powered fields) present in lamina propria
  • Neutrophilic inflammation may also be present in lamina propria; cryptitis and crypt abscess may be present but usually focal (J Clin Pathol 2013;66:8)
  • Lack of lamina propria endocrine cell aggregates
  • Architectural distortion
  • Pseudopyloric gland metaplasia can be seen (Int J Surg Pathol 2003;11:295)
  • Injured dilated eosinophilic crypts may be present but typically to a lesser degree than graft versus host disease
  • Can have Crohn's-like appearance with patchy inflammation and lymphoid hyperplasia (Int J Surg Pathol 2003;11:295)
  • Cases are not routinely graded but if necessary the Lerner system can be used (Am J Surg Pathol 2013;37:1319)
Microscopic (histologic) images

Contributed by Catherine E. Hagen, M.D.

Architectural distortion

Injured crypt

Apoptosis

Inflammation

Negative stains
  • CMV immunostain
Sample pathology report
  • Colon, biopsy:
    • Colonic mucosa with increased crypt apoptosis and mild architectural distortion (see comment)
    • Comment: Given the patient’s reported use of mycophenolate, the histologic features are suggestive of mycophenolate colitis (or give differential diagnosis as appropriate).
Differential diagnosis
Board review style question #1


A 45 year old man who underwent a kidney transplant one year ago presents complaining of watery diarrhea. A colonoscopy and biopsy are performed. CMV immunohistochemistry is negative. Based on the histologic findings, which of the following is the most likely diagnosis?

  1. Graft versus host disease
  2. Infectious colitis
  3. Mycophenolate toxicity
  4. Ulcerative colitis
Board review style answer #1
C. Mycophenolate toxicity

Comment Here

Reference: Mycophenolate mofetil colitis
Board review style question #2
Which of the following histologic features is not helpful in distinguishing mycophenolate toxicity from graft versus host disease?

  1. Apoptotic microabscesses
  2. Architectural distortion
  3. Endocrine cell aggregates
  4. Eosinophilic inflammation
Board review style answer #2
B. Architectural distortion

Comment Here

Reference: Mycophenolate mofetil colitis

Necrotizing enterocolitis
Definition / general
Etiology
  • Oral feeding of neonates with immature gut immune system causes release of proinflammatory cytokines; bacteria in food produce more cytokines which injure mucosa; intestinal blood flow may be disturbed
  • IL10 may prevent by attenuating intestinal inflammation (Am J Surg 2012;203:428)
Clinical features
  • Symptoms: abdominal distention, loss of bowel sounds, blood-stained stool
  • Complications: bowel perforation, short bowel syndrome, malabsorption (due to ileal resection), strictures, recurrence
Treatment
  • Fluids and surgery if gangrene/perforation
Gross description
  • Necrotic mucosa, submucosal gas-filled cysts
Gross images

Images hosted on other servers:

In situ

Thickened wall, mucosal ulceration, cyst-like spaces

Transverse section shows cyst-like spaces in wall

Alimentary tract

Small intestine

Microscopic (histologic) description
  • Early: mucosal edema, hemorrhage, necrosis, pneumatosis cystoides intestinalis
  • Late: hemorrhagic and gangrenous bowel wall with strictures
Microscopic (histologic) images

Images hosted on other servers:
Missing Image

Hemorrhagic necrosis

Missing Image

Mucosal necrosis and cyst-like spaces in submucosa

Missing Image

Submucosal cyst-like spaces

Missing Image

Lined by flattened cells

Missing Image

Subserosal lymph node with cyst-like spaces

Differential diagnosis

Neuroendocrine carcinoma
Definition / general
Essential features
  • Large cell or small cell neuroendocrine carcinoma
  • Rare and aggressive (distant metastases common, with poor survival rate)
  • Histology similar to neuroendocrine carcinomas elsewhere
Terminology
  • Full proper term is poorly differentiated neuroendocrine carcinoma
  • 2 subtypes, based on morphology: large cell neuroendocrine carcinoma and small cell carcinoma
Sites
  • Usually in cecum or right colon but can arise anywhere
Epidemiology
  • Disease typically presents at advanced stage
Clinical features
Prognostic factors
  • Good prognostic factors include peritumoral lymphocytes, microsatellite instability and pure large cell features; poor prognostic factors include CD117 positivity and vascular invasion (Am J Surg Pathol 2012;36:601)
  • Slightly improved prognosis if disease is not metastatic or patient responds to chemotherapy (Ann Surg Oncol 2014;21:2956)
Case reports
Radiology images

Images hosted on other servers:
Missing Image

Rectal mass

Missing Image

Noncontrast CT scans of abdomen

Microscopic (histologic) description
  • Large cell:
    • Organoid arrangement of cells larger than those in small cell carcinoma, with nuclear pleomorphism and hyperchromasia, prominent nucleoli, numerous mitoses and tumor necrosis
  • Small cell:
    • Resembles pulmonary small cell carcinoma, with sheets and nests of small ovoid cells with minimal cytoplasm, hyperchromatic nuclei with stippled chromatin, nuclear molding with peripheral palisading, brisk mitotic activity, apoptotic cells, necrosis and vascular invasion
    • No prominent nucleoli or pleomorphism
    • May have Azzopardi effect (encrustation of nuclear material around blood vessels)
Microscopic (histologic) images

Contributed by Raul S. Gonzalez, M.D., Aaron Huber, D.O. and Diana Agostini-Vulaj, D.O. (Case #531)
Large cell neuroendocrine carcinoma Large cell neuroendocrine carcinoma

Large cell neuroendocrine carcinoma


Synaptophysin

Chromogranin

INSM1

Positive stains
Electron microscopy description
  • Dense core secretory granules
Sample pathology report
  • Sigmoid colon, resection:
    • Poorly differentiated neuroendocrine carcinoma (see synoptic report and comment)
    • Comment: The tumor demonstrates large cell neuroendocrine carcinoma morphology. Immunostains show the tumor is positive for synaptophysin and chromogranin, with a Ki67 index of 85%. By definition, neuroendocrine carcinomas are WHO grade 3.
Differential diagnosis
Board review style question #1
    Which of the following is true about neuroendocrine carcinomas of the colorectum?

  1. Large cell and small cell subtypes have differences in prognosis and clinical management
  2. They are positive for synaptophysin but negative for chromogranin
  3. They can be any WHO grade
  4. They present at a younger average age than colorectal adenocarcinoma
Board review style answer #1
D. They present at a younger average age than colorectal adenocarcinoma

Comment Here

Reference: Neuroendocrine carcinoma
Board review style question #2
    A colorectal tumor is resected and appears to be a poorly differentiated adenocarcinoma, with focal but convincing gland formation. A synaptophysin immunostain is performed and shows patchy scattered positive staining in about 5% of cells. How should this tumor be diagnosed?

  1. Mixed neuroendocrine nonneuroendocrine neoplasm
  2. Poorly differentiated adenocarcinoma
  3. Poorly differentiated neuroendocrine carcinoma (large cell neuroendocrine carcinoma)
  4. Poorly differentiated neuroendocrine carcinoma (small cell carcinoma)
Board review style answer #2
B. Poorly differentiated adenocarcinoma

Comment Here

Reference: Neuroendocrine carcinoma
Board review style question #3
    Which of the following is true of poorly differentiated neuroendocrine carcinoma, large cell type in the colon?

  1. INSM1 expression is a sensitive and specific immunohistochemical marker for neuroendocrine differentiation
  2. Mixed neuroendocrine nonneuroendocrine neoplasm may be diagnosed if the poorly differentiated neuroendocrine carcinoma comprises only 5% of the total tumor volume
  3. This tumor has an overall good prognosis
  4. This tumor is a common morphologic subtype of colorectal cancer
Board review style answer #3
A. INSM1 (insulinoma associated protein 1) is a sensitive and specific marker of neuroendocrine differentiation in neuroendocrine neoplasms. Answer B is incorrect because the WHO criteria for mixed neuroendocrine nonneuroendocrine neoplasm requires at least 30% of each component. Answers C and D are incorrect because LCNEC of the colon is a rare highly aggressive subtype of colorectal cancer with a poor prognosis. See Case #531 for more information.

Comment Here

Reference: Neuroendocrine carcinoma

Neuroendocrine tumor
Definition / general
  • Well differentiated tumors demonstrating morphological and immunohistochemical neuroendocrine differentiation
Essential features
  • May be indolent and incidental or present with widespread metastases and have aggressive clinical course
  • The term neuroendocrine tumor (NET) comprises WHO G1, G2 and rare G3 lesions (depending on the proliferative activity)
Terminology
  • Previously termed carcinoid tumor (Arch Pathol Lab Med 2016;140:437)
  • According to the 2017 WHO classification, the term MiNEN (mixed neuroendocrine neoplasm) comprises mixed neuroendocrine and nonneuroendocrine proliferations (World J Gastrointest Oncol 2020;12:791)
    • The nonneuroendocrine component may be adenocarcinoma, squamous cell carcinoma, among others
    • Each component must be ≥ 30% to fall into the category of MiNEN
ICD coding
  • ICD-10: D3A.8 - other benign neuroendocrine tumors
Epidemiology
  • Incidence of colorectal neuroendocrine tumors has been continuously increasing over the past few decades (incidence: 1.04 per 100,000 persons) (JAMA Oncol 2017;3:1335)
  • Colorectal neuroendocrine tumors comprise roughly 30% of neuroendocrine tumors in the digestive system (Cancer 2003;97:934)
  • According to the WHO classification (5th edition), there is a slight male predominance and the median age of presentation is 56 years for rectal tumors and 65 years for colonic tumors
Sites
  • In the large intestine (excluding the appendix), the most frequent location is the rectum (34% of all gastrointestinal neuroendocrine tumors) (Ann Oncol 2013;24:3040)
  • In the colon, neuroendocrine tumors are more frequent in the cecum (69.6%), followed by sigmoid (13.0%), ascending colon (13.0%) and transverse colon (4.3%) (Arq Gastroenterol 2009;46:288)
  • Colon proper is the least common site for intestinal well differentiated neuroendocrine tumors; around 7.5% of all neuroendocrine tumors are of colonic origin (Surg Oncol Clin N Am 2020;29:267)
Pathophysiology
Etiology
  • Etiology remains unknown
Clinical features
  • Patients with colonic neuroendocrine tumors may present with pain, bleeding, altered bowel habits, weight loss, anorexia or even bowel obstruction (Best Pract Res Clin Gastroenterol 2012;26:775)
  • Carcinoid syndrome occurs in patients with liver metastasis
  • Prognosis largely depends on the grade and stage of the tumor; median overall survival ranges from 22 months in G1 - G2 tumors to 12 months in G3 tumors according to the WHO classification (5th edition)
  • Increased risk of lymph node metastasis according to size: around 1% in tumors < 10 mm, 23% in tumors from 10 - 20 mm and 50% in tumors > 20 mm (J Korean Soc Coloproctol 2011;27:17)
  • G3 colorectal neuroendocrine tumors show an intermediate behavior between G2 neuroendocrine tumors and neuroendocrine carcinomas (Cancer 2014;120:2814)
Diagnosis
Laboratory
Prognostic factors
Case reports
Treatment
Clinical images

Images hosted on other servers:

Small rectal carcinoids on colonoscopy

Gross description
  • Most colorectal neuroendocrine tumors appear as yellow or pale, polypoid or flat, doughnut shaped lesions or submucosal nodules (Neuroendocrinology 2016;103:139)
Gross images

Contributed by Carolina Martinez Ciarpaglini, M.D., Ph.D.

Flat polyp

Yellowish colored tumor

Microscopic (histologic) description
  • Well differentiated neuroendocrine pattern:
    • Monotonous regular cells with round or oval nuclei with salt and pepper chromatin and moderate eosinophilic granular cytoplasm
    • Organoid architecture: tumor cells arranged in nests, trabecular or insular pattern (Oncologist 2016;21:1191)
  • In some cases, nuclear pleomorphism may be encountered (endocrine atypia) but is not associated with tumor aggressiveness (Oncologist 2016;21:1191)
  • Rarely mucin secretion
  • Grading scheme is based on the proliferative rate (mitoses and Ki67 index) as follows:
    • Low grade or grade 1 (G1): mitoses < 2 per 2 mm2 and Ki67 index < 3%
    • Intermediate grade or grade 2 (G2): mitoses 2 - 20 per 2 mm2 or Ki67 index 3 - 20%
    • High grade or grade 3 (G3): mitoses > 20 per 2 mm2 or Ki67 index > 20%
  • Mitotic count should be evaluated in a 2 mm2 hotspot area (roughly equivalent to 10 high power fields with a 40x objective lens) (Arch Pathol Lab Med 2016;140:437)
  • Ki67 index should be estimated in ≥ 500 cells in the hotspot regions (Oncologist 2016;21:1191)
  • If there is any discrepancy between mitotic index and Ki67 index, the higher should be considered for the classification (World J Gastrointest Oncol 2020;12:791)
Microscopic (histologic) images

Contributed by Carolina Martinez Ciarpaglini, M.D., Ph.D. and Yuri Tachibana, M.D.

Polypoid neuroendocrine tumor

Organoid pattern

Transverse colon mass

Salt and pepper chromatin

Rectal WDNET

Nested pattern


Monotonous cells

Insular pattern

Pseudorosette formation

Somatostatin receptor 2 (SSTR2)

Rectal neuroendocrine tumor

Absence of chromogranin expression


Synaptophysin Synaptophysin

Synaptophysin

Proliferative activity

Low Ki67 index

Positive stains
Molecular / cytogenetics description
Sample pathology report
  • Ascending colon mass, endoscopic biopsy:
    • Well differentiated neuroendocrine tumor, WHO grade 2 (G2), 7 mm (see comment)
    • Comment: There are 2 mitotic figures in 2 mm2 and the Ki67 proliferation index is approximately 5%, consistent with G2 in this material.
Differential diagnosis
  • Neuroendocrine carcinoma (NEC):
    • High grade tumors with poorly differentiated morphology
    • Atypical small or large cells organized in diffuse sheets with little nesting
    • Apoptotic bodies and necrosis are usually observed
    • They are high grade by definition (mitoses > 20 per 10 high power fields or Ki67 > 20%); however, the Ki67 index is usually > 50% (Neuroendocrinology 2016;103:139)
    • Remember that there are some G3 neuroendocrine tumors; poorly differentiated morphology is the main feature to distinguish neuroendocrine carcinomas from G3 neuroendocrine tumors
    • BCL2 overexpression, loss of Rb expression and abnormal p53 expression are seen more frequently in neuroendocrine carcinomas and may be useful in the differential diagnosis with well differentiated neuroendocrine tumors (Arch Pathol Lab Med 2016;140:437)
Board review style question #1

A 5 mm polyp lesion located in the ascending colon was resected. The microscopic image is shown above. Which statement is true regarding this lesion?

  1. CD56 immunostaining is more specific than synaptophysin for its diagnosis
  2. CDX2 expression may indicate the site of origin of this type of tumor
  3. Presence / absence of necrosis is the main feature in defining the tumor grade
  4. Proliferative activity is not relevant in these tumors
Board review style answer #1
B. CDX2 expression may indicate the site of origin of this type of tumor

Comment Here

Reference: Neuroendocrine tumor
Board review style question #2
Which of the following is associated with aggressiveness in colon / rectal neuroendocrine tumors?

  1. CDX2 expression
  2. Cytologic atypia
  3. Lack of expression of chromogranin
  4. Mitotic activity
Board review style answer #2
D. Mitotic activity

Comment Here

Reference: Neuroendocrine tumor

NSAID associated colitis
Definition / general
  • Associated with bleeding, diarrhea, abdominal pain, although usual side effects are subclinical (Am J Med Sci 2009;338:96)
Clinical features
Case reports
Treatment
Microscopic (histologic) description
  • Usually mixed inflammatory infiltrate but may be predominantly neutrophilic or lymphocytic (Am J Clin Pathol 1998;110:622, Adv Anat Pathol 1999;6:213)
  • Often erosions, crypt architectural disarray; no granulomas
  • Incidental chronic colitis: cecal based chronic colitis associated with frequent NSAID use; basal plasmacytosis (94%) or crypt architectural distortion (94%) often disappears after NSAID withdrawal (Am J Surg Pathol 2010;34:463)
Microscopic (histologic) images

Images hosted on other servers:
Missing Image

Ulcer post NSAID therapy


Omphalocele
Definition / general
Case reports
Clinical images

Images hosted on other servers:

Multiple defects and gastrochisis

Differential diagnosis
  • Gastroschisis: not through umbilical cord, no membranous sac covering intestines

Perineurioma
Definition / general
  • Benign polypoid lesion occurring in the colonic mucosa or submucosa
  • Peripheral nerve sheath tumor, often with overlying serrated / hyperplastic epithelium
Essential features
  • Uncommon mucosal epithelial-mesenchymal lesion of the colon
  • Usually occurs in middle aged women
  • Benign, does not recur
Terminology
Sites
  • Usually in distal colon; rarely in small intestine
Clinical features
Treatment
  • Excision (benign behavior)
Gross description
  • Small sessile polyp (median size 0.4 cm)
Microscopic (histologic) description
  • Poorly circumscribed spindle cell proliferation, usually confined to the mucosa, with ovoid nuclei and pale indistinct cytoplasm, in a background of fine collagenous stroma
  • No atypia, no pleomorphism, no mitotic figures
  • Entrapped / overlying colonic crypts appear hyperplastic or serrated in the majority of cases
  • Perineurial proliferation sometimes focally present in otherwise typical sessile serrated polyp / adenoma (Am J Surg Pathol 2011;35:1373)
Microscopic (histologic) images

Contributed by Raul S. Gonzalez, M.D. and Christopher Hartley, M.D.

Medium power

High power

Weak positivity on EMA

Perineurioma

Positive stains
Electron microscopy description
  • Long bipolar cytoplasmic processes, prominent pinocytotic vesicles
Molecular / cytogenetics description
Sample pathology report
  • Ascending colon, polypectomy:
    • Perineurioma (see comment)
    • Comment: Immunohistochemical stains show the lesion is positive for EMA (focal) and negative for S100.
Differential diagnosis
Board review style question #1

Colonic perineuriomas are positive for which of the following immunohistochemical stains?

  1. CD117
  2. Claudin1
  3. Desmin
  4. S100
Board review style answer #1
B. Claudin1

Comment Here

Reference: Perineurioma

Peutz-Jeghers syndrome
Definition / general
  • Autosomal dominant polyposis syndrome with near complete penetrance characterized by hamartomatous gastrointestinal polyps and mucocutaneous melanin pigmentation
Essential features
  • WHO diagnostic criteria: (a) three or more histologically confirmed Peutz-Jeghers polyps; or (b) any number of Peutz-Jeghers polyps with a family history of Peutz-Jeghers syndrome; or (c) characteristic prominent mucocutaneous pigmentation with a family history of Peutz-Jeghers syndrome; or (d) any number of PJ polyps and characteristic prominent mucocutaneous pigmentation
  • Increased risk of GI and non-GI malignancies, with largest cohort putting the incidence at 23% (Clin Cancer Res 2006;12:3209)
  • Associated GI malignancies include colorectal, small bowel and pancreatic adenocarcinomas
  • Non-GI tumors include:
Sites
  • Polyps found throughout the GI tract but most commonly encountered in the small bowel
  • Polyps also described in gallbladder, bladder and nasopharynx (Int J Colorectal Dis 2000;15:118)
Pathophysiology
Clinical features
  • Mucocutaneous pigmented lesions are common and may be the first clue to an individual having Peutz-Jeghers syndrome; however, this pigmentation may fade over time
  • Presenting symptoms commonly include GI bleeding, abdominal pain or intussusception
Case reports
  • 43 year old man with Peutz-Jeghers syndrome, intussusception and malignant transformation of polyps (Oncol Lett 2015;10:1008)
Treatment
  • Following baseline colonoscopic examination and evaluation of the small bowel, patients should have repeat studies every 1 - 3 years, depending on previous findings (Gut 2010;59:975)
  • More frequent evaluation of the GI tract suggested after age 50, as most malignancies occur at that time
  • Surveillance thought to detect sizeable GI polyps which may cause intussusception / obstruction or bleeding and also detect early stage carcinomas
Clinical images

Images hosted on other servers:

Multiple gastric polyps on endoscopy

Characteristic mucocutaneous pigmentation

Microscopic (histologic) description
  • Hamartomatous mucosal polyps with characteristic central core of branching smooth muscle associated with a mucosa native to site of origin
  • Smaller polyps or those from the stomach and colon, may lack the prominent arborizing smooth muscle (Mod Pathol 2013;26:1235)
  • Epithelial misplacement is not uncommon and is likely secondary to prolapse changes
  • Dysplasia and adenocarcinoma can develop within polyps
Microscopic (histologic) images

Contributed by Michael Feely, D.O. and Christopher Hartley, M.D.

Colonic Peutz-Jeghers polyps

Duodenal Peutz-Jeghers polyps

Peutz-Jeghers polyp

Differential diagnosis
Board review style question #1
Which of the following is true about Peutz-Jeghers syndrome?

  1. 1 Peutz-Jeghers polyp alone meets criteria to diagnose a patient with the syndrome
  2. Patients have essentially a 100% risk of developing malignancy
  3. Patients may develop mucocutaneous pigmentation
  4. Peutz-Jeghers polyps are most common in the colon
Board review style answer #1
C. Patients may develop mucocutaneous pigmentation

Comment Here

Reference: Peutz-Jeghers syndrome

Pill fragment associated colitis
Definition / general
  • Sodium polystyrene sulfonate (Kayexalate) is an ion exchange resin that binds intraluminal potassium; it can cause ischemia and intestinal necrosis, especially (but not only) if given with sorbitol (Am J Surg Pathol 1997;21:60)
  • Sevelamer, used to treat hyperphosphatemia in patients with chronic kidney disease, binds phosphate and has may cause injury to the gastrointestinal tract (Am J Surg Pathol 2013;37:1686)
  • Bile acid sequestrants (e.g. cholestyramine) may microscopically mimic other pill fragments but do not appear to cause injury (Am J Surg Pathol 2014;38:1530)
Essential features
  • Kayexalate causes mucosal injury and sevelamer may also as well
  • Other pill fragments (such as bile acid sequestrants) can also be identified microscopically but do not cause mucosal injury
Sites
  • Colon is most commonly involved gastrointestinal organ but any can be affected
  • Resins can rarely be encountered outside the gastrointestinal tract (due to perforation, aspiration, etc.) (Diagn Pathol 2008;3:27)
Diagnosis
  • Microscopic examination and clinical confirmation of medication use
Case reports
Microscopic (histologic) description
  • Kayexalate: bright purple crystals with fish scale appearance
  • Sevelamer: yellow / pink crystals; also fish scale appearance
  • Bile acid sequestrants: opaque orange polygonal / rhomboid crystals; usually no fish scale appearance; may be spherical (Histopathology 2015;67:141)
Microscopic (histologic) images

Contributed by Raul S. Gonzalez, M.D.

Kayexalate and sevelamer in a rectal ulcer



Images hosted on other servers:

Incidental luminal polystyrene
sulphonate resin particles in
jejunal diverticular tissue

Particles at site of colonic
necrosis (direct Schiff stain
with light counterstain)

Particles at site of aspiration
pneumonia (Ziehl-Neelsen stain
with light counterstain)

Positive stains
  • AFB: Kayexalate appears black, sevelamer appears magenta and bile acid sequestrants appear dull yellow
Sample pathology report
  • Ascending colon, ulcer, biopsy:
    • Fibrinopurulent debris, consistent with ulceration (see comment)
    • Negative for malignancy.
    • Comment: Several fragments of pill material, consistent with sevelamer, are present within the ulcer debris. The pill material may be responsible for the ulceration or secondarily trapped within.
Differential diagnosis
  • Dystrophic calcification:
    • Can mimic Kayexalate but is a brighter purple
  • Bile:
    • Can mimic sevelamer but has no pink coloration
Additional references
Board review style question #1
Which of the following pill material fragments usually shows fish scales on histology?

  1. Colestipol
  2. Crospovidone
  3. Microcrystalline cellulose
  4. Sevelamer
Board review style answer #1

Pneumatosis cystoides intestinalis
Definition / general
  • Submucosal gas filled cysts within gastrointestinal tract
Essential features
  • Gas filled spaces throughout the colon, often lined with giant cells
  • May occur in children and adults and be benign or fulminant
  • May require surgery
Sites
  • Entire gastrointestinal tract can be affected
Etiology
  • In infants, fulminant form is associated with necrotizing enterocolitis and may be fatal
  • Also associated with cystic fibrosis or congenital heart defects
  • In adults, either idiopathic or associated with obstruction, chronic lung disease, scleroderma, chemotherapy, drugs, ischemic colitis, vascular insufficiency or infection with gas producing bacteria, such as Clostridia (Am J Med Sci 2006;332:100)
Clinical features
  • Either benign or fulminant form
  • Often indolent clinical course, although radiographically can resemble carcinoma
  • Patients may have nausea, vomiting, diarrhea
  • Considered a finding, not a diagnosis
Radiology description
  • Multiple, thin walled, air filled submucosal or subserosal cysts
  • Air may be present in peritoneum
Case reports
Treatment
  • Surgery if bowel necrosis, perforation or peritonitis
Gross description
  • Polypoid grape-like masses protruding through mucosa
Gross images

Images hosted on other servers:

Multiple subserosal bubbles

Diffuse hemorrhage

Pneumatosis intestinalis

Pneumatosis intestinalis and focal mucosal necrosis

Microscopic (histologic) description
  • Empty submucosal cysts lined by multinucleated giant cells
  • Mucosa may demonstrate cryptitis, crypt abscesses, granulomas
  • Can also resemble lipomatosis
Microscopic (histologic) images

Contributed by Raul S. Gonzalez, M.D.

Mural pneumatosis

Mucosal pneumatosis

Videos

Pneumatosis intestinalis

Sample pathology report
  • Descending colon, resection:
    • Segment of colon with numerous mural cystic spaces lined by foreign body giant cells, consistent with pneumatosis cystoides intestinalis
    • Margins of resection unremarkable.
    • Four benign lymph nodes.
Differential diagnosis
  • Crohn's disease:
    • Granulomas and inflammation may somewhat mimic Crohn’s but there would not be air filled spaces
  • Other granuloma forming diseases:
    • Limited biopsy material may raise this differential diagnosis but most other granuloma forming diseases would show true granulomas rather than numerous multinucleated giant cells (Am J Surg Pathol 2007;31:1469)
Board review style question #1

Which of the following is true about pneumatosis cystoides intestinalis?

  1. It can occur in children
  2. It can progress to malignancy
  3. The cystic spaces contain both air and fat
  4. Treatment is supportive rather than surgical
Board review style answer #1
A. It can occur in children

Comment Here

Reference: Pneumatosis cystoides intestinalis

POLD1 / POLE
Definition / general
  • Germline missense pathogenic variants in the exonuclease domain (ED) of polymerases epsilon (POLE) and delta (POLD1) affect their proofreading capabilities, predisposing to multiple colorectal adenomas and carcinomas
  • Polymerase proof reading associated polyposis (PPAP) has autosomal dominant inheritance
  • POLE / POLD1 germline and rare somatic mutations are also associated with endometrial, brain, breast, ovarian, stomach, pancreas and skin tumors, among others
Essential features
  • Deleterious mutations in the proofreading exonuclease domain of POLE occur in approximately 7 - 12% of endometrial carcinomas, 1 - 2% of colorectal carcinomas and a small number of other cancer types
  • Identifying pathogenic mutations in the POLE gene is clinically important; such mutations are associated with a more favorable prognosis and can impact oncologic management
Terminology
  • DNA polymerase epsilon (POLE) and delta 1 (POLD1)
  • Polymerase proof reading associated polyposis (PPAP)
  • Colorectal cancer (CRC)
Epidemiology
  • Early onset CRCs with POLE mutations occur in (Oncotarget 2016;7:68638)
    • Age group < 50 years
    • M > F
    • Right side > left side
Sites
  • POLE is associated with the following malignancies:
    • Colon
    • Pancreas
    • Small intestine
    • Cutaneous melanoma
    • Endometrial
    • Ovarian
    • Brain
    • Lung
  • POLD1 is associated with the following malignancies:
    • Colon
    • Endometrial
    • Breast cancer
Pathophysiology
  • POLE and POLD1 genes encode the major catalytic and proofreading subunits of the Polε (polymerase epsilon) and Polδ (polymerase delta) enzyme complexes, respectively
  • Germline or somatic mutations in POLE / POLD1 proofreading domain cause deficiencies in DNA repair (Ann Transl Med 2021;9:129)
  • DNA repair deficiencies lead to the formation of a DNA hypermutated molecular phenotype (J Clin Invest 2018;128:4179)
  • Predisposition to multiple polyps (both adenomatous and serrated), which can lead to carcinogenesis in the future
Clinical features
  • POLE / POLD1 mutated cancers are primarily microsatellite stable (MSS) (Nat Rev Cancer 2016;16:71)
  • POLE mutated endometrial and CRCs also display MSI-H
  • POLE gene is located in 12q24.33 and encodes the proofreading (exonuclease) subunit of polymerase epsilon (POLE) with 2286 amino acids
  • Most common POLE mutations are identified in P286R/H, V411L and S459F tumor genome (Nat Rev Cancer 2016;16:71)
  • Somatic POLD1 mutations are also found (Nat Rev Cancer 2016;16:71)
  • Hereditary CRCs associated with mutations POLD1 and POLE are termed polymerase proofreading associated polyposis (PPAP) (Nat Genet 2013;45:136)
  • Combined POLE / POLD1 defects and mismatch repair explain unresolved suspected Lynch syndrome cancers (Eur J Hum Genet 2016;24:1089)
  • POLE mutated CRCs have hypermutated phenotypes despite MMR intact status; mutation burdens are higher than that in microsatellite unstable CRCs (Genet Med 2016;18:325)
  • POLE mutations can co-occur with somatic MLH1 mutation in endometrial carcinomas and ovarian endometrioid or clear cell carcinomas; this is due to the hypermutated state and genomic instability of the tumor
Diagnosis
  • Molecular testing for POLE specific site analysis (PCR, next gene sequencing)
Laboratory
  • When to consider testing: the following initial guidelines for POLE and POLD1 genetic testing have been suggested (Genet Med 2016;18:325)
    • POLE
      • 20 - 100 adenomas
      • Family history that meets the Amsterdam I criteria (CRC only)
      • CRC and 5 - 20 adenomas, both diagnosed before age 50
      • CRC or 5 - 20 adenomas and a first degree relative with CRC before age 50
      • CRC or 5 - 20 adenomas and 2 or more first or second degree relatives with CRC, regardless of age
    • POLD1
      • 20 - 100 adenomas
      • Family history that meets the Amsterdam II criteria (only CRC and endometrial cancer [EC])
      • CRC before age 50 or EC before age 60 and 5 - 20 adenomas diagnosed before age 50
      • CRC or EC or 5 - 20 adenomas and a first degree relative with CRC before age 50 or EC before age 60
      • CRC or EC or 5 - 20 adenomas and 2 or more first or second degree relatives with CRC or EC, regardless of age
Prognostic factors
  • POLE / POLD1 mutations have beneficial clinical outcomes post immune checkpoint inhibitor (ICI) therapy in endometrial cancer, non small cell lung cancer, CRC and cervical carcinosarcoma (Nat Rev Cancer 2016;16:71)
  • POLE mutations are associated with better prognosis in endometrial cancer patients who underwent adjuvant treatment of chemotherapy or radiotherapy following surgery (J Natl Cancer Inst 2015;107:402)
  • POLE mutation is a good prognostic biomarker for patients with lung squamous cell carcinoma but not lung adenocarcinoma (Mol Cancer 2018;17:81)
Case reports
  • 29 year old Chinese man with advanced colon cancer harboring somatic POLE F367S mutation with microsatellite stability status (Onco Targets Ther 2021;14:1791)
  • 41 year old Chinese woman with a right sided colon adenocarcinoma who harbored a (p.P286R) POLE somatic mutation (BMC Gastroenterol 2020;20:255)
  • 45 and 54 year old men with advanced adenocarcinoma of the ileum and mixed neuroendocrine nonneuroendocrine neoplasm, both MSS and carrying a POLE mutation (Oncol Res Treat 2022;45:222)
Treatment
  • POLE / POLD is a recently described mutation so the guidelines regarding screening and management of POLE and POLD1 mutation carriers are not yet determined
  • POLE mutated tumors may also be eligible for immunotherapy (e.g., checkpoint inhibitors) given the strong correlation between these mutations and high tumor mutation burden (J Clin Invest 2016;126:2334)
  • Several POLE mutant cancers display increased tumor infiltrating lymphocyte density, a phenotype which shows better tumor immune response to immunotherapy
Microscopic (histologic) description
  • In a study, POLE mutated colorectal cancers frequently formed cribriform structures and intraluminal necrotic debris, which contained neutrophils and apoptotic bodies (Oncotarget 2016;7:68638)
  • Histologically, colonic adenocarcinoma and adenomas appear similar to sporadic cases
Negative stains
Videos

Understanding the cancer predisposing syndrome caused by defective POLE and POLD1

Board review style question #1
Which of the following is true about the POLE gene?

  1. POLE does not have an essential role in chromosomal DNA replication
  2. POLE gene is located in 12q24.33 and encodes the proofreading (exonuclease) subunit of polymerase epsilon (POLE) with 2286 amino acids
  3. POLE mutations harbor an unfavorable prognosis in endometrial cancer
  4. Mutations in the POLE gene do not have an increased risk for colon polyps or colon cancer
Board review style answer #1
B. POLE gene is located in 12q24.33 and encodes the proofreading (exonuclease) subunit of polymerase epsilon (POLE) with 2286 amino acids

Comment Here

Reference: POLD1 / POLE
Board review style question #2
Which of the following is associated with POLE / POLD1 mutation?

  1. Colon cancer
  2. Hepatocellular carcinoma
  3. Kaposi Sarcoma
  4. Salivary gland cancer
Board review style answer #2
A. Colon cancer

Comment Here

Reference: POLD1 / POLE

Polyp overview
Definition / general
  • A polyp is an exophytic or sessile lesion distinct from the mucosal surface and may be neoplastic or nonneoplastic
Essential features
  • > 95% of colorectal adenocarcinoma arises from polyps
  • Usually asymptomatic
ICD coding
  • Benign adenomatous:
    • D12.0 - benign neoplasm of cecum
    • D12.2 - benign neoplasm of ascending colon
    • D12.3 - benign neoplasm of transverse colon
    • D12.4 - benign neoplasm of descending colon
    • D12.5 - benign neoplasm of sigmoid colon
    • D12.6 - benign neoplasm of colon, unspecified; or polyposis of colon
    • D12.7 - benign neoplasm of rectosigmoid junction
    • D12.8 - benign neoplasm of rectum
  • Inflammatory:
    • K51.40 - inflammatory polyps of colon without complications
    • K51.411 - inflammatory polyps of colon with rectal bleeding
    • K51.412 - inflammatory polyps of colon with intestinal obstruction
    • K51.413 - inflammatory polyps of colon with fistula
    • K51.414 - inflammatory polyps of colon with abscess
    • K51.418 - inflammatory polyps of colon with other complication
  • Other:
    • K63.5 - polyp of colon (polyp not documented as adenomatous, benign or inflammatory; or hyperplastic polyp)
    • K62.1 - rectal polyp
Epidemiology
  • Varies by type of polyp
  • For more information on epidemiology of colon polyps, please refer to the individual chapters pertaining to each type of polyp in Pathology Outlines
Pathophysiology
  • Etiology of colonic polyps varies from neoplastic to nonneoplastic and can either arise sporadically or in syndromic context
Etiology
Clinical features
Diagnosis
Laboratory
Radiology description
Prognostic factors
Treatment
Gross description
  • Paris classification based on explicit recognition (Endoscopy 2005;37:570)
    • Polypoid type:
      • Protrudes at least 2.5 mm above mucosal layer
      • Sessile: no stalk (o-Is)
      • Pedunculated: with stalk, may be due to traction on the mass (o-Ip)
      • Semipedunculated (o-Isp)
    • Nonpolypoid type:
      • Less than 2.5 mm above mucosal layer
      • Slightly elevated (o-IIa)
      • Flat (o-IIb)
      • Slightly depressed lesions (o-IIc)
    • Excavated / ulcerated: o-III (Ann Gastroenterol 2017;30:592)
Microscopic (histologic) description
  • Generally depends on type of polyp (see specific topics for details)
  • Histology of submucosal lifting agents used during polypectomy
    • Some examples include ORISE and Eleview
    • Histologic appearance of lifting agents is temporally dependent, ranging from basophilic bubbly amorphous material (early) to eosinophilic hyalinized globules or ribbons (late)
    • Can mimic amyloid but differentiated based on presence of foreign body giant cell reaction to lifting agent that is not polarizable and Congo red negativity (Am J Surg Pathol 2020;44:793, Am J Clin Pathol 2020;153:630)

Portal hypertensive colopathy (pending)
[Pending]

Posttreatment changes
Essential features
  • Rectal tumors often receive neoadjuvant treatment before resection
  • Microscopic findings include fibrosis and acellular mucin
  • Acellular mucin does not equal viable residual tumor and serial sections to hunt for rare cancer cells are not warranted
Sites
  • Rectum only; neoadjuvant treatment is generally not given for colonic carcinoma
Clinical features
  • Treatment often shrinks tumors, reducing risk of local recurrence
  • Patients may experience treatment related complications or anorectal dysfunction (Dis Colon Rectum 2011;54:901)
Gross description
  • Flat, firm mass with central ulceration
  • May be no identifiable lesion
  • Unless residual tumor is grossly obvious, entire lesion site should be submitted for examination
  • Completeness of mesorectal excision should be assessed (J Clin Pathol 2007;60:849)
Microscopic (histologic) description
  • Microscopic changes include fibrosis, acellular mucin, necrotic tumor, ulceration and hemosiderin
  • Viable residual tumor may appear well, moderate or poorly differentiated
  • Lymph node metastases may remain viable, even if entire primary tumor is destroyed; viable tumor deposits also may be seen and portend poor outcome (Mod Pathol 2014;27:1281)
  • Various grading systems exist for tumor response, with poor concordance (Hum Pathol 2012;43:1917)
  • Acellular mucin should not be interpreted as residual tumor (Am J Surg Pathol 2011;35:127)
  • Deeper sections to search for rare residual tumor cells has little bearing on patient outcome (Histopathology 2011;59:650)
Microscopic (histologic) images

Contributed by Raul S. Gonzalez, M.D.

Posttreatment changes of rectal carcinoma

Positive stains
  • Cytokeratins may be used to detect rare residual tumor cells
Sample pathology report
  • Rectum, resection:
    • Segment of rectum with prominent reactive change and fibrosis, consistent with therapy effect (see comment)
    • Negative for residual malignancy.
    • Margins of resection unremarkable.
    • Comment: The entire grossly abnormal area was submitted for microscopic examination.
Board review style question #1
Neoadjuvant therapy is usually given for large intestine carcinomas in what location?

  1. Appendix
  2. Ascending colon
  3. Transverse colon
  4. Rectum
Board review style answer #1
D. Rectum

Comment Here

Reference: Posttreatment changes

Pseudomembranous colitis
Definition / general
  • Nonspecific pattern of injury occurring as a result of decreased oxygenation, disrupted blood flow and endothelial injury (Cleve Clin J Med 2016;83:361)
  • Often occurs following use of broad spectrum antibiotics
Essential features
  • Formation of a pseudomembrane resulting from multiple disease states and overlying colonic mucosa
  • Most commonly caused by Clostridioides difficile (formerly Clostridium difficile)
  • Often treated with antibiotics but may rarely require surgery
Terminology
  • Also termed antibiotic associated colitis
  • Pseudomembrane is a layer of fibropurulent exudate made of mucus and acute inflammatory cells originating from inflamed and erupting crypts
ICD coding
  • ICD-10: A04.7 - enterocolitis due to Clostridium difficile
Epidemiology
Sites
  • Colon
Pathophysiology
  • For cases caused by C. difficile
    • Usage of other antibiotics disrupts normal gut flora, allowing C. difficile to establish a presence
    • After colonization, the bacteria produce 2 large exoproteins, toxins A and B
    • Following release in the colon, the toxins bind to cell surface receptors and are internalized into the cell
    • Glycosylation of small proteins occurs inside the cells involved in signaling and regulating pathways
      • Leads to cytoskeleton disruption, causing morphologic cell changes, cytoskeleton activation and cell death
      • Infiltration of neutrophils and inflammatory cell reaction occurs due to tight junctions getting affected
    • Pseudomembranous formation occurs by toxin activation of the native immune system and by activation of macrophages and monocytes, leading to release of proinflammatory cytokines like interleukin 1, interleukin 8, tumor necrosis factor and leukotriene B4 (Dis Mon 2015;61:181)
Etiology

Drugs Medications Infections Inflammatory conditions
Cocaine
Glutaraldehyde
Cisplatin
Cyclosporine A
Docetaxel
5-Fluorouracil
Nonsteroidal anti-inflammatory drugs
Thomasclavelia ramosum
Cytomegalovirus
Entamoeba histolytica
Escherichia coli O157:H7
Klebsiella oxytoca
Plesiomonas shigelloides
Salmonella species
Schistosoma mansoni
Shigella species
Staphylococcus aureus
Strongyloides stercoralis
Behçet disease
Collagenous colitis
Inflammatory bowel disease
Ischemic colitis
Clinical features
  • Most common symptom is diarrhea
  • Acute symptoms like abdominal pain and cramping
  • Fever and leukocytosis indicate severe disease
  • Rarely, perforation, toxic megacolon, colonic ischemia and septic shock (hypotension) can occur (Crit Care Clin 2013;29:827)
Diagnosis
  • Clinical clues such as history of hospitalization and utilization of antibiotics are important
  • Lab investigations are used to confirm C. difficile associated disease (see Laboratory)
  • In more fulminant disease cases, endoscopy and radiology are useful
  • Endoscopic visualization of the colonic mucosa shows pseudomembrane formation
  • Computerized tomography (CT) scan of the abdomen / pelvis is the most common imaging modality used for pseudomembranous colitis (Crit Care Clin 2013;29:827)
Laboratory
  • Stool enzyme linked immunosorbent assay (ELISA) shows toxins A or B in the stool
  • PCR identifies the presence of DNA encoding the C. difficile toxins
Radiology description
  • CT scan of abdomen / pelvis shows pericolonic fat stranding, accordion sign (high attenuation oral contrast in the colonic lumen alternating with low attenuating inflamed mucosa) and target sign (varying degrees of attenuation of the IV contrast material in the mucosa) (Crit Care Clin 2013;29:827)
  • In some cases, pneumatosis or perforation can also be seen (Crit Care Clin 2013;29:827)
Radiology images

Contributed by Subramanya Sakaleshpura Mallikarjunappa, M.B.B.S., M.D.
Pseudomembrane colitis

Linear radiolucencies in left lumbar region

Pseudomembrane colitis

Extensive submucosal edema

Prognostic factors
  • Risk factors for recurrent disease include older age, female sex, additional courses of antibiotics / chemotherapy, use of gastrointestinal (GI) medications or procedures, prolonged hospital stays and prior episodes of recurrent pseudomembranous colitis (Dis Mon 2015;61:181)
Case reports
Treatment
  • Supportive care and antimicrobial therapy in most cases
    • Supportive care includes fluid resuscitation
    • Metronidazole and vancomycin are the main antibiotics used
  • Small number of fulminant cases need surgical management (Crit Care Clin 2013;29:827)
Gross description
  • White to yellow plaques and nodules on the colonic mucosa, which can coalesce
Gross images

Images hosted on other servers:
White to yellow plaques and nodules on the colonic mucosa.

White to yellow plaques and nodules on the colonic mucosa

Hyperemic mucosa and green-yellow exudate


Mucosal denudation and exudate

Microscopic (histologic) description
  • Mix of inflammatory and ischemic features
  • Volcano or mushroom-like eruption of fibrin, mucin and inflammatory cells, mainly neutrophils, forming a pseudomembrane overlying the injured mucosa
  • Epithelium shows ischemic appearance with attenuated / withered crypts
  • Superficial lamina propria contains dense neutrophils and some capillary fibrin thrombi
  • Adjacent epithelium may show active colitis or hemorrhage (Pathologe 2022;43:16)
Microscopic (histologic) images

Contributed by Raul S. Gonzalez, M.D. and Subramanya Sakaleshpura Mallikarjunappa, M.B.B.S., M.D.
Volcano shaped eruption

Volcano shaped eruption

Pseudomembrane

Pseudomembrane

Withered crypts

Withered crypts

Pseudo-signet ring change

Pseudo-signet ring change

Volcano shaped eruption

Volcano shaped eruption

Pseudomembrane

Pseudomembrane

Sample pathology report
  • Colon, biopsy:
    • Colonic mucosa with sloughed epithelium, neutrophils, mucin, fibrin deposition on the surface and crypts (see comment)
    • Comment: Given the reported history of recent antibiotic use and the appearance of a pseudomembrane on colonoscopy, the findings are most consistent with pseudomembranous / antibiotic associated colitis.
Differential diagnosis
  • Ischemic colitis:
    • Most often involves the watershed areas like splenic flexure and rectosigmoid
    • Lamina propria hyalinization and atrophic microcrypts are more common in ischemic colitis (Am J Surg Pathol 1997;21:706)
  • Infectious colitis by pathogens other than C. difficile:
    • Cannot be determined by histology alone
    • Identification of the responsible pathogens by lab investigations like stool assay, PCR or culture
  • Signet ring cell carcinoma:
    • Should show destructive invasion into the colonic mucosa or wall, not just signet ring shaped epithelial cells lining the native crypt spaces (Am J Surg Pathol 2004;28:1111)
Board review style question #1
Which of the following is a histologic feature of pseudomembranous colitis?

  1. Apoptosis in crypts
  2. Crypt branching
  3. Fibrin, mucin and inflammatory cells on the surface with ischemic epithelial changes
  4. Hyalinization of lamina propria
  5. Signet ring cells in the lamina propria
Board review style answer #1
C. Fibrin, mucin and inflammatory cells on the surface with ischemic epithelial changes. Fibrin, mucin and neutrophils in the epithelium forming a mushroom shape are classic features of pseudomembranous colitis. Answer A is incorrect because apoptosis of crypts is seen in medication induced colitis and graft versus host disease. Answer B is incorrect because crypt branching is a feature of chronic colitides such as inflammatory bowel disease. Answer D is incorrect because hyalinization of lamina propria is seen in ischemic colitis. Answer E is incorrect because invasive signet ring cells are found in adenocarcinoma.

Comment Here

Reference: Pseudomembranous colitis
Board review style question #2

Which of the following clinical attributes is associated with Clostridioides difficile colitis?

  1. Autoimmune conditions
  2. Copies of HLA DQ2
  3. TP53 germline mutation
  4. Use of broad spectrum antibiotics
  5. Young age
Board review style answer #2
D. Use of broad spectrum antibiotics. Broad spectrum antibiotic use leads to colonization of C. difficile, causing pseudomembranous colitis. Answer E is incorrect because it is seen more commonly in older individuals. Answers A, B and C are incorrect because autoimmune conditions, TP53 mutation and HLA DQ2 presence are not associated with pseudomembranous colitis.

Comment Here

Reference: Pseudomembranous colitis

Pulse granuloma
Definition / general
  • Granulomatous reaction to legume material that breaches the colonic mucosa
Essential features
  • Reactive process arising from foreign body (food material)
  • Usually clinically silent and incidental but may form a mass lesion
  • Various microscopic appearances but hyaline rings of pulse material must be present
Terminology
  • Pulse is the seed material of legumes (beans, peas, peanuts, etc.)
  • It has also been described as hyaline rings
Epidemiology
Sites
  • Can occur anywhere in gastrointestinal tract but may occur most commonly in colon
  • More commonly recognized in oral cavity, mandible / maxilla and lung
  • If colon perforates, pulse can access the abdominal cavity and deposit anywhere within (e.g. ovary)
Pathophysiology
  • Granulomatous reaction to foreign material
Etiology
  • Most often seen in colon damaged by perforation, diverticulitis, inflammatory bowel disease or malignancy
Clinical features
  • Generally an incidental finding but can may manifest as a mass lesion clinically
  • Associated with use of tobacco and NSAIDs (Am J Surg Pathol 2015;39:84)
Case reports
Microscopic (histologic) description
  • Hyaline predominant
    • More than half of lesion consists of pink hyaline rings / ribbons; other food present; inflammation and fibrosis minimal
  • Cellular predominant
    • Acute and chronic inflammation and fibrosis
    • Relatively less pulse and food material
  • Sclerosing mesenteritis-like
    • Rarest variant
    • Arises in mesentery and resembles sclerosing mesenteritis (inflammation, fibrosis) but pulse focally visible
    • Average size is 1.0 cm
Microscopic (histologic) images

Contributed by Raul S. Gonzalez, M.D.

Serosal hyaline
predominant
pulse granulomas

Pulse material
with numerous
foreign body
giant cells

Cellular predominant pulse granuloma

Scant pulse material embedded in colon wall



Contributed by @RaulSGonzalezMD on Twitter
Pulse granuloma

Pulse granuloma

Pulse granuloma

Pulse granuloma

Pulse granuloma

Pulse granuloma

Negative stains
Sample pathology report
  • Sigmoid colon, resection:
    • Diverticulosis with rupture, mural abscess formation and focal pulse granulomas.
Differential diagnosis
Board review style question #1

Which of the following is true about pulse granuloma in the colon?

  1. It always forms a mass lesion
  2. It has a few described histologic subtypes / forms
  3. It shows apple green birefringence on Congo red staining
  4. Pulse refers to any sort of vegetable material
Board review style answer #1
B. It has a few described histologic subtypes / forms

Comment Here

Reference: Pulse granuloma

Pyogenic granuloma
Definition / general
  • Benign capillary hemangioma forming a colonic polyp
Essential features
  • Very rare
  • Can cause rectal bleeding
Terminology
  • Also called lobular capillary hemangioma
Sites
  • Usually occurs in skin or oral cavity; gastrointestinal tract cases are very rare
Clinical features
  • Presents clinically as rectal bleeding and anemia
Case reports
Clinical images

Images hosted on other servers:

Jejunal pyogenic granuloma

Upper duodenum findings

Endoscopic duodenal mucosal resection

Gross description
  • Pedunculated or sessile polyp
  • May appear black and necrotic
Microscopic (histologic) description
  • Lobular arrangement of capillaries within edematous, inflamed stroma
  • Endothelial cells often swollen
  • Often shows surface ulceration; sometimes shows mitotic figures
Microscopic (histologic) images

Images hosted on other servers:

High power H&E

IHC for CD31 and CD34

Duodenum-edematous granulation tissue

Jejunum postsurgical pyogenic granuloma

Jejunum-CD34

Positive stains
Sample pathology report
  • Transverse colon, polypectomy:
    • Small focus of benign lobular vascular proliferation, most consistent with pyogenic granuloma
Differential diagnosis
Board review style question #1
A 21 year old man presents to his physician, complaining of hematochezia. Colonoscopy shows a small, pedunculated polyp. Microscopically, the polyp is mostly composed of inflammation and small blood vessels. Which of the following findings would favor a diagnosis of pyogenic granuloma?

  1. Dilated, ectatic crypts
  2. Lobular architecture
  3. Positivity for HHV8
  4. Surface adenomatous change
  5. Viral cytopathic effect
Board review style answer #1
B. Lobular architecture

Comment here

Reference: Pyogenic granuloma

Radiation enterocolitis
Definition / general
  • Radiation injury
Essential features
  • Injury process that occurs as a result of radiation damage and can be classified as acute (< 6 months) or chronic (> 6 months)
  • Acute injury findings include prominent eosinophils (cryptitis and crypt abscesses) and apoptosis; chronic injury findings result from damage to vasculature and include pronounced fibrosis and vascular changes (hyalinization and intimal thickening)
  • Should be differentiated from mimickers, such as ischemia, inflammatory bowel disease, collagenous colitis and mucosal prolapse
ICD coding
  • ICD-10: K52.9 - noninfective gastroenteritis and colitis, unspecified
Epidemiology
  • Over 100,000 American patients annually receive therapeutic pelvic radiation for malignancies of prostate, anus, rectum, uterine cervix, endometrium, urinary bladder and rarely other sites
  • Some acute radiation mucosal injury occurs in nearly all patients; most is self limited and medical management is effective in > 80% of symptomatic cases
  • Approximately 5 - 15% of patients develop chronic radiation proctitis
  • References: Gastroenterol Clin North Am 2013;42:913, Medicine (Baltimore) 2018;97:e13328
Pathophysiology
  • Ionizing radiation damages proteins, lipid bilayer and DNA of rapidly proliferating cells
  • Radiation changes are typically classified as acute (< 6 months) or chronic (> 6 months) (Curr Opin Support Palliat Care 2007;1:23)
  • Acute radiation injury (< 6 months): presents within hours to days, often self limited, due to ionizing radiation damage to mucosa
  • Chronic radiation injury (> 6 months): presents within months to years, due to damaged blood vessels:
    • Endothelial damage causes vascular sclerosis with obliterative endarteritis of small vessels, leading to chronic ischemia and fibrosis
    • Lack of perfusion causes fibrosis, ulcers, stenosis, strictures, fistulas, infection and bleeding
Clinical features
  • Acute radiation injury: diarrhea, pain, tenesmus, hematochezia
  • Chronic radiation injury: diarrhea, bleeding, pain secondary to stricture
  • Risk factors for developing radiation proctitis include: higher radiation doses, less fractionation, thinner patients, the elderly, concurrent or subsequent chemotherapy, collagen vascular disease, hypertension and diabetes mellitus
  • Radiation increases risk for secondary malignancies
  • Endoscopic findings:
    • Acute radiation proctitis shows edematous, dusky red rectal mucosa
    • Chronic radiation proctitis shows mucosal atrophy, ectatic superficial capillaries and variable stenosis, strictures and fistulas
  • References: Gastroenterol Clin North Am 2013;42:913, Medicine (Baltimore) 2018;97:e13328
Diagnosis
  • Acute radiation proctitis: clinical disease, rarely biopsied
  • Chronic radiation proctitis: typically biopsied; history is essential for diagnosis
Case reports
Treatment
Clinical images

Images hosted on other servers:
Edematous rectal mucosa

Edematous rectal mucosa

Rectal mucosa

Rectal mucosa

Microscopic (histologic) description
  • Acute radiation colitis (< 6 months after radiation treatment):
    • Epithelium can contain:
      • Reduced mitotic activity
      • Apoptosis
      • Reduced goblet cell mucin
      • Goblet cell swelling
      • Degenerative changes
      • Atypia (nuclear enlargement and malorientation)
    • Inflammation
      • Typically eosinophilic (characteristic)
        • Lamina propria eosinophilic infiltration
        • Eosinophilic cryptitis
        • Eosinophilic crypt abscesses
    • Can be fibroblast proliferation in lamina propria (reactive changes to radiation)
  • Chronic radiation colitis (> 6 months after radiation treatment):
    • Epithelium:
      • Paneth cell metaplasia
      • Ulceration
      • Mild epithelial atypia
    • Crypts:
      • Architectural changes and atrophy
    • Lamina propria:
      • Fibrosis
      • Mild chronic inflammation
      • Atypical fibroblasts / stromal cells with enlarged hyperchromatic nuclei
    • Submucosa and deeper layers:
      • Vasculature
        • Hyalinization and fibrosis of vessel wall
        • Intimal thickening
        • Vascular ectasia or narrowing
        • Thrombosis
        • Endothelial cell atypia
  • Reference: Am J Surg Pathol 2002;26:498
Microscopic (histologic) images

Contributed by Maryam Kherad Pezhouh, M.D., M.Sc.
Acute radiation associated injury Acute radiation associated injury

Acute radiation associated injury

Chronic radiation associated injury Chronic radiation associated injury

Chronic radiation associated injury

Chronic radiation associated injury Chronic radiation associated injury

Chronic radiation associated injury

Sample pathology report
  • Colon, biopsy:
    • Colonic mucosa with lamina propria fibrosis and ectatic, hyalinized vessels consistent with mild chronic radiation associated colitis
Differential diagnosis
  • Ischemia:
    • Present: hyalinized lamina propria, withered regenerative crypts, possible coagulative necrosis, thrombi, little if any inflammation, hemosiderin laden macrophages
    • Absent: eosinophilic inflammation, atypical fibroblasts / stromal cells with enlarged hyperchromatic nuclei and vascular changes (see Microscopic (histologic) description above for vasculature)
  • Inflammatory bowel disease:
    • Present: active chronic inflammation, including cryptitis, crypt abscesses with evidence of chronicity, such as glandular architectural distortion and increased basal lamina propria lymphoplasmacytosis
    • Absent: atypical fibroblasts / stromal cells with enlarged hyperchromatic nuclei and vascular changes (see Microscopic (histologic) description above for vasculature)
  • Collagenous colitis:
    • Present: thick subepithelial table collagenous band
    • Absent: atypical fibroblasts / stromal cells with enlarged hyperchromatic nuclei and vascular changes (see Microscopic (histologic) description above for vasculature) and submucosal fibrosis
  • Mucosal prolapse:
    • Present: telangiectatic vessels and fibromuscular hyperplasia, inflammation (typically noneosinophilic), erosion and ulceration
    • Absent: eosinophilic inflammation, atypical fibroblasts / stromal cells with enlarged hyperchromatic nuclei and vascular changes (see Microscopic (histologic) description above for vasculature) and submucosal fibrosis
  • Amyloidosis:
    • Present: vessels with eosinophilic amorphous material that is Congo red positive
    • Absent: eosinophilic inflammation, atypical fibroblasts / stromal cells with enlarged hyperchromatic nuclei and vascular changes (see Microscopic (histologic) description above for vasculature)
Board review style question #1
A 76 year old woman with a history of cervical cancer status post stereotactic radiation presents 7 months later with spotty rectal bleeding. Colonoscopy reveals erythema, which is biopsied. Histology demonstrates lamina propria fibrosis and mild chronic inflammation; the epithelium shows focal mild crypt architectural distortion and ectatic vessels. No significant activity was found. What is the most appropriate diagnosis?

  1. Inflammatory bowel disease
  2. Ischemia
  3. Metastatic cervical cancer, direct extension
  4. Radiation colitis
Board review style answer #1
D. Radiation colitis. Chronic radiation colitis usually presents later than 6 months and has the following histological changes that are secondary to vascular damage: epithelium with Paneth cell metaplasia, ulceration, mild atypia, crypt architectural distortion, lamina propria fibrosis, mild chronic inflammation and atypical fibroblasts with enlarged nuclei. Vascular changes, such as hyalinization and fibrosis of vessel wall, intimal thickening, vascular ectasia and endothelial cell atypia, can be seen in the submucosa. Inflammatory bowel disease (answer A) presents with active chronic colitis pattern of injury. The absence of activity is not a typical feature of emerging inflammatory bowel disease. Although radiation can also induce ischemia (answer B), features of radiation should be absent (example: atypical fibroblasts and hyalinization of lamina propria). Direct extension of cervical cancer (answer C) would show squamous cell carcinoma invading submucosal. The endoscopic findings would also likely show luminal narrowing secondary to mass effect, rather than a small ulcer.

Comment Here

Reference: Radiation enterocolitis
Board review style question #2

A 60 year old man with a prior history of prostate adenocarcinoma status postradiation 5 years ago presents with severe sigmoid colon stricture. The patient underwent a partial colonic resection and a section of the colonic wall is shown. What is the most likely diagnosis?

  1. Collagenous colitis
  2. Infection
  3. Mucosal prolapse
  4. Radiation colitis, acute
  5. Radiation colitis, chronic
Board review style answer #2
E. Radiation colitis, chronic. Chronic radiation colitis is expected at > 6 months postradiation therapy. Sections show ulcerations, mild epithelial atypia, architectural changes and atrophy. The lamina propria contains fibrosis, mild chronic inflammation and fibroblast atypia (enlarged hyperchromatic nuclei). The vasculature within submucosal and deeper layers contains hyalinization of vessel wall and intimal thickening. Vascular ectasia, narrowing and thrombosis can also be present. Collagenous colitis (answer A) typically presents with history of watery diarrhea and a > 10 μM subepithelial collagen table (collagenous band). Infection (bacterial) (answer B) will present with an acute clinical history (example: diarrhea and vomiting) after an inciting event (example: food consumption, travel). Acute colitis and granulomas can sometimes be seen. Viral infections (example: cytomegalovirus, Epstein-Barr virus) can occur in immunocompromised patients (example: HIV or transplant) and will contain atypical cells with nuclear changes. Mucosal prolapse (answer C) can present endoscopically as a polyp and contains ingrowth of thick muscle bundles that can mechanically squeeze ensnared epithelium. This is accompanied by variable inflammation, ulceration and reaction epithelial change. Acute radiation colitis (answer D) is expected at < 6 months postradiation therapy. The inflammation is typically eosinophilic (characteristic) with eosinophilic cryptitis, crypt abscesses and lamina propria eosinophilia. Sections demonstrate reduced mitotic activity, apoptosis, reduced goblet cell mucin, goblet cell swelling, degenerative changes and atypia (nuclear enlargement and malorientation).

Comment Here

Reference: Radiation enterocolitis

Reactive nodular fibrous pseudotumor
Definition / general
Essential features
  • Benign fibroinflammatory mass more common in men
  • Stains for AE1 / AE3, CD117 and muscle specific actin
  • Does not appear to recur
Sites
  • Usually arises in mesentery; may involve colon or small bowel
Etiology
Clinical features
  • Usually occurs in patients aged 40 - 60; more common in men
  • Associated with prior abdominal surgery
  • Can cause acute abdominal pain but sometimes discovered incidentally
Case reports
Treatment
  • Complete resection appears curative
Clinical images

Images hosted on other servers:

Protruding lesion on back wall of gastric cardia

Gross description
  • Solitary or multiple tumors, usually involving outer wall of small intestine or colon
  • Firm, tan white, well circumscribed
  • Usually around 6 cm but may measure up to 20 cm
Gross images

Images hosted on other servers:

Cut surface of lesion

Missing Image

Excised specimen in cut section

Missing Image

Fibrotic nodules in pouch of Douglas

Microscopic (histologic) description
  • Mildly to moderately cellular lesion composed of stellate or spindled fibroblasts arranged haphazardly or in intersecting fascicles
  • Stroma rich in collagen (wire-like, keloidal or hyalinized)
  • Sparse intralesional mononuclear cells and peripheral lymphoid aggregates usually present
  • May have infiltrative borders
Microscopic (histologic) images

Contributed by Raul S. Gonzalez, M.D.
Missing Image

Low power

Missing Image

Intermediate power

Missing Image

Interaction with adjacent fat

Missing Image

Entrapped nerves

Negative stains
Molecular / cytogenetics description
  • No substitutions, deletions or insertions in exons 9 or 11 of KIT gene
Sample pathology report
  • Mesentery, resection:
    • Reactive fibrotic nodule, most consistent with reactive nodular fibrous pseudotumor (2.2 cm)
    • Negative for malignancy.
    • Margins of resection unremarkable.
Differential diagnosis
Board review style question #1
Which of the following abdominal lesions stains positive for CD117 by immunohistochemistry?

  1. Desmoid fibromatosis
  2. Inflammatory myofibroblastic tumor
  3. Reactive nodular fibrous pseudotumor
  4. Retroperitoneal fibrosis
  5. Sclerosing mesenteritis
Board review style answer #1
C. Reactive nodular fibrous pseudotumor

Comment here

Reference: Reactive nodular fibrous pseudotumor

Salmonella (typhoid and nontyphoidal)
Definition / general
  • Disease caused by Salmonella species infection
  • While overlap exists, species causing typhoid fever and nontyphoidal species are often categorized separately
  • Salmonellosis is a food borne disease caused mainly by Salmonella typhi serotype, although Salmonella paratyphi A, B, C can cause similar disease
  • Salmonella penetrate the small bowel epithelium after ingestion, enter lymphoid tissue and disseminates via the lymphatic or hematogenous route
  • Typhoid fever is characterized by severe systemic illness with fever, relative bradycardia and considerable constitutional symptoms including abdominal pain (Med J Armed Forces India 2003;59:130)
  • Causes intestinal bleeding due to ulcers in distal ileum or proximal colon (Dig Liver Dis 2004;36:141)
  • Bleeding may be massive (Dis Colon Rectum 1986;29:511)
Essential features
  • Salmonellosis is considered a food borne disease
  • Typhoid fever:
    • Sometimes known as enteric fever, is a life threatening illness caused by Salmonella typhi sometimes called Salmonella enterica serotype Typhi or S. paratyphi
    • Infects from 20 to 30 million people a year, mostly in the developing world
    • In industrialized countries, it is usually encountered in travelers
  • Nontyphoid species:
    • Usually cause acute, self limited gastroenteritis
    • The Centers for Disease Control and Prevention (CDC) estimates that one million cases, 19,000 hospitalizations, and 450 deaths occur each year in the United States
    • Tens of millions of cases occur worldwide each year causing an estimated 100,000 deaths
    • Disease is spread through ingestion of contaminated food or water, most often in a sporadic fashion but sometimes through foodborne outbreaks that often are associated with suboptimal sanitary practices or food preparation
  • Typhoid colitis is caused by Salmonella enterica organisms, mainly S. typhi
  • S. typhi occurs only in humans
  • Salmonellosis is a food borne disease caused by ingestion of contaminated food
  • Salmonella penetrate the small bowel epithelium after ingestion, enter the lymphoid tissue and disseminates via the lymphatic or hematogenous route
  • Typhoid fever is characterized by severe systemic illness with fever, relative bradycardia and considerable constitutional symptoms including abdominal pain (Med J Armed Forces India 2003;59:130)
  • Salmonella typhi affects mainly the proximal colon causing ulceration and rarely massive bleeding (Dig Liver Dis 2004;36:141, Dis Colon Rectum 1986;29:511)
Terminology
  • Typhoid fever
  • Enteric fever
  • Salmonella is named after the veterinarian Elmer Daniel Salmon
  • Salmonella are gram negative rods
  • Salmonella species bacteria are serotyped based on the chemical content of the O antigens found on the surface and the protein content of part of the flagella known as the H antigen
  • Over 2500 serotypes have been described; however, under 100 are known to be associated with human disease
  • Serotyping is currently done by molecular methods
  • S. typhi and S. paratyphi are associated with typhoid fever
  • Some of the more common nontyphoidal species are:
    • S. enteritidis
    • S. typhimurium
    • S. muenchen
    • S. adnatum and S. give
    • S. paratyphi may also cause nontyphoidal disease
ICD coding
  • ICD-10: A01.09 - typhoid fever with other complications
Epidemiology
  • Worldwide over 20 million cases of typhoid fever occur each year, mostly in the developing world
  • Approximately 1800 cases of typhoid fever occur in the United States annually, mostly in travelers
  • From 1.0 to 1.2 million cases of nontyphoidal illness occur in the United States each year, causing about 19,000 hospitalizations and 450 deaths
  • Tens of millions of nontyphoidal illness occur worldwide, causing over 100,000 deaths each year
  • Disease occurs from ingestion of bacteria, usually in food or water, eggs, milk, meat, poultry
  • Vegetables contaminated with animal feces are commonly implicated
  • Food borne outbreaks, often associated with less than sanitary conditions, are common sources of disease
    • However, in the United States approximately 60% of cases occur sporadically
  • Fecal oral transmission may occur; disease after contact with infected animals is another source
  • Salmonellosis is a common cause of traveler diarrhea
  • All serotypes at least in theory may cause human disease
  • Serotypes may or may not be host specific
  • When serotypes specific to other species infect humans. severe disease may result
  • S. typhi is restricted to humans
  • Risk factors for infection include:
    • Lack of access to clean water
    • Poor sanitation. especially regarding food handling
    • Flooding
    • Ingestion of plants fertilized with sewage
    • Urban environments
    • Sexual transmission has been reported
  • Both typhoidal and nontyphoidal salmonellosis may occur in health care workers when proper hand hygiene is not practiced
  • Travelers to the developing world are often infected with S. paratyphi; vaccination against S. typhi is common prior to travel
  • Spreads through the oral-fecal route, mostly waterborne or via food infected by carriers, rarely through oral-anal sexual contact (Clin Infect Dis 2003;37:141)
  • More common in children and young adults
  • Most prevalent in developing countries and overcrowded areas with poor sanitation
  • High incidence (more than 100 cases per 100,000 person years) in South central Asia, Southeast Asia, southern Africa (Bull World Health Organ 2004;82:346, J Glob Health 2012;2:010401)
  • In developed countries, disease is usually limited to travelers to countries where typhoid fever is endemic (Lancet Infect Dis 2005;5:623)
  • The Vi polysaccharide typhoid vaccine is ineffective against most S. paratyphi, which lack the Vi antigen targeted by the vaccine (Lancet Infect Dis 2005;5:623)
Sites
  • Most common is terminal ileum (100%), followed by ileocecal valve (57%), ascending colon (43%), transverse colon (29%) (Dig Liver Dis 2004;36:141)
  • Nontyphoidal Salmonella infects the small and large bowel
  • S. typhi typically spreads to the liver, spleen, bone marrow and lymph nodes
  • Extraintestinal spread is uncommon in nontyphoidal Salmonella but may occur with severe disease
Pathophysiology
  • Salmonella possess cellular mechanisms enabling bacterial proteins to be transferred to enterocytes and M cells with subsequent growth within endosomes
  • Subsequent events include an inflammatory response with neutrophil recruitment and mucosal damage
  • The host immune response typically controls nontyphoidal infection; however, the very young, elderly, debilitated and immunosuppressed may lack the response necessary to control infection
  • S. typhi more commonly invades M cells
  • Bacteria are phagocytosed by histiocytes in underlying lymphoid tissue
  • The bacteria proliferate and widely disseminate through blood vessels and lymphatic channels
  • S. typhi is a gram negative bacillus typically transmitted via the fecal-oral route
  • Entry of S. typhi into the small bowel epithelium after ingestion appears to be mediated by the cystic fibrosis transmembrane conductance regulator (CFTR)
  • S. typhi proliferates intracellularly in the submucosa, initially in the second part of the Peyer patches, leading to hypertrophy via recruitment of mononuclear cells and lymphocytes
  • Over the next 7 - 21 days, further proliferation occurs in these organs, then organisms are released into the bile and reinfect the ileal lymphoid tissue, then organisms disseminate to the liver, spleen and reticuloendothelial system via the lymphatic system and bloodstream (Clin Exp Gastroenterol 2012;5:213)
  • Replication within the reticuloendothelial system is a hallmark of enteric fever and is responsible for the clinical findings of prostration, generalized sepsis and hepatosplenomegaly
  • An effective vaccine for S. typhi exists
Etiology
  • Salmonella are gram negative bacilli
  • S. typhi is found only in humans
  • Nontyphoidal species are found in humans, domestic and wild animals
  • In general, infection occurs through ingestion of contaminated food or water or the fecal-oral route
  • Nontyphoidal Salmonella are commonly found in food and companion animals including poultry, cattle, swine, parrots, cats, dogs and turtles
  • Eggs, milk, meat, poultry and contaminated vegetables are commonly implicated
  • Direct person to person transmission as well as transmission from pets may occur
    • In this setting, the source of the transmission may not appear ill
Clinical features
  • Symptoms of nontyphoidal salmonellosis generally start 12 - 36 hours after ingestion of bacteria but may occur after 6 - 72 hours
  • Ingestion of very few organisms may cause disease
  • Typically patients suffer self limited illness characterized by diarrhea, abdominal pain, fever, nausea and occasionally vomiting that is unpleasant but rarely life threatening
  • May be life threatening In infants, elderly, immunocompromised or debilitated patients, due to dehydration or dissemination of bacteria
  • Very rarely, toxic megacolon may complicate infection
  • Individuals with achlorhydria or hypochlorhydria from drugs, chronic Helicobacter pylori infection or other causes are at higher risk of infection
  • Some patients develop a reactive arthritis that may last for months and lead to chronic arthritis

  • Typhoid fever causes symptoms shortly after bacteria are ingested
  • Patients suffer from severe abdominal pain, bloody diarrhea, bloating, anorexia, nausea, vomiting, headache
  • A brief asymptomatic period is followed by bacteremia with fever and flu-like illness
  • Blood cultures are almost always positive at this time and antibiotic therapy may be lifesaving
  • With disease progression in untreated patients, high fever and abdominal pain occurs that may mimic appendicitis; lasts for about two weeks
  • If patients survive, symptoms slowly abate
  • Possible extraintestinal complications include CNS disease, endocarditis, myocarditis, pneumonia, cholecystitis, osteomyelitis (patients with sickle cell disease are particularly prone to osteomyelitis)
  • Disease relapses may occur
  • First week of illness: rising ("stepwise") fever associated with chills, although frank rigors are rare
    • Also relative bradycardia or pulse-temperature dissociation is common
  • Second week of illness: abdominal pain develops and "rose spots" (faint salmon colored macules on the trunk and abdomen) may be seen
  • Third week of illness: hepatosplenomegaly, intestinal bleeding and perforation due to ileocecal lymphatic hyperplasia of the Peyer patches may occur, together with secondary bacteremia and peritonitis (Lancet Infect Dis 2005;5:623)
  • Chronic Salmonella carriage is defined as excretion of the organism in stool or urine >12 months after acute infection
Diagnosis
  • Nontyphoidal salmonellosis traditionally has been diagnosed by stool culture although many patients do not seek medical attention
  • Recently a PCR based assay has been introduced that, in addition to Salmonella species bacteria, also detects Camplyobacter group, Shigella species, Vibrio group, Yersinia enterocolitica, Shiga toxin 1 and 2, Norovirus G1 / GII, Rotavirus A and Aeromonas species
  • Serotyping is done to characterize disease outbreaks
  • Typhoid fever is diagnosed by blood or stool culture
  • In the developing world where laboratory facilities may not be readily available, disease may be treated empirically
  • Bone marrow culture is the most sensitive test available, as it remains positive even after 5 days of antibiotic therapy (Lancet 1975; 1:1211)
Laboratory
  • CBC shows leukopenia and thrombocytopenia
  • Cultures can be from blood, stool, urine, rose spots, the blood mononuclear cell platelet fraction, bone marrow, gastric or intestinal secretions
  • A positive culture for S. typhi or S. paratyphi is obtained in more than 90% of patients if blood, bone marrow and intestinal secretions are all performed
  • Because almost all S. typhi organisms in blood are associated with the mononuclear cell platelet fraction, centrifugation of blood and culture of this fraction can reduce the time for isolation of the organism but does not increase the sensitivity
  • Children also have a higher incidence of positive stool cultures than adults (60% versus 27%)
  • The duodenal string test is useful for sampling the upper GI tract (J Infect Dis 1984;149:157)
Radiology description
  • Most common colonoscopic finding is multiple variably sized punched out ulcers with slightly elevated margins
  • Also edematous hyperemic mucosal patches with hemorrhagic spots or shallow erosions
Prognostic factors
  • Nontyphoidal salmonellosis is almost always a self limited infection, except in infants, the elderly, immunocompromised and debilitated patients
  • Antibiotic therapy is usually efficacious in these patients, although antibiotic resistance is a growing problem
  • Without treatment, typhoid fever may be lethal even in healthy individuals but the prognosis is worse in infants, the elderly, immunocompromised and debilitated patients
  • Prompt initiation of antibiotic therapy greatly improves patient outcome; although again, antibiotic resistance is a growing problem
Case reports
Treatment
  • Nontyphoidal Salmonella usually:
    • Only requires supportive care, particularly hydration
    • Antibiotics are indicated for severe disease and vulnerable patients
    • Ceftriaxone and ciprofloxacin are currently recommended by the CDC
    • Antibiotic resistance is a growing problem
  • Antibiotic therapy for typhoid fever is obligatory and should not be delayed for confirmatory laboratory testing
    • Prolonged course of antimicrobials and supportive care with transfusions
    • Antibiotic therapy for typhoid fever is obligatory and should not be delayed for confirmatory laboratory testing
    • Ceftriaxone and ciprofloxacin are currently recommended by the CDC (CDC: Typhoid Fever and Paratyphoid Fever [Accessed 28 November 2017], Lancet Infect Dis 2005;5:623)
    • Third generation fluoroquinolones are also recommended but resistance has been reported
    • In the past, chloramphenicol, trimethoprim-sulfamethoxazole, cephalosporins and first generation fluoroquinolones were used but antibiotic resistance developed
    • Surgery may be necessary for intestinal perforation or gallbladder disease
    • Selective angiography and platinum coil embolization is used for severe, life threatening lower gastrointestinal hemorrhage
  • WHO recommendations for the public and travelers include:
    • Ensure food is properly cooked and hot when served
    • Avoid raw milk and products made from raw milk
    • Avoid ice unless made from safe water
    • Use disinfecting tablets for water unless its purity is certain
    • Wash hands thoroughly with soap and water after contact with pets or farm animals and after using the toilet
    • Wash fruits and vegetables thoroughly and if possible, peel fruits and vegetables unless optimal sanitary practices are known to have taken place
  • Recommendations for food handlers and producers include:
    • Maintain a clean workspace
    • Separate raw and cooked food
    • Cook food thoroughly
    • Keep food at safe temperatures
    • Use safe water and raw materials
    • Practice good personal hygiene
    • Do not work if have fever, diarrhea, vomiting or infected skin lesions
    • Protect fields from animal contamination
Clinical images

Images hosted on other servers:
Missing Image

Typhoid rash

Gross description
  • Colonoscopy in patients with mild nontyphoidal salmonellosis shows nonspecific findings including edema and petechial hemorrhage; more severe disease is associated with friability and ulceration
  • Typhoid fever causes marked enlargement of Peyer patches and lymphoid tissue in the appendix and ascending colon leading to mucosal elevation along the axis of the ileum
  • Perforation may occur
  • Small gray, soft nodules (typhoid nodules) may be present in the liver
  • There is splenomegaly and lymphadenopathy
Microscopic (histologic) description
  • Nontyphoidal salmonellosis is rarely biopsied
  • The findings are a non-specific acute self limited colitis
  • The crypt architecture is maintained, there are increased mixed inflammatory cells within the lamina propria in a patchy distribution with foci of cryptitis and possible crypt abscesses
  • Long standing cases may demonstrate architectural distortion raising the spectre of idiopathic inflammatory bowel disease
    • This is also true of S. typhimurium infection
  • Knowledge of culture results is necessary to render a specific diagnosis

  • S. typhi shows neutrophils, histiocytes with cytoplasmic bacteria, nuclear debris and hemorrhage within the lamina propria associated with a lymphoplasmacytic infiltrate
  • The spleen and lymph nodes show sinus histiocytosis that distorts the normal architecture
  • Typhoid nodules in the liver show aggregates of histiocytes with necrotic debris
  • Typhoid nodules may also be found in bone marrow and lymph nodes
Microscopic (histologic) images

Contributed by Elliot Weisenberg, M.D.
Missing Image Missing Image

Acute self limited colitis



Images hosted on other servers:
Missing Image

Typhoid nodules

Differential diagnosis
  • The differential diagnosis of nontyphoidal salmonellosis includes other enteric pathogens that cause acute self limited colitis including Shigella and Campylobacter
  • Long standing cases may mimic idiopathic inflammatory bowel disease
Board review style question #1
Which of the following are common sources of infection for Salmonella?

  1. Food borne
  2. Food borne and water borne
  3. Sexual transmission
  4. Water borne
Board review style answer #1
B. Food borne and water borne

Comment Here

Reference: Salmonella (typhoid and nontyphoidal)
Board review style question #2
Which serotype of Salmonella occurs only in humans?

  1. S. choleraesuis
  2. S. typhi
  3. S. typhimurium
  4. None of the above
Board review style answer #2
Board review style question #3
Which site is most commonly involved by Salmonella?

  1. Colon
  2. Gall bladder
  3. Ileum
  4. Stomach
Board review style answer #3
Board review style question #4
Which is the most common site of localization of S. typhi in humans?

  1. Gastric epithelium
  2. Interstitial cell of Cajal
  3. Neurons
  4. Peyer patches
Board review style answer #4

Sapovirus
Definition / general
  • Acute gastroenteritis is one of the leading causes of death worldwide, mainly in children of developing countries
  • Sapovirus can cause acute gastroenteritis in animals and humans
  • Prevalence can reach 7.5% of pediatric acute gastroenteritis cases
  • No reports on histological features of disease
Essential features
  • Virus belonging to the Caliciviridae family, together with norovirus
  • Emerging cause of acute gastroenteritis, mainly in children, even in developed countries
  • Can sometimes explain both sporadic cases and outbreaks of norovirus negative acute gastroenteritis
Terminology
  • Formerly called Sapporo-like virus
  • The term typical human caliciviruses was discouraged by the 2002 International Committee on the Taxonomy of Viruses
ICD coding
  • ICD-10: A08.31 - calicivirus enteritis
Epidemiology
Pathophysiology
  • Has a single stranded RNA genome with 2 open reading frames (ORF), coding viral structural proteins (VP1 in the ORF1 and VP2 in the ORF2) (Curr Opin Infect Dis 2020;33:388)
  • 5 genogroups according to the sequence of VP1 and all but GIII can infect humans
  • 19 genotypes: GI, 8 genotypes; GII, 8 genotypes; GIV, 1 genotype; GV, 2 genotypes
  • Sapovirus particles bind to cell surface receptors then enter cells through complex, partially unknown processes and incite pathology (J Virol 2017;91:e00202)
  • Can disrupt epithelial tight junctions through the RhoA-ROCK-MLC signaling pathway and by using occludin as a coreceptor (J Virol 2021;95:e00051, J Virol 2019;93:e01773)
Clinical features
Diagnosis
  • Sapovirus was initially discovered in human diarrheic stool samples using electron microscopy in 1976 (Lancet 1976;1:199)
  • No animal model is available, which makes it difficult to obtain antibodies for immune assays (PLoS One 2018;13:e0178157)
  • Despite attempts to grow human sapovirus in cell culture systems, results are not reproducible yet (J Gen Virol 1984;65:1123)
  • Antigen detection methods using ELISA have been developed but lack sensitivity and are not commercially available (Arch Virol 2006;151:551)
  • Real time reverse transcription PCR has become the most widely used method for sapovirus detection (J Clin Virol 2006;35:321)
  • Multiplex studies that can detect several enteral virus are also available (Pathology 2021;53:290)
Laboratory
  • No specific abnormalities
Radiology description
Prognostic factors
Case reports
Treatment
  • No specific therapy is necessary, other than supportive measures for acute gastroenteritis
  • Usually self limited
  • Some cases require hospital admission (Infect Genet Evol 2005;5:281)
  • In transplanted patients with severe diarrhea due to sapovirus, nitazoxanide has been employed with good outcome (Case Rep Infect Dis 2018;2018:6846873)
Microscopic (histologic) description
  • No report of specific histologic changes in sapovirus infection
  • Biopsy is not indicated
Microscopic (histologic) images

Contributed by M.J. Fernández-Aceñero, M.D., Ph.D.
Slight mucosal changes in sapovirus infection

Slight mucosal changes in sapovirus infection

Electron microscopy description
  • Sapovirus was discovered in diarrheic human stool by electron microscopy
  • Morphology is similar to other caliciviruses
  • Small icosahedral particles (30 - 38 nm) with cup shaped depression on the surface
  • Typical Star of David morphology (Clin Microbiol Rev 2015;28:32)
  • Partially replaced by molecular techniques, although some outbreaks caused by previously unreported genotypes have been resolved with electron microscopy (Euro Surveill 2017;22:30543)
Electron microscopy images

Images hosted on other servers:
Typical morphology of sapovirus Typical morphology of sapovirus

Typical morphology of sapovirus

Molecular / cytogenetics description
  • There are many tools that can help identify the pathogens that lead to acute diarrhea
  • The performance of multiplex PCR testing is better than isolated tests
  • Assays that employ the FilmArrayTM technology can detect 22 of the most frequent gastrointestinal pathogens through rapid PCR multiplexing (BMC Microbiol 2017;17:111)
Sample pathology report
  • Colon, endoscopic biopsy:
    • Active colitis (see comment)
    • Comment: Colonic mucosa with inflammatory expansion of the lamina propria by lymphocytes and plasma cells, associated with slight crypt architectural distortion and few neutrophils. No granulomas or viral inclusions. No dysplasia. The histopathological image is nonspecific and can be associated with drugs, inflammatory bowel disease and bacterial, viral or parasitic infections. Diagnosis should be based on the synthesis of clinical, microbiological and endoscopic data.
Differential diagnosis
  • Inflammatory bowel disease (IBD):
    • Typical clinical and endoscopic features of IBD
    • Biopsy should show continuous crypt distortion, cryptitis and intraluminal microabscesses limited to the mucosa in ulcerative colitis
    • Typical skip lesions, with deep fibrosing ulcers associated or not associated with granulomas or pyloric metaplasia in Crohn's disease
  • Microscopic colitis:
  • Irritable bowel disease, diarrhea associated variant:
    • Largely a diagnosis of exclusion
    • No biopsy is indicated except to rule out other disorders
  • Drug adverse effects:
    • Clinical features are important
    • Apoptosis of epithelial cells and intraepithelial lymphocytes are more prominent (Am J Surg Pathol 2015;39:1653)
  • Other viral causes of diarrhea:
  • Parasites:
    • Can usually be identified in the biopsy samples
  • Bacterial causes of diarrhea:
    • Usually, active colitis with edema and hemorrhage
    • Clinical features and microbiological studies can help diagnosis
Board review style question #1
A 4 year old girl attending a daycare center is brought to hospital by her parents with diarrhea and abdominal pain lasting 2 days, without fever. She was vaccinated against norovirus. Her vital signs are normal. Which of the following is true regarding this patient?

  1. Biopsy is needed to confirm diagnosis
  2. Electron microscopy of a stool sample is mandatory for diagnosis
  3. She must be hospitalized immediately
  4. The symptoms will disappear in a few days with no therapy
Board review style answer #1
D. The symptoms will disappear in a few days with no therapy. Most viral infections disappear spontaneously with no further therapy. Answer A is incorrect because biopsy is not indicated in acute diarrheal diseases. Answer B is incorrect because while it was once the gold standard for diagnosis, electron microscopy has largely been replaced by molecular techniques that are more reliable. Answer C is incorrect because hospitalization need is guided by the clinical situation of the patient. In this case, vital signs were within normal limits.

Comment Here

Reference: Sapovirus
Board review style question #2
Which of the following techniques is most useful for diagnosis of sapovirus infection?

  1. Biopsy
  2. Complete blood count
  3. Molecular techniques
  4. Serology
Board review style answer #2
C. Molecular techniques. Answers A, B and D are incorrect because in the diagnosis of sapovirus infection, there are no serological assays, no specific analytical abnormalities and no indication for biopsy.

Comment Here

Reference: Sapovirus

Schistosomiasis
Definition / general
  • Schistosomiasis is a parasitic infection caused by a trematode blood fluke (WHO: Schistosomiasis [16 July 2018])
  • First identified by Theodor Bilharz in 1852, also known as "bilharziasis"
  • 5 different species: 3 major and 2 minor
    • 3 major species include:
      • Schistosoma mansoni
      • S. japonicum
      • S. haematobium
    • 2 minor species include:
      • S. mekongi
      • S. intercalatum
  • Only S. haematobium causes genitourinary tract disease; all other species affect intestine and liver
Essential features
  • Prevalence of disease highest in sub-Saharan Africa
Terminology
  • Bilharziasis
  • Snail fever
ICD coding
  • ICD-10: B65.1 - Schistosomiasis due to Schistosoma mansoni [intestinal schistosomiasis]
Epidemiology
  • Most patients are ages 15 - 20 years (J Med Case Rep 2014;8:331)
  • Prevalence of schistosomiasis is highest in sub-Saharan Africa (Lancet 2012;380:2163
  • There are three major species (J Adv Res 2013;4:445)
    • Schistosoma mansoni: Africa, South America, Middle East
    • S. japonicum: Southeast Asia
    • S. haematobium: Africa, Middle Eastern
Sites
  • Liver, intestine, bladder, lungs, spleen, brain or spinal cord; less common sites of embolization include the skin and peritoneal surfaces
Pathophysiology
  • Freshwater gets contaminated through feces (S. mansoni and S. japonicum) or urine (S. haematobium) from an infected person (Acta Trop 2000;77:41)
  • In water, eggs release miracidia, which enter freshwater snails, the intermediate hosts
  • Within snails, cercariae are formed and are released into the water after about four to six weeks
  • Cercariae can survive in water up to two days
  • Individuals become infected when skin contacts contaminated water and is penetrated by cercariae, which drop their tail and form schistosomulae
  • Schistosomulae enter the circulation and reach the liver, where they mature into adults over two to four weeks
  • Adult worms migrate against portal blood flow to the mesenteric venules of the small and large intestine (S. japonicum and S. mekongi), the mesenteric venules of the colon (S. mansoni, S. intercalatum) or the vesical venous plexus (S. haematobium)
  • Migrating eggs elicit a host immune response
  • Egg derived antigens induce a host Th2 immune response, leading to recruitment of eosinophils, granulomatous reaction and later fibrosis (Immunol Rev 2004;201:156, Immunol Cell Biol 2007;85:148, Front Immunol 2013;4:89)
Etiology
  • Blood fluke, schistosomiasis
Clinical features
  • Can be acute or chronic
  • Acute occurs in travelers of nonendemic region
  • Chronic disease occurs in individuals living in endemic regions
  • Acute manifestations include swimmer's itch and Katayama fever
  • Swimmer's itch causes itchy rash after swimming in water due to localized dermatitis
  • Katayama fever
    • Acute systemic hypersensitivity reaction to parasitic antigens and circulating immune complexes
    • Inflammation of bowel causes ulceration, blood loss and scarring
    • Within liver causes periportal fibrosis (or Symmers pipestem fibrosis) which can cause portal hypertension
  • Chronic disease occurs in endemic areas usually three to eight weeks after infection; can present as diarrhea, chronic or intermittent abdominal pain, poor appetite (World J Surg Oncol 2010;8:68)
  • Chronic colonic ulceration may lead to intestinal bleeding and iron deficiency anemia, intestinal polyps, dysplasia, bowel obstruction or acute appendicitis (Case Rep Infect Dis 2012;2012:896820)
Diagnosis
  • Based on two types of tests: direct assay and indirect assay
    1. Direct assay is either the demonstration of eggs in the stool or urine or demonstration of antigen or DNA of parasite in the blood, urine or stool
      • Circulating anodic antigen (CAA) and circulating cathodic antigen (CCA) excreted in urine are identified in antigen test (Lancet 2006;368:1106)
    2. Indirect assay is the demonstration of antibody to parasite in blood via serology
  • Intensity of intestinal infection is classified as light (up to 100 eggs per gram), moderate (100 to 400 eggs per gram) or severe ( > 400 eggs per gram) (World Health Organization 2002:912)
  • Most useful tests for returned travelers is serology
  • Most useful test for individuals living in endemic areas is determining the parasite burden by microscopy for egg detection and antigen detection
Laboratory
  • Egg identification in stool or urine by microscopy
  • Eosinophilia, anemia, thrombocytopenia
Radiology description
  • Colonoscopy can show congested mucosa / greyish yellow nodules / ulceration / polyps
  • USG shows periportal fibrosis in hepatic schistosomiasis
  • Grading of thickness of periportal fibrosis on ultrasonography (Am J Trop Med Hyg 1992;46:403)
    • Grade I: 3 - 5 mm
    • Grade II: 5 - 7 mm
    • Grade III: more than 7 mm
Radiology images

Images hosted on other servers:

USG showing pipestem fibrosis

Large pedunculated polyp

Prognostic factors
  • The adult worms live for 5 - 7 years but can remain in tissue for up to 30 years
  • Embolization of adult worms into the spinal cord or cerebral blood vessels can cause neuroschistosomiasis
Case reports
Treatment
Gross description
  • Acute schistosomiasis: colonic mucosa is edematous and congested with petechial hemorrhage
  • Chronic schistosomiasis: colonic mucosa has flat or elevated yellow nodules with vessels on surface, polyps and intestinal stricture
  • The most characteristic finding is the grayish yellow or yellowish white schistosomal nodules which mimics antibiotic associated / pseudomembranous enterocolitis (World J Gastroenterol 2010;16:723)
Microscopic (histologic) description
  • Ova are mainly seen in the loose submucosa of large intestine, usually with formation of granulomas and infiltration by eosinophils and inflammatory cells
  • Later the muscularis mucosa becomes involved and the overlying mucosa shows small superficial ulcers
  • Fibrosis develops in chronic stage, when only calcified eggs are seen
Microscopic (histologic) images

Contributed by Nalini Bansal Gupta, M.D., Lisa Cerilli, M.D. and @zaalruwai83 on Twitter

Calcified eggs of schistosomiasis


Missing Image Missing Image Missing Image Missing Image Missing Image Missing Image

Within colonic mucosa


Schistosomiasis Schistosomiasis Schistosomiasis Schistosomiasis

Schistosomiasis



Images hosted on other servers:

Several eggs noted in stroma of polyp

Missing Image

Eggs

Missing Image

S. mansoni eggs



S. japonicum:
Missing Image Missing Image

Eggs



Within tissue - species not specified:
Missing Image

intramucosal calcified bodies

Missing Image

Calcified bodies often clustered

Missing Image

Granuloma surrounding egg

Missing Image

Occasional forms
suggestive of spines
or hooks

Ziehl-Neelsen staining

Positive stains
Negative stains
  • PAS stain
  • Ziehl-Neelsen: Schistosoma haematobium shell is ZN negative
Molecular / cytogenetics description
Differential diagnosis
Board review style question #1
  1. How does the schistosomiasis parasite involve the intestines and bladder?
  2. How do humans acquire schistosomiasis infections?
  3. What type of parasite is schistosomiasis?
  4. What is the other name for schistosomiasis?
  5. Which species of schistosomiasis affects the urinary bladder?
Board review style answer #1
  1. By retrograde movement from portal vein to mesenteric vessels
  2. By swimming / contact with contaminated water
  3. Blood fluke
  4. Bilharziasis
  5. S. haematobium

Comment Here

Reference: Schistosomiasis

Schwannoma
Definition / general
  • Uncommon mesenchymal tumor of colon
Essential features
  • Benign spindle cell lesion of muscularis propria with prominent lymphoid cuff
  • Positive for S100, unlike gastrointestinal stromal tumor (GIST)
  • No syndromic association
Terminology
  • GANT is an outdated term for GIST, not schwannoma
Epidemiology
  • Rare; less common than colonic GIST
Sites
  • Colon is second most common site in gastrointestinal tract for schwannoma, after stomach
Clinical features
Case reports
Clinical images

Images hosted on other servers:

Submucosal colonic tumors

Gross description
  • Well circumscribed but usually not encapsulated
  • Polypoid intraluminal mass with mucosal ulceration, usually in right colon
Gross images

Contributed by @Andrew_Fltv on Twitter
Schwannoma Schwannoma

Schwannoma



Images hosted on other servers:

Various images

Microscopic (histologic) description
  • Nonencapsulated, polypoid lesion arising in muscularis propria (Mod Pathol 2015;28:S47)
  • Bland spindle cells with elongated, pointed nuclei; no or indistinct Verocay bodies and no prominent nuclear palisading
  • Stroma may be myxoid and contain prominent collagen bands
  • Surrounded by lymphoid cuff
  • May have microcystic or reticular features or may be epithelioid (Am J Surg Pathol 2008;32:1080)
  • May have focal nuclear atypia; mitotic figures rare
  • Usually no vascular hyalinization or xanthoma cells
Microscopic (histologic) images

Contributed by Raul S. Gonzalez, M.D. and @Andrew_Fltv on Twitter

H&E

Schwannoma Schwannoma

Schwannoma



Images hosted on other servers:

Various images


Various immunostains

Positive stains
Negative stains
Sample pathology report
  • Rectosigmoid colon, resection:
    • Schwannoma (4.1 cm) (see comment)
    • Margins of resection unremarkable.
    • Three benign lymph nodes.
    • Comment: An immunohistochemical stain for S100 is positive.
Differential diagnosis
Board review style question #1
Which of the following is a common feature of gastrointestinal schwannomas?

  1. Artifactual cytoplasmic vacuoles
  2. Dense cellularity and high mitotic rate
  3. Epicenter in the muscularis mucosae
  4. Peripheral lymphoid cuff
  5. Verocay bodies and thick walled vasculature
Board review style answer #1
D. Peripheral lymphoid cuff

Comment here

Reference: Schwannoma

Scleroderma (pending)
Table of Contents
Definition / general
Definition / general
[Pending]

Sclerosing mesenteritis
Definition / general
  • Uncommon nonneoplastic condition that causes thickening and shortening of colonic mesentery (JBR-BTR 2011;94:241)
Essential features
  • Benign fibrotic / inflammatory condition involving the mesentery
  • Some cases are IgG4 related
Terminology
  • Older terms include idiopathic retractile mesenteritis, mesenteric panniculitis and mesentery lipodystrophy (Am J Surg Pathol 1997;21:392)
Epidemiology
Sites
  • Affects the mesentery; small bowel mesentery is involved more often than colonic
Etiology
Clinical features
Radiology description
  • Mesentery can show fat attenuation or a fibrofatty mass
Case reports
Treatment
  • Various medications (including corticosteroids) and surgery, depending on clinical severity (Int Arch Med 2011;4:17)
Clinical images

Images hosted on other servers:

Thick mesentery with yellow nodules

Gross description
  • Markedly thickened and rubbery mesentery causing bowel twisting
Microscopic (histologic) description
  • Fibrosis with dense collagen, fat necrosis, chronic inflammation (especially around vessels) and variable focal calcification
  • Minimal atypia, no or few mitoses
  • IgG4 related cases lack fat necrosis and show obliterative phlebitis and increased inflammation
Microscopic (histologic) images

Contributed by Raul S. Gonzalez, M.D.

Fibrosis and chronic inflammation

Fibrosis and chronic inflammation, high power

Fibrosis, chronic inflammation and fat necrosis



Images hosted on other servers:

Fat necrosis, sclerosing fibrosis, chronic inflammation

Fat necrosis with numerous lipid laden macrophages

Positive stains
  • IgG4 (in some cases)
Negative stains
Sample pathology report
  • Transverse colon and mesentery, excision:
    • Prominent bland fibrotic process involving mesentery (see comment)
    • Segment of colon with reactive change.
    • Negative for malignancy.
    • Margins of resection unremarkable.
    • Comment: The overall findings are most consistent with sclerosing mesenteritis.
Differential diagnosis
Board review style question #1

Which of the following is true about sclerosing mesenteritis?

  1. IgG4 immunohistochemistry will always highlight numerous plasma cells
  2. It is a neoplastic process
  3. It is common in the pediatric population
  4. Treatment can involve medication rather than surgery
Board review style answer #1
D. Treatment can involve medication rather than surgery

Comment Here

Reference: Sclerosing mesenteritis

Secukinumab induced colitis (pending)
[Pending]

Serrated adenocarcinoma (SAC)
Definition / general
  • Morphologic variant first recognized in the fourth edition of WHO; accounts for approximately 9.1% of all colorectal carcinomas (Hum Pathol 2010;41:1359)
Essential features
  • Likely arises from malignant transformation of some sessile serrated polyps and traditional serrated adenomas (Mod Pathol 2019;32:1390)
  • Can be diagnosed on histologic grounds alone without having to resort to recognition of residual serrated lesion
  • Considered an end point carcinoma of the so called serrated pathway that can be reliably identified by morphologic features
  • Most carcinomas arising from serrated precursors do not represent serrated adenocarcinomas (Mod Pathol 2019;32:1390)
ICD coding
  • ICD-O: 8213/3 - serrated adenocarcinoma
Epidemiology
Sites
  • More often proximal colon than conventional carcinoma, only in men (this association is not observed in women) (Hum Pathol 2010;41:1359)
Clinical features
  • More frequently with lymph node metastases (51.8%) than conventional carcinomas (39.9%) (Hum Pathol 2010;41:1359)
  • Node positive serrated adenocarcinomas have worse survival than node positive conventional carcinomas; left sided ones have the worst prognosis (Hum Pathol 2010;41:1359)
  • Synchronous carcinomas are more often found in serrated adenocarcinoma than in conventional carcinoma (12.9% versus 3%) (Hum Pathol 2010;41:1359)
  • Frequently encountered in more advanced stages than conventional carcinomas
  • Less favorable 5 year survival than conventional carcinoma (especially left sided) (Hum Pathol 2010;41:1359)
Diagnosis
  • Generally discovered on colonoscopy and confirmed on biopsy
Prognostic factors
Case reports
Treatment
  • Surgical excision, with adjuvant chemotherapy for advanced / metastatic cases
  • No treatment related differences between serrated adenocarcinoma and conventional carcinoma
  • Treatment based on whether they are microsatellite stable or microsatellite instable (PD1 inhibitors but not 5FU are potential therapies if instable) and BRAF / KRAS / NRAS status (EGFR inhibitors not an option if any are mutated)
Clinical images

Contributed by David Hernandez Gonzalo, M.D. and Michael Feely, D.O.

Colonoscopy

Microscopic (histologic) description
  • Sawtoothed epithelial serrations
  • Abundant clear or eosinophilic cytoplasm
  • Vesicular nuclei with chromatin condensation at the nuclear envelope
  • Absence of necrosis or < 10% of the total surface area
  • Mucin production (cell balls and papillary rods in mucinous areas of tumor)
  • A serrated polyp with or without dysplasia can sometimes be seen around its edges (Histopathology 2007;50:131)
Microscopic (histologic) images

Contributed by David Hernandez Gonzalo, M.D. and Michael Feely, D.O.

Precursor lesions

Adjacent epithelium

Tumor and depth

Architectural changes

Cytologic changes

Positive stains
Molecular / cytogenetics description
  • KRAS mutations (45%)
  • BRAF mutations (33%)
  • Microsatellite instable - high (18.9%)
  • Microsatellite stable or microsatellite instable - low (81.1%) (Histopathology 2011;58:679)
Sample pathology report
  • Colon, transverse, resection:
    • Invasive serrated adenocarcinoma (see synoptic report)
    • 54 lymph nodes, negative for carcinoma (0/54)
Differential diagnosis
Board review style question #1

Which of the following is true about this variant of colorectal adenocarcinoma?

  1. Diagnosis requires the presence of a serrated polyp adjacent to the serrated adenocarcinoma
  2. Extensive necrosis is usually present
  3. Most carcinomas arising from serrated precursors have this morphology
  4. This variant accounts for approximately 9.1% of all colorectal carcinomas
Board review style answer #1
D. Serrated adenocarcinoma accounts for approximately 9.1% of all colorectal carcinomas. A precursor serrated lesion is not always identified (A). There is no or little necrosis (< 10%) in serrated adenocarcinoma (B). Most carcinomas arising from serrated polyps have a microsatellite instable - high histology (such as medullary, mucinous, signet ring cells, lack of dirty necrosis, tumor infiltrating lymphocytes) (C).

Comment Here

Reference: Serrated adenocarcinoma
Board review style question #2
Which of the following is true regarding the molecular aspect of serrated adenocarcinoma?

  1. Chemotherapy regimens are currently different for serrated adenocarcinoma compared to conventional adenocarcinoma
  2. GNAS mutant serrated adenocarcinoma has a better prognosis
  3. KRAS or BRAF mutations are present in over 75% of cases
  4. Most serrated adenocarcinomas are microsatellite instable - high
Board review style answer #2
C. Mutually exclusive mutations in KRAS or BRAF are present in over 75% of cases. There are no treatment related differences between serrated adenocarcinoma and conventional carcinoma (A). GNAS mutant serrated adenocarcinoma appears to be related to a poor prognosis and advanced stage (B). Microsatellite instability - high is only seen in 18.9% of serrated adenocarcinoma cases (D). Remember, in general, a serrated pathway can lead to microsatellite instable - high, microsatellite instable - low or microsatellite stable cancer.

Comment Here

Reference: Serrated adenocarcinoma

Serrated lesions
Definition / general
  • Lesions and polyps that are characterized by a serrated (sawtooth or stellate) architecture of the epithelium
  • Include hyperplastic polyps (HPs), sessile serrated lesions (SSLs), traditional serrated adenomas (TSAs) and serrated adenoma, unclassified
Essential features
  • Colorectal epithelial lesion with serrated architecture and arises from serrated pathway (BRAF or KRAS mutation)
  • Include hyperplastic polyps, sessile serrated lesions, traditional serrated adenomas and serrated adenoma, unclassified
  • Presence of dysplasia should be reported but grading of dysplasia is not recommended
  • Size or location alone is not diagnostic for either hyperplastic polyps or sessile serrated lesions
Terminology
  • Names that usually refer to sessile serrated lesions but are not recommended by WHO 5th edition (2019):
    • Sessile serrated adenoma or sessile serrated polyp
ICD coding
  • ICD-O: 8213/0 - serrated adenoma
  • ICD-11:
    • DB35.0 - hyperplastic polyp of large intestine
    • 2E92.4 & XH2F06 - benign neoplasm of the large intestine & sessile serrated adenoma
    • 2E92.4 & XH63V9 - benign neoplasm of the large intestine & sessile serrated polyp
    • 2E92.4 & XH9PD9 - benign neoplasm of the large intestine & traditional sessile serrated adenoma
Epidemiology
Sites
  • Hyperplastic polyps:
    • Include goblet cell rich hyperplastic polyps (GCHPs) and microvesicular hyperplastic polyps (MVHPs)
    • Mostly found in distal colon and rectum (left)
  • Sessile serrated lesions: mostly found in proximal colon (right), particularly sessile serrated lesions with dysplasia
  • Traditional serrated adenomas: mostly found in distal colon (left); exception: flat traditional serrated adenomas (more often in the proximal colon)
Pathophysiology
  • About 30% of all colorectal carcinomas arise via serrated neoplasia pathway (Histopathology 2015;66:49, J Gastroenterol 2013;48:287)
  • Lesions arise from BRAF serrated pathway:
    • BRAF mutation → microvesicular hyperplastic polyps → CpG island methylator phenotype (CIMP) high → sessile serrated lesions
      • Sessile serrated lesions → MLH1 methylation / WNT activation → MLH1 deficient sessile serrated lesions with dysplasia → BRAF mutated mismatch repair (MMR) deficient carcinoma
      • Sessile serrated lesions → WNT activation + TP53 mutation → MLH1 proficient sessile serrated lesions with dysplasia → BRAF mutated mismatch repair proficient carcinoma
    • BRAF mutation + CpG island methylator phenotype → BRAF mutated traditional serrated adenomas → WNT activation / TP53 mutation → BRAF mutated traditional serrated adenomas with high grade dysplasia → p16 silencing → BRAF mutated mismatch repair proficient carcinoma
  • Lesions arise from KRAS serrated pathway:
    • Goblet cell rich hyperplastic polyps → CpG island methylator phenotype low → KRAS mutated traditional serrated adenomas → WNT activation + TP53 mutation → KRAS mutated traditional serrated adenomas with high grade dysplasia → KRAS mutated mismatch repair proficient carcinoma
  • Activation of BRAF or KRAS is thought to be mutually exclusive (Mod Pathol 2015;28:414, Gastroenterology 2006;131:1400, Am J Surg Pathol 2014;38:1290, Am J Surg Pathol 2004;28:423)
Etiology
Diagrams / tables

Images hosted on other servers:

BRAF and KRAS serrated pathway

Histologic and molecular features of serrated polyps

Clinical features
Diagnosis
  • Usually asymptomatic and discovered at surveillance endoscopy (Am J Gastroenterol 2016;111:516)
  • Relevant endoscopy modalities:
    • Chromoendoscopy
    • Narrow band imaging
Prognostic factors
Case reports
Treatment
  • Endoscopic polypectomy
  • Lesions with low risk pathology features: repeat colonoscopy in 5 years
  • Lesions with high risk pathology features: repeat colonoscopy in 3 years
  • > 10 cumulative adenomatous polyps or SSL
    • Genetic testing for polyposis syndrome
    • If genetic testing is negative, repeat colonoscopy within 1 - 3 years
  • Incomplete or piecemeal polypectomy for large (> 1 cm) polyp and pathology shows no invasive cancer
    • Management depends on if high risk endoscopic features present
    • Piecemeal resection of sessile serrated lesions > 2 cm, repeat colonoscopy in 6 months
  • In general, SSLs without dysplasia are managed like tubular adenomas and SSLs with any grade dysplasia are managed like high risk adenomas but may need even more frequent surveillance
  • References: NCCN: NCCN Guidelines - Colorectal Cancer Screening [Accessed 12 November 2021], Gastroenterology 2020;158:1131
Clinical images

Images hosted on other servers:

Endoscopic appearance of SSLs

Endoscopic appearance of SSLs with dysplasia

Gross description
  • Endoscopic appearance:
  • Example of gross description in pathology report:
    • Specimen is received in formalin and consists of 3 white to red-tan, ovoid, soft tissue fragments, ranging from 0.2 cm up to 0.5 cm
Microscopic (histologic) description
  • Hyperplastic polyps:
    • Superficial serrated epithelium and funnel shaped, evenly spaced crypts
    • Proliferative zones confined to crypt bases
    • No basal dilatation, architecture distortion or submucosal misplacement
    • No cytologic dysplasia
    • Individual crypt branching may occur
    • Microvesicular hyperplastic polyps:
      • Microvesicular: abundant cytoplasm with fine apical vacuoles, occasional goblet cells present
      • Cross sections of crypts: stellate lumina (star shaped)
      • Diagnosis of exclusion: other criteria of sessile serrated lesions are not met
    • Goblet cell hyperplastic polyps:
      • Subtle superficial epithelial serrations, mimicking reactive epithelium or hyperplasia, thus easily overlooked
      • Goblet cell only in epithelium; no microvesicular cell
      • Cross sections of crypt: round, not stellate
  • Sessile serrated lesions:
    • Crypts with prominent serrations and overall architecture distortion, defined as unequivocal presence of at least 1 of the following features:
      • Horizontal growth along the muscularis mucosae
      • Dilation of the crypt base (basal third of the crypt)
      • Serrations extending into the crypt base
      • Asymmetrical proliferation
    • Boot shaped crypts
    • Branching of crypts is not sufficient for diagnosis
    • Size, location and endoscopic appearance alone should not be used to make the diagnosis of sessile serrated lesions but can be helpful when diagnostic features are ambiguous or in tangential sectioned specimen
      • Deeper levels may be helpful for poorly oriented specimen
    • Potential pitfall: mucosal herniation by smooth muscle fibers
      • Mucosal herniation often associated with lipomatous areas or lymphoid aggregates
  • Microvesicular hyperplastic polyps and sessile serrated lesions can be associated with stromal proliferation resembling perineural cells (Am J Surg Pathol 2011;35:1373, Ann Diagn Pathol 2018;35:48)
  • Sessile serrated lesions with dysplasia:
    • 2 - 5% of all sessile serrated lesions
    • Dysplastic areas are usually sharply demarcated from the nondysplastic areas (abrupt transition)
    • Heterogenous morphology (multiple dysplastic patterns) within single lesion
    • Architectural changes including:
      • Villous architecture
      • Crypt elongation
      • Crowding of crypts with complex branching
      • Cribriforming
      • Excessive or reduced luminal serration
    • Cytological changes including:
      • Intestinal dysplasia, mimicking tubular adenoma
      • Prominent nucleoli
      • Eosinophilic cytoplasm
      • Increased mitoses
    • 4 dysplasia patterns described in sessile serrated lesions (Mod Pathol 2017;30:1728):
      • Dysplasia, not otherwise specified:
        • Most common pattern
        • Easily identifiable architectural changes and obvious cytologic atypia
        • Frequently associated with MLH1 loss
      • Minimal deviation:
        • Subtle architectural changes (crypt crowding)
        • Cells with hypermucinous cytoplasm or gastric type change
        • Mild hyperchromasia
        • Mitotic figures present at higher part of the crypts
        • Requires MLH1 loss for diagnosis
      • Serrated dysplasia:
        • Dense eosinophilic appearance at low magnification
        • Closely packed small glands with reduced serration and cribriforming
        • Marked nuclear atypia
        • Rarely associated with MLH1 loss (retained MLH1 expression)
      • Adenomatous dysplasia: similar appearance to conventional adenomas; rarely associated with MLH1 loss
    • Multiple patterns are often found in a single lesion
    • Loss of MLH1 expression is not specific for sessile serrated lesions with dysplasia but can help when dysplastic features are subtle
    • Grading of dysplasia is not recommended because of low reproducibility, high heterogeneity of the lesion and lack of correlation of MLH1 expression loss (Mod Pathol 2017;30:1728, Histopathology 2015;66:49)
  • Traditional serrated adenomas:
    • 3 major morphologic features (Am J Surg Pathol 1990;14:524):
      • Slit-like serration (mimicking small intestinal mucosa)
      • Tall columnar cells with intense eosinophilic cytoplasm and bland elongated (pencillate) nuclei
      • Ectopic crypt formations
        • Epithelial buds not connected to the muscularis mucosae
        • Can have mild atypia; often with cell crowding and frequent mitoses
        • Always noted in polypoid traditional serrated adenomas; less common in flat traditional serrated adenomas
      • Diagnosis: ≥ 2 major morphologic features present in ≥ 50% of the polyp
      • Other common histological features: tubulovillous architecture, intraepithelial lymphocytes (Am J Surg Pathol 2008;32:21)
      • Often adjacent / associated with a precursor polyp (hyperplastic polyps or sessile serrated lesions)
      • Variants:
        • Goblet cell rich / mucinous traditional serrated adenomas: traditional serrated adenomas with a lot of goblet cells
        • Filiform traditional serrated adenomas: traditional serrated adenomas with bulbous and edematous tips of villi
  • Traditional serrated adenomas with overt dysplasia:
    • Intestinal type dysplasia: resembling dysplasia seen in conventional adenoma
    • Serrated type dysplasia: resembling serrated type dysplasia seen in sessile serrated lesions
    • No specific surveillance guidelines; however, traditional serrated adenomas with high grade dysplasia considered worrisome (often with molecular abnormalities), which should be reported and necessitate closer surveillance
  • Serrated adenoma, unclassified:
    • Serrated polyp with dysplasia and morphologically cannot fit into traditional serrated adenomas, sessile serrated lesions or conventional adenoma criteria
    • Serrated tubulovillous adenoma
    • Molecular features intermediate between traditional serrated adenomas and tubulovillous adenoma (TVA) (Histopathology 2018;73:444, Histopathology 2016;68:578)
Microscopic (histologic) images

Contributed by Chien-Kuang Cornelia Ding, M.D., Ph.D., Kwun Wah Wen, M.D., Ph.D., Raul S. Gonzalez, M.D. and Enoch Kuo, M.D.

Microvesicular hyperplastic polyp

Sessile serrated lesion

Sessile serrated lesion with adenomatous dysplasia

TSAs with pink, tall columnar cells

TSAs with ectopic crypt formation


Traditional serrated adenoma

Traditional serrated adenoma


Sessile serrated adenoma

MLH1 stain of SSL with dysplasia

Virtual slides

Images hosted on other servers:

Appendix, sessile serrated adenoma

Colon, traditional sessile serrated adenoma

Negative stains
  • MLH1 stain (Mod Pathol 2017;30:1728)
    • Although not specific, MLH1 expression loss can support the diagnosis of sessile serrated lesions with dysplasia
    • Recommended in:
      • Equivocal cytological atypia with inflammation, mucosal prolapse or cross section of crypt bases
      • Fragmented specimen with separate fragment of epithelial dysplasia from sessile serrated lesions
      • Sessile serrated lesions with morphology features support minimal deviation dysplasia (MLH1 loss is required for diagnosis)
Molecular / cytogenetics description
  • Molecular / cytogenetic testing is not required for establishing the diagnosis; however, the following tests could be helpful based on clinical settings:
    • DNA sequencing to determine:
      • BRAF status
      • KRAS status
      • Activation mutation of genes in WNT pathway
      • TP53 status
    • CpG methylation analysis
      • To determine if the tumor has CpG island methylator phenotype
      • Associated with MLH1 methylation and microsatellite instability
    • Microsatellite instability (MSI) test / DNA mismatch repair
      • MSI high (deficient mismatch repair)
      • MSI low (cannot be determined)
      • MSI stable (intact mismatch repair)
Videos

The problematic colorectal polyp by Dr. Rish K. Pai, on the serrated pathway

Sample pathology report
  • Transverse colon, polyp, endoscopic mucosal resection:
    • Sessile serrated lesion with dysplasia (see comment)
    • Comment: Immunohistochemical stain for MLH1 was performed and is intact. This does not exclude the diagnosis of dysplasia within sessile serrated lesion.
  • Sigmoid / rectum colon, polyps, biopsy:
    • Fragments of serrated polyp (see comment)
    • Comment: Sections show fragments of serrated polyp. Given the orientation of the specimen, the base of the serrated polyp is not well visualized to distinguish a hyperplastic polyp from sessile serrated lesion; however, the lack of lumina dilation and some narrow crypts favor a hyperplastic polyp. Level sections were performed to confirm the diagnosis.
Differential diagnosis
  • Sessile serrated lesion versus Microvesicular hyperplastic polyps:
    • Current recommendation: a polyp with single classical sessile serrated polyp type crypt is sufficient for a diagnosis of sessile serrated lesions over hyperplastic polyps (Am J Surg Pathol 2014;38:158, Am J Gastroenterol 2012;107:1315)
    • Deeper levels are often helpful
    • Size, location and endoscopic appearance are not part of the criteria for pathological diagnosis of hyperplastic polyps versus sessile serrated lesions
      • However, hyperplastic polyps are rarely > 10 mm
      • Presumption that hyperplastic polyps rarely exist in the proximal (right) colon is controversial
    • Tubular adenoma / tubulovillous adenoma
    • Reactive colonic mucosa
Board review style question #1

What is the most reliable morphologic feature for colorectal sessile serrated lesions?

  1. Architecture distortion of crypt base
  2. Branching of crypts
  3. Cytological atypia
  4. Proximal or distal location of the lesion
  5. Serrations of surface epithelium
Board review style answer #1
A. Architecture distortion of crypt base is the most reliable morphologic feature to diagnose colorectal sessile serrated lesions. Branching of crypts alone is not diagnostic for sessile serrated lesions. Cytological atypia does not always present in the sessile serrated lesions. Location is not part of the pathology diagnostic criteria of sessile serrated lesions. Serrations of surface epithelium can be seen in hyperplastic polyps.

Comment Here

Reference: Serrated lesions
Board review style question #2

When a colorectal polypectomy specimen shows no dysplasia but with equivocal features for either hyperplastic polyps or sessile serrated lesions, what is the right thing to do?

  1. Choose to call hyperplastic polyps or sessile serrated lesions based solely on clinical information (size and location)
  2. Consider taking deeper sections to look for definite diagnostic features
  3. Report as serrated adenoma, unclassified
  4. Sign out as serrated lesion and mention equivocal between hyperplastic polyps and sessile serrated lesions without deeper level sections
Board review style answer #2
B. Deeper sections of the tissue can often clarify some equivocal features and may demonstrate more definite diagnostic features, such as basally dilated crypts. Note in answer C, the diagnosis of serrated adenoma, unclassified is only reserved for serrated lesions with dysplasia and cannot fit into other diagnosis entities. Size or location alone should not be used to determine the diagnosis of hyperplastic polyps or sessile serrated lesions.

Comment Here

Reference: Serrated lesions
Board review style question #3
In colorectal sessile serrated lesions, which of the following dysplastic patterns requires loss of MLH1 for diagnosis?

  1. Adenomatous dysplasia
  2. Dysplasia, NOS
  3. Minimal deviation
  4. Serrated dysplasia
Board review style answer #3
C. Minimal deviation

Comment Here

Reference: Serrated lesions

Serrated pathway
Definition / general
  • CpG islands (also called CG islands, p indicates a phosphodiester bond connecting the C and G) are 500 to 2000 base pair regions rich in cytosine-guanosine dinucleotide repeats, present at 5' region in 50% of human genes (Wikipedia, Web-books)
  • Methylation of cytosine residues in promoter regions and proximal exons of these islands represses transcription
  • Hypermethylation of the CpG dinucleotide in the promoter regions of TP16 and CDH1 tumor suppressor genes is the mechanism to inactivate these genes
  • Promoter methylation of MLH1 may also occur
Terminology
  • Also called epigenetic changes / epigenetic silencing
Epidemiology
  • 30% of carcinomas have CpG island methylator phenotype (CIMP+)
  • Patients are usually female, have more advanced age, and have smoking history
Sites
  • Occurs predominantly in proximal colon
Etiology
  • An important mechanism of inactivating tumor suppressor genes in the chromosomal and microsatellite instability pathways (Curr Mol Med 2006;6:401, Cancer Metastasis Rev 2004;23:29)
  • However, CpG methylation can also act independently of chromosomal instability and microsatellite instability
Diagrams / tables

Images hosted on other servers:

DNA methylation

Prognostic factors
  • Although controversial, recent studies suggest that CIMP status is an independent predictor of a low colon cancer specific mortality and survival advantage and a high response rate to 5-fluorouracil based treatments (Gut 2009;58:90, Int J Clin Oncol 2008;13:498), although further studies are required

Serrated polyposis
Definition / general
  • Polyposis syndrome defined by the development of numerous sessile serrated polyps in the colon
Essential features
  • WHO diagnostic criteria: (a) at least 5 serrated polyps proximal to the rectum, all ≥ 5 mm in size, with 2 or more of these being ≥ 10 mm; or (b) > 20 serrated polyps of any size, distributed throughout the colon, with at least 5 proximal to the rectum (Gastroenterology 2020;158:1520)
  • Significantly increased risk for colorectal carcinoma and possibly extracolonic malignancies (Gut 2010;59:1094, Dis Colon Rectum 2011;54:164)
  • Family members also at increased risk for colorectal malignancies, suggesting an inherited component (Am J Gastroenterol 2012;107:770)
  • Likely represents a heterogeneous group of patients that includes several phenotypes of serrated polyposis
Terminology
  • Previously referred to as hyperplastic polyposis syndrome, although serrated polyposis is current preferred term by WHO
Sites
Pathophysiology
  • Likely consists of at least 2 groups: (type 1) patients with BRAF mutations and relatively few large right sided polyps and (type 2) patients with KRAS mutations and many small left sided polyps (J Pathol 2007;212:378)
  • Awaiting more definitive molecular genetic studies
Diagrams / tables

Images hosted on other servers:

Diagnosis and management of serrated polyposis

Clinical features
  • Mean age at diagnosis is 55 years, with overall equal distribution in males and females
  • Type 1 patients (with BRAF mutations) typically female smokers (United European Gastroenterol J 2016;4:305)
  • Typically asymptomatic and encountered on screening colonoscopy, although larger polyps may bleed
  • Most cases appear de novo, although a few familial cases have been described
Treatment
Gross images

Contributed by Raul S. Gonzalez, M.D.

Serrated polyposis

Microscopic (histologic) description
  • Lesions typically consist of sessile serrated adenomas / polyps that may have cytologic dysplasia
  • Hyperplastic polyps, typically of the microvesicular type, are also encountered and may be large
  • Conventional adenomas are occasionally present and may represent sessile serrated adenomas / polyps with cytologic dysplasia
Microscopic (histologic) images

Images hosted on other servers:

Sessile serrated
adenoma without
dysplasia (a) and with
low grade dysplasia (b)

Molecular / cytogenetics description
  • In addition to BRAF and KRAS mutations (as above), RNF43 has recently been implicated in sporadic and familial sessile serrated polyps (Gut 2017;66:1645)
Sample pathology report
  • Colon, total colectomy:
    • Colon with approximately two dozen sessile serrated lesions, some with focal cytologic dysplasia (see comment)
    • Negative for malignancy.
    • Sessile serrated lesion focally present at proximal resection margin; distal resection margin unremarkable.
    • Fourteen benign lymph nodes.
    • Comment: The findings are consistent with the patient's reported history of serrated polyposis. All grossly identifiable polyps were submitted for microscopy.
Differential diagnosis
Board review style question #1
Which of the following is true about serrated polyposis syndrome?

  1. It has been linked to MLH1 germline mutations
  2. Patients only develop sessile serrated adenomas
  3. Patients usually present with lower gastrointestinal bleeding
  4. There appear to exist 2 distinct subgroups
Board review style answer #1
D. There appear to exist 2 distinct subgroups

Comment Here

Reference: Serrated polyposis

Sessile serrated adenoma
Definition / general
  • Serrated neoplastic precursor lesion of colorectal cancer (Am J Gastroenterol 2012;107:1315)
  • Defined as having 2 or 3 contiguous crypts demonstrating features of sessile serrated adenoma, as discussed below (per Bosman: WHO Classification of Tumours of the Digestive System, 4th Edition, 2010)
  • Criteria for serrated polyposis syndrome include 1 of the following:
    • At least 5 serrated polyps proximal to the sigmoid colon with at least 2 greater than 1 cm in size
    • Any serrated polyp proximal to the sigmoid colon in a patient with a first degree relative with serrated polyposis syndrome
    • More than 20 serrated polyps of any size in the colon
Essential features
  • Neoplastic, premalignant lesion of the colorectum
  • If present, patient should undergo increased colonoscopy surveillance
  • May develop traditional cytologic dysplasia and progress to colorectal carcinoma with microsatellite instability
Terminology
  • Current official WHO designation is sessile serrated lesion; former WHO designation was sessile serrated adenoma / polyp
  • Examples that develop adenomatous epithelium are termed sessile serrated adenoma with cytologic dysplasia (Gastrointest Endosc 2014;80:307)
ICD coding
  • ICD-10: K63.5 - Colon polyp
Epidemiology
Sites
Etiology
Clinical features
  • Incidental findings on colonoscopy and have no clinical features unless they progress to malignancy
Diagnosis
Radiology description
Prognostic factors
Treatment
Gross description
Gross images

Images hosted on other servers:

Sessile serrated adenoma on colonoscopy

Sessile serrated
adenoma before
and after endoscopic
resection

Microscopic (histologic) description
  • Sawtooth serrations of the epithelium with abundant mucin, similar to hyperplastic polyps
  • Basal crypt dilation with mucous retention
  • Bases of the crypts are more serrated compared to the surface and have mature goblet cells and mucinous cells (Am J Gastroenterol 2012;107:1315)
  • Lateral spread of the crypt bases (commonly described as boot shaped or anchor shaped crypts)
  • May contain fibroblastic / perineuriomatous stroma (Am J Surg Pathol 2011;35:1373)
  • Conventional ("adenomatous") dysplasia may also be present, with retention of MLH1 staining; no need to grade per WHO
  • Nonconventional forms of dysplasia have also been described, including minimal deviation type (mild architectural and histologic changes, difficult to see on H&E but shows loss of MLH1), serrated type (tightly packed glands with rounded dysplastic nuclei and eosinophilic cytoplasm; MLH1 retained) and not otherwise specified (most common pattern; atypical changes not fitting into any other pattern; often MLH1 loss) (Mod Pathol 2017;30:1728, Mod Pathol 2019;32:1390)
Microscopic (histologic) images

Contributed by Monika Vyas, M.D. and Christopher Hartley, M.D.

SSA with dysplasia

MLH1 loss

Sessile serrated adenoma with adenomatous dysplasia



Contributed by Raul S. Gonzalez, M.D.

Sessile serrated adenoma / polyp


Contributed by @RaulSGonzalezMD on Twitter
Sessile serrated adenoma Sessile serrated adenoma

Sessile serrated adenoma



Images hosted on other servers:

Sessile serrated adenoma / polyp

Ascending colon


Sessile serrated adenoma


Cecum

Positive stains
Negative stains
Molecular / cytogenetics description
Sample pathology report
  • Ascending colon, polyp, endoscopic mucosal resection:
    • Sessile serrated lesion
    • Lateral margins of resection unremarkable.
Differential diagnosis
Additional references
Board review style question #1
What mutations can be seen in sessile serrated adenoma?

  1. Beta catenin and KRAS
  2. BRAF and beta catenin
  3. BRAF and CpG island methylator phenotype (CIMP)
  4. KRAS and CpG island methylator phenotype (CIMP)
Board review style answer #1
C. BRAF and CpG island methylator phenotype (CIMP). KRAS and beta catenin mutations are not associated with sessile serrated adenomas and are instead seen in traditional serrated adenomas and tubular adenomas.

Comment here

Reference: Sessile serrated adenoma

Sexually transmitted infectious colitis / proctitis
Syphilis
Definition / general
  • Disease of the colon caused by the spirochete Treponema pallidum

Essential features
  • With rare exceptions, syphilis is acquired by direct contact with an active syphilitic lesion during sex; anorectal disease is acquired secondary to receptive anal intercourse
  • The clinical manifestations occur after a variable incubation period
  • The primary stage is usually 1 or more nonpainful chancres associated with regional adenopathy
  • In this setting, secondary syphilis follows and may be associated with proctitis or condylomata lata
  • The secondary stage is followed by an asymptomatic latent stage lasting months to years
  • In the absence of treatment or if the patient is unable to clear the infection, gummas may involve the anorectum or colon as part of late or tertiary syphilis

Epidemiology
  • Worldwide, an estimated 12 million new cases of syphilis occur each year
  • In the United States in 2014, 63,450 new cases were reported to the CDC
  • There has been a substantial increase in the number of syphilis cases in the United States over the last 15 years, mainly in MSM; many are HIV+
  • The overwhelming majority of cases are acquired sexually, although there are reports of acquiring the disease through kissing or other close contact
    • Sexual transmission is most likely to occur early in the course of the disease during the primary and secondary stages; typically not transmissible by immunocompetent individuals after 4+ years of infection
    • Condoms protect against infection if contact with areas of active infection are avoided
  • Transplacental infection leads to congenital syphilis
  • Infections transmitted through blood transfusion and accidental inoculation have occurred
  • Two - thirds eventually clear their infection without treatment, although they may infect others

Clinical features
  • Spirochetes penetrate mucosa or damaged skin and enter lymphovascular spaces and disseminate
  • In primary syphilis, chancres occur 3 to 90 days (mean 21 days) after contact with an active lesion
    • Chancres are a superficial, nontender ulceration, associated with regional nonpainful lymphadenopathy
    • Have a smooth base and raised edges
    • They may be single or multiple and up to 2 cm
    • Multiple lesions are more common if HIV+
    • May be inconspicuous or absent, particularly in patients with a prior history of syphilis
    • If inoculum is small, only a bump may develop
    • May become secondarily infected, particularly if oral or anal
    • Many patients have mild proctitis and regional lymphadenopathy
    • Chancres typically heal spontaneously in 3 to 6 weeks, but also as quickly as 1 week or as slowly as 12 weeks
    • Chancres increase the risk of acquiring HIV and other sexually transmitted diseases
    • Anorectal chancres may resemble herpes simplex virus infection, chancroid, condyloma acuminatum, lymphogranuloma venereum, typical and atypical mycobacterial infection, anal fissures, anal fistulas, harmless bumps, ingrown hairs, abrasions and mucosal prolapse syndrome / solitary rectal ulcer
      • They are usually painless and asymptomatic, but some patients may experience pain especially during defecation or constipation
      • Discharge may be present
  • Secondary stage:
    • Secondary or disseminated syphilis develops 2 to 8 weeks after exposure, with myriad possible symptoms that usually includes mucocutaneous lesions and lymphadenopathy
    • Two - thirds spontaneous clear their infections
    • The inability of some patients to clear their infection is poorly understood, but likely involves the relatively small number of proteins and lipoproteins on the outer membrane of the spirochete and the change from a cellular (Th1) to a humoral (Th2) immune response on the part of the host
    • Usually the chancre is still present but not always
    • Most common symptoms are a rash, variable macular to papular to pustular, that commonly involves the palms and soles; also diffuse lymphadenopathy, fever, lethargy, anorexia, weight loss, pharyngitis, laryngitis and arthralgia
    • Condylomata lata, described as a condylomatous lesion that is moist, raised, smooth, itchy, mucin secreting and often malodorous, may occur
    • In secondary anorectal syphilis, proctitis is common, condyloma lata may occur and inguinal adenopathy is common
    • Other less common manifestations of secondary syphilis include mucous patches, erosions and aphthous ulcers of the mouth and throat, headache, meningismus or meningitis, diplopia, impaired vision, tinnitus, vertigo and involvement of cranial nerves, liver or kidneys
  • Tertiary / latent stage:
    • If disease progresses, the secondary stage is followed by an asymptomatic latent stage that may last for years
    • Occurs 5 to 30 years after initial infection
    • Asymptomatic and only detectable by laboratory studies
    • May progress to primarily affect the ascending aorta and CNS but gummas (soft, noncancerous growth, which is a form of granuloma) may be found in many locations including the anorectum
    • Usually subdivided into neurosyphilis, cardiovascular syphilis, gummatous syphilis and syphilitic (or luetic) osteitis
      • Gummatous syphilis, also known as late benign syphilis, may occur anywhere but most commonly in bone, skin and mucocutaneous tissue; rarely seen in contemporary medical practice

    Diagnosis
    • Since T. pallidum has never been successfully cultured on artificial media, diagnosis depends on (a) direct visualization of the organism in tissue with immunohistochemistry, immunofluorescence or Warthin-Starry stain, (b) dark field examination demonstrating spirochetes from an active lesion, (c) PCR testing or (d) serologic testing
    • In anorectal biopsies, the histologic findings are nonspecific; appropriate clinical history and a high index of suspicion are helpful for the pathologist

    Laboratory
    • In addition to dark field microscopy, the traditional diagnosis has been serologic testing, with a positive initial screening test for cross reacting nonspecific nontreponemal antibodies (either rapid plasma reagin (RPR) or the venereal disease research laboratory (VDRL) test ), followed by a confirmatory test for specific treponemal antibodies (either fluorescent treponemal antibody (FTA-ABS) or treponema pallidum particle agglutination assay (TPPA))
    • Recently, automated immunoassays (known as the syphilis IgG test, multiplex IgG test or T. pallidum IgG test) have been introduced to detect a specific IgG antibody to syphilis
      • It is now recommended that this be the initial screening test for syphilis and that RPR be the confirmatory test (called reverse sequence syphilis screening)
      • TPPA or FTA-ABS may be also be useful when there was prior treatment of syphilis, low titer RPR, suspicion of a false positive or a strong clinical suspicion of disease
    • PCR testing for syphilis is now available in some laboratories - material obtained from a chancre or other visible lesion as well as peripheral blood may be studied
    • Dark field microscopy of material from an active lesion is available in a limited number of laboratories

    Prognostic factors
    • Disease is generally more severe in immunocompromised patients; these patients also are less likely to clear infection and more likely to relapse during the latent stage

    Case reports

    Treatment
    • Parentally administered benzathine penicillin G (Bicillin LA) is the mainstay to treat all stages of syphilis
    • Treatment varies depending on the stage
    • For penicillin allergy, use doxycycline, tetracycline or azithromycin, but azithromycin should not be used in MSM, HIV+ patients or pregnant women
    • Sex partners should be treated

    Clinical images

    Images hosted on other servers:

    Rectal chancres located
    on posterior wall of rectum
    and regression



    Microscopic (histologic) description
    • Chancre: has a prominent mononuclear cell infiltrate dominated by plasma cells with scattered histiocytes; also obliterative endarteritis with thickened small vessels due to proliferation of endothelial cells and fibroblasts
    • Syphilis proctitis: has a dense mononuclear cell infiltrate with conspicuous plasma cells, often with cryptitis and crypt abscesses; granulomas (usually small) and obliterative endarteritis may be present; plasma cells may be less prominent
    • Condyloma lata: resembles condyloma acuminatum, with prominent epidermal hyperplasia, more edema, chronic inflammatory infiltrate, but less keratinization, minimal if any koilocytosis
    • Gummas: have caseous necrosis, chronic inflammatory cell infiltrate, obliterative endarteritis
    • Healed gummas: become noncontractile scars with arching fibrosis

    Microscopic (histologic) images

    Contributed by Rhonda Yantiss, M.D. and Raul Gonzalez, M.D.

    H&E: anal condyloma lata

    T. pallidum IHC stain



    Images hosted on other servers:
    Missing Image

    Obliterative endarteritis
    with heavy plasma
    cell infiltration



    For more micro images, see Penis / scrotum chapter

    Positive stains

    Differential diagnosis

    Additional references
  • Gonorrhea
    [Pending]
    Herpes
    [Pending]
    Chlamydia
    [Pending]
    Lymphogranuloma venereum
    [Pending]

    Shigella
    Definition / general
    • Infectious diarrhea and subsequent colitis caused by bacteria of the Shigella genus
    • Shigella species are virulent, invasive, nonmotile, noncapsulated, facultatively anaerobic, gram negative rods (StatPearls: Shigella [Accessed 20 February 2023])
    • 4 groups:
      • S. dysenteriae
      • S. flexneri
      • S. boydii
      • S. sonnei
    Essential features
    • Shigella are highly infective, virulent, invasive, gram negative rods
    • Causes watery diarrhea associated with fever and abdominal pain, which may progress to dysentery (severe diarrhea with blood or mucus in stool) (Clin Lab Med 2015;35:225)
    • Shigellosis is very common especially in the developing world and spreads via fecal oral route
    • Shiga toxin released by S. dysenteriae is associated with more severe disease and may be complicated by hemolytic uremic syndrome
    • Diagnosis is based on microbiological studies (culture and PCR); biopsy is not routinely performed
    • Infection is self limited in most cases but can be severe in children and immunocompromised
    • Antibiotic therapy is appropriate for moderate to severe disease or certain patient populations (StatPearls: Shigella [Accessed 20 February 2023])
    • Antibiotic resistance is a growing problem (Clin Lab Med 2015;35:225)
    Terminology
    • Shigellosis and related terms are named after Dr. Kiyoshi Shiga, a Japanese physician and microbiologist who discovered the bacterium in 1897 and made contributions to understanding the disease (EcoSal Plus 2018;8:10.1128)
    • Member of the Enterobacterales order and Enterobacteriaceae family and is closely related to and causes similar disease to enteroinvasive Escherichia coli
    ICD coding
    • ICD-10:
      • A03.0 - shigellosis due to Shigella dysenteriae
      • A03.1 - shigellosis due to Shigella flexneri
      • A03.2 - shigellosis due to Shigella boydii
      • A03.3 - shigellosis due to Shigella sonnei
      • A03.8 - other shigellosis
      • A03.9 - shigellosis, unspecified
    Epidemiology
    • According to 2010 data, an estimated 188 million cases occur worldwide annually (Lancet 2018;391:801)
    • Shigellosis is a leading cause of diarrheal death, with most deaths due to Shigella infection occurring in south Asia and sub-Saharan Africa
    • Infection occurs from ingestion of bacteria through fecal contamination of food or water and is linked to poor hygiene and sanitation, overcrowding and lack of access to clean water
    • The majority of cases occur in developing countries by serotype S. flexneri
    • The United States has an incidence of ~450,000 cases annually, most by S. sonnei (StatPearls: Shigella [Accessed 20 February 2023])
    • Infection in the U.S. often results from exposure / outbreak at daycare centers or institutional housing, is due to food borne transmission or in men who have sex with men
    • Shigella is acquiring resistance to antimicrobial therapy
    Sites
    • Colon
    Pathophysiology
    • Shigella are resistant to gastric acid, allowing them to pass through the stomach into the intestine after ingestion
    • Shigella bacterial cells invade M cells that are important for immune activation, are phagocytosed by macrophages which subsequently undergo apoptosis and lead to inflammation
    • Once bacteria transverse the gut epithelium, they can invade enteric epithelial cells from the basolateral side and by spreading between adjacent cells (Curr Trop Med Rep 2014;1:81)
    • Shigella can avoid immune recognition and destruction by escaping the phagocytic vacuole and replicating in the cytosol and by modifying its lipopolysaccharide structure
    • Shigella can produce toxins that lead to enterocyte cell death and increased production of inflammatory cytokines and chemokines: Shigella enterotoxin 1 (ShET1), Shigella enterotoxin 2 (ShET2) and Shiga toxin 1 (stx1) (Curr Trop Med Rep 2014;1:81)
    • Not all Shigella isolates produce Shiga toxin
    • Inflammation exacerbates tissue destruction
    • Less commonly, shigellosis can advance to extraintestinal disease, such as with S. dysenteriae serotype 1
    • Hemolytic uremic syndrome can occur in patients infected with Shiga toxin producing Shigella strains, leading to hemolytic anemia, thrombocytopenia and acute kidney injury (J Health Popul Nutr 2012;30:257)
    Etiology
    Clinical features
    • Most patients suffer a self limited diarrhea associated with fever and abdominal pain
    • Fever, nausea, vomiting, tenesmus
    • Fractional stools (an increase in the number of stools with smaller volumes)
    • May progress to bloody / mucoid diarrhea (dysentery)
    • Without treatment, gastroenteritis lasts ~7 days in an immunocompetent patient
    • Shedding may occur for up to 6 weeks (Mandell: Principles and Practice of Infectious Diseases, 6th Edition, 2004)
    • Incubation period is 1 - 4 days (Clin Lab Med 2015;35:225)
    • Complications include sepsis, perforation, toxic megacolon, hepatic dysfunction and toxic encephalopathy due to brain edema (Ekiri syndrome) (J Microbiol Immunol Infect 2012;45:147)
    Diagnosis
    • Traditionally, diagnosis was based on stool culture but cultures may be falsely negative as microorganisms are fastidious
    • Culture is still useful to rule out other pathogens and for susceptibility testing
    • Stool PCR panels for multiple organisms are faster and more sensitive than culture
    • Endoscopy and biopsy are rarely performed (Pathologe 2022;43:16)
    Prognostic factors
    Case reports
    Treatment
    • Disease may be self limited
    • Oral or intravenous hydration and electrolyte management are essential to replace losses
    • Antidiarrheal medication is contraindicated as it delays clearance of microorganisms and prolongs symptoms (StatPearls: Shigella [Accessed 20 February 2023])
    • Empiric fluoroquinolone is an acceptable therapy for adults and empiric azithromycin is acceptable for children (StatPearls: Shigella [Accessed 20 February 2023])
    • Local antibiogram should be consulted prior to initiation of empiric therapy
    • Antimicrobial susceptibility testing should be performed if feasible due to rising resistance
    • Other antibiotic choices include cephalosporins, trimethoprim / sulfamethoxazole, ampicillin and amoxicillin
    Gross description
    • Involved mucosa is edematous, hemorrhagic, inflamed and ulcerated depending on the severity of infection
    • Pseudomembranes may be present (Pathologe 2022;43:16)
    Microscopic (histologic) description
    • Biopsy is rarely performed
    • No specific histologic findings for Shigella versus other infectious bacterial colitides (StatPearls: Infectious Colitis [Accessed 20 February 2023])
    • Immunohistochemical staining for Shigella is not used
    • If performed, colon biopsy in the acute phase shows a neutrophilic inflammatory infiltrate with cryptitis and crypt abscesses similar to active inflammatory bowel disease (Virchows Arch 2018;472:135)
    • In the later phases of infection, there is an increase in other inflammatory cells, such as plasma cells and eosinophils (Pathologe 2022;43:16)
    • Features of chronicity such as significant crypt architectural distortion and Paneth cell metaplasia are absent
    Microscopic (histologic) images

    Contributed by Janina Markidan, M.D.
    Erosive colitis with inflammation

    Erosive colitis with inflammation



    Images hosted on other servers:
    Missing Image

    Bacterial dysentery

    Sample pathology report
    • Colon, rectosigmoid, biopsy:
      • Colon with pseudomembranous formation and active inflammation including cryptitis and crypt abscess (see comment)
      • Comment: Features of chronicity are not seen. Negative for granuloma or viral cytopathic effects. The histologic findings are most in keeping with an infectious colitis. Inflammatory bowel disease is less likely. Correlation with clinical history, microbiological and serologic findings is recommended.
    Differential diagnosis
    • Definitive diagnosis based on histopathology is not possible; correlation with history and presentation, microbiological, serological, radiographic or endoscopic findings is essential
    • Other infectious causes of colitis, including Escherichia coli, Campylobacter, nontyphoidal Salmonella, Clostridium difficile (Pathologe 2022;43:16)
    • Acute inflammatory bowel disease:
      • Granulomas are more specific to Crohn's disease; however, there is a large morphologic overlap
    • Drug induced colitis:
      • Many histologic patterns may overlap with Shigella colitis; however, increased intraepithelial apoptosis and the presence of pill fragments may be the clues to the diagnosis of drug induced colitis (Pathologica 2021;113:54)
      • Some small molecule inhibitors (ending in "ib" such as idelalisib) and monoclonal antibodies (ending in "mab" such as ipilimumab / nivolumab) may cause diarrhea, with a histologic triad of active colitis, intraepithelial lymphocytosis and crypt epithelial cell apoptosis (often popcorn type) (Am J Surg Pathol 2015;39:1661, Histopathology 2021;78:532)
    • Radiation enterocolitis:
      • In the acute radiation colitis phase, the inflammatory infiltrate is eosinophilic rather than neutrophilic, as expected with the acute phase of Shigella infection; however, there is a large morphologic overlap
      • Radiation atypia is seen only in radiation enterocolitis (Cancers (Basel) 2020;12:2571)
    • Ischemic colitis:
      • Features of acute ischemic colitis include superficial mucosal necrosis, crypt damage (withering) or loss and hyalinization of the lamina propria; acute cases may exhibit acute inflammatory exudate (pseudomembranes) on the mucosa, which can be indistinguishable based on histologic features (Pathologica 2021;113:54)
    Board review style question #1

    An otherwise healthy 5 year old girl presents to the emergency department with fever, abdominal pain and bloody diarrhea. She has felt unwell for the past 2 days and subsequently developed an initially watery but then bloody diarrhea. She has had many small volume bloody stools. History reveals that she attends a daycare where other children have experienced similar symptoms. Mucous membranes are dry, however, electrolytes are within normal limits. She is given an oral rehydration solution and is discharged. If biopsy were performed, random sections of colon would demonstrate the histology shown above. What is the most likely diagnosis?

    1. Active inflammatory bowel disease
    2. Cytomegalovirus (CMV) infection
    3. Giardiasis
    4. Pinworm infection
    5. Shigellosis
    Board review style answer #1
    E. Shigellosis. The patient presents with acute gastroenteritis that has progressed to dysentery. An infectious etiology is suspected given the history; therefore, active inflammatory bowel disease is unlikely. CMV colitis is unlikely in an otherwise healthy child and viral nuclear and cytoplasmic inclusions would be seen on histopathology. The history of daycare exposure would be consistent with shigellosis, pinworm infection / enterobiasis or giardiasis. Of the choices listed, the dysenteric symptoms are most consistent with shigellosis; enterobiasis presents with pruritis and giardiasis presents with watery diarrhea. Histologic sections of giardiasis show minimal inflammation and organisms present on the mucosal surface (Virchows Arch 2018;472:135).

    Comment Here

    Reference: Shigella
    Board review style question #2

    A 75 year old woman with relapsed refractory multiple myeloma undergoing stem cell transplant is seen in the emergency department presenting with fever, abdominal pain and bloody diarrhea. She is admitted and quickly develops hypotension, toxic megacolon and multiorgan failure. She is started on broad spectrum antibiotic therapy. A nucleic acid amplification test (NAAT) for the Clostridium difficile toxin gene is negative. Blood cultures are preliminarily positive for a gram negative bacillus. Stool cultures are pending. She is found to have an intestinal perforation on further imaging studies, then develops toxic encephalopathy. The decision is made to transition her to comfort care. An autopsy is performed and the findings above are seen in the colon. Which of the following is the most likely diagnosis?

    1. Clostridioides difficile infection
    2. CMV colitis
    3. Inflammatory bowel disease
    4. Ischemic colitis
    5. Shigellosis
    Board review style answer #2
    E. Shigellosis. The gross photograph shows the colonic mucosa covered in yellow-tan irregular plaques that coalesce in some areas. Pseudomembranous colitis is most commonly associated with Clostridioides difficile colitis. However, the nucleic acid amplification test for the C. difficile toxin gene is negative. In addition, the infection is rarely extraintestinal. Shigella associated colitis can form pseudomembranes and the fulminant disease exhibited in the case can be seen in immunocompromised patients. Ischemic colitis may also cause pseudomembranous colitis; however, the patient presented with infectious symptoms and sepsis. CMV pseudomembranous colitis is rare and has been reported; however, it would not fit with the blood culture positivity with gram negative rods (Dis Mon 2015;61:181). Inflammatory bowel disease also causes pseudomembranous colitis; however, it is unlikely in an older patient with the infectious clinical picture.

    Comment Here

    Reference: Shigella

    Signet ring cell carcinoma
    Definition / general
    • Rare subtype of colorectal carcinoma (< 1%) officially recognized by the WHO (Ann Surg 2013;258:775)
    • > 50% of the tumor must have signet ring cell morphology to be classified as signet ring cell carcinoma
    Essential features
    Terminology
    • Sometimes called linitis plastica but this typically refers to signet ring cell carcinoma of stomach
    Epidemiology
    Sites
    • Can occur anywhere in colon, though rectum is a less common location
    Etiology
    Clinical features
    Diagnosis
    Prognostic factors
    Case reports
    Clinical images

    Images hosted on other servers:

    Pedunculated polyp

    Stenotic rectal carcinoma

    Discolored lesion

    Gross description
    • May diffusely infiltrate colon wall, rather than forming an exophytic mass
    • Can rarely arise within a polyp
    Microscopic (histologic) description
    • Proliferation of signet ring cells with intracellular mucin that displaces nucleus to side
    • Pools of extracellular mucin often present as well
    • Diffuse tissue infiltration less common than in gastric signet ring cell carcinoma
    • May rarely be in situ or confined to a polyp (J Surg Res 2010;163:250)
    • No gland formation in signet ring cell component, although it may be present elsewhere in the same lesion
    Microscopic (histologic) images

    Contributed by Raul S. Gonzalez, M.D.

    Signet ring cell carcinoma

    Negative stains
    Molecular / cytogenetics description
    Sample pathology report
    • Ascending colon, resection:
      • Signet ring cell carcinoma, poorly differentiated (see synoptic report)
    Differential diagnosis
    Board review style question #1

      Colorectal carcinomas with signet ring cells may display which of the following features?

    1. Concurrent BRAF and KRAS mutations
    2. Inability to breach the basement membrane
    3. Loss of CDX2 by immunohistochemistry
    4. Microsatellite instability
    Board review style answer #1
    D. Microsatellite instability

    Comment Here

    Reference: Signet ring cell carcinoma

    Solitary rectal ulcer syndrome
    Definition / general
    • Solitary or multiple ulcerated or polypoid lesions 4 - 10 cm from anal margin
    Essential features
    • Not always solitary, not always rectal, not always ulcerated, not really a syndrome
    • Mucosal prolapse type change resulting in rectal lesions
    Terminology
    • Also called mucosal prolapse syndrome (may be a better term since not necessarily solitary, ulcerated or rectal)
    Epidemiology
    • Uncommon (incidence of 1 per 100,000 per year)
    • Usually third and fourth decade
    • More common in women
    • Rarely in children (Pediatrics 2002;110:e79)
    Sites
    • Usually in rectosigmoid colon
    Etiology
    • Abnormal function of anal and pelvic floor musculature during defecation, causing rectal mucosal prolapse or intussusception
    Clinical features
    Case reports
    • 45 year old man complaining of constipation and rectal bleeding is found to have an ulcerated rectal lesion on colonoscopy (Case of the Month #527)
    Treatment
    • High fiber diet, laxatives, topical steroids
    • Possibly resection
    Gross description
    • Well demarcated irregular ulcer(s) on rectal wall
    • Also polypoid, rough, erythematous lesions
    • Mucosal thickening
    Microscopic (histologic) description
    • Superficial mucosal ulceration and villiform change
    • Crypt hyperplasia and elongation with focal dilation (some glands diamond shaped)
    • Fibromuscular hyperplasia of lamina propria
    • Thickened muscularis mucosae with splayed fibers
    • Ectatic capillaries
    • Minimal inflammation
    • May have inflammatory pseudomembranes
    • Late changes resemble colitis cystica profunda
    Microscopic (histologic) images

    Contributed by Andrey Bychkov, M.D., Ph.D., Jian-Hua Qiao, M.D. and Raul S. Gonzalez, M.D. (Case #527)

    Polypoid shape

    Solitary rectal ulcer

    Crypt hyperplasia

    Disarrayed muscularis mucosa

    Haphazardly arranged benign colonic crypts

    Distorted diamond shaped glands


    Ulcerated, reactive mucosa Ulcerated, reactive mucosa

    Ulcerated, reactive mucosa

    Smooth muscle wisps

    Smooth muscle wisps

    Granulation tissue

    Granulation tissue

    Sample pathology report
    • Rectum, ulcer, biopsy:
      • Colonic mucosa with prolapse type change, mild acute inflammation and focal erosion (see comment)
      • Comment: The findings are compatible with so called solitary rectal ulcer syndrome.
    Differential diagnosis
    Board review style question #1

    Solitary rectal ulcer syndrome most frequently occurs in what patient population?

    1. Elderly men
    2. Female infants
    3. Middle aged women
    4. Young adult men
    Board review style answer #1
    C. Middle aged women

    Comment Here

    Reference: Solitary rectal ulcer syndrome
    Board review style question #2
    Which of the following is true about solitary rectal ulcer syndrome?

    1. Patients demonstrate typical associated systemic symptoms
    2. Prolapse of intestinal mucosa is specific to this disease entity
    3. Roughly 30% of patients have multiple lesions
    4. The diagnosis cannot be made without microscopic ulceration
    Board review style answer #2
    C. Roughly 30% of patients have multiple lesions. Despite the name, "solitary rectal ulcer syndrome" is not always a solitary finding. Approximately 30% of patients will demonstrate multiple rectal ulcers on colonoscopy. Answer A is incorrect because solitary rectal ulcer syndrome is not a systemic syndrome, meaning patients will not have associated systemic symptoms. Answer B is incorrect because prolapse of intestinal mucosa can be seen histologically in other diseases, such as diverticulosis. Answer D is incorrect because microscopic ulceration is often but not always encountered, meaning it is not required to establish the diagnosis.

    Comment Here

    Reference: Solitary rectal ulcer syndrome

    Squamous cell carcinoma
    Definition / general
    Essential features
    Sites
    Etiology
    Clinical features
    Prognostic factors
    Case reports
    Treatment
    Gross images

    Images hosted on other servers:

    Rectal squamous cell carcinoma

    Microscopic (histologic) description
    • Similar to squamous cell carcinoma in other organs (polygonal eosinophilic cells with intercellular bridges and keratin formation)
    • Can be poorly differentiated
    • May be basaloid or acantholytic
    Microscopic (histologic) images

    Contributed by Raul S. Gonzalez, M.D.
    Squamous cell carcinoma Squamous cell carcinoma

    Squamous cell carcinoma

    Positive stains
    Sample pathology report
    • Ascending colon, resection:
      • Squamous cell carcinoma of the colon (see synoptic report and comment)
      • Comment: An immunohistochemical stain for p40 is diffusely positive.
    Differential diagnosis
    Board review style question #1

      Colonic squamous cell carcinoma can arise in the setting of

    1. Colonic epidermal inclusion cysts
    2. Diverticulitis
    3. HPV infection
    4. Schistosomiasis
    Board review style answer #1
    D. Schistosomiasis

    Comment Here

    Reference: Squamous cell carcinoma

    Staging-carcinoma
    Definition / general
    • Adenocarcinoma, poorly differentiated, high grade neuroendocrine carcinoma and squamous cell carcinomas of the colon and rectum are covered by this staging system
    • These topics are not covered: appendiceal carcinoma, anal carcinoma and well differentiated neuroendocrine tumor
    Essential features
    • AJCC 7th edition staging was sunset on December 31, 2017; as of January 1, 2018, use of the 8th edition is mandatory
    ICD coding
    • ICD-10
      • C18.9 - malignant neoplasm of colon, unspecified
      • C19 - malignant neoplasm of rectosigmoid junction
      • C20 - malignant neoplasm of rectum
    Diagrams / tables

    Images hosted on other servers:
    Staging related diagrams from AJCC 7th edition (2006) Staging related diagrams from AJCC 7th edition (2006) Staging related diagrams from AJCC 7th edition (2006) Staging related diagrams from AJCC 7th edition (2006)

    Staging related diagrams from AJCC 7th edition (2006)

    Primary tumor (pT)
    • pTX: cannot be assessed
    • pT0: no evidence of primary tumor
    • pTis: carcinoma in situ, intramucosal carcinoma (involvement of lamina propria with no extension through muscularis mucosae)
    • pT1: tumor invades submucosa (through the muscularis mucosae but not into the muscularis propria)
    • pT2: tumor invades muscularis propria
    • pT3: tumor invades through the muscularis propria into the pericolorectal tissues
    • pT4:
      • T4a: tumor invades through the visceral peritoneum (including gross perforation of the bowel through tumor and continuous invasion of tumor through areas of inflammation to the surface of the visceral peritoneum)
      • T4b: tumor directly invades or adheres to other adjacent organs or structures

    Notes:
    • pTis
      • Tis (carcinoma in situ) refers to intramucosal carcinoma, which invades into the lamina propria and may involve but not penetrate through the muscularis mucosae (Ann Surg Oncol 2018;25:1454)
      • Intraepithelial carcinoma is synonymous with high grade dysplasia and should not be assigned to Tis, as these lesions lack potential for tumor spread (American Joint Committee on Cancer: AJCC Cancer Staging Manual, 8th edition, 2017)
      • Tumor extension through the muscularis mucosae into the submucosa is classified as pT1; a synoptic report is required for all cancers that are pT1 and beyond but not for pTis tumors
      • Tis includes intraepithelial carcinoma (tumor confined to epithelium by basement membrane) and intramucosal carcinoma (carcinoma invading lamina propria)
    • Carcinoma in polyps
    • pT4
      • If the tumor is grossly / macroscopically adherent to other organs, it is cT4; if no tumor is found microscopically within the adhesion, it should be best classified as pT3 with a note
      • Separation of T4 into 2 categories (T4a and T4b) is based on different outcomes in multiple datasets
      • T4a tumors directly invade the serosal surface (visceral peritoneum)
        • This includes tumors with perforation where the tumor cells are continuous with the serosal surface through inflammation and when tumor is present at the ink in serosal cleft
        • Even in the absence of readily demonstrable tumor cells on the serosal surface in histologic sections, grossly perforated cancers should be assigned pT4a stage (Surg Pathol Clin 2017;10:961)
        • Some but not all studies indicate that tumors that are < 1 mm from the serosal surface show a higher risk for peritoneal relapse
          • Multiple levels and additional sampling should be performed
          • In the absence of serosal surface involvement, the tumor should be considered pT3 (Mod Pathol 2021;34:131)
        • Utility of elastic stain to assess the peritoneal involvement is not universally accepted due to discontinuity of elastic lamina in colonic peritoneum (particularly absent in right colon) as well as difficulties in interpreting the stain (Mod Pathol 2015;28:S95)
        • pT4a should not be used in nonperitonealized portions of the colorectum (posterior aspects of ascending and descending colon, lower rectum)
      • T4b: transmural extension into another organ or site is a must for T4b designation; intramural extension of tumor from a segment of the large intestine into an adjacent subsite does not affect the pT classification
    Regional lymph nodes (pN)
    • pNX: cannot be assessed
    • pN0: no regional lymph node metastasis
    • pN1: metastasis in 1 - 3 regional lymph nodes
      • N1a: metastasis in 1 regional lymph node
      • N1b: metastasis in 2 - 3 regional lymph nodes
      • N1c: no regional lymph nodes are positive but there are tumor deposits in the subserosa, mesentery or nonperitonealized pericolic or perirectal / mesorectal tissues
    • pN2: metastasis in 4 or more regional lymph nodes
      • N2a: metastasis in 4 - 6 regional lymph nodes
      • N2b: metastasis in 7 or more regional lymph nodes

    Notes:
    • Minimum of 12 lymph nodes must be recovered for lymph node staging to be considered accurate in curative resections
    • Number of recovered nodes has been reported to correlate with better prognosis, likely due to more accurate staging or likely from implying better host response
    • N category is limited to only the regional nodes that are in the lymphatic drainage area of the tumor; metastasis to lymph nodes not found along vascular arcades of the marginal artery or pericolonic, perirectal or mesorectal nodes should be considered distant metastasis (M1a)
    • Lymph nodes with micrometastasis (0.2 - 2 mm) as well as macrometastasis (> 2 mm) are considered positive
    • Isolated tumor cells (single tumor cells or groups < 0.2 mm in maximum dimension) are classified as N0 as they were found to have no adverse prognosis (Eur J Surg Oncol 2014;40:263)
    • Tumor deposits are discrete tumor nodules of any shape, contour or size that lack associated lymphoid tissue, vascular structures or neural structures found within the lymph drainage area of the primary carcinoma
    • Only in the absence of unequivocal lymph node metastases, tumor deposits are recorded as N1c; in cases with lymph node metastasis, the number of tumor deposits is not added to the number of positive lymph nodes
    • There is no consensus opinion on assigning lymph nodes with acellular mucin pool deposits as positive (N1) or negative (N0) (Mod Pathol 2020;33:153)
    Distant metastasis (pM)
    • M0: no distant metastasis by imaging; no evidence of tumor in other sites or organs (this category is not assigned by pathologists)
    • M1: distant metastasis (Ann Surg Oncol 2018;25:1454)
      • M1a: metastasis confined to 1 organ or site without peritoneal metastasis
      • M1b: metastasis to 2 or more sites or organs is identified without peritoneal metastasis
      • M1c: metastasis to the peritoneal surface is identified alone or with other site or organ metastases

    Notes:
    • Metastasis to lymph nodes outside of the drainage area of the tumor (nonregional lymph nodes) should be considered distant metastasis (M1a)
    • Multiple metastases in an organ, even paired organs (ovaries, lungs), are still M1a disease
    • Pathologist should not assign the global designation pM0, as metastasis unknown to the pathologist may be present
    Prefixes
    • c: clinical (cTNM)
    • p: pathological (pTNM)
    • y: postneoadjuvant chemoradiation therapy (ycTNM or ypTNM)
    • r: recurrent tumor stage (rTNM)
    • a: cancer discovered incidentally during autopsy (aTNM)

    Notes:
    • Current clinical practice involves neoadjuvant chemoradiation for cT3 and cT4 rectal cancers
    • Microscopic evidence of residual disease has been shown to be associated with better prognosis than gross residual disease, which mandates thorough sectioning of specimens from neoadjuvant treated cancers
    • Tumor regression should be assessed in the primary tumor and not based on nodal metastasis assessment
    • Although acellular mucin is considered to represent completely eradicated tumor and recommendations are not to use mucin pools without viable tumor cells in assigning pT or pN category, there seems to be lack of consensus among pathologists across the world (Mod Pathol 2020;33:153)
    • When the mucin pools are present at the radial margin without any viable cells, the practice among most pathologists is to render the margin status negative (Mod Pathol 2020;33:153)
    Grading of quality and completeness of the mesorectum in a total mesorectal excision (TME)
    • Complete: intact and smooth mesorectum, defects (if present) are < 5 mm, there is no coning and the circumferential resection margin is smooth and regular
    • Nearly complete: mesorectum is moderately bulky and irregular with defects > 5 mm but none extending to muscularis propria, no areas of visibility of the muscularis propria except at the insertion site of the levator ani muscle, there is moderate coning and an irregular circumferential resection margin
    • Incomplete: little bulk to mesorectum, the muscularis propria is visible through defects, there is moderate to marked coning and on transverse sectioning, very irregular circumferential resection margin
    • Reference: CAP: Protocol for the Examination of Resection Specimens From Patients With Primary Carcinoma of the Colon and Rectum [Accessed 20 September 2023]
    Margin status assessment
    • Circumferential resection margin (CRM) or nonperitonealized margin involvement strongly predicts local recurrence and is found to be associated with poor survival (J Clin Oncol 2008;26:303, Ann Surg 2002;235:449)
    • Although there is some controversy in the definition of positive margin, if the tumor is identified within 1 mm of inked CRM / radial margin, it should be regarded as positive (Ann Surg 2022;275:e549)
    • Acellular mucin identified on the margin in treated / treatment naïve carcinomas are considered negative (Mod Pathol 2015;28:S95)
    • There is no consensus on the issue of positive lymph node at the CRM (Ann Surg 2022;275:e549)
    AJCC prognostic stage groups
    Stage 0: Tis N0 M0
    Stage I: T1 - T2 N0 M0
    Stage IIA: T3 N0 M0
    Stage IIB: T4a N0 M0
    Stage IIC: T4b N0 M0
    Stage IIIA: T1 - T2 N1 / N1c M0
    Stage IIIA: T1 N2a M0
    Stage IIIB: T3 - T4a N1 / N1c   M0
    Stage IIIB: T2 - T3 N2a M0
    Stage IIIB: T1 - T2 N2b M0
    Stage IIIC: T4a N2a M0
    Stage IIIC: T3 - T4a   N2b M0
    Stage IIIC: T4b N1 - N2 M0
    Stage IVA: any T any N M1a
    Stage IVB: any T any N M1b
    Stage IVC:  any T any N M1c
    Prognostic tools for colon and rectum cancer meeting all AJCC quality criteria
    Registry data collection variables (may or may not be responsibility of surgical pathologist)
    Histologic grade (G)
    Histopathologic type
    • Adenocarcinoma
    • Mucinous adenocarcinoma
    • Signet ring cell carcinoma (poorly cohesive carcinoma)
    • Medullary carcinoma
    • Serrated adenocarcinoma
    • Micropapillary carcinoma
    • Adenosquamous carcinoma
    • Undifferentiated carcinoma
    • Carcinoma with sarcomatoid component
    • Large cell neuroendocrine carcinoma
    • Small cell neuroendocrine carcinoma
    • Mixed neuroendocrine - nonneuroendocrine neoplasm (specify components)
    • Other histologic type not listed
    • Carcinoma, type cannot be determined
    Gross images

    Contributed by Vidya Arole, M.D.
    Complete TME (posterior)

    Complete TME (posterior)

    Nearly complete TME (posterior)

    Nearly complete TME (posterior)

    Incomplete TME Incomplete TME

    Incomplete TME

    Microscopic (histologic) images

    Contributed by Vidya Arole, M.D. and Wei Chen, M.D.
    pT1

    pT1

    pT1

    pT1

    pT2

    pT2

    pT3 invading subserosa

    pT3 invading subserosa

    pT3 invading pericolonic fat

    pT3 invading pericolonic fat

    pT4a

    pT4a


    pT4b

    pT4b

    Lymphovascular invasion (LVI) Lymphovascular invasion (LVI)

    Lymphovascular invasion (LVI)

    Extramural venous invasion (EMVI)

    Extramural venous invasion (EMVI)

    LVI, elastic stain

    LVI, elastic stain

    Videos

    Colorectal carcinoma staging challenges

    Board review style question #1
    Which of the following findings corresponds to T1 pathological stage (pT1) in colorectal carcinoma?

    1. Polyp with high grade dysplasia and no invasion
    2. Tumor invading and limited to lamina propria
    3. Tumor invading into muscularis mucosae
    4. Tumor invading through muscularis mucosae into submucosa
    Board review style answer #1
    D. Tumor invading through muscularis mucosae into submucosa. Answers B and C are incorrect because tumor invasion that is limited to lamina propria and invasion into muscularis mucosae describe pTis. Answer A is incorrect because a polyp with high grade dysplasia and no invasion is not invasive carcinoma.

    Comment Here

    Reference: Colon - Staging-carcinoma
    Board review style question #2


    What is the pathologic stage for this ascending colon cancer based on the above images?

    1. pT1
    2. pT2
    3. pT3
    4. pT4
    Board review style answer #2
    A. pT1. The tumor is invading and confined to the submucosa. Note the tumor glands are next to the thick walled vessels of the submucosa. Answer B is incorrect because pT2 would be tumor invading into (but not through) the muscularis propria. Answer C is incorrect because pT3 would be tumor invading through the muscularis propria into the pericolorectal soft tissue. Answer D is incorrect because pT4 would be invasion of serosa or adjacent structures.

    Comment Here

    Reference: Colon - Staging-carcinoma

    Staging-neuroendocrine

    Pathologic TNM staging of neuroendocrine tumors of the colorectum, AJCC 8th edition
    Definition / general
    • Well differentiated neuroendocrine tumors of the colorectum ("carcinoids") are covered by this staging system
    • Not covered by this staging system are poorly differentiated neuroendocrine carcinomas at this location (use the adenocarcinoma staging system instead)
    • Appendix tumors have their own separate staging system
    Essential features
    Terminology
    • "Carcinoid" is no longer an acceptable term
    ICD coding
    • C7A.02: Malignant carcinoid tumors of the appendix, large intestine and rectum (with subcodes depending on exact anatomic location)
    Primary tumor (pT)
    • TX: Primary tumor cannot be assessed
    • T0: No evidence of primary tumor
    • T1: Tumor invades the lamina propria or submucosa and is ≤ 2 cm
      • T1a: Tumor < 1 cm in greatest dimension
      • T1b: Tumor 1 - 2 cm in greatest dimension
    • T2: Tumor invades the muscularis propria or is > 2 cm with invasion of the lamina propria or submucosa
    • T3: Tumor invades through the muscularis propria into subserosal tissue without penetration of overlying serosa
    • T4: Tumor invades the visceral peritoneum (serosa) or other organs or adjacent structures
    Regional lymph nodes (pN)
    • NX: Regional lymph nodes cannot be assessed
    • N0: No regional lymph node metastasis has occurred
    • N1: Regional lymph node metastasis

    • Notes: Regional lymph nodes vary based on the precise lesional site within the colorectum but overall can include pericolic, ileocolic, right colic, middle colic, left colic, sigmoid, inferior mesenteric, superior rectal, inferior rectal, mesorectal and internal iliac nodes
    Distant metastasis (pM)
    • M0: No distant metastasis
    • M1: Distant metastasis
      • M1a: Metastasis confined to liver
      • M1b: Metastasis in at least one extrahepatic site (e.g. lung, ovary, nonregional lymph node, peritoneum, bone)
      • M1c: Both hepatic and extrahepatic metastases
    Prefixes
    • r: recurrent tumor stage
    Stage grouping
    Stage I: T1 N0 M0
    Stage IIA: T2 N0 M0
    Stage IIB: T3 N0 M0
    Stage IIIA: T4 N0 M0
    Stage IIIB: any T N1 M0
    Stage IV: any T any N M1
    Registry data collection variables
    • Tumor site
    • Size of tumor
    • Depth of invasion
    • Nodal status and number of nodes involved, if applicable
    • Sites of metastasis, if applicable
    • Ki67 index
    • Mitotic count
    • Histologic grade
    Histologic grade
    • Grading is not formally part of the staging system
    • Most pathologists use the ENETS / WHO grading criteria:
      • Grade 1: Mitotic rate < 2 per 10 high power fields and Ki67 rate < 3%
      • Grade 2: Mitotic rate 2 - 20 per 10 high power fields or Ki67 rate 3 - 20%
      • Grade 3: Mitotic rate > 20 per 10 high power fields or Ki67 rate > 20%
    Histopathologic type
    • Well differentiated neuroendocrine tumor, grade 1
    • Well differentiated neuroendocrine tumor, grade 2
    • Well differentiated neuroendocrine tumor, grade 3
    Board review style question #1
    Rectal neuroendocrine neoplasm with salt and pepper nuclei, no necrosis, a mitotic rate of 1 per 10 high power fields and a Ki67 index of 25% is best classified as

    1. Poorly differentiated neuroendocrine carcinoma
    2. Well differentiated neuroendocrine tumor, grade 1
    3. Well differentiated neuroendocrine tumor, grade 2
    4. Well differentiated neuroendocrine tumor, grade 3
    Board review style answer #1
    D. Well differentiated neuroendocrine tumor, grade 3

    Comment here

    Reference: Staging-neuroendocrine

    Stercoral ulcer
    Definition / general
    • Ulceration of colonic mucosa secondary to impacted feces
    Essential features
    • Inspissated feces can cause rectosigmoid ulceration in elderly patients
    • May perforate and lead to death
    Sites
    • Usually in rectosigmoid
    Etiology
    • Impacted feces stagnate, eventually eroding and ulcerating underlying mucosa
    Clinical features
    Diagnosis
    • Proposed criteria for stercoral perforation: round / ovoid antimesenteric perforation > 1 cm, associated with fecaloma, showing necrosis / ulcer / chronic inflammation microscopically (Dis Colon Rectum 2000;43:991)
    Case reports
    Gross description
    • Sharply demarcated ulcer(s)
    • Perforation may be present
    Microscopic (histologic) description
    • Colonic ulceration / perforation
    • Fecal material may or not may be observable microscopically
    Microscopic (histologic) images

    Contributed by Raul S. Gonzalez, M.D.
    Biopsy of colonic ulcer

    Biopsy of colonic ulcer

    Sample pathology report
    • Transverse colon, ulcer, biopsy:
      • Colonic mucosa with ulceration and embedded fecal material (see comment)
      • Comment: The findings are suggestive of a stercoral ulcer.
    Differential diagnosis
    Additional references
    Board review style question #1

    If left unattended, colonic stercoral ulcers can lead to

    1. Inflammatory bowel disease
    2. Malignancy
    3. Malnutrition
    4. Perforation
    Board review style answer #1
    D. Perforation

    Comment Here

    Reference: Stercoral ulcer

    Strongyloides stercoralis
    Definition / general
    • Infection of the colon by the intestinal nematode Strongyloides stercoralis
    Essential features
    • Strongyloides filariform larvae in fecally contaminated ground penetrate skin, enter the systemic circulation and migrate to the lungs where they induce inflammation
    • They climb the tracheobronchial tree and are swallowed
    • In the intestines they mature into adult worms
    • Eggs (made by mature worms) hatch in the intestines and release rhabditiform larvae that are excreted
    • Intestinal infection may be asymptomatic or be associated with diarrhea, weight loss, abdominal pain, malabsorption or other symptoms
    • Infection usually occurs in the small intestine but may occur in the stomach, colon or rarely elsewhere
    • Rhabditiform larvae may change into filariform larvae that penetrate the intestinal wall and enter the circulation, leading to autoinfection that may cause severe tissue and peripheral eosinophilia and infection lasting decades
    • Immunosuppressed patients may develop massive disseminated fatal infection from accelerated autoinfection
    Epidemiology
    • Strongyloides is found in all continents except Antarctica, but is most prevalent in warm and rainy parts of the tropics and subtropics
    • Worldwide estimates vary from 30 to 100 million infected people
    • Infection is more common in adults
    • In the United States, endemic regions include southeastern urban areas with large immigrant populations, mental institutions and Appalachia
      • Immigrants and military veterans who have served in endemic regions may harbor the parasite for up to 60 years
    • Worldwide, immigrants and refugees are more likely to be infected; endemic areas of southern and central Europe also exist
    • Transmission has been documented from close personal contact and from men who have sex with men
    Sites
    • Infection usually occurs in the lung and small intestine
    • The stomach, colon, appendix and liver may also be affected
    • Involvement of the central nervous system may occur with disseminated disease
    • Esophageal disease is rare
    Pathophysiology
    • Strongyloides filariform larvae in fecally contaminated ground penetrate skin, enter the systemic circulation and migrate to the lungs where they induce inflammation
    • They climb the tracheobronchial tree and are swallowed
    • In the intestines they mature into adult worms
    • Eggs (made by mature worms) hatch in the intestines and release rhabditiform larvae that are excreted
    • Rhabditiform larvae may change into filariform larvae that penetrate the intestinal wall and enter the circulation, leading to autoinfection
    • Free living adult male and female worms may be found in soil; they mate and produce rhabditiform larvae that may become infective filariform larvae
    Clinical features
    • Migration of larvae through the lungs may be associated with wheezing, migratory pneumonia or cough
    • Involvement of the intestines may be asymptomatic but may be associated with abdominal pain, diarrhea, weight loss, malabsorption, nausea, GI bleeding or vomiting
    • Involvement of the appendix may cause acute appendicitis
    • Immunosuppressed patients are at risk of disseminated disease, especially those receiving corticosteroids, chronically ill and hospitalized, or with HTLV I infection
      • Those with disseminated disease may lack peripheral eosinophilia
      • Untreated disseminated infection is essentially 100% fatal and even with treatment up to 25% of patients will die
      • Interestingly, patients with HIV/AIDS are not at high risk for disseminated infection
    Diagnosis
    • In biopsy specimens, larvae and adult worms are identified within crypts
    • If the diagnosis is suspected or confirmation is necessary, stool ova and parasite examination and serologic testing is available
    • Ova and parasite testing is not highly sensitive and multiple examinations may be necessary to detect the parasite
    • PCR testing of stool is extremely sensitive and specific
    Case reports
    Treatment
    • Infection should always be treated
    • Ivermectin is the first line treatment in most cases
    • Albendazole or thiabendazole are other options
    • Disseminated disease should be treated with prompt antihelmintic therapy, broad spectrum antibiotics and decrease of immunosuppression if possible
    Gross description
    • Endoscopy may show small ulcers or hyperplastic mucosa
    • Adult female worms approximately 2 mm in length may be seen
    • The mucosa may resemble pseudomembranous colitis
    • Fulminant infection may appear similar to idiopathic inflammatory bowel disease
    Microscopic (histologic) description
    • Tissue eosinophilia and granulocytic infiltration that may cause abscess formation are present
    • Adult worms or larvae may be found within crypts
    • Necrosis or granulomas may be present
    • Ulcers with fissuring similar to Crohn’s disease may occur
    Cytology images

    Contributed by Elliot Weisenberg, M.D.

    Strongyloides larvae in BAL

    Differential diagnosis
    • Capillaria, Trichuris trichuria and other infections, especially other nematodes, pseudomembranous colitis, inflammatory bowel disease

    Tactile corpuscle-like bodies
    Definition / general
    • Tactile corpuscle-like bodies (TCLBs) are Schwannian mechanoreceptors found primarily in the skin and not normally present in the gastrointestinal tract
    Essential features
    • Very rarely seen in the gastrointestinal tract
    • Always benign, incidental
    • Likely a reactive / reparative process
    • Histologically characterized as unencapsulated nodules within the lamina propria with distinctive eosinophilic fibrillary cytoplasm
    • S100 protein positive by IHC
    Terminology
    • Tactile corpuscle-like bodies is the preferred current term
    • Other names / historic terms: Wagner-Meissner corpuscles, Wagner-Meissner-like corpuscles, tactoid bodies, Wagner-Meissner bodies, pseudo-Meissner corpuscles, Meissneroid corpuscles
    Epidemiology
    Sites
    • Colon > esophagus / gastroesophageal junction > stomach > cricopharynx (within gastric heterotopia) / rectum
    Etiology
    Diagnosis
    • Endoscopic findings are incidental and nonspecific
      • Involved mucosa is usually normal
      • May appear as a colon polyp
    Prognostic factors
    • Benign, excellent prognosis
    Case reports
    Treatment
    • TCLBs require no treatment
    Microscopic (histologic) description
    Microscopic (histologic) images

    Contributed by Aaron R. Huber, D.O.

    TCLBs within colonic mucosa

    TCLBs within the gastroesophageal junction mucosa

    Sample pathology report
    • Colon, biopsy:
      • Colonic mucosa with tactile corpuscle-like bodies (see comment)
      • Comment: This finding is incidental, benign and of no clinical consequence.
    Differential diagnosis
    Board review style question #1


    A 65 year old man underwent screening colonoscopy. A small polypoid area was biopsied, as seen above. An immunostain for S100 protein is positive. What is the correct diagnosis?

    1. Amyloid deposition
    2. Granulomas
    3. Mucosal Schwann cell hamartoma
    4. Tactile corpuscle-like bodies (TCLBs)
    Board review style answer #1
    D. TCLBs (formerly known as Wagner-Meissner corpuscles) are Schwannian mechanoreceptors found primarily in the skin and not normally present in the gastrointestinal tract (GIT). TCLBs in the GIT likely arise as a response to mucosal injury. TCLBs are incidentally discovered during endoscopy where they are most often associated with normal mucosa; however, they can also present as polyps. Histologically, TCLBs are seen as nodules within the lamina propria with lamellated or fibrillary eosinophilic cytoplasm. TCLBs are S100 protein positive, which helps distinguish them from some of their histologic mimics, including amyloid (Congo red+, S100-) and granulomata (CD68+, S100-). No treatment is required as TCLBs are benign.

    Comment Here

    Reference: Tactile corpuscle-like bodies
    Board review style question #2
    Which of the following is true regarding tactile corpuscle-like bodies (TCLBs) in the gastrointestinal mucosa?

    1. They always present as polyps on screening colonoscopy
    2. They are neoplastic and require extensive resection
    3. They are never associated with conditions that injure the mucosa such as inflammatory bowel disease
    4. They may be confused with amyloid deposition or mucosal granulomata but are S100 protein positive
    Board review style answer #2
    D. They may be confused with amyloid deposition or mucosal granulomata but are S100 protein positive. TCLBs are benign lesions that likely represent a reactive / reparative process as they are often associated with injured mucosa (B and C). TCLBs are most often incidental findings associated with an endoscopically normal mucosal appearance; however, in some cases they may appear as polyps (A). Histologically, TCLBs have a nodular architecture, composed of cells with abundant eosinophilic, fibrillary cytoplasm and peripherally placed small nuclei. These findings share morphologic characteristics with amyloidosis or granulomata, which can be distinguished by Congo red and CD68, respectively.

    Comment Here

    Reference: Tactile corpuscle-like bodies

    Traditional serrated adenoma
    Definition / general
    Essential features
    • Neoplastic polyp of the colon
    • Shows dysplastic appearing nuclei and prominent eosinophilic cytoplasm
    Terminology
    ICD coding
    • ICD-10: K63.5 - Colon polyp
    • ICD-10: D12 - Benign neoplasm of colon, rectum, anus and anal canal (subcodes depending on site of adenoma)
    Epidemiology
    Sites
    Pathophysiology
    • May arise from microvesicular hyperplastic polyps (MVHP) and sessile serrated adenomas (SSA) (Hum Pathol 2015;46:933)
    • Generally considered to be intrinsically dysplastic, though some argue against this view (Hum Pathol 2015;46:933)
    Etiology
    Clinical features
    • Incidental finding on colonoscopy
    Diagnosis
    Radiology description
    • Screening CT colonography may identify large (~14 mm) traditional serrated adenomas (Radiology 2016;280:455)
    Prognostic factors
    Treatment
    • Endoscopic removal of the adenoma
    • US Multi Society Task Force on Colorectal Cancer recommends a 3 year surveillance interval after initial diagnosis of traditional serrated adenomas (Gastroenterology 2012;143:844)
    Gross description
    Microscopic (histologic) description
    • Protuberant villiform growth pattern with slit-like serrations
    • Pseudostratified epithelial columnar cells with eosinophilic cytoplasm and dark, pencillate, dysplastic-like nuclei
    • Ectopic crypt foci / formations are a key feature (abnormally positioned crypts with bases not seated at the muscularis mucosae) (Am J Surg Pathol 2008;32:21)
    • High grade adenomatous dysplasia may be present
    • Goblet cells present
    • Mucin rich variant is characterized by > 50% goblet cells and fewer ectopic crypt formations (Histopathology 2017;71:208)
    Microscopic (histologic) images

    Contributed by Enoch Kuo, M.D.
    Missing Image Missing Image

    Traditional serrated adenoma



    Contributed by Christopher Hartley, M.D.

    Traditional serrated adenoma

    Positive stains
    Negative stains
    Molecular / cytogenetics description
    • MAPK pathway activation is a critical initiating event, either by a BRAF or KRAS mutation (Hum Pathol 2015;46:933)
    • MAPK pathway activation is followed by CpG island methylator phenotype (CIMP)
    • BRAF mutated traditional serrated adenomas are more frequently CpG island methylator phenotype high (Mod Pathol 2015;28:414, Hum Pathol 2015;46:933)
    • Most cases have no defects in mismatch repair (microsatellite stable)
    • PTPRK-RSPO3 gene fusions may be present (Histopathology 2017;71:601)
    Sample pathology report
    • Rectum, polypectomy:
      • Traditional serrated adenoma
    Differential diagnosis
    • Hyperplastic polyp and sessile serrated adenoma:
      • Cytologic features of pseudostratification and nuclear elongation are most useful in distinguishing traditional serrated adenomas from hyperplastic polyps and sessile serrated adenomas (Virchows Arch 2012;461:495)
      • Ectopic crypts, goblet cells and eosinophilic cytoplasm are not distinguishing features since they may be present in sessile serrated adenomas and hyperplastic polyps as well (Am J Gastroenterol 2012;107:1315, Diagn Pathol 2014;9:212)
      • Traditional serrated adenomas can be distinguished from sessile serrated adenomas and hyperplastic polyps by their low Ki67 staining, specifically within the ectopic crypts (Am J Surg Pathol 2008;32:21)
    • Tubulovillous adenoma:
      • Definitive dysplastic nuclei, no eosinophilic cytoplasm, no serrated architecture or ectopic crypts
      • Tubulovillous adenomas often have a component resembling traditional serrated adenoma, with significance unclear
    Board review style question #1
    Which mutation is most common in traditional serrated adenomas?

    1. BRAF mutation
    2. CDKN2A mutation
    3. KRAS mutation
    4. TP53 mutation
    Board review style answer #1
    A. BRAF and KRAS mutations in traditional serrated adenomas are almost mutually exclusive with approximately 67% having BRAF mutations and 22% with KRAS mutations. BRAF mutated traditional serrated adenomas are more likely to be CpG island methylator phenotype high. TP53 mutation and CDKN2A silencing by methylation are seen in areas with conventional dysplasia (Mod Pathol 2015;28:414).

    Comment here

    Reference: Traditional serrated adenoma

    Trichuriasis
    Definition / general
    • Colonic infection with the nematode Trichuris trichiura (whipworm)
    Essential features
    • Infection via the fecal oral route
    • More common in children
    • Symptoms vary from none / mild to abdominal pain, diarrhea, growth retardation, anemia and others in children
    ICD coding
    • B79
    Epidemiology
    • The CDC estimates that 604 to 795 million people are infected worldwide
    • Disease more common in children
    • Infection via the fecal oral route and correlates with poor sanitation and poverty
    • Infection most common in the tropics where human feces is used as fertilizer or where open defecation occurs
    • May also be common in temperate zones during warm and humid weather
    • Infection may occur via ingesting eggs in contaminated soil via hands, as commonly occurs in children or eating contaminated, inadequately cleaned fruits or vegetables
    • Genetic factors likely impact susceptibility to infection
    • Disease is uncommon in the United States but is found occasionally in the Southeast
    • Incidence of infection is decreasing with globally improved sanitation
    Sites
    • Worms usually reside in the cecum and ascending colon
    • More distal colon may be involved with more severe infection
    • No migratory phase through the lungs
    Pathophysiology
    • Excreted eggs become embryonated and infective in 15 to 30 days after passing through a 2 cell stage
    • Embryonated eggs are ingested and the larvae hatch in the small intestine
    • Larvae migrate to the cecum and ascending colon and penetrate crypts
    • Worms molt several times and embed their whip-like anterior into mucosal crypts
    • Females oviposit after two to three months
    • Female egg output is high, 3000 to 20,000 eggs per day or about 200 eggs per gram of stool
    • Worms live one to three years
    • Worms do not invade but infection may lead to mucosal production of anti-inflammatory cytokines; this property has led to the controversial use of Trichuris suis to treat inflammatory bowel disease, although most authorities do not recommend this treatment
    Diagrams / tables

    Images hosted on other servers:

    Life cycle


    Clinical features
    • Majority of carriers are asymptomatic or minimally symptomatic
    • Peripheral blood eosinophilia may occur
    • More severe disease tends to occur in children who often have higher parasite burdens and lesser immunity
    • Heavy infections may be associated with chronic abdominal pain, diarrhea, tenesmus, chronic anemia, nocturnal incontinence, growth retardation that may be due in part to vitamin A deficiency and rectal prolapse
    • Patients may also suffer from other parasitic infections
    Diagnosis
    • Diagnosis is made by seeing worms on mucosal surfaces or by identifying the characteristic lemon shaped eggs in stool
    • Stool examination has a very high yield due to high egg output
    Laboratory
    • Eggs are 50 - 55 × 20 - 25 micrometers and have a characteristic barrel shape with thick shells and polar plugs at each end
    Prognostic factors
    • Disease is more severe in children
    • Infection clears in essentially all patients with appropriate treatment
    Case reports
    Treatment
    • Three to seven day courses of albendazole, mebendazole or ivermectin, depending on the severity of the infection
    Clinical images

    Images hosted on other servers:

    Endoscopic image of adult worm

    Gross description
    • Worms display a characteristic shape with a slender whip-like anterior that embeds in the colonic mucosa and a thicker posterior that is visible during endoscopy or on the surface of prolapsed rectal mucosa
    • Adult worms are about 4 cm in length
    Gross images

    Images hosted on other servers:

    Adult T. trichiura removed during a colonoscopy

    Infestation with mucosal hemorrhage

    Microscopic (histologic) description
    • Mucosa in the vicinity of embedded worms is edematous and friable with increased histiocytes within the laminal propria
    Microscopic (histologic) images

    Images hosted on other servers:

    Ova stained with iodine

    Differential diagnosis

    Tuberculosis
    Definition / general
    • Infectious disease of the colon or small intestine by acid fast bacilli, Mycobacterium tuberculosis
    • Second leading infectious killer after COVID-19 (BMC Gastroenterol 2022;22:89)
    • 10% of all cases of extrapulmonary tuberculosis are intestinal tuberculosis (BMC Gastroenterol 2022;22:89)
    Essential features
    • Infection occurs through ingestion of mycobacteria in sputum or infected milk, direct extension from adjacent infected lymph node or hematogenous spread
    • Ileocecal region is the most common site
    • Common symptoms include weight loss, anorexia, fever, abdominal pain, diarrhea, constipation and palpable mass
    • Endoscopic findings comprise strictures, ulcers and mucosal hypertrophy
    • Colonoscopic biopsies show granulomatous inflammation with caseous necrosis
    • Special stain, culture or PCR is used to detect the organisms
    • Treatment is with combination of antibiotics and more aggressive treatment is essential for multidrug resistant tuberculosis
    • Surgery may be necessary to treat complications such as perforation, obstruction, fistula or bleeding
    ICD coding
    • ICD-11: 1B12.7 - tuberculosis of the digestive system
    Epidemiology
    Sites
    Pathophysiology
    • Most commonly, Mycobacterium bacilli enter the gastrointestinal tract through the ingestion of infected milk or sputum (World J Emerg Surg 2019;14:33)
    • Second pathway is hematogenous spread from tubercular focus elsewhere in the body
    • Ileocecum is a favored site in GI tract due to abundant lymphoid tissue
    • Nonspore forming, obligate aerobic, catalase negative, nonmotile bacillus with high lipid and mycolic acid content of cell wall (World J Emerg Surg 2019;14:33)
    • Cell mediated delayed type IV hypersensitivity reaction develops that helps to contain the bacteria
    • Bacilli are phagocytosed by macrophages, phagolysosome blockage occurs, further replication and bacteremia develops leading to primary TB (J Clin Tuberc Other Mycobact Dis 2022;27:100300)
    • Caseating granulomatous inflammation and tissue damage occurs
    • Reactivation may occur when immune status is compromised
    • 5 - 10% of immune competent patients develop active disease while 30% of the patients with HIV develop active disease (Emerg Microbes Infect 2016;5:e10)
    Etiology
    • Causative agent is Mycobacterium (M) tuberculosis
      • Mycobacterium (M) tuberculosis complex includes M. tuberculosis, M. africanum, M. bovis, M. microti (Ig Sanita Pubbl 2020;76:59)
    Clinical features
    • Colon
      • Chronic disease is usually present for months
      • Most common symptoms include anorexia, fever, abdominal pain, diarrhea, constipation, abdominal distension and a palpable abdominal mass (BMC Gastroenterol 2022;22:89)
      • Other symptoms may include night sweats, rectal bleeding and malabsorption
      • Perforation, intestinal obstruction and lymphadenopathy may also occur in complicated cases (Saudi J Gastroenterol 2021;27:261)
    • Small intestine
      • Disease occasionally seen in less developed countries
      • Small intestinal disease may be transmitted via contaminated milk
      • Common symptoms encompass abdominal pain, malabsorption, strictures, perforation and fistula (BMC Gastroenterol 2022;22:89)
    Diagnosis
    • Gold standard diagnosis is demonstration of microorganisms with culture of Mycobacterium tuberculosis using intestinal mucosal tissue (BMC Gastroenterol 2022;22:89)
    • Histopathological examination of the intestinal tissue, acid fast bacilli (AFB) staining and nucleic acid amplification tests (MMWR Morb Mortal Wkly Rep 1996;45:950)
    • GI TB is a paucibacillary disease and acid fast bacilli may not be isolated from the clinical specimens
    • Colonoscopic biopsies have 80% diagnostic accuracy (BMC Gastroenterol 2022;22:89)
    Laboratory
    • Blood tests help in detecting erythrocyte sedimentation rate (ESR) and anemia
    • Gene Xpert MTB / RIF test detects acid fast bacillus and rifampicin resistance, polymerase chain reaction (PCR), multiplex PCR and immunological markers (BMC Gastroenterol 2022;22:89)
    • One of the conventional tests used to detect TB is intestinal biopsy
    Radiology description
    • Cross sectional imaging with ultrasonography, multidetector CT and MRI play an important role in the diagnosis (World J Gastroenterol 2014;20:14831)
    • CT scan is the modality of choice; it detects nodular mucosal thickening with loss of symmetry in the fold pattern, thickened omentum and peritoneum
    • Mesenteric lymphadenopathy with necrotic lymph node can be associated with GI TB in children (World J Emerg Surg 2019;14:33)
    • May see fissures, fistulae, sinus tracts, linear and transverse ulcers perpendicular to long axis or perforation (World J Gastroenterol 2014;20:14831)
    • Obliteration of ileocecal angle with widely patent ileocecal valve
    • Rapid emptying of cecum with passage of barium, from terminal ileum to the ascending colon (Stierlin sign) (Indian J Radiol Imaging 2016;26:161)
    Radiology images

    Images hosted on other servers
    CT scan of the abdomen

    CT scan of the abdomen

    Prognostic factors
    • Overall prognosis is not very good, owing to delayed diagnosis and usual occurrence in immunocompromised persons (Cureus 2021;13:e13058)
    • Specifically untreated GI TB causes significant morbidity, leading to prolonged hospitalization and may require surgical intervention
    • Poor prognosis is seen in cases of life threatening complications, such as intestinal stricture, obstruction, bleeding and perforation (Cureus 2021;13:e13058)
    • HIV patients with low CD4 counts, other immunosuppressed states, elderly and the debilitated also tend to have a poorer prognosis
    Case reports
    • 37 year old Mexican man with history of heavy alcohol abuse and progressive diffuse abdominal pain (Cureus 2021;13:e13058)
    • 47 year old man, originally from India, residing in Portugal for a year, presented to emergency with diarrhea and diffuse abdominal pain (Cureus 2022;14:e30600)
    • 81 year old immunocompetent woman from a nonendemic area who developed intestinal TB (Eur J Gastroenterol Hepatol 2011;23:1074)
    Treatment
    • 2 months of 4 drug therapies (rifampicin, isoniazid, pyrazinamide, ethambutol) followed by 4 months of 2 drug therapies (rifampicin, isoniazid) (BMC Gastroenterol 2019;19:106)
    • Drug resistant TB is defined as resistant to at least 1 first line drug, extensively drug resistant is resistant to isoniazid and rifampin, plus at least 1 of 3 injectable second line drugs (amikacin, kanamycin or capreomycin)
    • WHO recommends directly observed therapy (DOT)
    • CDC recommends expert consultation to treat drug resistant and extensively drug resistant TB
    • Most common complication is perforation, which requires surgical intervention (Abdom Imaging 1999;24:32)
    • Inflammatory cecal masses can mimic acute appendicitis, leading to surgery (Abdom Imaging 1999;24:32)
    Gross description
    Gross images

    Images hosted on other servers:
    Ileocecal TB

    Ileocecal tuberculosis

    Microscopic (histologic) description
    • Granulomas with caseating necrosis, conglomerate epithelioid histiocytes, cuff of lymphocytes, Langhans giant cells, aphthous to deep ulcers, with architectural distortion of normal colon mucosa (BMC Gastroenterol 2022;22:89)
    • AFB stains may highlight acid fast bacillus in necrotic areas and histiocytes
    • Disproportionate submucosal inflammation (BMC Gastroenterol 2022;22:89)
    • Hyalinization, fibrotic strictures and calcified granulomas within the lesions may occur in chronic cases (World J Gastroenterol 2014;20:14831)
    • Destruction of muscularis propria, along with scarring in long term can be encountered
    Microscopic (histologic) images

    Contributed by Safina Ahmed, M.B.B.S., Saroona Haroon, M.B.B.S. and Hanni Gulwani, M.B.B.S.
    Colonic mucosa with ill defined granuloma

    Colonic mucosa with ill defined granuloma

    Noncaseating granulomas Noncaseating granulomas

    Noncaseating granulomas

    Caseating granulomas

    Caseating granulomas

    Positive acid fast stain Positive acid fast stain

    Positive acid fast stain


    Ileocecal tuberculosis and adjoining draining lymph node

    Positive stains
    • There are 3 common acid fast staining methods: Ziehl-Neelsen (ZN) (hot), Kinyoun (cold) and auramine-rhodamine fluorochrome (Truant method)
    • Bacteria stain bright red by the Ziehl-Neelsen / Kinyoun method
    • Fluorochrome method is used by large laboratories that have a fluorescent (ultraviolet) microscope
    • Finding a small acid fast bacillus in a ZN stained slide is a cumbersome procedure for a pathologist; therefore, an artificial intelligence based method can be used to detect microorganisms (Diagnostics (Basel) 2022;12:1484)
    Molecular / cytogenetics description
    • World Health Organization (WHO) has recommended the use of molecular nucleic acid amplification tests (NAATs) tests for TB detection as they are able to detect TB more accurately, particularly in patients with paucibacillary disease (J Clin Microbiol 2020;58:e01582)
    • Gene Xpert / MTB and Gene Xpert / MDR are WHO endorsed tests (J Clin Microbiol 2020;58:e01582)
    Videos

    Endoscopy of TB of ascending colon

    Sample pathology report
    • Colon, biopsy:
      • Colonic mucosa showing necrotizing granulomatous inflammation; Ziehl-Neelsen (ZN) stain is negative for AFB and periodic acid-Schiff with diastase (PASD) is negative for fungal elements (see comment)
      • Comment: Correlation with clinical and microbiological culture findings is advised.

    • Colon, resection:
      • Colonic mucosa showing effacement of architecture with numerous caseating granulomatous inflammation and AFB positive bacilli
      • Features are consistent with tuberculosis
      • Resection margins are free of inflammation
      • Total of 14 benign reactive lymph nodes recovered, none of which show granuloma
    Differential diagnosis
    • Crohn's disease:
      • Linear and serpiginous ulcers
      • Noncaseating granulomas
      • No bacteria identified
    • Intestinal sarcoidosis:
      • Nonnecrotizing granulomas
      • No bacteria identified
    • Foreign body granuloma:
      • Noncaseating foreign body type granulomas
      • Foreign material identified
      • No bacteria identified
    • Intestinal malignancy:
      • Atypical cells / adenocarcinoma glands seen
      • No granuloma identified
    • Yersiniosis:
      • Neutrophilic infiltrate
      • Suppurative granulomas
    Board review style question #1

    A 45 year old HIV positive man has presented with abdominal pain, low grade fever and change in bowel habits. CT of the abdomen shows an ileocecal mass. Endoscopic biopsy is taken then processed into a histological slide as shown in the picture. Special stain Ziehl-Neelsen shows positive AFB. Which of the following is the likely diagnosis?

    1. Crohn's disease
    2. Fungal infection
    3. Sarcoidosis
    4. Tuberculosis
    Board review style answer #1
    D. Tuberculosis. Tuberculosis is the correct answer because caseating granulomas with positive AFB are seen in tuberculosis. Answer B is incorrect because fungi will stain by PASD and no AFB will be identified. Answer C is incorrect because with sarcoidosis, no caseous necrosis will be seen and no AFB will be present. Answer A is incorrect because no caseous necrosis will be seen and no AFB are identified. Also, typical fissures and lymphoid aggregates are seen in a beaded appearance.

    Comment Here

    Reference: Tuberculosis
    Board review style question #2
    A 66 year old woman was diagnosed with intestinal tuberculosis. She underwent a colonoscopic biopsy. Which one of the following types of necrosis will you see in her biopsy?

    1. Caseous
    2. Coagulative
    3. Fibrinoid
    4. Liquefactive
    Board review style answer #2
    A. Caseous. Caseous is the correct answer because it is seen in tuberculosis. Answer B is incorrect because coagulative necrosis is seen in myocardial infarction or renal infarction. Answer C is incorrect because fibrinoid necrosis is present in autoimmune diseases and in peptic ulcer. Answer D is incorrect because liquefactive necrosis is seen in the central nervous system, such as in brain infarcts or brain abscesses (Histopathology.guru: Types of Necrosis with Examples [Accessed 10 August 2023]).

    Comment Here

    Reference: Tuberculosis

    Tubular adenoma
    Definition / general
    • Neoplastic colon polyp with at least low grade dysplasia
    • Precursor to invasive adenocarcinoma
    Essential features
    • Dysplastic nuclei (elongated and hyperchromatic pseudostratification)
    • < 25% villous component
    Terminology
    ICD coding
    • ICD-10: K63.5 - colon polyp
    • ICD-10: D12.6 - adenomatous polyp
    Epidemiology
    Sites
    • Found more commonly in the left colon and rectum compared with other polyps
    Pathophysiology
    • Has a well documented adenoma to carcinoma sequence that involves mutations in KRAS, TP53, APC and beta catenin (Cell 1990;61:759)
    • Activation of KRAS leads to downstream signaling that influences survival, antiapoptosis and proliferation, though this event is typically seen in advanced rather than early adenomas (J Gastroenterol Hepatol 2012;27:1423, PLoS One 2017 27;12)
    • Wnt pathway: activation of Wnt pathway leads to beta catenin accumulation in the nucleus and subsequent transcription and cell proliferation
    • Mutations include loss of function of APC or beta catenin mutations that resist degradation by APC (J Gastroenterol Hepatol 2012;27:1423)
    Etiology
    Diagrams / tables

    Images hosted on other servers:
    Missing Image

    Wnt / beta catenin pathway

    Clinical features
    • < 1 cm: usually asymptomatic and detected by screening colonoscopy
    • > 1 cm: may bleed, lead to iron deficiency anemia, obstruction; increased risk of progressing to carcinoma
    • Mostly left sided in spontaneous polyps and in familial adenomatous polyposis; right sided in Lynch syndrome (Gut 2002;50:382)
    Diagnosis
    • Diagnosis is made by histologic confirmation
    Case reports
    Treatment
    • Colonoscopy is diagnostic and potentially curative (via polypectomy)
    • High grade dysplasia and large polyps may warrant endoscopic mucosal resection or partial colectomy
    Clinical images

    Images hosted on other servers:
    Missing Image

    Large sigmoid polyp

    Gross description
    • May be sessile or pedunculated
    • Typically dark red compared with mucosa
    • Features concerning for high grade dysplasia or malignancy include size > 1 cm, villous architecture and ulceration / friability
    Gross images

    Contributed by Laleh Montaser, M.D. and @Andrew_Fltv on Twitter

    Colonic tubular adenoma

    Tubular adenoma Tubular adenoma Tubular adenoma

    Tubular adenoma



    Images hosted on other servers:

    Tubular adenoma

    Hemorrhagic surface

    Multiple cecal polyps

    Microscopic (histologic) description
    • Polypoid colonic mucosa covered with dysplastic epithelium comprised of hyperchromatic, elongated nuclei arranged in a pseudostratified manner
    • Dysplasia is typically low grade but may also be high grade, with architectural (cribriforming, luminal necrosis) and cytologic changes (vesicular chromatin, nucleoli, loss of basal polarity)
    • Abrupt transition from normal to dysplastic mucosa is commonly present
    • Variable amounts of mucin loss
    • Metaplasia may be present: osseous, squamous or Paneth cells (J Clin Pathol 2005;58:220, J Surg Oncol 1984;26:130)
    • Pseudoinvasion can mimic progression to adenocarcinoma but displaced glands are benign and surrounded by lamina propria and often hemosiderin (Mod Pathol 2015;28:S88)
    • May rarely show clear cell features (Am J Surg Pathol 2010;34:1344)
    Microscopic (histologic) images

    Contributed by Andrew L.J. Dunn, M.D. and Christopher Hartley, M.D.

    Nuclear crowding

    Transition to dysplasia

    Dark nuclei with pseudostratification

    Tubular adenoma



    Contributed by @Andrew_Fltv and @liverwei on Twitter
    Tubular adenoma Tubular adenoma Tubular adenoma Tubular adenoma Tubular adenoma

    Tubular adenoma



    Images hosted on other servers:

    Attached on a stalk

    Compared to normal mucosa

    Pseudocarcinomatous
    invasion

    Positive stains
    • BCL2 (almost all cases), increased CEA (in atypical areas)
    Negative stains
    • p53 (usually)
    Molecular / cytogenetics description
    • 33% are aneuploid
    • Depending on the villous component, 2 types of tubular adenomas can be identified (Am J Surg Pathol 2011;35:212):
      • TA1: less than 1% villous component, lower rate of p53 overexpression, KRAS mutation and MGMT loss
      • TA2: 1 - 20% villous component, higher rate of TP53 and KRAS mutation and MGMT loss
    • KRAS mutations may account for up to 60% of mutations in tubular adenomas (J Gastroenterol Hepatol 2012;27:1423)
    Sample pathology report
    • Colon, splenic flexure, polypectomy:
      • Tubular adenoma
    Differential diagnosis
    Board review style question #1
    Which of the following gene mutations is implicated in a majority of tubular adenomas?

    1. ALK
    2. BAP1
    3. KRAS
    4. MDM2
    Board review style answer #1
    C. KRAS mutations are implicated in up to 60% of adenomas and adenocarcinomas. BAP1 mutations are present in a subset of melanocytic tumors and mesotheliomas. MDM2 is amplified in liposarcoma and some of its variants. ALK mutations can be seen in inflammatory myofibroblastic tumor, anaplastic large cell lymphoma and a subset of lung adenocarcinomas.

    Comment here

    Reference: Tubular adenoma

    Tubulovillous / villous adenoma
    Definition / general
    Essential features
    • Recognized as the precursor lesion of chromosomal unstable colorectal cancer (Pathologica 2021;113:218)
    • Low grade dysplasia is an intrinsic feature
    • Associated with oncogene KRAS mutation in 50% of cases
    ICD coding
    • ICD-10: D12.6 - benign neoplasm of colon, unspecified
    Sites
    Pathophysiology
    • Conventional adenoma carcinoma sequence with oncogene (KRAS) activation and tumor suppressor (APC, SMAD4 and TP53) inactivation
    • Associated with chromosomal unstable colorectal cancer (Cancer Biol Med 2016;13:120)
    Clinical features
    Diagnosis
    • Screening or surveillance colonoscopy identifies and removes lesion, followed by tissue diagnosis
    Prognostic factors
    • Villous or tubulovillous histology is associated with increased risk of colorectal neoplasia: 16.8% versus 9.7% compared with tubular adenomas
    • Increased risk of malignant transformation in cases with high grade dysplasia (↑ 1.77x) (Gastroenterology 2012;143:844)
    • Risk of recurrence with malignancy after excision is associated with piecemeal endoscopic resection (Surg Endosc 2021;35:2500)
    Case reports
    Treatment
    • Endoscopic resection
    • When possible, en bloc resection should be the goal for the management (Surg Endosc 2021;35:2500)
    Gross description
    • Pedunculated or sessile polypoid lesions with macroscopic finger-like projections
    Gross images

    Contributed by Carolina Martinez Ciarpaglini, M.D., Ph.D.
    Large villous polyp

    Large villous polyp

    Giant villous adenoma

    Giant villous adenoma



    Images hosted on other servers:
    Sessile villous adenoma

    Sessile villous adenoma

    Pedunculated villous adenoma

    Pedunculated villous adenoma

    Villous surface

    Villous surface

    Detailed view of villous projections

    Detailed view of villous projections

    Adenocarcinoma arising in villous adenoma

    Adenocarcinoma arising in villous adenoma

    Microscopic (histologic) description
    • Low grade dysplasia is a constituent feature
    • Epithelial, finger-like projections, away from the muscularis mucosae, formed by fibrovascular cores lined by dysplastic epithelium (villous architecture)
      • Percentage of villosity defines diagnostic terminology
      • 20 - 80% = tubulovillous adenoma
      • > 80% = villous adenoma
    • Crowded pseudostratification of cells with elongated nuclei occupying the basal half of the cytoplasm
    • Pleomorphism and atypical mitoses should be absent or minimally present
    • Mitotic activity and minimal loss of cell polarity are allowed
    • Architecturally, the crypts should maintain a resemblance to normal colon, without significant crowding, cribriform or complex forms
    • Squamous metaplasia as solid nests of squamous cells in direct continuity with adenomatous glands may be seen in about 0.4% of colorectal adenomas (Histopathology 2021;78:348)
    • Paneth cell metaplasia is a frequent finding (17 - 23%), especially in proximal locations; osseous metaplasia has been reported (Histopathology 2021;78:348)
    • Neuroendocrine metaplasia refers to the presence of scattered foci of neuroendocrine cells that lack significant nuclear atypia, mitotic activity or necrosis; these cells comprise < 30% of the lesion (Ann Diagn Pathol 2019;42:69)
    • Clusters of neuroendocrine metaplasia are usually < 2 mm and limited to the lamina propria without disturbing the overall architecture (Histopathology 2021;78:348)
    • Ectopic crypt foci are a frequent finding (J Clin Pathol 2016;69:1063)
    • High grade dysplasia can be seen (carcinoma in situ or intraepithelial / intramucosal carcinoma are not recommended terms)
      • Increased nucleus to cytoplasm ratio
      • Significant loss of cell polarity: nuclear stratification through the entire thickness of the epithelium
      • Round nuclei with open appearing chromatin and increasingly prominent nucleoli
      • Significant pleomorphism and atypical mitoses
      • Abnormal architecture includes cribriform structures with back to back glands, prominent glandular budding and intraluminal papillary tufting (StatPearls: Villous Adenoma [Accessed 31 May 2022])
    • Poor interobserver agreement for assessment of the villous component and high grade dysplasia have been demonstrated (Am J Surg Pathol 2013;37:427)
    Microscopic (histologic) images

    Contributed by Carolina Martinez Ciarpaglini, M.D., Ph.D., Jijgee Munkhdelger, M.D., Ph.D. and Andrey Bychkov, M.D., Ph.D.
    Neuroendocrine metaplasia Neuroendocrine metaplasia

    Neuroendocrine metaplasia

    Squamous metaplasia Squamous metaplasia Squamous metaplasia

    Squamous metaplasia


    High grade dysplasia High grade dysplasia High grade dysplasia

    High grade dysplasia

    Low grade dysplasia

    Low grade dysplasia

    Villous architecture

    Villous architecture


    Giant rectal villous adenoma

    Giant rectal villous adenoma

    Focal eosinophilic change

    Focal eosinophilic change

    Ectopic crypt foci

    Ectopic crypt foci

    Cut section

    Cut section

    Pedunculated polyp

    Pedunculated polyp


    Neuroendocrine metaplasia (marker expression)

    Neuroendocrine metaplasia (marker expression)

    Neuroendocrine metaplasia (marker expression)

    Neuroendocrine metaplasia (Ki67)

    Squamous metaplasia (CK5/6)

    Squamous metaplasia (CK5/6)

    Squamous metaplasia (beta catenin)

    Squamous metaplasia (beta catenin)

    Virtual slides

    Images hosted on other servers:
    Villous adenoma

    Villous adenoma

    Positive stains
    Negative stains
    Molecular / cytogenetics description
    Sample pathology report
    • Colon, polyp at 12 cm, endoscopic polypectomy:
      • Tubulovillous adenoma with high grade dysplasia

    • Colon, polyp at 14 cm, endoscopic polypectomy:
      • Villous adenoma
    Differential diagnosis
    Board review style question #1

    A colon polyp is resected. The image shown above is representative of the lesion. What is your diagnosis?

    1. Composite adenoma neuroendocrine carcinoma
    2. Composite adenoma neuroendocrine tumor
    3. Tubulovillous adenoma with high grade dysplasia
    4. Tubulovillous adenoma with squamous metaplasia
    Board review style answer #1
    D. Tubulovillous adenoma with squamous metaplasia

    Comment here

    Reference: Tubulovillous / villous adenoma
    Board review style question #2
    Which of the following is true about villous adenoma?

    1. Frequently associated with KRAS mutations
    2. Precursor lesion in the serrated pathway of colorectal development
    3. Presents the same risk of malignant transformation as tubular adenoma
    4. Routine screening of MMR proteins for Lynch syndrome in this lesion is useful
    Board review style answer #2
    A. Frequently associated with KRAS mutations

    Comment here

    Reference: Tubulovillous / villous adenoma

    Turcot syndrome
    Definition / general
    Essential features
    Terminology
    • Pronounced with silent second t (Turcot was French Canadian)
    Clinical features
    • Same as FAP or Lynch Syndrome, but additionally with central nervous system tumor (as above)
    Case reports
    Differential diagnosis
    • Constitutional mismatch repair deficiency syndrome: biallelic mismatch repair gene mutation; patients develop brain tumors, hematopoietic malignancies and Lynch syndrome associated tumors (Haematologica 2010;95:699)
    Additional references

    Typhlitis
    Definition / general
    • Means inflammation of cecum (eMedicine); a 19th century term for severe acute/chronic inflammation of cecum/appendix, probably due to untreated acute appendicitis
    • Currently, represents a life threatening acute inflammatory destruction of cecal mucosa in neutropenic patients, possibly due to impaired mucosal immunity and compromised blood flow; also associated with secondary bacteria and fungal overgrowth but minimal active inflammation (due to neutropenia)
    • Also called hemorrhagic necrosis of bowel wall, neutropenic colitis
    • Often leukemia or lymphoma patients after immunosuppressive therapy
    • Mechanism (not known), but probably due to a combination of ischemia, infection (especially with cytomegalovirus), mucosal hemorrhage, and perhaps neoplasia
    Clinical features
    • Present with fever, watery or bloody diarrhea, and abdominal pain, may be localized in right lower quadrant (Radiographics 2000;20:399)
    • May cause perforation and require surgery
    • Severity in childhood cancer is related to duration of neutropenia and presence of fever or abdominal tenderness (Cancer 2005;104:380)
    Diagnosis
    • Usually diagnosed by CT scan due to thickening of cecal wall
    Prognostic factors
    Case reports
    Gross description
    • Inflamed cecum and proximal right colon with hemorrhage and mucosal edema
    Microscopic (histologic) description
    • Adherent fibrinopurulent exudate over necrotic and edematous mucosa

    Ulcerative colitis
    Definition / general
    • Idiopathic chronic inflammatory disorder of the colon mucosa
    • Generally starts in the rectum and extends proximally in a continuous pattern
    Essential features
    • Begins in the rectum and continues proximally in a continuous pattern to include up to the entire colon (pancolitis)
    • Histologically characterized by active chronic colitis, with inflammation limited to mucosa / submucosa, absence of granulomas and absence of fissuring ulcers
    • Complex multifactorial process involving an overwhelming T helper type 2-like immune response, leading to mucosal injury in response to gut microbial dysbiosis in genetically predisposed patients
    • Risk of colorectal carcinoma, compared with the general population, is increased with duration of disease and severity of inflammation (Gastroenterol Hepatol (N Y) 2017;13:357)
    ICD coding
    • ICD-10: K51 - ulcerative colitis
    Epidemiology
    • Higher incidence (9 - 20/100,000 person years) and prevalence (156 - 291/100,000 people) in populations of North American and Northern European descent (Lancet 2012;380:1606)
    • Incidence increased in industrialized countries and urban versus rural locations, suggestive of environmental triggers, such as improved sanitation, reduced exposure to childhood enteric infections and mucosal immune system maturation (Lancet 2012;380:1606)
    • Bimodal age distribution with peaks at 15 - 30 years and 50 - 70 years (Lancet 2012;380:1606)
    • Family history of inflammatory bowel disease, particularly that of a first degree relative (5.7 - 15.5%) and Ashkenazi Jewish descent (3 - 5x) show higher risk of disease development (Lancet 2012;380:1606)
    • Gastrointestinal infections with Salmonella spp, Shigella spp and Campylobacter spp have twice the risk of developing ulcerative colitis postinfection (Lancet 2012;380:1606)
    • M = F
    • Former cigarette smoking is strong risk factor (Lancet 2017;389:1756)
    Sites
    Pathophysiology
    • Not fully known but appears to be a complex multifactorial process involving an overwhelming T helper type 2-like immune response, leading to mucosal injury in response to gut microbial dysbiosis in genetically predisposed patients
    • Proposed mechanisms include:
      • Damage to the colonic epithelial barrier due to dysregulation of epithelial tight junctions, which provide a physical barrier between the immune cells and the luminal microbes, leads to increased permeability (Lancet 2012;380:1606)
      • Colonic epithelium upregulation of antimicrobial peptides, known as beta defensins (Lancet 2012;380:1606)
      • Disruption in the homeostatic balance of the mucosal immunity and the enteric nonpathogenic bacteria, resulting in the patient's aberrant immune response to the enteric commensal bacteria (Lancet 2012;380:1606, Front Microbiol 2018;9:2247)
      • Increased number of colonic epithelium activated and mature dendritic cells with increased stimulatory capacity (Lancet 2012;380:1606)
      • Increased expression of TLR4 by lamina propria cells and TLR4 polymorphism, which can alter susceptibility to enteric infections and tolerance to commensal bacteria (Lancet 2012;380:1606)
      • Disruption in the homeostatic balance between regulatory and effector T cells, leading to a nonclassic natural killer T cell production of IL5 and IL13, which have cytotoxic effects on epithelial cells, mediating an atypical Th2 response
        • IL13 can induce a positive feedback system on the natural killer T cells, leading to increased tissue injury (Lancet 2012;380:1606)
      • Increase in proinflammatory cytokines, chemoattractants such as CXCL8 and adhesion molecules such as MadCAM1 recruit increased leukocytes to the colonic mucosa (Lancet 2012;380:1606)
      • Other genetic risk loci include IL23 and IL10, JAK2 kinase pathway genes, hepatocyte nuclear factor 4α, CDH1 and laminin β1 (Lancet 2012;380:1606)
    Etiology
    • Idiopathic
    Clinical features
    • Clinical symptoms include bloody diarrhea, abdominal pain, mucus discharge, fecal urgency, tenesmus; in severe cases, symptoms may include weight loss, fever or colonic perforation (Mayo Clin Proc 2019;94:1357)
    • Characterized by alternating periods of clinical relapse and remission
    • At diagnosis, most patients have mild to moderate symptoms, with fewer than 10% having severe disease
      • Patients presenting with severe disease are usually those diagnosed at young ages (15 - 30 years of age) or with simultaneous PSC (Lancet 2012;380:1606)
    • 30 - 50% of patients will present with disease of the rectum or sigmoid colon and only approximately 20% of patients will present with pancolitis (Lancet 2012;380:1606)
    • Appendectomy due to acute appendicitis before age 20 has been shown to be protective against ulcerative colitis (Lancet 2012;380:1606)
    • Fulminant colitis, known as acute, clinically severe colitis involving the entire colon and requiring surgical resection, can be seen (Histopathology 2014;64:317)
    • Toxic megacolon (marked colonic dilation with signs of systemic toxicity) can occur and requires surgical intervention (BMJ Case Rep 2018;2018:bcr2018227121)
    • May have iron deficiency anemia
    • Increased risk of hypercoagulability and thrombosis
    • Disease severity via endoscopy is stratified as remission, mild, moderate or severe
      • Numerous severity indices exist
      • Goal of endoscopic remission following therapy
    Diagnosis
    • Correlation of clinical symptoms with endoscopic and histological examination
    • Exclusion of other etiologies for colitis (infection, drug, etc.)
    • Colonoscopy with biopsy is essential
      • Endoscopic findings include erythema, loss of vascular pattern, granularity, friability and erosion / ulceration
      • Often a sharp demarcation between inflammation and normal mucosa (Lancet 2017;389:1756)
    • High definition colonoscopy or chromoendoscopy are preferred over traditional white light endoscopy due to higher sensitivity (93 - 97%) and specificity (93%) (Dig Endosc 2016;28:266, Histopathology 2015;66:37)
    • Targeted biopsies of mucosal abnormalities and random biopsies at each segment of the colon help determine microscopic extent of disease (Gastroenterol Hepatol (N Y) 2017;13:357, Dig Endosc 2016;28:266)
    • Esophagogastroduodenoscopy to rule out upper gastrointestinal tract involvement
    Laboratory
    • Overall nonspecific
    • Markers of inflammation
      • Erythrocyte sedimentation rate ≥ 30 mm/h
      • C reactive protein > 8 mg/L
      • Leukocytosis and thrombocytosis
    • Antineutrophil cytoplasmic antibodies
    • Fecal calprotectin > 50.0 mcg/g
    • References: Pathologica 2021;113:39, Diagnostics (Basel) 2021;11:207
    Radiology description
    • Magnetic resonance imaging (MRI) and computed tomography (CT) may be useful in identifying bowel wall thickening and ahaustral colon but are not sensitive or specific for diagnosis of acute disease
    • Plain upright abdominal Xray can be performed in patients with severe colitis to assess for toxic megacolon
    • Target or double halo sign can be seen in cases of advanced disease
    Radiology images

    Images hosted on other servers:
    Toxic megacolon

    Toxic megacolon: radiograph

    Toxic megacolon: contrast enhanced CT Toxic megacolon: contrast enhanced CT

    Toxic megacolon:
    contrast enhanced CT

    Prognostic factors
    • Colorectal carcinoma is the cause of death in an estimated 15% of inflammatory bowel disease patients; risk factors for developing colorectal carcinoma include:
      • Duration of disease (increased risk of up to 2% after 10 years, 8% after 20 years and 18% after 30 years)
      • Extent of disease, with pancolitis carrying the highest risk
      • Simultaneous PSC, severity of colitis, psuedopolyps, family history of sporadic colorectal carcinoma and male sex
    • Risk factors for aggressive or complicated disease include:
      • Young age at onset, pancolitis, lack of endoscopic healing, deep ulcerations and high concentrations of antineutrophil cytoplasmic antibodies
    • References: Lancet 2012;380:1606, Lancet 2017;389:1756
    Case reports
    Treatment
    • 5-aminosalicylate agents are first line therapy for mild to moderate disease
    • Corticosteroids
    • Patients with moderate to severe disease may require thiopurines or biologic agents (anti-TNF therapy or anti-integrin therapy) (Mayo Clin Proc 2019;94:1357)
    • Patients with proctitis only may be treated with topical agents
    • Colorectal carcinoma surveillance at 8 - 10 years after the onset of symptoms and fixed interval surveillance every 1 - 2 years afterward (Gastroenterol Hepatol (N Y) 2017;13:357)
    • Surgery will eventually be required in 20 - 30% of patients with ulcerative colitis that has become refractory to medical management or who have developed dysplasia or colorectal carcinoma (Lancet 2012;380:1606)
      • Total colectomy with ileal pouch - anal anastomosis is preferred surgical intervention
    Clinical images

    Images hosted on other servers:
    Pseudopolyps

    Pseudopolyps

    Endoscopic inflammation

    Endoscopic inflammation

    Gross description
    • Findings generally progress in a continuous pattern from distal to proximal along the specimen
      • Absence of skip lesions
    • Mucosal erythema and granularity with superficial ulceration
      • Usually no deep, fissuring ulceration
    • Absence of strictures, fistulas and creeping fat
    • Pseudopolyp formation due to ulceration with intervening areas of preserved mucosa
      • Filiform polyposis is an exaggerated form of pseudopolyp formation and shows elongated, finger-like mucosal projection
    • Toxic megacolon shows marked dilation and thinning of the colonic wall
    • References: Virchows Arch 2014;464:511, Virchows Arch 2018;472:81
    Gross images

    Contributed by Daniel Soderberg, M.S., P.A.
    Colectomy

    Colectomy

    Proctectomy

    Proctectomy



    Images hosted on other servers:

    Inflammatory pseudopolyps

    Toxic megacolon

    Transition point

    Microscopic (histologic) description
    • Untreated disease characteristically shows active chronic colitis
    • Features of chronicity include:
      • Crypt architectural distortion including crypt atrophy, irregular spacing and size of crypts, crypt shortening and crypt branching
      • Inflammatory expansion of the lamina propria with basal lymphoplasmacytosis
      • Paneth cell metaplasia or hyperplasia
    • Features of activity
      • Neutrophilic inflammation with cryptitis, crypt abscess or ulceration
    • Patients who have had symptoms for a short duration, longstanding disease or who have undergone therapy may have histological changes that may make microscopic diagnosis difficult (Histopathology 2014;64:317)
      • Early / evolving cases may lack features of chronicity
      • Treated cases may show chronic inactive (quiescent) colitis
    • Superficial inflammation limited to mucosa / submucosa, with lack of transmural inflammation
    • Lack of granulomas
      • Crypt rupture granulomas may be noted
    • Lack of fissuring ulcers
    • Biopsies are limited in determining the depth of inflammation and the exact anatomical distribution of the inflammation (Histopathology 2014;64:317)
    • Dysplasia (low or high grade) may be present, especially in patients with longstanding disease (Histopathology 2014;64:317)
    • Activity is usually graded (Gastroenterology 2007;133:1099):
      • Inactive: absence of neutrophils
      • Mild: activity involving < 50% of the mucosa
      • Moderate: activity involving > 50% of the mucosa; crypt abscesses usually seen at this grade
      • Severe: presence of surface ulceration or erosion
    • More complex grading systems, including Geboes and Robarts, exist but are not typically used in everyday practice (Gut 2000;47:404, Gut 2017;66:50)
    Microscopic (histologic) images

    Contributed by Rachel Horton, D.O., Catherine E. Hagen, M.D. and @AnaPath10 on Twitter
    Biopsy Biopsy

    Biopsy

    Cryptitis

    Cryptitis

    Colectomy specimen

    Colectomy specimen

    Severe activity

    Severe activity

    Inactive

    Inactive


    Crypt rupture granuloma

    Crypt rupture granuloma

    Ulcerative colitis Ulcerative colitis

    Ulcerative colitis

    Ulcerative colitis Ulcerative colitis

    Ulcerative colitis

    Virtual slides

    Images hosted on other servers:
    Ulcerative colitis resection

    Ulcerative colitis resection

    Negative stains
    • CMV immunohistochemical stain
    Molecular / cytogenetics description
    • Association with major histocompatibility complex class 2 region near HLA-DRA
    • DRB1*0103 HLA haplotype is associated with disease susceptibility and extensive disease, as well as higher risk of colectomy (Lancet 2012;380:1606)
    Sample pathology report
    • Colon, total colectomy:
      • Severely active chronic colitis, compatible with ulcerative colitis; no dysplasia (see comment)
      • Comment: Sections show continuous involvement of the colon by active chronic colitis. No transmural inflammation, deep fissuring ulcers or granulomas are seen. The findings are compatible with a diagnosis of ulcerative colitis.
    • Colon, biopsy:
      • Moderately active chronic colitis; no granulomas (see comment)
      • Comment: The histologic features are compatible with idiopathic inflammatory bowel disease provided other etiologies have been excluded. Clinical correlation with distribution of disease is required to distinguish Crohn's disease from ulcerative colitis.
    Differential diagnosis
    Board review style question #1
    Which of the following pathologic features, if present in a colectomy specimen, favors a diagnosis of ulcerative colitis over Crohn's disease?

    1. Fissuring ulcers
    2. Granulomas
    3. Inflammation limited to mucosa
    4. Skip lesions
    Board review style answer #1
    C. Inflammation limited to mucosa

    Comment Here

    Reference: Ulcerative colitis
    Board review style question #2

    A 22 year old man reports to the emergency department complaining of abdominal pain and bloody diarrhea. He is diagnosed with fulminant colitis and requires colectomy. Histologic sections of the sigmoid colon are most consistent with which diagnosis?

    1. Acute infectious colitis
    2. Collagenous colitis
    3. Ischemic colitis
    4. Ulcerative colitis
    Board review style answer #2
    D. Ulcerative colitis

    Comment Here

    Reference: Ulcerative colitis

    Ulcerative proctitis
    Definition / general
    • Subgroup of idiopathic ulcerative colitis wherein the extent of disease is anatomically limited to the rectum (i.e., distal to the rectosigmoid junction)
    • Corresponds to E1 in the Montreal classification of ulcerative colitis by extent of disease (Can J Gastroenterol 2005;19:5A)
    • Comprises approximately 30% of patients with ulcerative colitis (Clin Gastroenterol Hepatol 2018;16:343)
    Essential features
    • Diagnosis is based on constellation of symptoms, endoscopic findings, histopathology and exclusion of alternative diagnoses (Lancet 2017;389:1756)
      • Symptoms: rectal bleeding, tenesmus, passage of mucus, urgency
      • Endoscopic findings: erythema, granularity, friability, erosions, ulcerations
      • Histopathology:
        • Active mucosal inflammation (cryptitis, crypt abscess, erosion, ulceration)
        • Chronic mucosal inflammation (mucosal expansion, basal plasmacytosis)
        • Evidence of chronic injury (crypt architecture distortion, Paneth cell metaplasia)
      • Exclusion of alternative diagnoses:
        • Infectious proctitis, including sexually transmitted (e.g., syphilis proctitis)
        • Ischemic proctitis
        • Therapy related proctitis, including drug induced (e.g., immune checkpoint inhibitor associated proctitis or NSAID injury) and radiation proctitis
        • Segmental colitis associated with diverticulosis
        • Fecal impaction (e.g., stercoral ulcer)
        • Prolapse related (e.g., solitary rectal ulcer syndrome)
    Terminology
    • Idiopathic ulcerative proctitis
    ICD coding
    • ICD-10:
      • K51.20 - ulcerative (chronic) proctitis without complications
      • K51.211 - ulcerative (chronic) proctitis with rectal bleeding
      • K51.212 - ulcerative (chronic) proctitis with obstruction
      • K51.214 - ulcerative (chronic) proctitis with abscess
      • K51.219 - ulcerative (chronic) proctitis with complication
    Epidemiology
    • Similar to ulcerative colitis:
      • No sex predilection (M = F)
      • Incidence and prevalence is increasing worldwide (Gastroenterology 2012;142:46)
      • Highest prevalence in Europe, Canada and United States
    Sites
    • Rectum
    Pathophysiology
    • Same as ulcerative colitis (results from abnormalities in immune regulation, epithelial barrier functions and host - microbiome interactions)
    Etiology
    Diagrams / tables

    Images hosted on other servers:

    Phenotypes of ulcerative colitis

    Clinical features
    • Symptoms:
      • Rectal bleeding, tenesmus, passage of mucus, urgency
      • Up to 10% show paradoxical constipation (Lancet 2017;389:1756)
      • Ulcerative proctitis less likely to show constitutional symptoms (such as fatigue and fever) when compared to extensive ulcerative colitis
    • Endoscopic findings (limited to rectum):
      • Erythema, granularity, friability, erosions, ulcerations
    Diagnosis
    • Diagnosis is based on constellation of symptoms, endoscopic findings, histopathology and exclusion of alternative diagnoses
    Laboratory
    Prognostic factors
    • Factors associated with poor outcomes (i.e., complicated or aggressive disease requiring immunosuppressants, biologics [e.g., antibody based drugs]) in adults include (Clin Gastroenterol Hepatol 2015;13:635):
      • Young age of onset (< 40 years)
      • Nonsmokers and ex smokers
      • Deep ulcerations
    • Extent of disease (ulcerative proctitis is more associated with lower risk of colectomy than left sided and extensive colitis):
    • Colitis related dysplasia and carcinoma:
    Case reports
    Treatment
    • Same as ulcerative colitis
    • Because disease in ulcerative proctitis is limited to within 15 - 20 cm of the anal verge, medical therapies (such as mesalamine) may be administered topically (i.e., by suppository or enema)
    • Reference: Am J Gastroenterol 2019;114:384
    Gross description
    • Diffuse, continuous involvement of the rectal mucosa by:
      • Congestion, granularity and edema
      • Shallow, broad based ulcerations
      • Inflammatory pseudopolyps (islands of regenerating mucosa amid shallow ulcers)
      • Mucosal bridges (fused inflammatory pseudopolyps)
    • Mucosal disease may be patchy after medical treatment (J Clin Pathol 2004;57:1233)
    • Longstanding chronic disease may show flattened, featureless and atrophic mucosa (burned out proctitis)
    Gross images

    Contributed by John D. Paulsen, M.D. and Alexandros D. Polydorides, M.D., Ph.D.
    Ulcerative proctitis

    Ulcerative proctitis

    Ulcerative proctitis, inflammatory pseudopolyps

    Ulcerative proctitis, inflammatory pseudopolyps

    Ulcerative proctitis, treated

    Ulcerative proctitis, treated

    Ulcerative proctitis, atrophic

    Ulcerative proctitis, atrophic

    Microscopic (histologic) description
    • Same as ulcerative colitis; however, only rectum is involved
      • Active mucosal inflammation (J Clin Pathol 1995;48:749):
        • Cryptitis (neutrophilic infiltration of crypt epithelium)
        • Crypt abscess (crypt with intraluminal aggregates of neutrophils)
        • Erosions
        • Ulceration
      • Chronic mucosal inflammation (J Clin Pathol 1995;48:749):
        • Mucosal expansion: increased lymphoplasmacytic inflammation in the lamina propria with loss of gradient (i.e., lack of increased infiltrates near lumen)
        • Basal plasmacytosis (so called crypt shortfall)
      • Evidence of chronic injury:
        • Paneth cell metaplasia (Paneth cells are normally absent distal to the mid transverse colon)
        • Crypt architectural distortion (branched, dilated and disorganized crypts)
      • Cryptolytic granulomas (granulomas in association with ruptured crypts) may be seen (Histopathology 2002;41:50)
      • Inactive (quiescent) proctitis in treated ulcerative proctitis may show subtle signs of chronicity (mild architecture distortion, focal Paneth cell metaplasia) or appear with near normal histology
      • Dysplasia / carcinoma
    Microscopic (histologic) images

    Contributed by John D. Paulsen, M.D. and Alexandros D. Polydorides, M.D., Ph.D.
    Ulcerative proctitis, mucosal biopsy

    Ulcerative proctitis, mucosal biopsy

    Cryptitis

    Cryptitis

    Crypt abscesses

    Crypt abscesses

    Inactive (quiescent) proctitis

    Inactive (quiescent) proctitis

    Ulcerative proctitis, proctectomy specimen

    Ulcerative proctitis, proctectomy specimen

    Sample pathology report
    • Rectum, endoscopic biopsy:
      • Moderately active chronic proctitis; negative for dysplasia (see comment)
      • Comment: The proctitis is characterized by cryptitis and crypt abscesses, mucosal expansion by full thickness lymphoplasmacytosis, which is most prominent basally, crypt distortion, including cystic dilation and crypt shortfall and evidence of chronic mucosal injury, including atrophy and Paneth cell metaplasia. In the appropriate clinical setting, these findings are suggestive of idiopathic inflammatory bowel disease.
    • Rectum, completion proctectomy:
      • Segment of rectum with mildly active chronic idiopathic ulcerative proctitis; negative for dysplasia (see comment)
      • Reactive lymph nodes
      • Comment: There is also prominent follicular lymphoid hyperplasia, possibly representing a (superimposed) component of diversion proctitis.
    Differential diagnosis
    • Crohn's disease:
      • Nonnecrotizing epithelioid cell granulomas (not associated with ruptured crypts)
      • Transmural lymphoid aggregates in areas of intact (nonulcerated) mucosa
      • Fistulas or strictures
      • Perianal disease
    • Sexually transmitted proctitis (i.e., syphilis and lymphogranuloma venereum):
      • Mucosal expansion by plasma cells and histiocytes
      • Granulomas (rare)
      • Relatively less crypt architecture distortion and Paneth cell metaplasia when compared to ulcerative proctitis (Am J Clin Pathol 2015;144:771)
      • Positive spirochete immunohistochemical stain in cases of syphilis proctitis
      • Correlation with gold standard clinical tests (rapid plasma reagin [RPR], Treponema specific serology, nucleic acid amplification [NAAT] using rectal swabs)
      • Clinical history of receptive anal intercourse
    • Acute infectious (self limited) proctitis:
      • Neutrophils predominate in lamina propria
      • Lack of crypt architecture distortion
      • Lack of basal plasmacytosis
      • Lack of Paneth cell metaplasia
      • Correlation with clinical tests on stool samples (such as multiplex PCR panels)
    • CMV proctitis:
      • May complicate ulcerative proctitis in patients on immunosuppressive treatment
      • Endothelial cells, mesenchymal stromal cells or glandular epithelial cells may show viral cytopathic effect, including enlargement (cytomegaly) and inclusions (owl eye intranuclear inclusions or pink cytoplasmic inclusions)
      • CMV immunohistochemical stain is an extremely useful adjunct
    • Diversion proctitis:
      • Prominent mucosal / submucosal lymphoid hyperplasia
      • History of fecal stream diversion (i.e., ileostomy or colostomy proximal to the rectum)
      • Often seen in completion proctectomy specimens in patients with idiopathic inflammatory bowel disease undergoing multistage ileal pouch anal anastomosis (IPAA) procedures
    • Immune checkpoint inhibitor associated proctitis:
      • Several patterns of injury have been described, including active colitis, lymphocytic colitis, collagenous colitis and apoptosis with increased intraepithelial apoptosis, apoptotic crypt abscesses and multifocal crypt injury (withered / shrunken crypts) (Am J Surg Pathol 2022;46:e15)
      • Mucosal expansion, crypt distortion, cryptitis and crypt abscesses may also be seen, mimicking idiopathic ulcerative proctitis
      • Clinical history of immune checkpoint inhibitor use
    • Ischemic proctitis:
      • Uncommon in rectum due to dual blood supply from inferior mesenteric and iliac arteries
      • Surface active inflammation / necrosis
      • Hyalinized lamina propria
      • Withered / shrunken crypts
      • Ischemic type changes may be seen in stercoral ulceration, trauma and solitary rectal ulcer syndrome
    • Solitary rectal ulcer syndrome (SRUS):
      • Microscopic changes imparted by rectal prolapse: fibromuscular hyperplasia of lamina propria splaying basal crypts (resulting in angulated or diamond shaped crypts), ectatic and congested subepithelial capillaries, surface ulceration, pseudomembranes
    • Stercoral ulcer:
      • Ulceration secondary to impacted fecal material
      • Microscopically shows ulcer and focal ischemic changes variable embedded fecal material
      • Background mucosa should be relatively uninflamed
    Board review style question #1

    A 63 year old woman presents to her primary care physician with a 3 month history of bloody diarrhea and tenesmus. She is referred for a colonoscopy, which demonstrates an erythematous friable mucosa with erosions in the rectum, as demonstrated in the figure above. Biopsies are performed. Which of the following histologic findings would most suggest idiopathic ulcerative proctitis in this case?

    1. Follicular lymphoid hyperplasia and cryptitis
    2. Increased intraepithelial apoptosis and multifocal crypt injury
    3. Lamina propria hyalinization and withered / shrunken crypts
    4. Mucosal expansion, crypt architecture distortion and crypt abscesses
    5. Prominent lamina neutrophils and preserved crypt architecture
    Board review style answer #1
    D. Mucosal expansion, crypt architecture distortion and crypt abscesses are all features characteristic of idiopathic ulcerative proctitis. Answer A describes diversion proctitis. Answer B describes histology that can be seen in immune checkpoint inhibitor associated colitis. Answer C describes ischemic proctitis. Answer E describes acute infectious (self limited) proctitis.

    Comment Here

    Reference: Ulcerative proctitis
    Board review style question #2
    A 22 year old man with a history of ulcerative proctitis undergoes surveillance colonoscopy. The rectum appears erythematous and granular. Biopsies are taken of the rectal mucosa and are sent for histopathologic evaluation. Which of the following, if found in the biopsy, would provide the best histologic evidence of active disease?

    1. Basal plasmacytosis (crypt shortfall)
    2. Crypt architecture distortion
    3. Cryptitis
    4. Mucosal expansion
    5. Paneth cell metaplasia
    Board review style answer #2
    C. Cryptitis (neutrophilic infiltration of the crypt epithelium) is a form of active mucosal inflammation and is evidence of active disease. The other forms of injury listed (basal plasmacytosis, crypt architecture distortion, mucosal expansion and Paneth cell metaplasia) are evidence of chronicity in ulcerative proctitis.

    Comment Here

    Reference: Ulcerative proctitis

    Vascular ectasia
    Definition / general
    Essential features
    • Cause of lower GI tract bleeding
    • More common in right colon and in older patients
    • Can be subtle and focal on histology
    Terminology
    • Also called angiodysplasia, arteriovenous malformation
    Epidemiology
    • < 1% prevalence but accounts for 20% of patients with lower GI bleeding (#2 most common cause, after diverticulitis)
    • Incidence increases with age (J Clin Pathol 1982;35:824)
    Sites
    Etiology
    • Acquired changes in colonic extracellular matrix which distort veins and capillaries, disposing them to bleed
    • Changes may be secondary to chronic vascular obstruction
    Clinical features
    • Rectal bleeding, often in elderly
    • Bleeding episodes typically cease spontaneously but recur
    • May be associated with aortic stenosis or von Willebrand disease
    Diagnosis
    • Colonoscopy, angiography
    Case reports
    Treatment
    Clinical images

    Images hosted on other servers:

    Friable telangiectatic mucosal lesions

    Gross description
    • Tortuous dilation of multiple small submucosal and mucosal blood vessels
    • Easier to identify by angiography than in a surgical specimen unless injected with silicone rubber and cleared with methyl salicylate
    Microscopic (histologic) description
    • Dilated and thin walled vessels (arteries, veins and capillaries) in mucosa and submucosa, often clustered
    • Overlying mucosa may be eroded
    • Changes can be subtle and focal
    Microscopic (histologic) images

    Contributed by Raul S. Gonzalez, M.D.

    Vascular ectasia

    Sample pathology report
    • Ascending colon, resection:
      • Segment of colon with submucosal angiodysplasia and focal overlying mucosal erosions
      • Margins of resection unremarkable.
      • Two benign lymph nodes.
    Differential diagnosis
    • Colonic or anal varices:
      • Due to portal hypertension
    • Hemangioma:
      • Discrete lesion
    Board review style question #1

      Which of the following is true about vascular ectasia (angiodysplasia) of the colon?

    1. Can be found in roughly 20% of resected colons
    2. Indistinguishable from hemangioma
    3. More prevalent in younger patients
    4. Usually found in the right colon
    Board review style answer #1
    D. Usually found in the right colon

    Comment Here

    Reference: Vascular ectasia

    Vasculitis
    Definition / general
    Essential features
    • Essentially any systemic vasculitis can involve the GI tract, with varying histologic features
    Epidemiology
    • Depends on particular vasculitis
    Sites
    • Colon can be affected as well as remainder of gastrointestinal tract
    Clinical features
    • Colonic manifestations may range from clinically silent to causing intestinal ischemia
    Diagnosis
    • Small vessel vasculitis may be observed on biopsy, medium vessel vasculitis on resection
    Radiology images

    Images hosted on other servers:

    Occlusion of inferior mesenteric artery

    Small intestinal edema

    Clinical images

    Images hosted on other servers:

    Marked narrowing of sigmoid colon

    Circumferential sigmoidal ulceration

    Inflammation, submucosal hemorrhage, and small ulceration

    Hyperemic, edematous colonic mucosa

    Microscopic (histologic) images

    Contributed by Raul S. Gonzalez, M.D.
    Vasculitis Vasculitis

    Vasculitis

    Sample pathology report
    • Ascending colon, ulcer, biopsy:
      • Colonic mucosa with focal submucosal eosinophilic inflammation surrounding blood vessels, suggestive of involvement by the patient's known Churg-Strauss syndrome
    Differential diagnosis
    • Main consideration is what form of vasculitis is present
      • Requires careful pathologic analysis and clinical correlation
    • Crohn’s disease or ulcerative colitis:
      • Severe cases of inflammatory bowel disease may show focal inflammation of vessels as part of the overall inflammatory component
      • True vasculitis may rarely coexist (Semin Arthritis Rheum 2016;45:475)
    Additional references
    Board review style question #1
      Which of the following is true about vasculitis involving the gastrointestinal tract?

    1. It always occurs in the setting of inflammatory bowel disease
    2. It is sometimes clinically silent
    3. Medium vessel vasculitides can often be observed on biopsy
    4. The rest of the gastrointestinal tract is never affected
    Board review style answer #1
    B. It is sometimes clinically silent

    Comment Here

    Reference: Vasculitis

    Vibrio cholerae
    Definition / general
    • Infection by toxigenic strains of Vibrio cholerae O1 or V. cholerae O139
    Essential features
    • Infection is caused by ingestion of toxigenic bacteria in food or water
    • Most cases are asymptomatic or cause mild diarrhea
    • Severe disease consists of a precipitous onset of acute watery diarrhea that rapidly may lead to dehydration, shock, electrolyte disturbances, renal failure and death
    • Endemic and epidemic disease can occur
    • Public health measures to prevent disease are centered on providing safe water and sanitation with vaccination in disease "hotspots" and in high risk individuals
    • Oral rehydration is the mainstay of treatment; in severe cases intravenous fluid and antibiotics are also employed
    ICD coding
    Epidemiology
    • World Health Organization estimates that there are between 1.3 and 4.0 million cases annually with 21,000 - 143,000 deaths (WHO: Cholera [Accessed 15 February 2018])
    • Centers for Disease Control and Prevention estimates there are between 3 and 5 million cases annually with over 100,000 deaths (CDC: Cholera - Vibrio cholerae Infection [Accessed 15 February 2018])
    • V. cholerae has an aquatic reservoir and attaches to algae, crustacean shells or zooplankton
    • It can be metabolically active or be dormant during adverse conditions
    • Bacteria may infect seafood or be spread to human water sources causing disease; fecal oral spread also occurs
    • V. cholerae is killed by high temperatures and tolerates acid poorly
    • In the developed world, cholera is very rare
    • Endemic disease is defined as disease that occurs during 3 of the past 5 years where there is no evidence of imported disease
      • Cholera is endemic in around 50 countries mostly in Africa, South and Southeast Asia
    • Outbreaks or epidemics are defined by the occurrence of at least 1 confirmed case with endemic local transmission where the disease is not usually encountered
    • Endemic disease:
      • Affects populations with preexisting immunity
      • More often affects children from 2 to 15 years old
      • May be spread by ingesting food or water or the fecal oral route
      • Has an aquatic reservoir
      • Is more likely to be asymptomatic
    • Epidemic disease:
      • Affects all ages of a nonimmune population where the bacteria does not have a reservoir
      • Spread is fecal oral with high secondary spread
      • Asymptomatic infection is less common
    • Disease is most common in the developing world; in the developed world disease is generally encountered in travelers and immigrants
    • Rarely disease occurs from ingesting contaminated seafood, a phenomenon rarely associated with seafood from the Gulf of Mexico
    • Hypochlorhydria caused by drugs or chronic Helicobacter pylori infection is a risk factor, as is HIV infection
    • Infection is more common in summer months
    • In endemic areas, breast milk contains protective IgA antibodies
    Sites
    • Small intestine
    Pathophysiology
    • Symptomatology occurs due to production of an exotoxin encoded by a virulence phage that causes the small intestine to secrete fluids and electrolytes
    • V. cholerae is a noninvasive pathogen
    • Toxin has two A subunits and five B subunits
      • Two A subunits, A1 and A2, are linked by a disulfide bond
    • Through several steps, intracellular cyclic adenosine monophosphate opens the cystic fibrosis transmembrane regulator and chloride ions are released into the lumen while chloride and sodium ion reabsorption is inhibited by cAMP
    • There is affinity of the toxin to the ganglioside receptor on enterocytes that is higher in patients with blood type O
    • Hemagglutinin is necessary for bacterial detachment and shedding in stool
    Etiology
    • Infection by toxigenic strains of Vibrio cholerae O1 or V. cholerae O139
    Clinical features
    • Most cases are asymptomatic or cause mild diarrhea
    • Symptomatic cases have an incubation period of 12 hours to 5 days
    • Severe disease is characterized by sudden onset of profuse watery diarrhea often described as having a "rice water" character sometimes preceded and usually followed by vomiting
    • Patients are anxious and rapidly become dehydrated with sunken eyes, dry mucous membranes, thirst and loss of skin turgor
    • Other signs and symptoms include tachycardia, hypotension and muscle cramps
    • Untreated cases may rapidly progress to electrolyte disturbances, renal failure, shock and death
    • Rarely "cholera sicca" occurs where patients have ileus and abdominal distention but lack diarrhea; fever is uncommon
    • Children may suffer from hypoglycemia, seizures, fever and altered mental status
    • Coinfection with other pathogens may occur
    Diagnosis
    • Microbiologic culture is the gold standard and may confirm other testing methods
    • PCR and real time nucleic acid sequencing are available
    • In epidemic settings, bacteria have characteristic chaotic movements when viewed by darkfield microscopy and an immunochromographic dipstick test is available
    • Testing to determine serotype and antibiotic sensitivity is necessary
    • In the United States, all cell isolates should be sent to the CDC via state health departments
    Prognostic factors
    • Elderly and debilitated patients have a worse prognosis
    • Pregnant patients do worse and have a high rate of fetal loss
    • HIV infection confers an increased risk of acquiring disease
    • Subjects with blood group O are at lower risk of acquiring disease but it is worse when it occurs
    Case reports
    Treatment
    • Most patients are successfully treated with oral rehydration solution (ORS); treatment should begin promptly
    • Patients with moderate to severe disease or who cannot tolerate ORS should receive intravenous hydration
    • With severe disease, antibiotic treatment is indicated to reduce fluid requirements and disease duration
    • Quinolones, trimethoprim sulfamethoxazole, tetracycline, doxycycline, erythromycin and azithromycin have been used; antibiotic resistance is common and antimicrobial sensitivity should guide treatment
    • Three oral vaccines are effective; vaccinations should be given to populations in disease "hotspots" and travellers at high risk of acquiring disease
    • Zinc supplementation reduces disease severity in children in resource limited areas
    Clinical images

    Images hosted on other servers:

    Adult with severe dehydration due to cholera

    Gross description
    • Gross changes are rarely encountered
    Microscopic (histologic) description
    • Biopsy is rarely performed
    • Mucosa is normal or may show a mild neutrophilic infiltrate of the lamina propria
    Videos

    Small intestine - cholera and Vibrio cholerae

    Differential diagnosis
    • Clinically mild to moderate disease mimics many types of gastroenteritis but the precipitous onset of profuse watery diarrhea is very characteristic of severe cholera
    Board review style question #1
    Which statement regarding Vibrio cholerae infection is correct?

    1. Severe disease is due to bloody diarrhea resembling inflammatory bowel disease
    2. Most cases are asymptomatic or cause mild diarrhea
    3. Antibiotics are the initial treatment in most cases
    4. Infection is primarily caused by exposure to household contacts
    Board review style answer #1
    B. Most cases are asymptomatic or cause mild diarrhea. A is incorrect: severe disease consists of a precipitous onset of acute watery diarrhea that rapidly may lead to dehydration, shock, electrolyte disturbances, renal failure and death. C is incorrect: oral rehydration is the mainstay of treatment; in severe cases intravenous fluid and antibiotics are also employed. D is incorrect: infection is caused by ingestion of toxigenic bacteria in food or water.

    Comment Here

    Reference: Vibrio cholerae

    Visceral myopathy (pending)
    Table of Contents
    Definition / general
    Definition / general
    [Pending]

    Volvulus
    Definition / general
    • Complete twisting of loop of bowel around its mesenteric base of attachment, causing obstruction and infarction
    Essential features
    • Clinical torsion of colon leading to colonic necrosis
    • High mortality rate
    Terminology
    • Also called torsion (volvulus is a more specific term referring to gastrointestinal torsion)
    Sites
    Etiology
    • Potentially due to adhesions from prior surgery
    Clinical features
    • More common in men
    • Patients typically demonstrate abdominal pain, distension, hyperactive bowel sounds
    • High rate of mortality, particularly in older patients (Eur Rev Med Pharmacol Sci 2013;17:127)
    Radiology description
    • Dilated loops of intestine
    • Volvulus may be visible on CT
    Radiology images

    Images hosted on other servers:

    Volvulus as seen on CT

    Case reports
    • 16 year old boy with sigmoid volvulus with an associated anomalous congenital band (Am Surg 2013;79:1140)
    Treatment
    Gross description
    Microscopic (histologic) description
    • Colon may be ischemic or gangrenous
    Sample pathology report
    • Cecum and ascending colon, resection:
      • Segment of colon with patchy transmural necrosis and serosal adhesions (see comment)
      • Necrosis extends to distal resection margin; proximal margin unremarkable.
      • Four benign lymph nodes.
      • Comment: The findings are consistent with the clinical impression of volvulus as reported in the operative note.
    Board review style question #1
    Which of the following is true about volvulus of the colon?

    1. It can only be identified via surgical exploration
    2. It has a low rate of morbidity / mortality
    3. The affected colon appears normal microscopically
    4. The cecum is a common site
    Board review style answer #1
    D. The cecum is a common site

    Comment Here

    Reference: Volvulus

    WHO classification
    Table of Contents
    Definition / general | WHO (2019)
    Definition / general
    • WHO classification of tumors of the colon and rectum
    • Currently on 5th edition, published in 2019
    • Based on histologic appearance, not molecular characteristics
    WHO (2019)


    Xanthoma
    Definition / general
    • Localized mass forming collection of lipid bearing histiocytes
    Essential features
    • Rare but typically incidental and benign polypoid collection of histiocytes
    Terminology
    • Also called xanthomatous polyp, xanthogranulomatous inflammation, xanthelasma
    Epidemiology
    • Very rare in colon - stomach is most common gastrointestinal site
    Sites
    • Colonic examples almost always rectosigmoid
    Etiology
    • May be due to prior injury
    Clinical features
    • Colon polyp
    • May clinically resemble carcinoma
    • May cause intestinal obstruction if more extensive and involving deeper layers
    Case reports
    • 38 year old woman with xanthogranulomatous inflammation presenting as a submucosal mass of the sigmoid colon (Pathol Int 2005;55:440)
    • 68 year old man with xanthoma disseminatum with large plaques confined to the back, pulmonary involvement and multiple intestinal xanthomas (Dermatology 2004;208:164)
    Microscopic (histologic) description
    • Collection of foamy cells without mucin, typically limited to lamina propria
    • Associated with surface hyperplastic change (APMIS 2004;112:3)
    Microscopic (histologic) images

    Contributed by Raul S. Gonzalez, M.D. and Jian-Hua Qiao, M.D.

    Xanthoma

    Prominent clusters of foamy histiocytes in colon polyp

    Positive stains
    Negative stains
    Electron microscopy description
    • Electron dense globules
    Sample pathology report
    • Descending colon, polypectomy:
      • Xanthoma
    Differential diagnosis
    Board review style question #1

      Xanthomas of the colon are positive for which of the following stains?

    1. CD1a
    2. CD68
    3. S100
    4. Mucicarmine
    Board review style answer #1
    B. CD68

    Comment Here

    Reference: Xanthoma

    Yersiniosis
    Definition / general
    • Infection by Yersinia enterocolitica and less commonly Y. pseudotuberculosis, pleomorphic, facultative gram negative bacilli from the family Enterobacteriaceae
    Essential features
    • Zoonotic infection whose reservoir consists of a variety of domestic and wild animals
    • Infection is usually acquired by ingesting the microorganisms; disease rarely occurs after blood transfusion
    • Vast majority of cases occur after ingestion of contaminated food or water; infection rarely occurs subsequent to contact with animals or patients
    • Most common in childhood
    • Most common manifestation is enterocolitis lasting one to three weeks with fever, diarrhea and abdominal pain; other patients present with mesenteric lymphadenitis or terminal ileitis with symptomatology mimicking acute appendicitis that includes right lower quadrant pain and tenderness with leukocytosis; involvement of the appendix may occur
    • Associated suppurative pharyngitis is not uncommon and may occur in the absence of gastrointestinal illness
    • Most cases are self limiting but on occasion, mesenteric vein thrombosis, bowel rupture or sepsis may occur, especially in patients with comorbidities
    • Noninfectious complications that occur in a minority of patients include reactive polyarthritis and polyarteritis nodosa
    • Relatively common in parts of northern Europe
    • Uncommon in the U.S., it is NOT an important cause of tropical diarrhea
    ICD coding
    • ICD-10:
      • A04.6 - enteritis due to Yersinia enterocolitica
      • A04.8 - other specified bacterial intestinal infections
    Epidemiology
    • Y. enterocolitica is widely distributed
      • Animal reservoir includes pigs, rodents, birds, cattle, horses, sheep, goats, foxes, porcupines, dogs and cats
      • Uncommon in U.S., the CDC estimates it causes 117,000 illnesses, 640 hospitalizations and 35 deaths in U.S. annually (CDC - Yersiniosis) with an incidence of 0.28 cases per 100,000 population in 2014 (CDC - Yersiniosis)
      • More common in parts of Northern Europe including Germany, Denmark and Finland
      • Does occur in the tropics but it is not considered an important cause of tropical diarrhea
      • Has over 60 serotypes based on O and H antigens and 6 biotypes
        • Most pathologic strains are serotypes O:3, O:5.27, O:8 and O:9 and biotypes 2, 3 and 4
        • Serotypes O:3 and O:9 cause most disease in Europe, serotypes O:3 and O:8 cause most disease in U.S. and Canada
        • Y. pseudotuberculosis has a separate system; there are 6 serotypes and 4 subtypes
    • Infection is usually fecal-oral and occurs after ingestion of contaminated food or water; however, occurs less commonly after contact with an infected animal or carcass
      • Butchers and workers in slaughterhouses are at increased risk
    • Disease has occurred after contact with feces from household pets and from environmental surfaces
    • Yersinia can grow at 4°C and infections have occurred after blood transfusion
    • Outbreaks have been linked to contaminated milk (pasteurized and unpasteurized), raw vegetables and chitterlings (raw pork intestines)
    • Rarely, transmission from a mother to her newborn has occurred
    • Y. enterocolitica has been cultured from lakes, streams and drinking water; however, this is only a rare cause of disease
    • Infections in the northern hemisphere are more common in November through January
    • Children especially under 4 years are affected more commonly than adults
    • Y. pseudotuberculosis is widely distributed and is widespread in nature; most reports of disease are from Europe
      • Animal reservoir includes rodents, deer, birds and farm animals
      • More common in winter
      • Outbreaks have been linked to contaminated milk and produce
      • Patients are usually 5 to 14 years old and males are affected three times more frequently than females
    • A chronic carrier state for Yersinia is not described but patients may shed bacteria for weeks to months after yersiniosis
    Sites
    • Most disease in children under 4 is associated with an enterocolitis affecting the terminal ileum and proximal colon
    • Disease in older patients generally involves the terminal ileum and mesenteric lymph nodes and may affect the appendix
    • A minority of patients have suppurative pharyngitis, rarely without gastrointestinal illness
    • A reactive polyarthritis occurs in 15% of patients, especially those with HLA B27
    • Polyarteritis nodosa is an uncommon complication
    • In septic patients, involvement of the lungs, liver, spleen and central nervous system has been reported
    Pathophysiology
    • An inoculum of 109 organisms is believed necessary to cause disease that occurs after a 4 to 7 day incubation period
    • Microorganisms invade the intestine via M cells, which are involved in antigen presentation and are prominent over Peyer patches and prefer sites dense with lymphoid tissue, accounting for predilection for the terminal ileum, proximal colon and occasionally the pharynx
    • Invasive proteins and enterotoxin are produced
    • Plasmid mediated virulence factors include resistance to phagocytosis by neutrophils, interference with platelet aggregation, interference with complement activation and suppression of tumor necrosis factor α which causes enterocytes to undergo apoptosis
    • Invasion of the mucosa leads to active, chronic or granulomatous inflammation with necrosis and ulceration that varies in extent depending on the severity of the disease
    • Yersiniae are unable to chelate iron and rely on siderophores produced by other bacteria
    • Iron overload as seen in chronic hemolytic states greatly increases the pathogenicity of Yersinia infections and patients being treated with deferoxamine are at particularly high risk
    • Reactive polyarthritis is likely related to molecular mimicry and is especially common in those with HLA B27
    Clinical features
    • Necrotizing enterocolitis has been described in infants
    • Children under 4 usually suffer an acute self limited enterocolitis lasting 1 to 3 weeks associated with fever, diarrhea, nausea and vomiting in 15 to 40% of cases
    • In severe cases, bloody diarrhea or (rarely) ileal perforation may occur
    • In older patients. the terminal ileum and mesenteric lymph nodes are involved, causing right lower quadrant pain and tenderness with leukocytosis that mimics acute appendicitis that in many cases has led to appendectomies
    • The appendix may be uninvolved or show variable inflammatory changes, including granulomatous appendicitis
    • Exudative pharyngitis occurs in up to 8% of patients, sometimes without gastrointestinal disease
    • A reactive polyarthritis occurs in approximately 15% of patients and is especially common in patients with HLA B27; it may last for months but eventually resolves
    • Polyarteritis nodosa occurs in up to 30% of cases, especially in women; it generally resolves within one month
    • It is unusual for yersiniosis to last longer than a month but chronic infections occur
    • In severe cases, mesenteric vessel thrombosis with intestinal necrosis and hemorrhage may occur
    • Septicemia is more common in patients with comorbidities such as immunosuppression, underlying malignancy, old age or chronic hemolysis, especially with deferoxamine therapy and may involve the liver, spleen, lungs or central nervous system
    • Yersinia sepsis is often fatal even with antibiotic treatment
    • Y. pseudotuberculosis usually causes mesenteric lymphadenitis; it has been linked to a scarlet fever-like illness. and there are reports linking it to Kawasaki disease
    Diagnosis / laboratory
    • PCR based testing or isolation of the organism from stool (the CDC recommends cefsulodin-irgasan-novobiocin (CIN) agar and incubation at 25°C), lymph nodes, bile, pharyngeal exudate, blood, abscesses or peritoneal fluid
    • Serologic testing is available but generally not indicated in the clinical setting
    Radiology description
    • Imaging studies may show thickening of the terminal ileum and ascending colon with mesenteric lymphadenopathy
    • Severe disease may lead to bowel perforation with peritonitis and free air under the diaphragm
    Prognostic factors
    • Most patients suffer mild symptoms that are self limiting
    • Disease is more severe in immunocompromised patients, debilitated patients and patients with iron overload, especially with deferoxamine therapy
    Case reports
    Treatment
    • Self limited infections are usually not treated
    • Patients with persistent infection or sepsis usually receive aminoglycosides, sulfamethoxazole / trimethoprim, doxycycline or ciprofloxacin
    • Patients with persistent infection or sepsis due to Y. pseudotuberculosis are usually treated with ampicillin, tetracycline or an aminoglycoside
    • Patients with reactive arthritis usually are treated with NSAIDs, intra-articular corticosteroid injections and physical therapy
    Gross description
    • In uncomplicated cases, the bowel wall is thickened and edematous and demonstrates ulcers over Peyer patches or lymphoid aggregates
    Microscopic (histologic) description
    • Affected mucosa variably demonstrates lymphoid hyperplasia, cryptitis and crypt abscesses, generally without significant architectural changes typical of acute self limited colitis and granulomas with central abscess formation (suppurative granulomas)
    • Chronic infection may be associated with architectural changes
    • Lymphoid hyperplasia, neutrophilic infiltrates and suppurative granulomas may involve mesenteric lymph nodes
    Microscopic (histologic) images

    Contributed by Elliot Weisenberg, M.D.

    Lymphoid hyperplasia and suppurative granulomas in the appendix and a mesenteric lymph node of a 16 year old boy with symptoms of acute appendicitis



    Images hosted on other servers:

    Granulomas

    Suppurative granuloma

    Mesenteric lymphadenitis

    Back to top
    Recent Colon Pathology books

    Arnold: 2019

    Arnold: 2014

    Greenson: 2019

    IARC: 2019

    Jouret-Mourin: 2018

    Montgomery: 2017

    Montgomery: 2017

    Odze: 2022

    Srivastava: 2023

    Yantiss: 2021



    Find related Pathology books: GI, liver
    Image 01 Image 02